Hechos Geométricos en el Triángulo (2013-14)

Angel Montesdeoca

(Última actualización: )

Hechos Geométricos en el Triángulo
(2013) (2014) (2015) (2016) (2017) (2018) (2019) (2020) (2021) (2022) (2023) (2024)

Cómo es el enlace a un Hecho Geométrico correspondiente a un día concreto:
http://amontes.webs.ull.es/otrashtm/HechosGeometricos.htm#HGddmmaa
EJEMPLO: Domingo, 3 de noviembre del 2013
http://amontes.webs.ull.es/otrashtm/HechosGeometricos.htm#HG031113

  • martes 23 de diciembre del 2014

    Triángulos pedales ortológicos. Cúbica K389. Cuártica Q098


      If P and Q are two points collinear with the circumcenter O of ABC, then the pedal triangles of P and Q wrt ABC are orthologic.
    (Luis González. Art of Problem Solving)

      If P and Q are points with the midpoint O, the circumcenter, then their pedal triangles are orthologic and their orthology centers share the same midpoint O. (Luis González. Art of Problem Solving)

      Si ABC es triángulo, se denota por O el circuncentro, por PaPbPc y por QaQbQc los triángulos pedales de dos puntos P y Q, respectivamente.

      Dado un punto P, el lugar geométrico de los puntos Q tal que los triángulos pedales de P y Q son ortológicos es la recta OP.


      Si P(p:q:r), las coordenadas baricéntricas de Pa son:

    (0 : (b^2-c^2)p + a^2(p+2q) : (-b^2+c^2)p + a^2(p+2r)).

      La perpendicular por Pa al lado XbXc, si Q(u:v:w), es:

    (b^2-c^2)p(c^2v+b^2w)+a^2(c^2(p+2q)v-b^2(p+2r)w)x -
    ((-b^2+c^2)p+a^2(p+2r))(c^2v+b^2w)y +
    ((b^2-c^2)p+a^2(p+2q))(c^2v+b^2w)z = 0.

    Y análogamente se obtienen las perpendiculares por Pb al lado XcXa y por Pc al lado XaXb.
      Estas tres perpendiculares son concurrentes (PaPbPc y QaQbQc son ortológicos) si y sólo si

    a^2(b^2r-c^2q) - (b^2-c^2)(c^2q+b^2r)u+... =0,

    es decir, el punto Q está en la recta OP.


      Sean P un punto con triángulo pedal PaPbPc y Q un punto variable sobre la recta OP, con triángulo pedal QaQbQc.

     • El lugar geométrico del centro de ortología X de QaQbQc respecto a PaPbPc es una recta (L) que pasa por el ortocentro HP de PaPbPc.
      La recta (L) pasa por O si y solo si P está en la cúbica K389.
      La recta (L) pasa por P si y solo si P está en la cuártica Q098.

     • El lugar geométrico del centro de ortología Y de PaPbPc respecto a QaQbQc es la hipérbola equilátera (H) circunscrita a PaPbPc que pasa por P.

      Si P(p:q:r), la ecuación de la recta (L), lugar geométrico del centro de ortología X de QaQbQc respecto a PaPbPc, es

    (L):    (b^2SBr-c^2SCq)x ⁄ (c^2q+b^2r)+... = 0.


      La recta (L) pasa por el ortocentro HP de PaPbPc:

    HP (a^2(c^2q+b^2r) (SA(c^2p q+b^2p r+a^2q r) - b^2c^2p(p+q+r)): ... : ... )


      La cúbica K389 ("Kosnita nodal cubic") es el lugar geométrico de los puntos P tales que la recta (L) pasa por el circuncentro.

      Si Q=P' es el simétrico de P respecto O, X e Y son simétricos respecto a O e Y es el segundo punto de interseccion D de (L) y (H).
    ( Mostrar/Ocultar figura )
      TriangulosPedalesOrtologicosK389.png
    Descargar fichero GeoGebra


      La cuártica Q098 ("An inversible circular quartic, the isogonal transform of Q044") es el lugar geométrico de los puntos P tales que la recta (L) pasa por P.
    ( Mostrar/Ocultar figura )
      TriangulosPedalesOrtologicosQ098.png
    Descargar fichero GeoGebra


    C A S O   G E N E R A L

      El lugar geométrico de los puntos P tales que la recta (L) pasa por el punto P0, que divide al segmento OP en la razón OP0 : P0P = m : n, es la cuártica

    m a^2b^2c^2 Q098 + n R^2 (x+y+z) K389 = 0.

    R es el radio de la circunferencia circunscrita a ABC. Todas las cuárticas de este haz están circunscritas a ABC, tiene al circuncentro como punto singular, pasan por los vértices del triángulo tangencial y por los puntos en el infinito de la hipérbola de Jerabek.
    ( Mostrar/Ocultar figura )
      TriangulosPedalesOrtologicosmn.png
    Descargar fichero GeoGebra


    CASO m:n=-1:1
      El lugar geométrico de los puntos P tales que las rectas OP y (L) son paralelas es una cuártica con punto triple en el circuncentro.
    Pasa además por los centros X186, X1319, X1691, X2574, X2575 (estos últimos en el infinito, conjugados isogonales de los puntos en que la recta de Euler corta a la circunferencia circunscrita).
    ( Mostrar/Ocultar figura )
      TriangulosPedalesOrtologicosQnnn.png
    Descargar fichero GeoGebra


    CASO m:n=-1:2
      El lugar geométrico de los puntos P tales que la recta (L) pasa por el simétrico de P respecto al circuncentro es una cuártica, que pasa por los centros X3, X1498, X2574, X2575, X3532.
    ( Mostrar/Ocultar figura )
      TriangulosPedalesOrtologicosQnnnb.png
    Descargar fichero GeoGebra


  • viernes 19 de diciembre del 2014

    Cúbicas circulares circunscritas con punto nodal en el ortocentro


      Si ABC es triángulo, sean DEF su triángulo medial y A'B'C' su triángulo antimedial. Dado un punto X, sean X' su anticomplemento y Xa , Xb, Xc las reflexiones de X' en D, E, F, respectivamente,

      • Las circunferencias circunscritas a los triángulos BCXa, CAXb y ABXc son concurrentes en un punto Y, sobre la circunferencia circunscrita al triángulo XaXbXc.
    ( Mostrar/Ocultar figura )
      CircularNodalCubic.png

      Como los triángulos ABC y XaXbXc son simétricos respecto a X, ellos son ciclológicos.
      Un centro ciclológico es Y, el cuarto punto de intersección de la circunferencia circunscrita a XaXbXc con la cónica circunscrita a ABC con centro en X.
      El otro centro ciclológico, Z, es el cuarto punto de intersección de la circunferencia circunscrita a ABC con la citada cónica.

      Si X=(u:v:w), las coordenadas baricéntricas de Y son:

    F(X) = Y = ((u-v-w) /(2 a^2 v w+(b^2 w+c^2 v) (u-v-w)) : ... : ...)

    Para X el circuncentro, F(X) no está definido. Si X está en el infinito F(X)=X

      • Si X recorre una recta d: px+qy+rz=0, que no pasa por el circuncentro, F(X) queda sobre en una quíntica circunscrita con puntos dobles en los vértices de ABC y en el ortocentro, con una asíntota paralela a d.
    ( Mostrar/Ocultar figura )
      CircularNodalCubic0.png


    ( Hyacinthos #9381, Jean-Pierre Ehrmann)
      Los puntos Xa , Xb, Xc son las reflexiones de A, B, C en X, respectivamente. Al punto Y, de concurrencia de las circunferencias circunscritas a los triángulos BCXa, CAXb y ABXc, se le denomina "syngonal conjugate" of X.
      Así, Y=F(X) es el antigonal (conjugado isogonal del inverso en la circunferencia circunscrita del conjugado isogonal) del anticomplento de X.

      • La imagen de una recta d que pasa por el circuncentro, mediante la aplicacion XF(X), es una cúbica circular Kd circunscrita a ABC con punto nodal el ortocentro.
    ( Mostrar/Ocultar figura )
      CircularNodalCubicOP.png

      Si la recta d corta a BC en el punto que divide al segmento BC en la razón m:n, el lugar geométrico de Y=F(X) cuando X se mueve sobre d es la cúbica K(d):

    2x(SB((a^2-b^2-a c)(a^2-b^2+a c)m+ b^2c^2n) y^2+SC(- b^2 c^2m-(a^2-a b-c^2) (a^2+a b-c^2)n)z^2)+
    2y(SC(a^2 c^2m-a^2 (a^2-b^2)n) z^2 +SA(-(a^2-b^2-b c)(a^2-b^2+b c)m -c^2 (b^2-c^2)n)x^2)+
    2z(SA(-b^2 (b^2-c^2)m+ (a^2-b c-c^2) (a^2+b c-c^2)n)x^2 +SB(a^2(a^2-c^2)m- a^2 b^2 n)y^2 )+
    ((a^2-b^2)(a^4+b^4-c^4)m-(a^2-c^2)(a^4-b^4+c^4)n) x y z =0,

    O bien, en función de las coordenadas (p:q:r), p+q+r=0, de su punto del infinito:

    2 x (SB (b^2 c^2 (a^2 q SA - b^2 p SB) + (a^2 - b^2 - a c) (a^2 - b^2 + a c) (a^2 r SA - c^2 p SC)) y^2 + SC (-(a^2 - a b - c^2) (a^2 + a b - c^2) (a^2 q SA - b^2 p SB) - b^2 c^2 (a^2 r SA - c^2 p SC)) z^2) +
    2 y (SA (-c^2 (b^2 - c^2) (a^2 q SA - b^2 p SB) + (a^2 - b^2 + b c) (-a^2 + b^2 + b c) (a^2 r SA - c^2 p SC)) x^2 + SC (-a^2 (a^2 - b^2) (a^2 q SA - b^2 p SB) + a^2 c^2 (a^2 r SA - c^2 p SC)) z^2) +
    2 z(SA ((a^2 - b c - c^2) (a^2 + b c - c^2) (a^2 q SA - b^2 p SB) - b^2 (b^2 - c^2) (a^2 r SA - c^2 p SC)) x^2 + SB (-a^2 b^2 (a^2 q SA - b^2 p SB) + a^2 (a^2 - c^2) (a^2 r SA - c^2 p SC)) y^2) +
    (-(a^2 - c^2) (a^4 - b^4 + c^4) (a^2 q SA - b^2 p SB) + (a^2 - b^2) (a^4 + b^4 - c^4) (a^2 r SA - c^2 p SC)) x y z = 0,
    donde SA=(b^2+c^2-a^2)/2,...

     - Si la recta d corta a la circunferencia de Euler en X1 y X2, el punto F(X1)=F(X2)=H es crunodal.
     - Si la recta d es tangente a la circunferencia de Euler en X, F(X)=H es tacnodal (de retroceso de primera especie).
     - Si la recta d no corta a la circunferencia de Euler, H es acnodal (aislado).

    Cúbicas K(d) particulares:

     • Si d es la recta de Euler, K(d) = K025 Ehrmann strophoid
    ( Mostrar/Ocultar figura )
      CircularNodalCubicK025.png

     • Si d es la recta de X3X523, K(d) = K186 Iona cubic
    ( Mostrar/Ocultar figura )
      CircularNodalCubicK186.png

     • Si d es la recta paralela por X3 a X4X6, K(d) = K288 Gigha cubic
    ( Mostrar/Ocultar figura )
      CircularNodalCubicK288.png

     • Si d es el eje de Brocard, K(d) = K289 Canna cubic
    ( Mostrar/Ocultar figura )
      CircularNodalCubicK289.png

     • Si d es la recta X3X1, la ecuación baricéntrica de K(d) es:

    c (a^2 - b^2 - c^2) (a^3 - a^2 b - a b^2 + b^3 + a b c - b c^2) x^2 y + c (a^2 - b^2 + c^2) (a^3 - a^2 b - a b^2 + b^3 + a b c - a c^2) x y^2 - b (a^2 - b^2 - c^2) (a^3 - a^2 c + a b c - b^2 c - a c^2 + c^3) x^2 z - (a - b) (a - c) (b - c) (a^3 + a^2 b + a b^2 + b^3 + a^2 c + 2 a b c + b^2 c + a c^2 + b c^2 + c^3) x y z + a (a^2 - b^2 + c^2) (-b^3 + a^2 c - a b c + b^2 c + b c^2 - c^3) y^2 z - b (a^2 + b^2 - c^2) (a^3 - a b^2 - a^2 c + a b c - a c^2 + c^3) x z^2 - a (a^2 + b^2 - c^2) (a^2 b - b^3 - a b c + b^2 c + b c^2 - c^3) y z^2=0.
    ( Mostrar/Ocultar figura )
      CircularNodalCubicOI.png



  • domingo 14 de diciembre del 2014

    Quártica Q23 y séptica asociada


    ( Anopolis #2155, Antreas P. Hatzipolakis)

      Sea ABC un triángulo, O su circuncentro y P un punto, el lugar geométrico de P tal que el ortocentro de su triángulo pedal queda en la recta OP es la cuártica Q023, del catálogo de Bernard Gibert).

      Si (x:y:z) son las coordenadas baricéntricas de P el ortocentro del triángulo pedal A'B'C' es:

    H' (a^2(c^2y+b^2z) (yza^4 + (b^2z(x-y)+c^2y(x-z))a^2 - x(b^4z+c^4y-b^2c^2(2x+y+z))) : ... : ...)

      Los puntos P que están alineados con H' y O (a^2(b^2+c^2-a^2):...:...) satisfacen a la ecuación de la cuártica Q023.
    ( Mostrar/Ocultar figura )
      Anopolis2155.png



      El lugar geométrico del ortocentro del triángulo pedal de un punto P que varía en la cuártica Q023 es una séptica (ecuación obtenida por Bernard Gibert), con el circuncentro como punto quíntuple, puntos dobles en los puntos medios de los lados de ABC y tangentes en los vértices las recta OA, OB y OC.

  • domingo 30 de noviembre del 2014

    Cúbica de Bataille como un caso particular

      Sean ABC un triángulo y DEF el triángulo ceviano de un punto U. Tomemos los puntos D1=BC∩EF, E1=CA∩FD y F1=AB∩DE y sus reflexiones D2, E2, F2 respecto a los puntos medios D0, E0, F0 de los lados de DEF.
      Sea Ka el punto de intersección de la paralela por D1 a AU con la recta AD0.

    NOTA:
      Las rectas KaD1 y KaD2 son las asíntotas de la hipérbola Φa tangente en E, F a AC, AB y con una asíntota paralela a AU. Así, lo expuesto da un procedimiento para construir la hipérbola dado la dirección de una asíntota y dos tangentes de dos de sus puntos. Ver otros en It_Pt_P, It_Pt_P2, (3P_12T)_2, este último de Paris Pamfilos, omitido en A Gallery of Conics by Five Elements. Forum Geometricorum, Volume 14 (2014) 295–348.

    ( Mostrar/Ocultar figura )
      BatailleCubicsdadbdc.png

      Se denota por da la recta paralela a KaD2 por A. Similarmente, se definen las rectas db y dc.

      Las tres rectas da, db y dc son concurrentes.

      Si U(u:v:w), las coordenadas baricéntricas del punto V, de intersección de las tres rectas da, db, dc, son:

    F(U) = V = (u(v+w)^2 : v(u+w)^2 : w(u+v)^2)


      A un punto U y a su conjugado isotómico U les corresponde el mismo punto V.

      Cuando U recorre la cónica circunscrita de perspector P(p:q:r), pyz+qzx+rxy=0, o la recta px+qy+rz=0, V=F(U) describe la cúbica:

    ( Mostrar/Ocultar figura )
      BatailleCubics.png

      El punto singular de esta cúbica es:

    D = (p/(q + r - p) : q/(r + p - q) : r/(p + q - r)).

    Es el punto que correponde, mediante la aplicación F, a los puntos de intersección de la cónica circunscrita de perspector P y la tripolar de P.

      Los puntos de intersección de las tripolares de P(p:q:r) y del punto (1/(-p+q+r)^2:1/(p-q+r)^2:1/(p+q-r)^2) con los lados de ABC, están en la cúbica.

    CASO PARTICULAR

      Si U(u:v:w) se mueve sobre la recta del infinito, x+y+z=0, o sobre la elipse circunscrita de Steiner, yz+zx+xy=0, V (u^3:v^3:w^3) o V (1/u^3:1/v^3:1/w^3) describe la cúbica de Bataille (K656):
    x^3 + 3 x^2 y + 3 x y^2 + y^3 + 3 x^2 z - 21 x y z + 3 y^2 z + 3 x z^2 + 3 y z^2 + z^3 =0
    ó (x + y + z)^3 = 27 x y z.

    El punto singular de K656 es el baricentro (aislado) y pasa por X3081=F(X30)=F(X1494).

  • sábado 29 de noviembre del 2014

    Cuadrados inscritos en un triángulo y cúbicas asociadas

    a Lolilla, por su "cumple"



      Sean ABC un triángulo, P un punto y DEF su triángulo ceviano, Se denota por Ai el centro del cuadrado inscrito en ABC con un lado sobre BC y los otros dos vértices sobre los segmentos AB y AC. Ae el centro del cuadrado inscrito en ABC con un lado sobre BC y los otros dos vértices fuera de los segmentos AB y AC. Similarmente, se define los centros Bi, Be, Ci, Ce.
    ( Mostrar/Ocultar figura )
      CentrosCuadradosInscritos.png

      En coordenadas baricéntricas, Ai=(a^2:SC+S:SB+S) y Ae=(a^2:SC-S:SB-S).

      Los triángulos AiBiCi y AeBeCe son perspectivos con centro de perspectividad el simediano.

    ( Mostrar/Ocultar figura )
      CuadradosInscritosABC.png

      El lugar geométrico de los puntos P tales que los triángulos DEF y AiBiCi son perspectivos es la cúbica K070-b, "Shoemaker's (inner) cubic" = pK(X4, X1585). El centro de perspectividad Qi queda sobre la cúbica isogonal de pivote X3068 (K424a, del catálogo de cúbicas de Bernard Gibert).

    ( Mostrar/Ocultar figura )
      CuadradosInscritosCevianoK070b.png
    Descargar fichero GeoGebra

      Si P=(p:q:r) la ecuación baricéntrica de la recta DAi es (q(S+SB)-r(S+SC))x + a^2ry - a^2qz = 0.
      Las rectas DAi, EBi, FCi son concurrentes si y solo si las coordenadas de P satisfacen a la ecuación: Dividiendo por S SASBSC: que es la ecuación de la cúbica pivotal pK(X4,X1585).

      La primera coordenada de Qi es:

    a^4(-p^2+p(q+r)+3q r) + 2a^2(p(c^2(p+q)+b^2(p+r))+2(p-q)(p-r)S) - b^4(p+q)(p-r) - c^4(p-q)(p+r) + 2b^2c^2(p^2-qr) - 4pS(c^2(p-q)+b^2(p-r))-4(p-q)(p-r)S^2

    Este punto está, cuando P=(p:q:r) recorre K070-b, en la cúbica K424a que pasa por los centros X(1), X(2), X(6), X(371), X(485), X(493), X(3068), por los excentros Ia, Ib, Ic y por los vértices de AiBiCi.


      Cambiando en las expresiones algebraicas anteriores S (doble del área de ABC) por -S, se obtiene:

      El lugar geométrico de los puntos P tales que los triángulo DEF y AeBeCe son perspectivos es la cúbica K070-a, "Shoemaker's (outer) cubic" = pK(X4, X1586). El centro de perspectividad Qe queda sobre la cúbica isogonal de pivote X3069 (K424b).

    ( Mostrar/Ocultar figura )
      CuadradosInscritosCevianoK070a.png
    Descargar fichero GeoGebra


    ∗  ∗  ∗  ∗  ∗  ∗  ∗  ∗  ∗  ∗ 



      Planteamos el estudio anterior sustituyendo el triángulo ceviano de P por su triángulo anticeviano.


      El lugar geométrico de los puntos P tales que su triángulo anticeviano DEF es perspectivo con AiBiCi y con AeBeCe, es la cúbica K006, "Orthocubic" = pK(X6, X4). Los centros de perspectividad Qi y Qe quedan, respectivamente, sobre la cúbicas isogonales K424a y K424b.

    ( Mostrar/Ocultar figura )
      CuadradosInscritosAntiCevianoK006.png
    Descargar fichero GeoGebra

    Los centros de perspectividad Qi y Qe están alineados con el baricentro y están sobre la hipérbola rectangular que pasa por P y circunscrita a su triángulo anticeviano DEF y al triángulo excentral IaIbIc.

  • lunes 24 de noviembre del 2014

    Cúbica asociada a una recta. Cúbica de Allardice

      Sean ABC un triángulo, U un punto y DEF su triángulo ceviano. Los puntos D1=BC∩EF, E1=CA∩FD y F1=AB∩DE están en el eje d de perspectividad ("perspectrix") de los triángulos ABC y DEF. Denotamos por d‍′ la recta que pasa por las reflexiones D2, E2, F2 de D1, E1, F1 respecto a los puntos medios de los lados de DEF
      Si U=(u:v:w), las coordenadas baricéntricas de V=d∩d‍′ son:

    V = (u(v-w)(2u+v+w) : v(w-v)(u+2v+w) : w(u-v)(u+v+2w)).

      El lugar geométrico del punto de intersección V de las la rectas d y d‍′, cuando U se mueve sobre una recta p : px+qy+rz=0 es la cúbica:

    ( Mostrar/Ocultar figura )
      HG241114.png
    Descargar fichero GeoGebra


      El punto singular de esta cúbica es S=(3p-2(q+r) : 3q-2(r+p) :3r-2(p+q)).
      Si P es el punto de coordenadas (p:q:r), es decir, el conjugado isotómico del polo trilineal de la recta p : px+qy+rz=0, el punto singular S es la imagen de P mediante la homotecia h(G,-5), de centro el baricentro y razón -5.

      La correspondencia

    p(p:q:r)  ↦  S(3p-2(q+r) : 3q-2(r+p) :3r-2(p+q))

    es una correlación. Si la recta p es tangente a la elipse x^2+y^2+z^2+4xy+4xz+4yz=0, homotética de la elipse inscrita de Steiner en la homotecia h(G,Sqrt[10]), el punto singular S es el punto de tangencia.
    ( Mostrar/Ocultar figura )
      HG241114T.png
    Descargar fichero GeoGebra


    CASOS PARTICULARES

     • Cuando el punto U está en la recta del infinito p : x+y+z=0, el punto V es el baricentro de las trazas (D1, E1, F1) de la polar trilineal de U en los lados de ABC. Así. el lugar geométrico de V es la cúbica de Allardice (K219, del catálogo de cúbicas de Bernard Gibert)

     • Cuando el punto U está en la recta de Euler el punto V describe la parábola, cuyo eje tiene la dirección de punto X525, de ecuación:

    (a^4+a^2(-3b^2+2c^2)+2b^4+b^2c^2-3c^4) (a^4+a^2(2b^2-3c^2)-3b^4+b^2c^2+2c^4)x^2+ (-12a^8 + 12a^6(b^2+c^2) + a^4(11b^4-34b^2c^2+11c^4) - 10a^2(b^2-c^2)^2(b^2+c^2) - (b^2-c^2)^2(b^4-10b^2c^2+c^4) )yz+.... =0
    ( Mostrar/Ocultar figura )
      HG241114Euler.png


     • Cuando el punto U está en la recta con coeficientes las coordenadas del ortocentro, p : x/SA+y/SB+z/SC=0, el punto V describe una cúbica, con punto doble X3529 y pasa por X1650 (ambos en la recta de Euler).
    ( Mostrar/Ocultar figura )
      HG241114X4.png


  • sábado 15 de noviembre del 2014

    Séptica que pasa por los "equilateral cevian points"



    ( Anopolis #2003, Antreas P. Hatzipolakis)

    Equilateral cevian points, X(370) and related curves (Bernard Gibert)
      A point P is an equilateral cevian point if the cevian triangle of P is equilateral (TCCT, p.267). The corresponding center of the triangle is denoted by Q. There are 6 such points P which lie on the Neuberg cubic: 4 are always real, one of them being X(370), the only one inside ABC. Each line through P and the corresponding Q passes through O.


      Sean ABC un triángulo, O el circuncentro, P un punto y A'B'C' el triángulo ceviano de P.
      El lugar geométrico de P, tal que el centro N' de la circunferencia de los nueve puntos de A'B'C' esté sobre la recta OP, es una séptica que pasa por los vértices de los triángulos ABC (triples), medial y excentral, y por X1, X2, X3, X370 (y los otros cinco -tres siempre reales- "equilateral cevian points"), X1138.


      Si P(u:v:w), la primera coordenada baricéntrica de N' es:

    a^2 c^2 u^3 v^3 - b^2 c^2 u^3 v^3 + c^4 u^3 v^3 - 3 a^4 u^3 v^2 w +
    6 a^2 b^2 u^3 v^2 w - 3 b^4 u^3 v^2 w + 5 a^2 c^2 u^3 v^2 w +
    5 b^2 c^2 u^3 v^2 w - 2 c^4 u^3 v^2 w - a^4 u^2 v^3 w +
    2 a^2 b^2 u^2 v^3 w - b^4 u^2 v^3 w + 4 a^2 c^2 u^2 v^3 w +
    2 b^2 c^2 u^2 v^3 w - c^4 u^2 v^3 w - 3 a^4 u^3 v w^2 +
    5 a^2 b^2 u^3 v w^2 - 2 b^4 u^3 v w^2 + 6 a^2 c^2 u^3 v w^2 +
    5 b^2 c^2 u^3 v w^2 - 3 c^4 u^3 v w^2 - 4 a^4 u^2 v^2 w^2 +
    6 a^2 b^2 u^2 v^2 w^2 - 2 b^4 u^2 v^2 w^2 + 6 a^2 c^2 u^2 v^2 w^2 +
    4 b^2 c^2 u^2 v^2 w^2 - 2 c^4 u^2 v^2 w^2 - a^4 u v^3 w^2 +
    a^2 b^2 u v^3 w^2 - a^2 c^2 u v^3 w^2 + a^2 b^2 u^3 w^3 +
    b^4 u^3 w^3 - b^2 c^2 u^3 w^3 - a^4 u^2 v w^3 + 4 a^2 b^2 u^2 v w^3 -
    b^4 u^2 v w^3 + 2 a^2 c^2 u^2 v w^3 + 2 b^2 c^2 u^2 v w^3 -
    c^4 u^2 v w^3 - a^4 u v^2 w^3 - a^2 b^2 u v^2 w^3 +
    a^2 c^2 u v^2 w^3 - 2 a^4 v^3 w^3.


      La condicion necesaria y suficiente para que P, el circuncentro y N' estén alineados, es que las coordenadas de P satisfagan a la ecuación:

    -c^4 x^4 y^3 + c^4 x^3 y^4 - a^2 c^2 x^4 y^2 z - 2 b^2 c^2 x^4 y^2 z + c^4 x^4 y^2 z - 2 a^2 c^2 x^3 y^3 z + 2 b^2 c^2 x^3 y^3 z + 2 a^2 c^2 x^2 y^4 z + b^2 c^2 x^2 y^4 z - c^4 x^2 y^4 z + a^2 b^2 x^4 y z^2 - b^4 x^4 y z^2 + 2 b^2 c^2 x^4 y z^2 - a^2 b^2 x^3 y^2 z^2 + b^4 x^3 y^2 z^2 + a^2 c^2 x^3 y^2 z^2 - c^4 x^3 y^2 z^2 - a^4 x^2 y^3 z^2 + a^2 b^2 x^2 y^3 z^2 - b^2 c^2 x^2 y^3 z^2 + c^4 x^2 y^3 z^2 + a^4 x y^4 z^2 - a^2 b^2 x y^4 z^2 - 2 a^2 c^2 x y^4 z^2 + b^4 x^4 z^3 + 2 a^2 b^2 x^3 y z^3 - 2 b^2 c^2 x^3 y z^3 + a^4 x^2 y^2 z^3 - b^4 x^2 y^2 z^3 - a^2 c^2 x^2 y^2 z^3 + b^2 c^2 x^2 y^2 z^3 - 2 a^2 b^2 x y^3 z^3 + 2 a^2 c^2 x y^3 z^3 - a^4 y^4 z^3 - b^4 x^3 z^4 - 2 a^2 b^2 x^2 y z^4 + b^4 x^2 y z^4 - b^2 c^2 x^2 y z^4 - a^4 x y^2 z^4 + 2 a^2 b^2 x y^2 z^4 + a^2 c^2 x y^2 z^4 + a^4 y^3 z^4=0.

    ( Mostrar/Ocultar figura )
      Anopolis2001.png
    Descargar fichero GeoGebra


      Los 21 puntos comunes de esta séptica y la cúbica de Neuberg (K001, del catálogo de cúbicas de Bernard Gibert) son: los vértices de los triángulos ABC (triples) y excentral, X1, X3, X370 (y los otros cinco -tres siempre reales- "equilateral cevian points") y X1138.

  • martes 11 de noviembre del 2014

    Una nónica con puntos cuádruples en los vértices del triángulo de referencia



      Dados un triángulo ABC, un punto P y su triángulo ceviano DEF, sean Da, Eb y Fc los puntos medios de los segmentos EF, BE y CF.
      La circunferencia circunscrita a DaEbFc vuelve a cortar a la recta EF en el punto D'. Se denota por A' la reflexión de D' en la recta EbFc. El punto A' está en la circunferencia de los nueve puntos de ABC.
    ( Mostrar/Ocultar figura )
      Q066_P3p.png

      Procediendo cíclicamente, de definen los puntos B' y C'.

      Los triángulos ABC y A'B'C' son ortológicos si sólo si el punto P está en una nónica con puntos cuádruples en los vértices de ABC, puntos dobles los vértices del triángulo antimedial y que pasa por el baricentro, ortocentro y punto de Gergonne.

    ( Mostrar/Ocultar figura )
      HG111114Math.png


      Si (u:v:w) son las cordenadas baricéntricas de P, la ecuación de la circunferencia DaEbFc es:
    HG111114circDaEbFc.png

      La circunferencia circunscrita a DaEbFc vuelve a cortar a la recta EF en el punto D':

    D' = (u(c^2uv(v-w)+ w(b^2u(-v+w)+a^2v(2u+v+w))) : v(c^2uv(u+w)+ w(-b^2u(u+2v-w)+a^2v(u+w))) : -w(-(u+v)(b^2u+ a^2v)w+c^2uv(u-v+2w))).

      La reflexión de D' en la recta EbFc es:

    A' = (-u(v-w)(-b^2(u+v)w+c^2v(u+w)) : -v(u+ w)(c^2u(v-w)+a^2(u+v)w) : -(u+v)w(a^2v(u+w)+ b^2u(-v+w)).


      Los puntos para los cuales las perpendiculares por los vértices de A'B'C' a los lados de ABC son concurrentes, están en una nónica que, además de las propiedades enunciadas arriba, pasa por los pies de las cevianas de X264 (conjugado isotómico del circuncentro). Estos son los únicos puntos de intersección de la nónica con los lados de ABC, a parte de los vértices.
    Las rectas que unen los vértices del triángulo antimedial con el ortocentro, cortan a sus lados opuestos en puntos sobre la nónica. Estos son los únicos puntos de intersección de la nónica con los lados del triángulo antimedial, a parte de sus vértices.
      Tres de las tangentes en los vértices del triángulo antimedial se cortan el retrocentro (isotómico conjugado del ortocentro, simediano del triángulo antimedial, el punto X69 de ETC).
      Las otras tres tangentes delimitan un triángulo perspectivo con ABC, con centro de perspectividad en X253 (conjugado cicloceviano del X69).

      Los centros de ortología de ABC respecto a A'B'C' correpondientes a los puntos X2, X4 y X7 son X4, X3 y X1, respectivamente.
      Los centros de ortología de A'B'C' respecto a ABC correpondientes a los puntos X2, X4 y X7 son X3, X4 y X5, respectivamente.

  • domingo 9 de noviembre del 2014

    Otra caracterización de la cuártica de Stammler



    Art of Problem Solving
    Let triangle ABC, circrumcircle (O). UV is a chord of (O), UV cuts AB,AC at Q,P. Denote M,N,J,R is midpoint of BP,CQ,PQ,UV.
    a. Prove MNJR is cyclic
    b. Prove that the symmetry point of R with mirror MN is lies on Euler circle of ABC
    (a appear in a contest in Vietnam and b is i propose when solve a)

    Synthetic solution by Luis González
    ( Mostrar/Ocultar figura )
      HCM.png


      Dados un triángulo ABC, un punto P y su triángulo ceviano DEF, sean Da, Eb y Fc los puntos medios de los segmentos EF, BE y CF.
      La circunferencia circunscrita a DaEbFc vuelve a cortar a la recta EF en el punto D'. Se denota por A' la reflexión de D' en la recta EbFc. Por el resultado enunciado: el punto A' está en la circunferencia de los nueve puntos de ABC.
      Procediendo cíclicamente, de definen los puntos B' y C'.
    ( Mostrar/Ocultar figura )
      Q066_P3p.png

      Los triángulos ABC y A'B'C' son perspectivos si sólo si el punto P está en la circunferencia circunscrita o en la cuártica de Stammler (Q066)

      Si (u:v:w) son las coordenadas baricéntricas de P:

    D' = (u(c^2uv(v-w)+w(b^2u(-v+w)+a^2v(2u+v+w))) : v(c^2uv(u+w)+w(-b^2u(u+2v-w)+a^2v(u+w))) : -w(-(u+v)(b^2u+a^2v)w+c^2uv(u-v+2w))).

      El simétrico de este punto respecto a EF es:

    A' =( u(v-w)(-b^2(u+v)w+c^2v(u+w)) : v(u+w)(c^2u(v-w)+a^2(u+v)w) : (u+v)w(a^2v(u+w)+b^2u(-v+w)) ).

      Las rectas AA', BB', CC' son concurrrentes si las coordenadas de P satifacen a una de las ecuaciones:

    a^2 yz + b^2 zx + c^2 xy =0,
    Q066:   a^2(b^2-c^2)y^2z^2 + b^2(c^2-a^2)z^2x^2 + c^2(a^2-b^2)x^2y^2= 0.

      La primera es la circunferencia circunscrita y la segunda es la cuártica de Stammler, con puntos dobles en los vérices de ABC y que pasa por los centros X1, X2, X4, X254, X1113, X1114, X1138, X2184, X3223, X3346, X3459, X8049, X9510.

      Si el punto P recorre la cuártica de Stammler Q066, el lugar geométrico del centro de perspectividad de ABC y A'B'C' es la cuártica (Q) de ecuación:

    ( Mostrar/Ocultar figura )
      Q066_P3.png
    Descargar fichero GeoGebra

      Como la cuártica Q066 es una curva algebraica de orden 4 de género cero (los vértices de ABC son tres puntos nodales) es unicursal, es decir, se puede parametrizar en función racional de un parámetro (§9.Curvas unicursales).
      Esto se puede lograr tomando un haz de cónicas pasando por los tres puntos dobles y por un punto más (el baricentro) de la cuártica Q006: y(x-z) + tz(y-xy)=0.
      A cada "t" corresponde una cónica del haz y ésta tiene con Q066 ocho puntos comunes, de los cuales ya se conocen siete (tomando cada punto doble dos veces); lo cual indica que a cada "t" corresponde otro único punto de intersección de ambas curvas, a parte de los elegidos. Este parámetro nos va a determinar todos los puntos de Q066.

    Pt = (1/(b^2(a^2-c^2)-c^2(a^2-b^2)t^2) :
    1/(b^2(c^2- a^2)-2c^2(b^2-a^2)t+c^2(b^2-a^2)t^2) :
    1/(b^2(a^2-c^2)-2b^2(a^2-c^2)t+c^2(a^2-b^2)t^2)).

      Tomando este punto, la intersección de las rectas AA', BB', CC' es el punto:

    Qt = ((b^2-c^2)^2t(-a^2t+b^2(1+t))(a^2-c^2(1+t)):
    (a^2-c^2)^2(1+t)(b^2+c^2t)(a^2t-b^2(1+t)) :
    (a^2-b^2)^2t(1+t)(b^2+c^2t)(a^2-c^2(1+t))).

      Eliminando el parámetro "t" obtenemos la ecuación implícita de la cuártica (Q), que tiene puntos dobles en los vértices de ABC y que pasa por los centros X2, X4, X12, X68, X252, X1312, X1313, X5627.

      La cuártica (Q) es conjugada isogonal de la cónica, que pasa por X3, X6, X24, X60, X143, X1511, X1986:




      Si el punto P recorre la la circunferencia circunscrita, el lugar geométrico del centro Q de perspectividad de ABC y A'B'C' es la circunferencia de los nueve puntos. El punto Q es la imagen de P en la homotecia de centro el baricentro y razón -1/2.

    ( Mostrar/Ocultar figura )
      Q066_P3O(R).png
    Descargar fichero GeoGebra

    Cada par que se muestra a continuacion representa los índices en la Enciclopedia de los Centros del Triangulo (ETC) de centros en la circunferencias circunscrita (el primero) y de los nueve puntos (el segundo), correspondientes a P y Q, respectivamente:
    {74, 113}, {98, 114}, {99, 115}, {100, 11}, {101, 116}, {102, 117}, {103, 118}, {104, 119}, {105, 120}, {106, 121}, {107, 122}, {108, 123}, {109, 124}, {110, 125}, {111, 126}, {112, 127}, {476, 3258}, {675, 5513}, {691, 5099}, {805, 2679}, {815, 5509}, {835, 5515}, {901, 3259}, {925, 136}, {927, 1566}, {930, 137}, {932, 5518}, {934, 5514}, {1113, 1313}, {1114, 1312}, {1141, 128}, {1290, 5520}, {1292, 5511}, {1293, 5510}, {1294, 133}, {1296, 5512}, {1297, 132}, {1298, 129}, {1300, 131}, {1303, 130}, {1305, 5190}, {1309, 10017}, {1310, 5517}, {1379, 2039}, {1380, 2040}, {2373, 1560}, {3565, 5139}, {5606, 5952}, {5951, 5950}, {5970, 9152}, {6078, 5519}, {6079, 5516}, {6093, 6092}, {9080, 9193}, {9150, 9151}.

  • viernes 7 de noviembre del 2014

    Conjugado antigonal del incentro respecto a su triángulo ceviano



    ( Anopolis #1978, Antreas P. Hatzipolakis)

    Antigonal conjugate
    Let P be a point not on a sideline of ABC and not X(4). The antigonal conjugate of P is the point h(P) isogonal conjugate of the inverse-in-circumcircle of the isogonal conjugate of P. If P lies on the circumcircle then h(P)=X(4); otherwise, h(h(P))=P. If P lies on the line at infinity, then h(P)=P.


      Sean ABC un triángulo y A'B'C' el triángulo ceviano del incentro X1. El conjugado antigonal Q de X1 respecto A'B'C' está en la recta X1O', donde O' es el circuncentro de A'B'C'. Se verifica que
             Q = 3R X1 + 2r X399,
    donde R y r son los radios de las circunfencias circunscrita e inscrita a ABC, respectivamente.

      X399 es el punto de concurrencia de las circunferencias circunscritas a los triángulos TaB'C', A'TbC', A'B'Tc; donde, TaTbTc es el triángulo tangencial y A'B'C' es el triángulo obtenido reflejando cada vértice de ABC en el lado opuesto (Cosmin Pohoata, "On the Parry reflection point," Forum Geometricorum 8 (2008), 43-48).

      La primera coordenada baricéntrica de Q es:

    a^2 (a^5 + a^4(b+c) - 2a^3(b^2+c^2) - a^2(2b^3-b*c(b+c)+2c^3)+ a(b^4+b^2c^2+c^4) + (b-c)^2(b^3+c^3))

    con (6,9,13)-número de búsqueda en ETC: 0.812149174855219822306671631


    Una propiedad geométrica de Q:

    ( Mostrar/Ocultar figura )
      Anopolis1975.png
    Descargar fichero GeoGebra

    Hyacinthos #21642 (Feb 26, 2013)
    Let ABC be a triangle and A'B'C' the cevian triangle of I.
    Denote:
    B'a, C'a = the reflections of B',C' in AA', resp.
    A'a = (perpendicular to BB' from B'a) /\ (perpendicular to CC' from C'a).
    
    C'b, A'b = the reflections of C',A' in BB', resp.
    B'b = (perpendicular to CC' from C'b) /\ (perpendicular to AA' from A'b).
    
    A'c, B'c = the reflections of A',B' in CC', resp.
    C'c = (perpendicular to AA' from A'c) /\ (perpendicular to BB' from B'c).
    
    Are the circumcircles of A'aB'aC'a, B'bC'bA'b, C'cA'cB'c 
    concurrent?.
    
     Antreas P. Hatzipolakis
    
    Hyacinthos #21651 (Feb 28, 2013)
    The circumcircles are concurrent at non-ETC 0.812149174855220, which is, 
    the antigonal conjugate, wrt incentral triangle, of X(1)
    
    Randy Hutson
    


  • jueves 6 de noviembre del 2014

    Ejes radicales concurrentes y cúbica de Neuberg



    ( Anopolis #1973, Antreas P. Hatzipolakis)

      Sean ABC un triángulo, P un punto y Pa, Pb, Pc las reflexiones de P en BC, CA, AB, respectivamente.
      Denotamos por (Oab) y (Oac) las circunferencias circunscritas a los triángulos ABPb y ACPc, respectivamente; y por ea el eje radical de (Oab) y (Oac). Similarmente, se consideran los ejes radicales eb y ec.

      Los ejes radicales ea, eb y ec concurren en un punto Q si y solo si P está en la cúbica de Neuberg (K001, del catálogo de cúbicas de Bernard Gibert).

    ( Mostrar/Ocultar figura )
      Anopolis1972.png
    Descargar fichero GeoGebra

      El lugar geométrico del punto Q de intersección de los radicales ea, eb y ec,cuando P recorre la cúbica Neuberg, es la cúbica pK(X50, X3) (K073, del catálogo de cúbicas de Bernard Gibert).
      El punto Q es el inverso de P respecto a la circunferencia circunscrita a ABC.

  • miércoles 5 de noviembre del 2014

    Reflexión de la recta de Euler y triángulo tangencial



    ( Complemento a Anopolis #1971, Antreas P. Hatzipolakis)

      Sean ABC un triángulo, TaTbTc el triángulo tangencial y P un punto. Se denota por da, db y dc las reflexiones de la recta de Euler en los lados BC, CA y AB, respectivamente; y por Pa, Pb y Pc las reflexiones de P en las rectas da, db y dc, respectivamete.

      Los triángulos TaTbTc y PaPbPc son perspectivos si P está sobre el díámetro de ABC que pasa por X110 (foco de la parábola de Kiepert) o en la circunferencia con centro en la reflexión de X110 en X156 (centro de la circunferencia de los nueve puntos de TaTbTc) y que pasa por X155 (ortocentro de TaTbTc).


      La ecuación de la reflexión da de la recta de Euler en el lado BC es:

    ((b^2-c^2)^3-a^2(b^4-c^4))x - a^2(a^2-c^2)(a^2-b^2+c^2)y + a^2(a^2-b^2)(a^2+b^2-c^2)z=0.

      Si P(u:v:w), las coordenadas baricéntricas de la reflexión Pa de P en da son:

    (-a^2(a^8u - a^6(b^2(u+v-w)+c^2(u-v+ w)) - a^4(b^4(u-v)+c^4(u-w)+b^2c^2(-3u+v+w)) - a^2(b^2-c^2)^2(c^2v+b^2w) + (b^2-c^2)^4u ) :
    (b^2-c^2)^5u - a^10w - a^2(b^2-c^2)^3(c^2(u+v)+b^2w) + a^8(b^2(-v+w)+c^2(v+w)) - a^4(b^2-c^2)(c^4(u-v-w)+b^4(2u+w)+b^2c^2(-u+2v+w)) + a^6(-c^4(u+v-w)-4b^2c^2w+b^4(u+v+2 w)) :
    -(b^2-c^2)^5u - a^10v+ a^2(b^2-c^2)^3(c^2v+b^2(u+w)) + a^8(c^2(v-w)+b^2(v+w)) + a^6(-4b^2c^2v-b^4(u-v+w)+c^4(u+2v+w)) + a^4(b^2-c^2)(c^4(2u+v)+b^4(u-v-w) + b^2c^2(-u+v+2w)))

      Las coordenadas de los vértices del triángulo tangencial TaTbTc son:

    {(-a^2:b^2:c^2), (a^2:-b^2:c^2), (a^2:b^2:-c^2)}


      La condición analítica para que los triángulos TaTbTc y PaPbPc sean perspectivos es que P satisfaga a la ecuación de la recta:

    d:  b^2c^2(b^2-c^2)(2a^4 - a^2(b^2+c^2) - (b^2-c^2)^2x+...=0

    o a la ecuación circunferencia Γ con centro en P0 (reflexión de X110 en X156, dato proporcionado por Peter Moses) y pasa por X155 (ortocentro de TaTbTc).

    •   La circunferencia Γ tiene centro en P0 ((reflexión de X110 en X156):

    P0 = ( a^2 (a^14 - 4a^12(b^2+c^2) + a^10(5b^4+11b^2c^2+5c^4) - 13a^8b^2c^2(b^2+c^2) + a^6(-5b^8+12b^6c^2+2b^4c^4+12b^2c^6-5c^8) + 2a^4(2b^10-4b^8c^2+b^6c^4+b^4c^6-4b^2c^8+2c^10) - a^2(b^2-c^2)^2(b^8+b^6c^2-6b^4c^4+b^2c^6+c^8) + b^2c^2(b^2-c^2)^4(b^2+c^2)) : ... : ... ).

      Cuando el punto P recorre la circunferencia Γ, los triángulos TaTbTc y PaPbPc son perspectivos y SEMEJANTES. Si T1 es uno de los puntos de intersección de la recta d con Γ (el antipodal de X155 en Γ), el centro de semejanza S es el segundo punto de intersersección de la recta PT1 con la circunferencia circunscrita al triángulo tangencial Γ'. El centro de pesrpectividad es el punto Q (distinto de S) de intersección de las circunferencias circunscritas a los triángulos TaTbTc y PaPbPc.


    ( Mostrar/Ocultar figura )
      Anopolis1970Tangencial.png
    Descargar fichero GeoGebra


    •   La recta d contiene al diámetro de la circunferencia circunscrita a ABC con extremos en X74 (conjugado isogonal del punto del infinito de la recta de Euler) y X110 (foco de la parábola de Kiepert, centro de la hipérbola equilátera que pasa por el centro de la circunferencia circunscrita y por los vértices del triángulo tangencial).
      En este caso, los TaTbTc y PaPbPc son HOMOTÉTICOS y, cuando P varía sobre d, el lugar geométrico de los centros de homotecia Q es la recta d' que pasa por X110 y por X26 (centro de la circunferencia circunscrita al triangulo tangencial).
      La correspondencia:

    P ∈ d ↦ Q ∈ d'

    es una perspectividad con centro en T1, el antipodal de X155 en Γ:

    T1 = (a^2(a^10 - a^8(b^2+c^2) + a^6(-2b^4+5b^2c^2-2c^4) + 2a^4(b^2-c^2)^2(b^2+c^2) + a^2(b^2-c^2)^2(b^4-b^2c^2+c^4)- b^10+b^2c^2(b^6+c^6)-c^10) : ... : ... ).

    con (6,9,13)-número de búsqueda en ETC: -6.312998437457754226552932181
    ( Mostrar/Ocultar figura )
      Anopolis1970Tangencial0.png
    Descargar fichero GeoGebra


      El conjunto d∪Γ se tansforma en d'∪Γ' mediante una semejanza directa de centro en T2, el otro punto de intersección (distinto de T1) de las circunferencia Γ y Γ'.

    T2 = (a^2(a^2+b^2-c^2)(a^2-b^2+c^2) (a^12 - 4a^10(b^2+c^2) + a^8(7b^4+11b^2c^2+7c^4) - a^6(8b^6+11b^4c^2+11b^2c^4+8c^6) + a^4(7b^8+3b^6c^2+8b^4c^4+3b^2c^6+7c^8) - a^2(4b^10-3b^8c^2+3b^6c^4+3b^4c^6-3b^2c^8+4c^10) + (b^6-b^4c^2+b^2c^4-c^6)^2): ... : ...).

    con (6,9,13)-número de búsqueda en ETC: -1.88300482666727983296569296

  • domingo 26 de octubre del 2014

    Una propiedad del centro X3574



    ( Anopolis #1947, Antreas P. Hatzipolakis)

    X(3574) = X(21)-of-the-orthic-triangle if ABC is acute. Let A'B'C' be the orthic triangle, which is the pedal (and cevian) triangle of the orthocenter, H = X(4) . The Euler lines of the triangles HB'C', HC'A', HA'B' concur in X(3574). (Michel Garitte, July 7, 2009)


      Sea ABC un triángulo y A'B'C' el triángulo ceviano de un punto P.
    Se denota por:
      (Nab) y (Nac) las circunferencias de los nueve puntos de los triángulos BC'P y CB'P, respectivamente.
    da el eje radical de las circunferencias (Nab) y (Nac). Similarmente, db y dc.

      El lugar geométrico de los puntos P tal que los ejes radicales da, db y dc son concurrentes es una séptica circunscrita a los triángulos ABC y antimedial, y que pasa por el baricentro y ortocentro.


    ( Mostrar/Ocultar figura )
      Anopolis1947.png


      La ecuación baricéntrica de la circunferencia de los nueve puntos de BC'P es:
    2(u+v)(u+v+ w)(a^2yz+b^2zx+c^2xy) -
    (x+y+z) ((u(c^2v+SAw)+2v(c^2v+SAw))x + u(c^2u+SBw)y + (2SCu^2+u(2a^2v+SC(2v+w))+a^2v(2v+w))z) = 0.
      La ecuación de la circunferencia de los nueve puntos de CB'P, sesulta de ésta permutando y por z, b por c, SB por SC, v por w.

      El eje radical da de las circunferencias (Nab) y (Nac) es:
    (a^2u^2v-b^2u^2v+c^2u^2v+a^2uv^2-b^2uv^2+3c^2uv^2-a^2u^2w-b^2u^2w+c^2u^2w-2b^2uvw+2c^2uvw+2a^2v^2w-2b^2v^2w+2c^2v^2w-a^2uw^2-3b^2uw^2+c^2uw^2-a^2vw^2-2b^2vw^2+2c^2vw^2)x +
    (-2a^2u^3+2b^2u^3-3a^2u^2v+3b^2u^2v-3c^2u^2v-a^2uv^2+b^2uv^2-c^2uv^2-5a^2u^2w+b^2u^2w+c^2u^2w-8a^2uvw+2b^2uvw-2c^2uvw-2a^2v^2w-3a^2uw^2-b^2uw^2+c^2uw^2-4a^2vw^2)y +
    (2a^2u^3-2c^2u^3+5a^2u^2v-b^2u^2v-c^2u^2v+3a^2uv^2-b^2uv^2+c^2uv^2+3a^2u^2w+3b^2u^2w-3c^2u^2w+8a^2uvw+2b^2uvw-2c^2uvw+4a^2v^2w+a^2uw^2+b^2uw^2-c^2uw^2+2a^2vw^2)z = 0.

    Las escuaciones de los ejes radicales db y dc surgen por permutación cíclica.

      Los ejes radicales da, db y dc son concurrentes si y sólo si P está en una séptica circunscrita al triángulo ABC y a su triángulo antimedial; que pasa por el baricentro y el ortocentro. Los vértices A, B y C son puntos dobles.
    Las tangentes en los vértices del triángulo antimedial concurren en el retrocentro (el simediano del triángulo antimedial, el punto X69 de ETC).

      Cuando P es el ortocentro la intersección de los ejes radicales da, db y dc es el centro X3574,


    ( Mostrar/Ocultar figura )
      Anopolis1947H.png
    Descargar fichero GeoGebra



    CONSIDEREMOS OTROS EJES RADIACALES DE PARES DE CIRCUNFERENCIAS DE LOS NUEVE PUNTOS DESCRITAS ARRIBA


    •  Sean los ejes radicales ea de (Nba) y (Nca), eb de (Ncb) y (Nab), y ec de (Nac) y (Nbc).

      El lugar geométrico de los puntos P tal que los ejes radicales ea, eb y ec son concurrentes es una séptica circunscrita a los triángulos ABC, órtico, antimedial y excentral, y que pasa por el incentro, ortocentro y X1138. Los vértices de ABC son puntos triples.


    ( Mostrar/Ocultar figura )
      Anopolis1947a.png


      Cuando P = X1, X4 y X1138, los ejes radicales ea, eb y ec concurren en los centros X3649, X5 y X30 (es decir, son paralelos a la recta de Euler), respectivamente.

      There are only two points X such that the pedal triangle of X is similar to the cevian triangle of X. They are X(4) and X(1138). (Jean-Pierre Ehrmann, 1/4/03)

      Let X(1) be the incenter and A'B'C' is a central triangle. Let LA be the line through A' parallel to the Euler line of triangle BCX, let LB be the line through B' parallel to the Euler line of CXA, and let LC be the line through C' parallel to the Euler line of AXB. If the lines LA, LB, LC concur, the point of concurrence is the Kirikami-Schiffler point of the triangle A'B'C', denoted by KS(A'B'C'). Seiichi Kirikami (February 1, 2011)
    X(3649) = KS(Intouch Triangle)



    •  Sean los ejes radicales fa de (Nbc) y (Ncb), fb de (Nca) y (Nac), y fc de (Nab) y (Nba).

      El lugar geométrico de los puntos P tal que los ejes radicales fa, fb y fc son concurrentes es la "O(X5) orthopivotal cubic" (K060, del catálogo de cúbicas de Bernard Gibert).


    ( Mostrar/Ocultar figura )
      Anopolis1947K060.png
    Descargar fichero GeoGebra

    Si P está sobre la cúbica K060, se designa por Q el punto de intersección de los ejes radicales fa, fb y fc.
    Pares de centros del triángulo {P,Q} son:
    {X(4), X(4)}, {X(5), X(140)}, {X(13), X(396)}, {X(14), X(395)}, {X(79), X(3649)},
    {X(80), ((b+c)(-2a+b+c)/(b+c-a) : .. : ..)},
    {X(265), X(30)}, {X(621), X(298)}, {X(622, X(299},
    {X(1141), ((2a^8+(b^2-c^2)^4-4a^6(b^2+c^2)-2a^2(b^2-c^2)^2(b^2+c^2)+3a^4(b^4+c^4)) / (-(b^2-c^2)^2+a^2(b^2+c^2)) : ... : ...)},
    {X(5627), (2a^8-(b^2-c^2)^4-2a^6(b^2+c^2)+4a^2(b^2-c^2)^2(b^2+c^2)+a^4(-3b^4+8b^2c^2-3c^4)) / (2a^4-(b^2-c^2)^2-a^2(b^2+c^2))}

  • viernes, 24 de octubre del 2014

    Bisectriz interior, altura y mediana concurrentes

      Dado un triángulo ABC, sea MaMbMc el su triángulo medial.
    Lab es la recta que pasa por Ma y por el punto de intersección de la bisectriz interior en B y la altura desde C.
    Lac es la recta que pasa por Ma y por el punto de intersección de la bisectriz interior en C y la altura desde B.
    Ab es el punto de intersección de AB con Lab y Ac es el punto de intersección de AC con Lac.
     Similarmente se definen los puntos Bc, Ba, Ca y Cb.

      Las mediatrices de los segmentos BcCb, CaAc y AbBa son concurrentes.

    ( Mostrar/Ocultar figura )
      AGBICH.png
    Descargar fichero GeoGebra

      En coordenadas baricéntricas:
    Ab = (-a(a^2-b^2+c^2) : a^3+a^2c-a(b^2+c^2)-b^2c+c^3 : 0),
    Ac = (a(a^2+b^2-c^2) : 0 : -a^3-a^2b+a(b^2+c^2)-b^3+bc^2 ).
      Permutando cíclicamente se tienen las coordenadas de los puntos Bc, Ba, Ca y Cb.
      La mediatriz de BcCb (las otras dos mediatrices en cuestión, se obtienen por permutación cíclica) es:
    b(a^4+2a^3(b-c)+2ac(b^2-bc+c^2)+b^4-2b^3c-2b^2c^2-c^4)x
    +(a-c)(a^4-2a^2c^2+2ab^3+b^4+2b^3c+c^4)y
    - b(-a^4+2a^3c-2a^2b(b+c)-2a(b^3-b^2c+c^3)+b^4+2bc^3+c^4)z = 0.

      El punto M de intersección de estas mediatrices es:

    (a(a^6 - a^4(b+c)^2 + 2a^3(b^3+b^2c+bc^2+c^3) - 2a(b^5+b^4c+bc^4+c^5) - (b-c)^4(b+c)^2+a^2(b^2+c^2)^2) : ... : ... )

    con (6,9,13)-número de búsqueda en ETC: -88.3239018332220355783645503
      El punto M está, al menos, sobre las rectas X3X1104, X4X345.

  • jueves, 23 de octubre del 2014

    Parábolas asociadas al triángulo de reflexión

      Dado un triángulo ABC, sea A'B'C' el triángulo de reflexión que resulta de reflejar cada vértice de ABC en su lado opuestos. Se denota por:
      ℘ab la parábola de foco C y directriz BC', Dab el polo de CB' respecto a ℘ab.
      ℘ac la parábola de foco B y directriz CB', Dac el polo de BC' respecto a ℘ac.
      Similarmente, se definen las parábolas ℘bc, ℘ba, ℘ca y ℘cb, y los puntos Dbc, Dba, Dca y Dcb.

      Las rectas DabDac, DbcDba y DcaDcb delimitan un triángulo perspectivo con ABC.

    ( Mostrar/Ocultar figura )
      TrianguloReflexionParabolas.png
    Descargar fichero GeoGebra
      Las coordenadas baricéntricas de los vértices del triángulo de reflexión son:

    A' (-a^2 : 2SC : 2SB),   B' (2SC : -b^2 : 2SA),   C' (2SB : 2SA : -c^2).

      Ecuaciones de las parábolas (R radio de la circunferencia circunscrita):

    ab : b^2R^2SB^2x^2 + a^2c^2SC^2y^2+a^4b^2R^2z^2 + 2a^2b^2R^2SBzx - a^2b^2c^2SCxy= 0,
    ac : c^2R^2SC^2x^2 + a^4c^2R^2y^2 + a^2b^2SB^2z^2 - a^2b^2c^2SBzx + 2a^2c^2R^2SCxy = 0.

      El polo de la directriz de ℘ab respcto a ℘ac es:

    Dab ((a^2-b^2+c^2)(a^4-2a^2c^2+(b^2-c^2)^2) : b^2(-a^4+(b^2-c^2)^2) : -c^2(a^4-2a^2c^2+(b^2-c^2)^2)).

      El polo de la directriz de ℘ac respcto a ℘ab es:

    Dac (-(a^2+b^2-c^2)(a^4-2a^2b^2+(b^2-c^2)^2) : b^2(a^4-2a^2b^2+(b^2-c^2)^2) : -c^2(-a^4+(b^2-c^2)^2)).

      La ecuación de la recta DabDac es:

    -b^2c^2(a^4(b^2+c^2)+(b^2-c^2)^2(b^2+c^2)-2a^2(b^4-b^2c^2+c^4))x + c^4(-a^6-a^4(b^2-3c^2)-(b^2-c^2)^3-a^2(b^4-4b^2c^2+3c^4))y + b^4(-a^6+(b^2-c^2)^3+a^4(3b^2-c^2)-a^2(3b^4-4b^2c^2+c^4))z = 0.

      El centro de perspectividad P de ABC y el triángulo delimitado por las rectas DabDac, DbcDba, DcaDcb tiene primera coordenada:

    a^2 / (a^14(b^2+c^2) -
      a^12(5b^4+3b^2c^2+5c^4)+
      2a^10(5b^6+2b^4c^2+2b^2c^4+5c^6) -
      a^8(10b^8+3b^6c^2-2b^4c^4+3b^2c^6+10c^8) +
      a^6(5b^10+b^8c^2-2b^6c^4-2b^4c^6+b^2c^8+5c^10)-
      a^4(b^4-c^4)^2(b^4+b^2c^2+c^4)+
      2a^2b^2c^2(b^2-c^2)^4(b^2+c^2)-b^2c^2(b^2-c^2)^6 )

    con (6,9,13)-número de búsqueda en ETC: -3.347657323661639630845858

  • domingo, 19 de octubre del 2014

    Bisectriz interior, altura y simediana concurrentes

    a Kake, por su "cumple"

      Dado un triángulo ACUTÁNGULO ABC, sean:
    Ab es el único punto sobre AB tal que, en el triángulo AbBC, la simediana en Ab, la bisectriz interior en B y la altura en C son concurrentes.
    Ac es el único punto sobre AC tal que, en el triángulo AcBC, la simediana en Ac, la bisectriz interior en C y la altura en B son concurrentes.
      Similarmente se define los puntos Bc, Ba, Ca y Cb.

      Las mediatrices de los segmentos BcCb, CaAc y AbBa son concurrentes.


    ( Mostrar/Ocultar figura )
      AK-BI-CHmediatrz.png
    Descargar fichero GeoGebra

      Dado un punto en el lado AB de coordenadas baricéntricas A'(t:1-t:0), la condición para que, en el triángulo AbBC, la simediana en Ab, la bisectriz interior en B y la altura en C son concurrentes es:

    (a-b-c)(a+b-c)c^2t^2 - (a^2-b^2+c^2)(a^2-b^2-ac+c^2)t + a^2(a^2-b^2+c^2) = 0.

      El único valor de 't' que anula esta ecuación que determina un punto Ab sobre AB tal que, en el triángulo AbBC, la simediana en Ab, la bisectriz interior en B y la altura en C son concurrentes es:

    t = (a^2-b^2+c^2)(a^2-ac+c^2-b^2) - ((a^2-b^2+c^2)f2(a,b,c))½

    donde
    f2(a,b,c)= a^6-2a^5c-3a^4b^2+ 4a^3c(b^2+c^2)+3a^2b^2(b^2-c^2)-2ac(b^2-c^2)^2-(b^2-c^2)^3.
      Se tiene:

    Ab = ((a^2-b^2+c^2)(a^2-ac+c^2-b^2) - ((a^2-b^2+c^2)f2(a,b,c))½ :
    -a^4+a^3c+2a^2b^2-ac(b^2+3c^2)-b^4+c^4 + ((a^2-b^2+c^2)f2(a,b,c))½ : 0).

    Permutando cíclicamente las coordenadas de Ab y permutando 'b' y 'c', se tiene:

    Ac = ( a^4-a^3b+2a^2(b^2-c^2)+a(-b^3+bc^2)+(b^2-c^2)^2 -((a^2+b^2-c^2)f3(a,b,c))½ : 0 :
    -a^4+a^3b+2a^2c^2+b^4-ab(3b^2+c^2)-c^4)-c^2 +f3(a,b,c))½).

    donde
    f3(a,b,c)= a^6-2a^5b-3a^4c^2+ 4a^3b(b^2+c^2)+a^2c^2(-3b^2+3c^2)-2ab(b^2-c^2)^2+(b^2-c^2)^3 .

      Procediendo cíclamente sobre los vértices de ABC, obtennemos las coordenadas de los puntos Bc, Ba, Ca y Cb.

      Las mediatrices de los segmentos BcCb, CaAc y AbBa (cuyas ecuaciones son muy largas para exponerlas aquí) concurren en el punto de primera coordenada baricéntrica:

    f0(a,b,c) - 2a^2(b+c-a)((b^2+c^2-a^2)f1(a,b,c))½ + (a-b+c)(a^2+b^2-c^2)((a^2-b^2+c^2)f2(a,b,c))½ + (a+b-c)(a^2-b^2+c^2)((a^2+b^2-c^2)f3(a,b,c))½

    donde
    f0(a,b,c)= -2a^7+a^6(b+c) + a^5(b^2+c^2) - 2a^4(b^3+c^3) + 2a^3(b^4-b^3c-2b^2c^2-bc^3+c^4) - a^2(b^5-b^4c-4b^3c^2-4b^2c^3-bc^4+c^5) - a(b-c)^4(b+c)^2 + 2(b-c)^4(b+c)^3,

    f1(a,b,c)=-a^6 + a^4(3b^2-2bc+3c^2) + a^2(-3b^4+4b^3c-3b^2c^2+4bc^3-3c^4) + b^6-2b^5c+4b^3c^3-2bc^5+c^6.

  • domingo 12 de octubre del 2014

    Centros de circunferencias de Euler y circunferencias concurrentes

    ( Anopolis #1891, Antreas P. Hatzipolakis)

      Sea ABC un triángulo y O su circuncentro. Se denota por Ab, Ac las reflexiones de A en OB y OC, respectivamente. Na es el centro de la circunferencia de los nueve puntos del triángulo AAbAc. Similarmente se definen Nb y Nc.

      Las circunferencias circunscritas a los triángulos ABC, ANbNc, BNcNc y CNaNb son concurrentes en X930.
      Las circunferencias circunscritas a los triángulos NaNbNc, NaBC, NbCA y NcAB son concurrentes en:

    Q = (a^2(a^6(b^2+c^2)- a^4(3b^4+4b^2c^2+3c^4)+ a^2(3b^6+2b^4c^2+2b^2c^4+3c^6)-b^8+b^6c^2+b^2c^6-c^8) : ... : ...)

    con (6,9,13)-número de búsqueda en ETC: 8.07910137097857081131659206
      Este punto ha sido inculido en como el X6102.

    En coordenadas baricéntricas:
    Ab = (a^2(a-c)(a+c):-(a^2-c^2)(a^2+b^2-c^2):-c^2(a^2+b^2-c^2))
    Ac = (a^2(a-b)(a+b):-b^2(a^2-b^2+c^2):-(a^2-b^2)(a^2-b^2+c^2)
    Na = (a^2(-b^8+b^6c^2+b^2c^6-c^8+a^6(b^2+c^2)- a^4(3b^4+4b^2c^2+3c^4)+ a^2(3b^6+2b^4c^2+2b^2c^4+3c^6)) :
      -a^8c^2+(b^2- c^2)^3(b^4+c^4)+a^6(-b^4+b^2c^2+4c^4)+ a^4(3b^6+b^4c^2+b^2c^4-6c^6)+ a^2(-3b^8+2b^6c^2+2b^4c^4-5b^2c^6+4c^8) :
     -a^8b^2+ a^6(4b^4+b^2c^2-c^4)-(b^2-c^2)^3(b^4+c^4)+ a^4(-6b^6+b^4c^2+b^2c^4+3c^6)+ a^2(4b^8-5b^6c^2+2b^4c^4+2b^2c^6-3c^8) ).

    ( Mostrar/Ocultar figura )
      Anopolis1891.png


      Los triángulos ABC y NaNbNc son ortológicos.
    • El centro de ortología de NaNbNc respecto a ABC es el punto, sobre la circunferencia circunscrita a NaNbNc:

    D = (a^2(a^6(b^2+c^2)- 3a^4(b^4+c^4)+a^2(3b^6-2b^4c^2-2b^2c^4+3c^6)-b^8+b^6c^2+b^2c^6-c^8) : ... :...)

    con (6,9,13)-número de búsqueda en ETC: 5.485624417414120257712782094

    • El centro de ortología de ABC respecto a NaNbNc es X1141 (sobre la circunferencia circunscrita a ABC).
    X(1141) = GIBERT POINT
    X(1141) was first noted (Hyacinthos #1498, September 25, 2000) by Bernard Gibert as a point of intersection of the circumcircle and certain cubic, denoted Kn. To define Kn, note first that the Neuberg cubic is the locus of a point M such that the reflections of M in the sidelines of triangle ABC are the vertices of a triangle perspective to ABC. The locus of the perspector is the cubic Kn, and X(1141) is the point, other than A,B,C, in which Kn meets the circumcircle.




  • sábados 11 de octubre del 2014

    Triángulos precevianos y rectas de Euler concurrentes

      Sean ABC un triángulo, P un punto y PaPbPc el triángulo preceviano de P.
      Se denotan por da, db y dc las rectas de Euler de los triángulos PaBC, PbCA y PcAB, respectivamente.

      Las rectas de Euler da, db y dc son concurrentes si y solo si P está en una nónica (ver su ecuación baricéntrica), que pasa por los vértices de los triángulos ABC (dobles), medial y excentral, por los centros X1, X2 y X6.

    Las rectas da, db y dc son concurrentes cuando los triángulos precevianos son los triángulos excentral, antimedial y tangencial. En el primer caso las rectas concurren en X100, en el segundo son paralelas a la recta de Euler de ABC y en el tercero concurren en el circuncentro.
    ( Mostrar/Ocultar figura )
      Anopolis1884math.png

      La ecuación de la recta de Euler del triángulo PaBC es:
    a^4v(v-w)w-(b^2-c^2)u(b^2(u^2+v^2-w^2-u(2v+w))+ c^2(u^2-v^2+w^2-u(v+2w)))+ a^2(-c^2(u^3-3vw(v+w)-u^2(3v+2w)+ u(3v^2+4vw+w^2))+ b^2(u^3-3vw(v+w)-u^2(2v+3w)+ u(v^2+4vw+3w^2)))x + ((-b^2+c^2)u+ a^2(u-2w))(-u(c^2(u-w)+b^2w)+ a^2(u^2-u(2v+w)+v(v+2w)))y + ((b^2-c^2)u+ a^2(u-2v))(u(b^2(u-v)+c^2v)- a^2(u^2+w(2v+w)-u(v+2w)))z = 0.

  • jueves 9 de octubre del 2014

    Cónicas homofocales y parábola envolvente de polares

      Sean ABC un triángulo, MaMbMc el triángulo medial y P un punto, la recta da que une P con Ma vuelve a cortar a la circunferencia circunscrita a PBC en Pa; sea Fa la reflexión de Pa en la mediatriz de BC. Similarmente, procediendo cíclicamente, se consideran los puntos Fb y Fc.

      Los triángulos ABC y FaFbFc son perspectivos si y sólo si el punto P está sobre la quíntica Θ de ecuación baricéntrica:

    b^2 c^4 x^4 y + 2 a^2 c^4 x^3 y^2 + b^2 c^4 x^3 y^2 - 2 c^6 x^3 y^2 - a^2 c^4 x^2 y^3 - 2 b^2 c^4 x^2 y^3 + 2 c^6 x^2 y^3 - a^2 c^4 x y^4 - b^4 c^2 x^4 z - b^4 c^2 x^3 y z + b^2 c^4 x^3 y z + 4 a^4 c^2 x^2 y^2 z - 4 b^4 c^2 x^2 y^2 z - 4 a^2 c^4 x^2 y^2 z + 4 b^2 c^4 x^2 y^2 z + a^4 c^2 x y^3 z - a^2 c^4 x y^3 z + a^4 c^2 y^4 z - 2 a^2 b^4 x^3 z^2 + 2 b^6 x^3 z^2 - b^4 c^2 x^3 z^2 - 4 a^4 b^2 x^2 y z^2 + 4 a^2 b^4 x^2 y z^2 - 4 b^4 c^2 x^2 y z^2 + 4 b^2 c^4 x^2 y z^2 - 4 a^4 b^2 x y^2 z^2 + 4 a^2 b^4 x y^2 z^2 + 4 a^4 c^2 x y^2 z^2 - 4 a^2 c^4 x y^2 z^2 - 2 a^6 y^3 z^2 + 2 a^4 b^2 y^3 z^2 + a^4 c^2 y^3 z^2 + a^2 b^4 x^2 z^3 - 2 b^6 x^2 z^3 + 2 b^4 c^2 x^2 z^3 - a^4 b^2 x y z^3 + a^2 b^4 x y z^3 + 2 a^6 y^2 z^3 - a^4 b^2 y^2 z^3 - 2 a^4 c^2 y^2 z^3 + a^2 b^4 x z^4 - a^4 b^2 y z^4=0.

    ( Mostrar/Ocultar figura )
      parabolaFFPmath.png

      Si P=(u:v:w), la recta da es (v-w)x-uy+uz=0, que vuelve a corta a la circunferencia

    a^2 y z + b^2 z x + c^2 x y - ((c^2 u v + b^2 u w + a^2 v w) x (x + y + z))/( u (u + v + w))=0

    en el punto

    Pa = ( a^2u(u+v+w) : -b^2u(u+v-w)-(u-v+w)(c^2u+a^2w) : -b^2u(u+v-w)-a^2v(u+v-w)-c^2u(u-v+w)).

      La reflexión de Pa en la mediatriz de BC es

    Fa = ( a^2u(u+v+w) : -2b^2u(u+v)+v(2c^2u-a^2(u+v-w)) : -2c^2u(u+w)+w(2b^2u-a^2(u-v+w)) ).

      Finalmente, la recta AFa tiene por ecuación:

    (2c^2u(u+w)+ w(-2b^2u+a^2(u-v+w)))y+(-2b^2u(u+v)+ v(2c^2u-a^2(u+v-w)))z=0.

    ( Mostrar/Ocultar figura )
      parabolaFFP1.png


      Puntos en la quíntica Θ:
    Los vértices de los triángulos ABC, medial MaMbMc y circunceviano del simediano A(K)B(K)C(K) (indicación de Bernard Gibert).
    Los centros X3, X6, X15, X16, X187, X524.
    Los puntos del infinito de la quíntica, además de X524, son los puntos cíclicos (dobles).
    Las tangentes en los vértices de ABC son la simedianas.
    La asíntota con punto en el infinito X524 es:

    (b^2-c^2)(11a^6 - 6a^4(b^2+c^2) - 3a^2(5b^4-7b^2c^2+5c^4) + 2b^6+3b^4c^2+3b^2c^4+2c^6)x+....=0

    Si concideramos los pares {P,Q}, siendo Q el centro de perspectividad de los triángulos ABC y FaFbFc, cuando son perspectivos, tenemos:

    {X3, X6}; {X6,X25}; {X15, X62}; {X16, X61}; {X187, X6}; {X524,X2}.

      El eje de Brocard interseca a la quíntica en los cinco puntos: X3, X6, X15, X16 y X187.

    ==============================================================



      Otra interpretación geométrica de los puntos Fa, Fb y Fc, surge del siguiente resultado, que ha compartido conmigo Francisco Javier García Capitán:
      Problem. Prove an generalise that the envolope of the polar of a given point for a system of confocals is a parabola touching the axes of the confocals and having the given point on its directrix.
    E. H. Askwith, A Course of Pure Geometry, p. 255, problem 21.


      Podemos aplicarlo a la Geometría del Triángulo cuando tenemos un triángulo ABC y un punto P.

      Sea ℘a la parábola envolvente de las polares de P respecto a las cónicas homofocales de focos en los vértices B y C; su foco lo denotamos por Fa. Similarmente, procediendo cíclicamente, se consideran los focos Fb y Fc de las parábolas ℘b y ℘c, respectivamente.
      Los triángulos ABC y FaFbFc son perspectivos si y sólo si el punto P está sobre la quíntica Θ.

    ( Mostrar/Ocultar figura )
      parabolaFFPPa.png

      Para determinar la cónica envolvente de las polares de P, respecto a las cónicas homofocales de focos B y C, nos basta con conocer cinco de ellas:
    Las dos tangentes, en P, a las dos cónicas de focos B y C y que pasan por P.
    Y las tres rectas (cónicas degeneradas de la familia de cónicas homofocales de focos B y C) BC, mediatriz de BC y la recta del infinito.

    Tenemos, así, cinco tangentes y la cónica queda determinada; tratándose de una parábola ℘a, al ser tangente a la recta del infinito.

    De la parábola ℘a conocemos dos pares de tangentes ortogonales, por lo que cada par se cortan en su directriz. Esta es, por tanto, la recta da que pasa por P y el punto medio Ma de BC. El foco Fa es el punto de intersección de las polares de P y Ma respecto a la parábola ℘a.
    ( Mostrar/Ocultar figura )
      parabolaFFP.png


  • miércoles 8 de octubre del 2014

    Rectas de Euler concurrentes

    ( Anopolis #1872, Antreas P. Hatzipolakis)

      Sean ABC un triángulo y A'B'C' el triángulo ceviano del incentro.
    Oab y Oac son los circuncentros de los triángulos ABA' y ACA', respectivamente.
    Oa es el circuncentro del triángulo AOabOac. Ob y Oc se definen de manera similar.

      Las recta de Euler de los triángulos ABC, AOabOac, BObcOba, COcaOcb concurren en el inverso del ortocentro en la circunferencia circunscrita, X186.
      El ortocentro del triángulo OaObOc es el punto sobre la recta de Euler:

    X = (r^2 + 2 r R - R^2 + s^2)*X(3) + R^2*X(4),
    R y r, radios de las circunferencias circunscrita e inscrita, y s el semiperímetro.

    ( Mostrar/Ocultar figura )
      Anopolis1872.png

      El circuncentro del triángulo ABA' es:

    Oab = (a^2(bc^2+cSA) : b^2cSB + b(S^2+SASB)) : c^2(-bSB+cSC)),

      El circuncentro del triángulo ACA' es:

    Oac = (a^2(cb^2+bSA) : b^2(-cSC+bSB): c^2bSC + c(S^2+SASC)) ),

      El circuncentro del triángulo AOabOac es:

    Oa = (a^2(b(3SA+SB)+c(3SA+SC)) : b^2(2bSB+c(SB-SC)) : -c^2(b(SB-SC)-2cSC) ).


      El baricentro del triángulo AOabOac es:

    ((-2a^2+(b-c)^2)(b+c) : b(-a^2+2b(b-c)) : c(-a^2+2c(-b+c))).


      La ecuación de la recta de Euler de AOabOac es:
    -a^2bc(b-c)(a^2-b^2-c^2)x - c(a^2-b^2+c^2)(a^4+a^2c(b-2c)+c(b-c)^2(b+c))y + b(a^2+b^2-c^2)(a^4+a^2b(-2b+c)+b(b-c)^2(b+c))z = 0.

      Cíclicamente, se obtienen las ecuaciones de las rectas e Euler de los triángulos BObcOba y COcaOcb. Las tres concurren en el punto X186},

    (a^2(a^2-b^2+c^2)(a^2+b^2-c^2)(a^2-b^2-bc-c^2)(a^2-b^2+bc-c^2) : ... : ...).




      El ortocentro del triángulo OaObOc

    (a^2(a^2-b^2-bc-c^2)(a^5(b+c)+bc(b^2-c^2)^2 - 2a^3(b^3+c^3) - a^2bc(b^2+c^2) + a(b^5-b^4c-bc^4+c^5)) : ... : ...).

    con (6,9,13)-número de búsqueda en ETC: 3.5037197568390148045586626416,

    Nota: El punto X ha sido incluido en ETC como X(6097) = 14th Hatzipolakis-Montesdeoca Point

  • martes 7 de octubre del 2014

    Rectas de Euler y triángulos ortológicos

    ( Anopolis #1881, Antreas P. Hatzipolakis)

      Sean ABC un triángulo, Q un punto sobre la recta de Euler y P su conjugado isogonal (queda sobre la hipérbola de Jerabek).
    Sean Qa, Qb, Qc los mismos puntos que Q en los triángulos PBC, PCA, PAB, respectivamente.
    Se denota por Mqa, Mqb, Mqc los puntos medios de AQa, BQb, CQc, respectivamente.

      Los triángulos ABC y MqaMqbMqc son ortológicos.
      El lugar geométrico de los centros de ortología de ABC con respcto a MqaMqbMqc, cuando Q varía en la recta de Euler, es la hipérbola de Jerabek.
      El lugar geométrico de los centros de ortología de MqaMqbMqc con respcto a ABC, cuando Q varía en la recta de Euler, es una cúbica, con punto doble en el centro de la circunferencia de los nueve pountos, X5, con puntos en la recta del infinito los dos de la hipérbola de Jerabek (X2574, X2575) y el conjugado isogonal (X5663) del antipodal del punto de Tixier; esta cúbica también pasa por el conjugado isogonal (X2883) del ortocentro respecto al triángulo medial.

    ( Mostrar/Ocultar figura )
      Anopolis1881.png

      Un punto Q sobre la recta de Euler tal que GQ:QO=m:n, tiene coordenadas baricéntricas:

    (a^4(3m+n) - a^2(b^2+c^2) (3m+2n)+(b^2-c^2)^2n:...:...)

      Para un punto P(u:v:w) arbitrario del plano, el punto Qa respecto al triángulo PBC, que es el mismo que Q respecto al triángulo ABC es:

    Qa = (-(b^2-c^2)^2nu^2 + a^2u(b^2(2nu+3m(u+v-w)) + c^2(2nu+3m(u-v+w))) - a^4(nu^2+3m(u^2+2vw+u(v+w))):
    -a^4(-3mvw + nu(u+2v+w)) - (b^2-c^2)u(-c^2(3mv+n(u+2v+w)) + b^2(3m(u+v)+n(u+2v+w))) + a^2(c^2(3mv(u-w) + 2nu(u+2v+w)) + b^2(2nu(u+2v+w) + 3m(u^2+uv+2uw+vw))):
    -a^4(-3mvw+nu(u+v+2w)) - (b^2-c^2)u(b^2(3mw+n(u+v+2w)) - c^2(3m(u+w) + n(u+v+2w))) + a^2(b^2(3m(u-v)w + 2nu(u+v+2w)) + c^2(2nu(u+v+2w) + 3m(u^2+2uv+uw+vw)))).

      Las coordenadas de los puntos Qb y Qc se obtienesn por permutación cíclica.
      El punto medio de A y Qa es:

    Mqa = (-(b^2-c^2)^2u(3m(u+v+w)+n(4u+3(v+w)))- a^4(6m(u+v)(u+w)+nu(4u+3(v+w)))+ a^2u(b^2(3m(3u+3v+w)+2n(4u+3(v+w)))+ c^2(3m(3u+v+3w)+ 2n(4u+3(v+w)))) :
    -a^4(-3mvw+ nu(u+2v+w))-(b^2- c^2)u(-c^2(3mv+n(u+2v+w))+ b^2(3m(u+v)+n(u+2v+w)))+ a^2(c^2(3mv(u-w)+2nu(u+2v+w))+ b^2(2nu(u+2v+w)+ 3m(u^2+uv+2uw+vw))) :
    -a^4(-3mvw+ nu(u+v+2w))-(b^2- c^2)u(b^2(3mw+n(u+v+2w))- c^2(3m(u+w)+n(u+v+2w)))+ a^2(b^2(3m(u-v)w+2nu(u+v+2w))+ c^2(2nu(u+v+2w)+3m(u^2+2uv+uw+vw))))



     • El centro de ortología de MqaMqbMqc con respecto a ABC, para cualquier punto P del plano, es:

    ( a^4n(v+w) + a^2(b^2+c^2)(3m(u+v+w)+2n(2u+v+w)) - (b^2-c^2)^2(3m(u+v+w)+n(4u+3(v+w))) : ... : ...).


      En particular, si P es el conjugado isogonal de Q, poniendo m=tn:

    ρ(x:y:z) = ( - a^8(b^2+c^2)(5+6t) + a^6(2b^2c^2(2+3t)^2+3b^4(4+5t)+3c^4(4+5t)) - a^4(b^2+c^2)(3c^4(2+3t)+b^4(6+9t) - a^2(b^2-c^2)^2(3b^2c^2(2+t)(2+3t)^2+b^4(4+3t) +3(b^2-c^2)^4(b^2+c^2)(1+t) -b^2c^2(28+78t+72t^2+27t^3)) + c^4(4+3t)) : ... : ...)

      Eliminado ρ y t, resulta la ecuación implícita de una cúbica.


     • El centro de ortología de ABC con respecto a MqaMqbMqc , para cualquier punto P del plano, es:

    (u/(a^2(3m(u+v)(u+w)+2nu(u+v+w))- u(b^2(3m(u+v)+2n(u+v+w))+ c^2(3m(u+w)+2n(u+v+w)))):...:...).

      En particular, si P es el conjugado isogonal de Q, poniendo m=tn:

    ρ(x:y:z) = (1/(2(a-b-c)(a+b-c)(a-b+c)(a+b+c)(a^2+b^2- c^2)(a^2+b^2+c^2) + 3(a^8-3a^6b^2+4a^4b^4-3a^2b^6+b^8-2a^6c^2- 12a^4b^2c^2-12a^2b^4c^2-2b^6c^2+13a^2b^2c^4+ 2a^2c^6+2b^2c^6-c^8)t - 9a^2b^2(a^4-2a^2b^2+b^4+3a^2c^2+3b^2c^2-4c^4)t^2) : ...:...).

      Eliminado ρ y t, resulta la ecuación implícita de la hipérbola de Jerabek:

    a^2(b^2-c^2)(-a^2+b^2+c^2)yz+ b^2(-a^2+c^2)(a^2-b^2+c^2)zx+(a^2-b^2)c^2(a^2+b^2-c^2)xy = 0.



  • domingo 5 de octubre del 2014

    Circunferencias tangentes a los lados de un triángulo

    (Inspirado en Problem JP27, Ercole Suppa,)

      Sean ABC un triángulo, P un punto y A'B'C' su triángulo ceviano.
    Se denota por:
      Φab la circunferencia tangente en B a BC y que pasa por C'.
      Φac la circunferencia tangente en C a BC y que pasa por B'.

    t'a la tangente exterior (distinta de BC) a las circunferencias Φab y Φac.

      De forma similar, procediendo cíclicamente, se definen las tangentes exteriores t'b y t'c a las correspondientes circunferencias.

      Las tangentes t'a, t'b y t'c delimitan un triángulo DEF perspectivo con ABC.

    ( Mostrar/Ocultar figura )
      SuppaP27Ceviano.png
      En coordenadas baricéntricas, si P(u:v:w), se tienen las ecuaciones de las circunferrencias:

    Φab: a^2yz+b^2zx+c^2xy-(x+y+z)(c^2vx/(u+v)+a^2z)=0
    Φac: a^2yz+b^2xz+c^2xy-(x+y+z)(b^2wx/(u+w)+a^2y)=0.

    Y la ecuación de la tangente exterior (distinta de BC) a estas circunferencias es:
    t'a: (a^4(u+v)^2(u+w)^2+(b^2w(u+v)-c^2v(u+w))^2- 2a^2(u+v)(u+w)(b^2w(u+v)+c^2v(u+w)))x + 4a^2c^2u(u+v)(u+w)^2y + 4a^2b^2u(u+v)^2(u+w)z=0.

      El centro de perspectividad Q de los triángulos ABC y DEF tiene primera coordenada baricéntrica:

    u/((v+w)(b^4(u+v)^2w^2 + c^4v^2(u + w)^2 + a^4(u+v)^2(u+w)^2 - 2a^2(u+v)(u+w)(b^2(u+v)w+c^2v(u+w))- 2b^2c^2(u+v)(u+w)(vw+2u(v+w)))).



      El centro de perspectividad Q de los triángulos ABC y DEF coincide con P si P está sobre la elipse circunscrita de Steiner.
      Además, en este caso:
      • Las seis circunferencias Φab, Φac, Φbc, Φba, Φca, Φcb concurren en el punto T donde la recta X99P vuelve a corta a la circunferencia circunscrita.
      • Los puntos de tangencia de las tangentes t'a, t'b y t'c con las circunferencias Φab, Φac, Φbc, Φba, Φca, Φcb están en la circunferencia reflexión de la circunferencia circunscrita en el punto T.

    ( Mostrar/Ocultar figura )
      SuppaP27Steiner.png

      La correspondencia P↦T, entre centros del triángulo incluidos actualmente en ETC, sobre la elipse circunscrita de Steiner y centros en la circunferencia circunscrita, está dada por los siguientes pares {P,T} de sus índices:
    {99,110}, {190,101}, {290,98}, {648,112}, {664,109}, {666,919}, {668,100}, {670,99}, {671,111}, {886,?}, {889,898}, {892,691}, {903,106}, {1121,2291}, {1494,74}, {2479,?}, {2480,?}, {2481,105}, {2966,2715}, {3225,699}, {3226,727}, {3227,739}, {3228,729}, {4555,901}, {4562,813}, {4569,934}, {4577,827}, {4586,825}, {4597,4588}, {5641,842}.

      Los tres puntos T, sobre la circunferencia circunscrita, que no figuran en ETC, son:
      • Si P= X886,

    (1/((b^2-c^2)(a^2(b^2+c^2)-2b^2c^2)) : ... : ....),

    con (6,9,13)-número de búsqueda en ETC: 6.784805747166842285745353786
      • Si P= X2479 (de los dos puntos de intersección de la recta de Euler con la elipse circunscrita de Steiner, X2479 es la más cercano a X3).

    (a^2 (a^8 - a^6(b^2+c^2) + a^4b^2c^2 - a^2(b^2-c^2)^2(b^2+c^2) + b^8- b^6c^2-b^2c^6+c^8 + (-2a^4+a^2(b^2+c^2)+(b^2-c^2)^2)Λ) : ... : ....),

    donde Λ=[a^8 - a^6(b^2+c^2) + a^4b^2c^2 - a^2(b^6-b^4c^2-b^2c^4+c^6) + b^8-b^6c^2-b^2c^6+c^8]½ y con (6,9,13)-número de búsqueda en ETC: 6.770997724920497989820816646
     • Si P= X2480 (de los dos puntos de intersección de la recta de Euler con la elipse circunscrita de Steiner, X2480 es la más alejado de X3).

    (a^2 (a^8 - a^6(b^2+c^2) + a^4b^2c^2 - a^2(b^2-c^2)^2(b^2+c^2) + b^8- b^6c^2-b^2c^6+c^8 - (-2a^4+a^2(b^2+c^2)+(b^2-c^2)^2)Λ) : ... : ....),

    donde Λ=[a^8 - a^6(b^2+c^2) + a^4b^2c^2 - a^2(b^6-b^4c^2-b^2c^4+c^6) + b^8-b^6c^2-b^2c^6+c^8]½ y con (6,9,13)-número de búsqueda en ETC: 0.11661014590270525314320708434


      Consideremos ahora los segundos puntos A'1, B'1, C'1 de intersección de los pares de circunferencias Φbc y Φcb, Φca y Φac, Φab y Φba, respectivamente.

      El triángulo A'1B'1C'1 es perpectivo con ABC, con centro de perspectividad en el producto baricéntrico Y de P y el simediano.
      Tambien los triángulos A'1B'1C'1 y DEF son perspectivos.

    ( Mostrar/Ocultar figura )
      SuppaP27Ceviano1.png

      Se tiene que A'1 = (-a^2vw/(v+w) : b^2v : c^2w ); B'1 = (a^2u : -b^2uw/(u+w) : c^2w); y C'1 = (a^2u : b^2v : -c^2uv/(u+v)). Por lo que el centro de perspectividad de A'1B'1C'1 y ABC es:

    (a^2u : b^2v : c^2w).


      El punto de concurrencia de las rectas DA'1, EB'1, FC'1 tiene coordenadas (a^2u f(a,b,c,u,v,w):...:...), donde f(a,b,c,u,v,w) es un polinomio de grado 12 en las variables (a,b,c) y (u,v,w).

      En el caso particular de P=X69 los triángulos A'1B'1C'1 y DEF coinciden con la reflexión de ABC respecto al circuncentro.

  • jueves 25 de septiembre del 2014

    Lugares geométricos asociados a ejes radicales

    ( Anopolis #1845, Antreas P. Hatzipolakis)

      Sean ABC un triángulo, P un punto y A'B'C' su triángulo ceviano.
    Se denota por:
    Ab y Ac las proyecciones ortogonales de B' y C' sobre AA'.
      (Oab) y (Oac) las circunferencias circunscritas a los triángulos BB'Ab y CC'Ac, respectivamente.
      Similarlente se definen las circunferencias (Obc), (Oba), (Oca) y (Ocb).
      Ra es el eje radical de las circunferencias (Oab) y (Oac). Rb es el eje radical de las circunferencias (Obc) y (Oba). Rc es el eje radical de las circunferencias (Oca) y (Ocb).
      Finalmente, A* = Ra ∩ BC, B* = Rb ∩ CA, C* = Rc ∩ AB.

      Si (u:v:w) son las coordenadas baricéntricas del punto P,
    Ab = (c^2v(2u+w)(v+w)-w(a^2v(2u-v+w)-b^2(2u-v)(v+w)) : -vw(a^2v-c^2v-b^2(v+2w)) : w^2(-a^2v+c^2v+b^2(v+2 w))),
    Ac = (c^2v(2u-w)(v+w)+w(-a^2v(2u+v-w)+b^2(2u+v)(v+w)) : v^2((-a^2+b^2)w+c^2(2v+w)) : vw((-a^2+b^2)w+c^2(2v+w))).

      El punto de intersección del eje radical Ra con lado BC es:
    A* = (0 : (u+v)(c^2SCuv-b^2SBuw+SBSCvw-a^2b^2w^2)(b^2uw+v(SAu-SBv+SCw)) : (u+w)(-c^2SCuv-a^2c^2v^2+b^2SBuw+SBSCvw)(c^2uv+w(SAu+SBv-SCw)).

      El lugar geométrico de los puntos P tales que A*B*C* es el triángulo ceviano de un punto Q es una séxtica con puntos dobles en los vértices de ABC, pasa por los vértices del triángulo excentral IaIbIc y por los pies de las bisectrices interiores y exteriores.

    ( Mostrar/Ocultar figura )
      Anopolis1845.png
    ( Mostrar/Ocultar figura )
      Anopolis1845m.png

      Esta séxtica pasa por los centros del triángulo X1, X2, X3, X15, X16.
      Si P es el incentro, el punto Q es el X21, punto de Schiffler.
      Si P es el baricentro, el punto Q es el punto de primera coordenada

    (5a^2-b^2-c^2) (a^4-6a^2b^2+b^4+2a^2c^2+2b^2c^2-3c^4) (a^4+2a^2b^2-3b^4-6a^2c^2+2b^2c^2+c^4),

    con (6,9,13)-número de búsqueda en ETC: 0.9249381788103438608692305
      Si P es el circuncentro, el punto Q es el punto de primera coordenada

    a^2(a^4-(b^2-c^2)^2)^2 (a^12 - 5a^10(b^2+c^2) + 5a^8(2b^4+3b^2c^2+2c^4) - 2a^6(5b^6+7b^4c^2+7b^2c^4+5c^6) + a^4(5b^8+2b^6c^2+4b^4c^4+2b^2c^6+5c^8) - a^2(b^2-c^2)^4(b^2+c^2) - b^2c^2(b^2-c^2)^2(b^4+c^4) ),

    con (6,9,13)-número de búsqueda en ETC: -6.0305312940593606748466345


      El lugar geométrico de los puntos P tales que los ejes radicales Ra, Rb y Rc son concurrentes es una curva algebraica de grado once con puntos cuádruples en los vértices de ABC, pasa por los vértices del triángulo excentral IaIbIc y por los pies de las alturas.

    ( Mostrar/Ocultar figura )
      Anopolis1845C11.png

      Esta curva pasa por los centros del triángulo X1, X4, X110.
      Si P es el incentro, el punto de concurrencia de los ejes radicales es el circuncentro.
      Si P es el ortocentro, el punto donde se cortan los tres ejes radicales es también el ortocentro
      Si P es el X110 (foco de la parábola de Kiepert), los ejes radicales se corta en el punto Y, de primera coordenada

    a^2(3a^8 - 7a^6(b^2+c^2) + a^4(3b^4+13b^2c^2+3c^4) + a^2(3b^6-7b^4 c^2-7b^2c^4+3c^6) - (b^2-c^2)^2(2b^4+3b^2c^2+2c^4))

    con (6,9,13)-número de búsqueda en ETC: 4.824096254164793916494256409

  • miércoles, 24 de septiembre del 2014

    Propiedad geométrica del punto X2136

     The eigencenter of a triangle is defined in the Glossary and in TCCT, p. 192.
     Let T be a central triangle, and let U(T) be the unary cofactor triangle of T. Then T and U(T) are perspective, and their perspector is the eigencenter of T.
      In case the triangle is the cevian triangle of a point U = (x:y:z), in barycentric coordinates, the eigencenter is given by
    (a^2yz(a^2y^2z^2-b^2z^2x^2-c^2x^2y^2) : b^2zx(b^2z^2x^2-c^2x^2y^2-a^2y^2z^2) : c^2xy(c^2x^2y^2-a^2y^2z^2-b^2z^2x^2))
    This point is now named the eigentransform of U, denoted by ET(U).

    In particular:
    If X(57) is the perspector of the intouch triangle and excentral triangle, X(2136) is the eigentransform of X(57).


      Sean ABC un triángulo, IaIbIc el triángulo excentral, Φa la hipérbola circunscrita a IaIbIc y de asíntotas paralelas a las bisectrices interiores en B y C, y D el centro de Φa. De foma similar, de consideran las hipérbolas Φb y Φc y sus respectivos centros E y F.

    Las rectas DIa, EIb, FIc concurren en X2136.

    ( Mostrar/Ocultar figura )
      X(2136).png

      La ecuación de la hipérbola Φa es:
    b(a-b-c)cx^2 + 2a^2cy^2 + 2a^2bz^2 + a^2(a+b+c)yz + ab(a+b+c)zx + ac(a+b+c)xy=0.
    Y su centro D tiene coordenadas:
    (a(a^2+2ab+b^2+2ac-6bc+c^2) : -b(a^2+2ab+b^2-2ac+6bc-3c^2) : -c(a^2-2ab-3b^2+2ac+6bc+c^2))

      El punto donde concurren las rectas DIa, EIb, FIc es X2136: (a(a-b-c)(a^2+2a(b+c)+b^2-6bc+c^2) : ... : ... ).


      En general, si se toma el tiángulo anticeviano PaPbPc de un punto P=(p:q:r), y D es ahora el centro de la hipérbola circunscrita a PaPbPc con asíntotas paralelas a PPb y PPc, y lo mismo para E y F, las rectas DPa, EPb, FPc concurren en el punto:
    Q=(p(p-q-r)(p^2+2p(q+r)+q^2-6qr+r^2) : ... : ... ).

    Pares de centros del triángulo {P,Q} son:
    {X(1), X(2136)}, {X(2),X(2)}, {X(3), X(1498)}, {X(5), X(52)}, {X(9), X(2951)}, {X(10), X(72)}, {X(141), X(3313)}, {X(178), X(177)}, {X(1125), X(4065)}

  • viernes 18 de septiembre del 2014

    Hipérbolas circunscritas con asíntotas paralelas a cevianas



      Dados un triángulo ABC y un punto P, denotamos por Φa la hipérbola circunscrita a ABC y de asíntotas paralelas a PB y PC; sea Oa su centro (ver construccción en Pt_Pt_P). Similarmente, se consideran las hipérbolas Φb y Φc, y sus respectivos centros Ob y Oc.

      Las rectas AOa, BOb y COc son concurrentes, para cualquier punto P.

    ( Mostrar/Ocultar figura )
      HiperbolasCevianas.png

      Si las coordenadas baricéntricas de P son (u:v:w), la ecuación de la hipérbola Φa es u^2yz-v(u+w)zx-w(v+u)xy=0, y su centro es:

    Oa(u(u^2+u(v+w)+2vw) : v(u+w)(u+v-w) : w(v+u)(u-v+w))

      El punto Q de intersección de las rectas AOa, BOb y ACc es:

    (u(v+w)(u^2-(v-w)^2) : v(w+u)(v^2-(w-u)^2 : w(u+v)(w^2-(u-v)^2).

      Pares de puntos (P,Q) que son ambos centros del triángulo que figuran actualmente en ETC:

    (X(1),X(65))  (X(2),X(2))  (X(3),X(5562))  (X(4),X(64))  (X(6),X(1843))  (X(7),X(3062))  (X(8),X(3680))  (X(9),X(3059))  (X(10),X(4647))  (X(37),X(2667))



      Los centros Oa, Ob y Oc de las hipérbolas Φa, Φb y Φc están alineados si y solo si P está en la cúbica isotómica no-pivotal de raíz el baricentro y parámetro k=6 del triángulo medial.
      La envolvente de las rectas que contienen los centros, cuando P varía en esta cúbica, es la elipse inscrita de Stenier.

      La ecuación de la cúbica, en coordenadas baricéntricas respecto al triángilo medial, es x(y^2+z^2)+y(z^2+x^2)+z(x^2+y^2)+6xyz=0.

    ( Mostrar/Ocultar figura )
      HiperbolasCevianasCentrosAlineados.png



      Denotemos por Pa el cuarto punto de intersección de la hipérbola Φa con la circunferencia circunscrita. Similarmente se definen Pb, Pc.
      Si P(u:v:w), las coordenadas baricéntricas de Pa son:

    Pa = (1/(b^2(u+v)w-c^2v(u+w)) : -1/(c^2u^2+a^2w(u+v)) : 1/(b^2u^2+a^2v(u+w)).

      Si P está sobre la circunferencia circunscrita, las rectas AOa, BOb, COc son paralelas, de dirección perpendicular a la del diámetro que pasa por P.

    ( Mostrar/Ocultar figura )
      HiperbolasCevianasCuartoPuntoO(R).png

      Si P está sobre la cúbica isogonal no-pivotal Kjp= K024 = nK0+(X6,X6) , las rectas AOa, BOb, COc concurren en el conjugado isogonal P' de P, sobre dicha cúbica.

    ( Mostrar/Ocultar figura )
      HiperbolasCevianasCuartoPuntonK.png

      Tenemos así la siguiente caractización de la cúbica nK0+(X6,X6):

      Kjp = K024=nK0+(X6,X6) is the locus of point P such that AP and APa are isogonal lines with respect to the sidelines AB and AC, BP and BPb are isogonal lines with respect to the sidelines BC and BA, and CP and CPc are isogonal lines with respect to the sidelines CA and CB.



  • martes 16 de septiembre del 2014

    La isocúbica no-pivotal nK(X577,X2,X3)

    (Variación del Problema, propuesto por Dao Thanh Oai)

      Dado un triángulo ABC, sean DEF su triángulo antimedial y una recta δ de ecuación baricéntricas px+qy+rz=0, cuyos puntos de intersección con los lados EF, FD, DE son, respectivamente, A0(q-r:-p:p), B0(q:-p+r:-q), C0(-r:r:p-q).
      Sean M((q-r)(qr+μ) : (r-p)(rp+μ) : (p-q)(pq+μ)) y P((q-r)(qr+t) : (r-p)(rp+t) : (p-q)(pq+t) dos puntos sobre la recta δ.
      El inverso del punto A0 en la circunferencia C(M,P), de centro M que pasa por P, es:

    A1 = ((q-r) (-p^2q r + p q^2r + p q r^2 - p^2μ + p q μ + p r μ + q r μ + 2 t μ- t^2) :
    (p-r) (p^3r - p^2q r - p^2r^2 + p^2μ - p q μ - 2 p r μ - 2 t μ+ t^2) : -(p-q) (p^3q - p^2q^2 - p^2q r + p^2μ - 2 p q μ - p r μ - 2 t μ+ t^2)).

      Los inversos B1, C1 de los puntos B0, C0 se obtienen por permutación cíclica.

      Las afirmaciones siguientes surgen del estudio hecho en HG140914:

    ∗  Las rectas DA1, EA1, FA1 concurren en un punto N.

    ∗   Cuando el punto P queda fijo y M varía sobre la recta δ, el punto N describe la cónica δP, circunscrita a DEF tangente a δ en P.

    ∗ El lugar geométrico del centro Q de la cónica δP, cuando P se mueve sobre la recta δ es la cuártica Φδ con puntos singulares en los vértices de ABC y tangente a los lados de DEF en los puntos de corte con la recta δ, de ecuación:

      El cuarto punto común de la cuártica Φδ y la recta δ es el punto D0 de coordenadas baricéntricas:

    D0 = (qr(q-r) : pr(r-p) : pq(p-q)).

      Cuando la recta δ gira alrededor del circuncentro, el punto D0 recorre la isocúbica no-pivotal nK(X577,X2,X3).

    x (c^4 (a^2 + b^2 - c^2)^2 y^2 + b^4 (a^2 - b^2 + c^2)^2 z^2) + y (a^4 (b^2 + c^2 - a^2)^2 z^2 + c^4 (b^2 - c^2 + a^2)^2 x^2) + z (b^4 (c^2 + a^2 - b^2)^2 x^2 + a^4 (c^2 - a^2 + b^2)^2 y^2) + 2 (b^2 c^2 (b^2 - c^2)^2 + a^6 (b^2 + c^2) + a^2 (b^2 - c^2)^2 (b^2 + c^2) - a^4 (2 b^4 + b^2 c^2 + 2 c^4)) x y z =0
    ( Mostrar/Ocultar figura )
      ADGEOM1694AntiMedial.png

      La cúbica nK(X577,X2,X3) pasa por X3 (doble) y por X4240 (que es el cuarto punto D0, cuando δ es la recta de Euler). Sus asíntotas son paralelas a los lados de ABC, por lo que delimitan un triángulo homotético a ABC con centro de homotecia X577 (polo de la cúbica). nK(X577,X2,X3) no figura actualmente en el catálogo CTC, de Bernard Gibert

      Cuando δ es la recta de Euler, la cuártica Φδ y la circunferencia circunscrita tienen un sólo punto común X925 (aparte de A, B, C), con tangente común. En este caso, la cuártica contiene a X648, tripolo de la recta de Euler.

  • domingo 14 de septiembre del 2014

    Inversos de los puntos de corte de una recta y un triángulo

    ( ADGEOM #1698, Dao Thanh Oai)

    Problem , Dao Thanh Oai

    Let ABC be a triangle, let three points A0, B0, C0 lie on BC, CA, AB such that A0, B0, C0 are collinear. Let circle (P) which center P lie on the line A0B0C0. Denote A1, B1, C1 are the inverse (or inversion, or reflection) of A0, B0, C0 in (P), resp. Show that AA1, BB1, CC1 are concurrent.


      Dado un triángulo ABC y una recta δ de ecuación baricéntricas px+qy+rz=0, los puntos de intersección con los lados BC, CA, AB son, respectivamente, A0(0:-r:q), B0(r:0:-p), C0(-q:p:0).
      Sean M((q-r)(qr+μ) : (r-p)(rp+μ) : (p-q)(pq+μ)) y P((q-r)(qr+t) : (r-p)(rp+t) : (p-q)(pq+t) dos puntos sobre la recta δ.
      El inverso del punto A0 en la circunferencia C(M,P) de centro M que pasa por P es:

    A1 = ((q-r)(qr+t)(qr-t+2μ) : (r-p)(pqr^2-t^2+prμ+qrμ+2tμ) : (p-q)(pq^2r-t^2+pqμ+qrμ+2tμ)).

      Los inversos B1, C1 de los puntos B0, C0 se obtienen por permutación cíclica.
      Las rectas AA1, AA1, AA1 concurren en el punto:

    N = ((q-r)/(p^2qr+p(q+r)μ-t(t-2μ)) : ... : ...).

    ( Mostrar/Ocultar figura )
      ADGEOM1694.png

      Cuando el punto P queda fijo y M varía sobre la recta δ, el punto N describe la cónica δP, circunscrita a ABC tangente a δ en P:

    δP: p(q-r)^2+(qr+t)^2yz + q(r-p)^2+(rp+t)^2zx + r(p-q)^2+(pq+t)^2)xy=0.

      El centro de δP es el punto:

    Q = (p(q-r)(qr+t)^2 (p^2qr(q+r)-pqr(q^2-2qr+r^2-4t)+(q+r)t^2) : ... : ...).


      El lugar geométrico del centro de la cónica δP, cuando P se mueve sobre la recta δ es la cuártica Φδ con puntos singulares en los vértices del triángulo medial y tangente a los lados de ABC en los puntos de corte con la recta δ, de ecuación:

    p^2x^4 + (-2p^2+2pq)x^3y + (p^2-4pq+q^2)x^2y^2 + (2pq-2q^2)xy^3 + q^2y^4 + (-2p^2+2pr)x^3z + (2p^2-2qr)x^2yz + (2q^2-2pr)xy^2z + (-2q^2+2qr)y^3z + (p^2-4pr+r^2)x^2z^2 + (-2pq+2r^2)xyz^2 + (q^2-4qr+r^2)y^2z^2 + (2pr-2r^2)xz^3 + (2qr-2r^2)yz^3 + r^2z^4=0.


    ( Mostrar/Ocultar figura )
      ADGEOM1694N1.png

      El cuarto punto común de la cuártica Φδ y la recta δ es el punto D0 de coordenadas baricéntricas:

    D0 = (p(q^2-r^2) : q(r^2-p^2) : r(p^2-q^2)).

      Si D1 es el inverso de D0 respecto a C(M,P) la recta ND1 pasa por un punto fijo N1 sobre la cónica δP, cuando P queda fijo y M varía.

      Las coordenadas de D1 y N1 son

    D1 = ((q-r)(p(q+r)(qr+μ)+(qr+t)(qr-t+2μ)) : ... : ...),
    N1 = (qr(q-r)(qr+μ)/(q+r) : rp(r-p)(rp+μ)/(r+p) : pq(p-q)(pq+μ)/(p+q)).

      El lugar geométrico del punto N1, cuando P, es la cónica inscrita en ABC tangente a δ en D0.

    Esta cónica inscrita es la envolvente de las rectas PN1 y su perspector es:

    P1 = (q^3r^3/(q^2-r^2) : r^3p^3/(r^2-p^2) : p^3q^3/(p^2-q^2)).



    CASOS PARTICULARES:

     • Cuando δ es la recta de Euler, la cuártica Φδ la corta en X1650, que es la posición del centro de δP cuando P tiende al punto del infinito de la recta de Euler. El punto X136 también está en Φδ, que es el centro de δP cuando P es el ortocentro. En X136 la circunferencia de Euler y la cuártica asociada a la recta de Euler son tangentes, por lo que no tienen más puntos comunes que éste y los puntos medios de los lados.
      Cuando P es el ortocentro la cónica δP es la hipèrbola equilátera de centro X136, pasando por los centros de ETC con índices 4, 93, 225, 254, 264, 393, 847, 1093, 1105, 1179, 1217, 1300, 1826. Su ecuación es:

    (b^2-c^2)yz/SA + (c^2-a^2)zx/SB + (a^2-b^2)xy/SC = 0.

    Su perspector es el centro radical de las circunferencias circunscrita, de los nueve puntos y de Taylor, X2501.
    ( Mostrar/Ocultar figura )
      ADGEOM1694Euler.png


     • Cuando δ es la eje de Brocard, la cuártica Φδ lo corta en el punto:

    (a^2(b^2-c^2)^2(b^2+c^2-a^2)(a^2(b^2+c^2)-b^4-c^4) : ... : ...)

    (con (6,9,13)-número de búsqueda en ETC: 2.9801797228438530320754966389) que es la posición del centro de δP cuando P tiende al punto del infinito del eje de Brocard.
    El punto X1084 también está en Φδ, que es el centro de δP cuando P es el simediano.
      Cuando P es el simediano la cónica δP es la hipèrbola circunscrita que pasa por el baricentro y simediano, además de los centros de ETC con índices 25, 37, 42, 111, 251, 263, 308, 393, 493, 494, 588, 589, 694, 941, 967, 1169, 1171, 1218, 1239, 1241, 1383, 1400, 1427, 1880, 1976, 1989, 2054, 2165, 2248, 2350, 2395, 2433, 2963, 2981, 2987, 2998, 3108, 3228, 3444, 3457, 3458, 3572, 5638, 5639. Su ecuación es:

    a^2(b^2-c^2)yz + b^2(c^2-a^2)zx + c^2(a^2-b^2)xy = 0.

    Su perspector es el centro radical de las circunferencias circunscrita, de los nueve puntos y de Taylor, X2501.
    ( Mostrar/Ocultar figura )
      ADGEOM1694Brocard.png


     • Cuando δ es una recta que pasa por X5 (centro de la circunferencia de los nueve puntos), el cuarto punto D0 de interseccion de δ y la cuártica Φδ recorre la cúbica nK(X577,X2,X3) no-pivotal del triángulo medial, que pasa por X5 (singular) y por X1650 (X4240 del triángulo medial). Las asíntotas de esta cúbica son paralelas a los lados de ABC. El centro de homotecia de ABC y el triángulo delimitado estas las asíntotas es X233, cociente ceviano del baricentro y el centro de la circunfencia de los nueve puntos del triángulo medial.
    ( Mostrar/Ocultar figura )
      ADGEOM1694X5.png


  • viernes 12 de septiembre del 2014

    "Porismcircles" y ejes radicales



    The porismcircles, Floor van Lamoen

    The midcircle (circle of antisimilitude) of two circles is the circle that swaps these two circles by inversion.
    Inversion in the midcircle of incircle and circumcircle maps the sides of a triangle to a triple of circles tangent to the circumcircle and intersecting on the incircle. I call them porismcircles.


    ( Mostrar/Ocultar figura )
      porismcircles.png

      La circunferencia de antisimilitud ("midcircle") de dos circunferencias es una circunferencia respecto a la cual las dos son mutuamente inversas.
      Sean ABC un triángulo y Γ la circunferencia de antisimilitud de sus circunferencias inscrita y circunscrita que tiene centro en el centro de homotecia interior, X55, de estas dos circunferencias.
      Se denota por (Ka) la circunferencia inversa del lado BC en la inversión respecto a Γ, por (Ja) la circunferencia que pasa por B y C y es tangente a la circunferencia inscrita y por ea el eje radical de (Ka) y (Ja). Similarmente se toman los ejes radicales eb y ec.

      El triángulo delimitado por los ejes radicales ea, eb y ec es perspectivo con ABC con centro de perspectividad el punto de coordenadas baricentricas:

    (a/(a^4-2a^2b c-2a^3(b+c)+2a(b^3+c^3)-(b-c)^4) : ... : ...)

    con (6,9,13)-número de búsqueda en ETC: 2.2548851539334829652783884
    ( Mostrar/Ocultar figura )
      HG120914.png


  • miércoles 10 de septiembre del 2014

    Lugares geométricos asociados a rectas de Euler

    ( Anopolis #1834, Antreas P. Hatzipolakis)

      Sean ABC un triángulo, P un punto y A'B'C' su triángulo ceviano.
    Se denota por:
    Pab y Pac las paralelas por A a PB y PC, resp.
    Ab, Ac las proyecciones ortogonales de A' sobre Pab y Pac, resp.
    La, Lb, Lc las rectas de Euler de los triángulos AAbAc, BBcBa, CCaCb, resp.

    Si (u:v:w) son las coordenadas baricéntricas,

    Ab = (w(u+v+w)((-b^2+c^2)u+a^2(u+2w)) :
    (u+w)(-w(a^2(u-v+w)+b^2(-u+v+w))+c^2(w(v+w)+u(2 v+w))) :
    w(w(a^2(u-v+w)+b^2(-u+v+w))-c^2(w(v+w)+u(2v+w))) )

    Las coordenadas de Ac se obtienen partir de las de Ab, permutando sus dos últimas componentes y permutando "b" por "c" y "v" por "w".

      El lugar geométrico de los puntos P tales que las rectas La, Lb, Lc son paralelas es una séxtica con puntos dobles en los vértices de ABC.

    ( Mostrar/Ocultar figura )
      Anopolis1834lg6.png

      El lugar geométrico de los puntos P tales que las rectas La, Lb, Lc son concurrentes una séptica con puntos dobles en los vértices de ABC y que pasa por los centros X3, X4, X13, X14, X20.

    ( Mostrar/Ocultar figura )
      Anopolis1834lg7.png

    En particular, cuando P es el ortocentro (A'B'C' triángulo órtico) el punto donde concurren las rectas La, Lb, Lc es X973 (1st Ehrmann Point, nombre propuesto en Hyacinthos #3694, Antreas P. Hatzipolakis).
    
    
    Dear Jean-Pierre,
    
    [APH]:
     So, we have the Theorem:
    
     Let AA', BB', CC' be the three altitudes of ABC, and
     Let Ab, Ac be the orth. proj. of A' on AB, AC resp.
     Bc, Ba " B' BC, BA
     Ca, Cb " C' CA, CB
    
     Then the Euler lines of A'B'C', A'AbAc, B'BcBa, C'CaCb are
     concurrent.
    
     Which is the point of concurrence [a point lying on the Euler
     line of the orthic triangle A'B'C' of ABC]? Is it in ETC?
    
     
     [JPE]:
      X(442) of the orthic triangle (not in ETC, I think)
    
    [JPE]:
    Let's name them temporarily (or not!) as:
     
     Triangles A'AbAc, B'BcBa, C'CaCb = Orthiac Triangles
     
    [APH]:
    
      A PS for Clark Kimberling:
     
     Dear Clark,
     
     If the above  point of concurrence of the Euler lines
     of the Orthiacs (ie X442 of Orthic)  is not already
     in your list, then you may of course include them, but with these 
    name:
     1st Ehrmann Point.
     

      El lugar geométrico de los puntos P tales que los triángulos ABC y el delimitado por las rectas La, Lb, Lc son ortológicos es una curva algebraica de grado ocho, con puntos triples en los vértices de ABC y pasa por el foco de la parábola de Kiepert, X110..

    ( Mostrar/Ocultar figura )
      Anopolis1834lg8Orth.png


  • domingo 7 de septiembre del 2014

    El centro X4240 y cuatro rectas de Euler

    ( ADGEOM #1629, Dao Thanh Oai)

    Problem. Let ABC be a triangle, let the Euler line of the triangle ABC meets the sidelines BC, CA, AB at A1,B1,C1 respectively. Let Ab,Ac are reflection of A in B1,C1 respectively. Let Ba,Bc are reflection of B in A1,C1 respectively. Let Ca,Cb are reflection of C in A1,B1 respectively. Show that the Euler lines of four triangles ABC, AAbAc, BBcBa, CCaCb are concurrent.


      En coordenadas baricéntricas, usando la notación de Conway, se tiene:

    Ab = ((a^2-c^2) SB : 0 : -2(b^2-c^2)SA)
    Ac = ((a^2-b^2)SC : -2(c^2-b^2)SA : 0)

      Las coordenadas de los puntos Bc, Ba, Ca, Cb se obtienen permutando cíclicamente las de Ab y Ac.

      La ecuación de la recta de Euler del triángulo AAbAc es:

    2 (b^2-c^2)^2SA^2x + (a^2-b^2)(a^2-c^2)SBSC(y + z) = 0.

    Por permutación cíclica resultan las ecuaciones de las rectas de Euler de los triángulos BBcBa y CCaCb.
    Estas tres rectas concurren el centro X4240:

    ( (2a^4-a^2(b^2+c^2)-(b^2-c^2)^2)/((b^2-c^2)SA) : ... : ...).

    ( Mostrar/Ocultar figura )
      ADGEOM1625.png


      Los seis puntos Ab, Ac, Ba, Bc, Ca, Cb están en una misma cónica, tangente en X4240 a la recta de Euler y que pasa por X648 (tripolo de la recta de Euler). Estos dos puntos son antipodales en esta cónica.

      La ecuación de la cónica es:
    (a^2-b^2)(a^2-c^2)SBSC(8(b^2-c^2)^2SA^2x^2 + (4a^8- 4 a^6 (b^2+c^2) + a^4 (-5b^4+14b^2c^2-5c^4) + 6 a^2(b^2-c^2)^2(b^2+c^2) - (b^2-c^2)^2(b^4+ 6b^2c^2 +c^4)) y z) + cicl. = 0.

  • lunes 1 de septiembre del 2014

    Involución sobre la recta de Euler y centros ortológicos

    ( Anopolis #1822, Antreas P. Hatzipolakis)

      Sean ABC un triángulo, H su ortocentro y P un punto. Se denota por:
    Pa, Pb, Pc las reflexiones de P en los lados BC, CA, AB, respectivamente.
    Pab, Pac las reflexiones de Pa en BH, CH, respectivamente.
    Pbc, Pba las reflexiones de Pb en CH, AH, respectivamente.
    Pca, Pcb las reflexiones de Pc en AH, BH, respectivamente.
    A'B'C' el triángulo delimitado por las rectas PabPac, PbcPba y PcaPcb.

      Los triángulos ABC y A'B'C' son ortológicos si y solo si P está sobre la recta de Euler.
      El centro de ortología de A'B'C' con respecto a ABC es P; el centro de ortología de ABC con respecto a A'B'C' es un punto Q sobre la recta de Euler.
      La correspondencia P ↦ Q es una involución de centro en el inverso del ortocentro respecto a la circunferencia circunscrita de ABC.

    ( Mostrar/Ocultar figura )
      Anopolis1822.png
    En coordenadas baricéntricas si P=(x:y:z), la expresión del lugar geométrico de los puntos P tales que los triángulos ABC y A'B'C' sean ortológicos es:

    (x+y+z)*
    ((b^2-c^2)(b^2+c^2-a^2)x+(c^2-a^2)(c^2+a^2-b^2)y+(a^2-b^2)(a^2+b^2-c^2)z)*
    (a^2yz+b^2zx+c^2xy)*
    (a^4b^2c^2(b^2+c^2-c^2)^2x^2 + a^2(b^2+c^2-a^2)(c^2+a^2-b^2)(a^2+b^2- c^2)(a^4-2a^2(b^2+c^2)+b^4-b^2c^2+c^4 )yz+ ...)=0.

    • El primer factor representa la recta del infinito, para cuyos puntos la construcción no tiene sentido.
    • El segundo factor es la ecuación de la recta de Euler.
    • El tercer factor es la ecuación de la circunferencia circunscrita. En este caso, las tres rectas PabPac, PbcPba y PcaPcb concurren en P. Las perpendiculares a ellas desde los vértices de ABC concurren el punto antipodal de P.
    • Y el último fractor, igualado a cero, es la ecuación de una cónica imaginaria, degenerada en el producto de dos rectas imaginarias conjugadas, con punto de intersección X186 (inverso del ortocentro respecto a la circunferencia circunscrita).


      Si P es un punto sobre la recta de Euler que divide al segmento X3X4 en la razón t, el correspondiente centro de ortología Q, del triángulo ABC respecto a A'B'C', divide a X3X4 en la razón:

    t' = -(a^2b^2c^2)/((a^2+b^2-c^2)(a^4-2a^2b^2+b^4-c^4)t).


    Así, la correspondencia P ↦ Q (t↦t') es una involución sobre la recta de Euler.

    P=(a2SA+2SBSC t:...:...) ↦ Q=(a2b2c2SBSC+4a2SA2SBSC t:...:...).


    Los puntos fijos corresponden a los valos de t:

    ±abc/((-a^2+b^2+c^2)(a^2-b^2+c^2)(a^2+b^2-c^2))½,

    que serán reales si el triángulos ABC es acutángulo (involución hiperbólica) e imaginarios conjugados cuando es obtusángulo.
      El centro de esta involución (punto medio de los fijos) es X186, inverso del ortocentro respecto a la circunferencia circunscrita:

    Anopolis1822prod.png
    Anopolis1822prod2.png



      Pares de puntos conjugados en esta involución, que son centros del triángulo que figuran actualmente en la Enciclopedia ETC, son los que corresponden a los índices:
    {2, 378}, {3, 4}, {5, 3520}, {20, 24}, {25, 376}, {28, 3651}, {30, 186}, {403, 2071}, {550, 3518}, {631, 1593}, {1006, 4219}, {1113, 1114}, {1597, 3524}, {1598, 3528}, {3090, 3516}, {3515, 3529}, {4220, 4227}, {4221, 4231}.

  • sábado 30 de agosto del 2014

    Problema de Apolonio relativo a las circunferencias exinscritas

    ( Forum Geometricorum Volume 10 (2010) 35–40, Boris Odehnal)

    Se cree que fue Isaac Newton el primero
    que resolvió, por medio de la regla y el compás,
    el problema de encontrar la circunferencia tangente
    a otras tres circunferencias.



      Dado un triángulo ABC, las ocho circunferencias tangentes a las circunferencias exinscritas son:
    • La circunferencia de Apolonio (Ap) de ABC
    • La circunferencia de Euler (N)
    • Los tres lados del triángulo ABC, circunferencias con centro en la recta del infinito
    • La circunferencia (Ja) que encierra la A-exinscrita (Ia) y toca a las otras dos por el exterior
    • La circunferencia (Jb) que encierra la B-exinscrita (Ib) y toca a las otras dos por el exterior
    • La circunferencia (Jc) que encierra la C-exinscrita (Ic) y toca a las otras dos por el exterior
    ( Mostrar/Ocultar figura )
      Exinscritas Problema de Apolonio_0.png

      La circunferencia (Ja) es la inversa del lado BC en la inversión respecto a la circunferencia radical de las exinscritas (Re). En esta inversión (Ia) se transforma en sí misma.
      Las circunferencias (Ia) y (Ja) son tangentes en el punto Aa de coordenadas baricéntricas:

    Aa = ((a+b-c)(a-b+c)(b+c)^2 : -c^2(a-b+c)(a+b+c) : -b^2(a+b-c)(a+b+c)).

    Permutando cíclicamente estas coordenadas obtenemos las de los puntos de tangencia Bb de (Ib) y (Jb), y Cc de (Ic) y (Jc).

      El centro de la circunferencia (Ja) es:
    Ja = ( * : 1/(a^3(b^2+2bc+2c^2)+ a^2(b^3+b^2c+2bc^2+2c^3) + ab(-b^3-2b^2c+bc^2+2c^3) - b^2(b-c)(b+c)^2) : 1/(a^3(2b^2+2bc+c^2) + a^2(2b^3+2b^2c+bc^2+c^3) + ac(2b^3+b^2c-2bc^2-c^3) - c^2(c-b)(b+c)^2)).

    Por permutación cíclica, se deducen las coordenadas de los centros Jb y Jc de las circunferencias (Jb) y (Jc).

      De las expresiones de las coordenadas obtenidas se deduce que:

    Los triángulos ABC y AaBbCc son perspectivos con centro de perspectividad en X3596 (1st Odehnal Point):
    ((-a+b+c)/a^2 : (a-b+c)/b^2 : (a+b-c)/c^2).

    Los triángulos ABC y JaJbJc son perspectivos con centro de perspectividad en X3597 (2nd Odehnal Point):
    (1/(a^5+a^4(b+c) - a^3(b-c)^2-a^2(b^3+b^2c+bc^2+c^3) - 2abc(b^2+bc+c^2) - 2b^2c^2(b+c)) : .. : .. ).

    *************************************



     Las tangentes en los puntos de contacto Aa, Bb y Cc de las circunferenicias (Ia), (Ib) y (Ic) con las (Ja), (Jb) y (Jc) delimitan un triángulo T1T2T3 perspectivo con ABC. El centro de perspectividad T tiene coordenadas baricéntricas:

    T = ((b+c)/(a^3(b+c) + a^2bc - a(b-c)^2(b+c) - bc(b+c)^2) : ... : ...)

    con (6,9,13)-número de búsqueda en ETC: 3.2570453258373956346802382
    ( Mostrar/Ocultar figura )
      Exinscritas Problema de Apolonio_3.png
    La ecuación de la tangente común en Aa a las circunferencias (Ia) y (Ja) es:

    bc(a+b+c)x + b(a+b-c)(b+c)y + c(a-b+c)(b+c)z = 0.



    *************************************



      Sean Ab y Ac los puntos de contacto de la circunferencia (Ja) con (Ib) y (Ic), respectivamente. Sus coordenadas son:

    Ab = ((-a+b+c)(a+b-c)(b+c)^2 : c^2(a-b-c)(a+b+c) : (b(b+c)+a(b+2c))^2),
    Ac = ((a-b-c)(a-b+c)(b+c)^2, : -(c(b+c)+a(2b+c))^2 : b^2(-a+b+c)(a+b+c)).

    Procediendo cíclicamente, de deducen las coordenadas de los puntos Bc y Ba de contacto de la circunferencia (Jb) con (Ic) y (Ia), respectivamente; y las de los puntos Ca y Cb de contacto de la circunferencia (Jc) con (Ia) y (Ib), respectivamente.

     Las rectas AbAc, BcBa y CaCb delimitan un triángulo P1P2P3 perspectivo con ABC. El centro de perspectividad es:

    P = ( (b+c)(b+c-a)/(a^2(b^2+3bc+c^2) + a(b^3+b^2c+bc^2+c^3) + bc(b+c)^2) : ... : ... )

    con (6,9,13)-número de búsqueda en ETC: 4.57674437778156865008060726
    ( Mostrar/Ocultar figura )
      Exinscritas Problema de Apolonio_1.png
    La ecuación de la recta AbAc es:
    (b^3+2b^2c+2bc^2+c^3+a(b^2+3bc+c^2))x - c(b+c-a)(b+c)y - b(b+c)(b+c-a)z = 0.

     Las rectas BaCa, CbAb y AcBc delimitan un triángulo Q1Q2Q3 perspectivo con JaJbJc. El centro de perspectividad Q tiene primera coordenada baricéntrica:

    a^10 (b + c)^3 + a^9 (b + c)^2 (5 b^2 - b c + 5 c^2) -
      a^8 (5 b^5 + 28 b^4 c + 42 b^3 c^2 + 42 b^2 c^3 + 28 b c^4 + 5 c^5) -
    a^7 (33 b^6 + 101 b^5 c + 182 b^4 c^2 + 243 b^3 c^3 + 182 b^2 c^4 +
      101 b c^5 + 33 c^6) -
    a^6 (21 b^7 + 86 b^6 c + 257 b^5 c^2 + 411 b^4 c^3 + 411 b^3 c^4 +
      257 b^2 c^5 + 86 b c^6 + 21 c^7) -
    a^5 (-27 b^8 - 27 b^7 c + 83 b^6 c^2 + 290 b^5 c^3 + 454 b^4 c^4 +
     290 b^3 c^5 + 83 b^2 c^6 - 27 b c^7 - 27 c^8) +
    a^4 (29 b^9 + 104 b^8 c + 160 b^7 c^2 - 53 b^6 c^3 - 372 b^5 c^4 -
      372 b^4 c^5 - 53 b^3 c^6 + 160 b^2 c^7 + 104 b c^8 + 29 c^9) +
    a^3 (b^2 - c^2)^2 (b^6 + 73 b^5 c + 182 b^4 c^2 + 239 b^3 c^3 +
      182 b^2 c^4 + 73 b c^5 + c^6) -
    a^2 (b - c)^2 (b + c)^3 (4 b^6 - 11 b^5 c - 49 b^4 c^2 -
      99 b^3 c^3 - 49 b^2 c^4 - 11 b c^5 + 4 c^6) -
    a b c (b - c)^2 (b + c)^4 (8 b^4 - 21 b^3 c - 9 b^2 c^2 -
      21 b c^3 + 8 c^4) -
    b^2 c^2 (b - c)^2 (b + c)^5 (4 b^2 - 11 b c + 4 c^2)

    con (6,9,13)-número de búsqueda en ETC: -1.35127668285956378285916015
    ( Mostrar/Ocultar figura )
      Exinscritas Problema de Apolonio_2.png


      La circunferenica (J) tangente a las (Ja), (Jb) y (Jc), abarcándolas, es tangente a la circunferencia de Apolonio de ABC en el centro del triángulo X3030.

    ( ADGEOM #1541)
    ( Mostrar/Ocultar figura )
      X(3030).png
    Si D es el punto de contacto de la circunferencia inscrita con el lado BC, el punto de tangencia Sa de las circunferencias (J) y (Ja) es el inverso de D en la inversión respeto a la circunferencia (Re) radical de las exinscritas; sus coordenadas son:

    Sa = (-(b+c)(a^2(b+c)-2a(b-c)^2+(b-c)^2(b+c)) : (a(2b-c)+(b-c)c)(a(2b-c)+c(b+c)) : (a(b-2c)+b(b-c))(a(b-2c)-b(b+c))).

    Similarmente se obtienen las coordenadas de los puntos Sb y Sb de contacto de (J) con (Ja) y (Jc), respectivalente.

    La ecuación de la circunferencia (J), inversa de la circunferencia inscrita a ABC mediante la inversión respecto a la circunferencia radical de las circunferencias exinscritas, es;

    k(a,b,c)(a^2yz+b^2zx+c^2xy)-(x+y+z)(f(a,b,c)x+f(b,c,a)y+f(c,a,b)z)=0

    donde
    k(a,b,c) = 4(a^2(b+c)+a(b^2-7bc+c^2)+bc(b+c)) y
    f(a,b,c) = a^4(b+c) - a^3(b^2-5bc+c^2) - a^2(b^3+c^3) + a(b^4-7b^3c+16b^2c^2-7bc^3+c^4) + bc(b+c)^3.

    El centro de esta circunferencia es el punto J de coordenadas:

    J = (a(a^2(b+c)-3bc(b+c)+a(b^2+bc+c^2)) : ... : .. )

    con (6,9,13)-número de búsqueda en ETC: 166.4953881815177657693948105

      El triángulo tangencial de SaSbSc es perspectivo con ABC, con centro de perpectividad el punto de Spieker X10.



  • martes 26 de agosto del 2014

    Puntos de Feuerbach y circunferencia de Apolonio

    ( ADGEOM #1525, Tran Quang Hung)

    a Clara, por su 13 "cumple", con 9 días de retraso



      Sean ABC un triángulo, (Ia), (Ib), (Ic) sus circunferencia exinscritas y Fa, Fb, Fc los puntos de tangencia con la circunferencia de los nueve puntos (puntos de Feuerbach).
      Se denota por (Ka) la circunferencia (distinta de la circunferencia de los nueve puntos) tangente a (Ia) y que pasa por Fb, Fc. Similarmente se definen las circunferencias (Kb) y (Kc).

    La construcción de la circunferencia (Ka) es el problema de Apolonio PPC.

    Los pasos para la construcción de (Ka) son los siguientes:
    • La recta que une los puntos Fb y Fc es el eje radical de todas las circunferencias que pasan por ellos.
    • A continuación dibujamos una circunferencia auxiliar que pase por los puntos Fb y Fc y que corte a la circunferencia dada: tomamos la circunferencia de los nueve puntos. Trazamos la tangente común a ésta y a (Ia). En la intersección de esta tangente con la recta FbFc está el centro radical La.
    • La otra tangente desde La a la circunferencia (Ia) es también tangente a la circunferencia (Ka), que estamos buscando.
    ( Mostrar/Ocultar figura )
      ADGEOM1525a.png


    Si consideramos los puntos Lb y Lc necerarios para construir las circunferencias (Kb) y (Kc), de forma similar a la construcción de (Ka), se verifica:

    Los puntos La, Lb y Lc están en una misma recta y su tripolo L tiene coordenadas baricéntricas:
          ((b+c)/(a^2+bc) : (a+c)/(b^2+ac) : (a+b)/(ab+c^2))

    con (6,9,13)-número de búsqueda en ETC: 3.4313407904038455162362

      Se conoce como circunferencia de Apolonio del triángulo ABC a la círcunferencia que toca sus tres circunferencias exinscritas y las abarca (Kimberling.-TCCT 1998, p. 102).
      La recta que une el centro radical (punto de Spieker, incentro del triángulo medial) de las circunferencias exinscritas con el punto de Feuerbach Fa, vuelve a corta a la circunferencia A-exinscrita en el punto F'a de tangencia con la circunferencia de Apolonio de ABC. Similarmente, se determinan los puntos de contacto F'b y F'c.

      Los ejes radicales de la circunferencia de Apolonio de ABC y cada una de las circunferencias (Ka), (Kb), (Kc) delimitan un triángulo perspectivo con ABC. El centro de perspectividad es X6042:
       (a(b+c)^2(a(b+c)+b^2+c^2)^2 : b(c+a)^2(b(b+a)+c^2+a^2)^2 : c(a+b)^2(c(a+b)+a^2+b^2)^2).

    ( Mostrar/Ocultar figura )
      ADGEOM1525b.png
      Las ecuación baricéntrica del eje radical de la circunferencia de Apolonio de ABC y la circunferencia (Ka) es (las ecuaciones de los otros dos ejes radicales se obtienen por permutación cíclica):

    bc(a+b+c)(4a^2bc+a(b+c)^3+(b^2+c^2)^2)x + ac(b+c)(b^2+c^2+a(b+c))^2y + ab(b+c)(b^2+c^2+a(b+c))^2z = 0.

    X(6042) in ETC

    X(6042) = Perspector of Montesdeoca-Hung Triangle and ABC
    Let (Ap) be the Apollonius circle, and let (KA), (KB), (KC) be the circles described at X(5973) in association with the Hung-Feuerbach circle at X(5974). Let LA be the radical axis of (Ap) and (KA), and define LB and LC cyclically. The lines LA, LB, (LC form a triangle T (here named the Montesdeoca-Hung triangle) that is perspective to ABC, and the perspector is X(6042). (Tran Quang Hung, ADGEOM #1506; Angel Montesdeoca, ADGEOM #1525, August 24, 2014)


    X5975, Touchpoint Hung-Feuerbach circle and Apollonius circle (Peter Moses, ADGEOM #1508)
    ( Mostrar/Ocultar figura )
      ADGEOM1525c.png
      Para construir la circunferencia que toca a las tres circunferencias (Ka), (Kb), (Kc) y las abarca (Hung-Feuerbach circle), consideramos la recta q que contiene a los tres centros externos, Qa, Qa, Qa de homotecia de las circunferencias (Ka), (Kb), (Kc).
    Sean Pa, Pb, Pc los polos de la recta q respecto a las circunferencias (Ka), (Kb), (Kc). Si R es el centro radical de las circunferencias (Ka), (Kb), (Kc), las semirrectas RPa, RPb, RPc las cortan en los puntos de tangencia con la circunferencia de Hung.

  • miércoles 13 de agosto del 2014

    Descripciones geométricas del centro del triángulo X542

    ( Anopolis #20, Antreas P. Hatzipolakis)

    X(542) in ETC

    Let ABC ge a triangle and X(98) the Tarry point (the point of intersection, other than A, B, and C, of the circumcircle and Kiepert hyperbola), then X(542) is the point at infinity defined by the direction of the vector AX(98) + BX(98) + CX(98).

    &ensp  Sean ABC un triángulo, P un punto y PaPbPc el triángulo pedal de P. Consideremos P'a, P'b, P'c los puntos sobre las rectas PPa, PPb, PPc tales que:

    PP'a / PPa = PP'b / PPb = PP'c / PPc = t, (t≠1)

    La paralela por P'a a BC interseca a BA, CA en Ba, Ca, respectivamente.
    La paralela por P'b a BA interseca a CB, AB en Cb, Ab, respectivamente.
    La paralela por P'c a AB interseca a AC, BC en Ac, Bc, respectivamente.

    El lugar geométrico de los puntos P tales que las mediatrices de BaCa, CbAb, AcBc son concurrentes es la recta que pasa por el baricentro y el simediano.


    ( Mostrar/Ocultar figura )
      Anopolis20a.png
    La ecuación baricéntrica de la mediatriz da de BaCa (las de las otras mediatrices db y dc se obtiene por permutación cíclica) es, si P=(u:v:w):

    (b^2-c^2)(tu+v+w)x + ((b^2-c^2)(t-1)u+a^2(u+v+w))y + ((b^2-c^2)(t-1)u-a^2(u+v+w))z = 0.

    La condición para que las tres mediatrices sean concurrentes es:

    (b^2-c^2)u + (c^2-a^2)v + (a^2-b^2)w = 0.

    Es decir, el lugar geométrico es la recta que pasa por el baricento G(1:1:1) y el simediano K(a^2:b^2:c^2).


    Si P es el punto Pt de intersección de las rectas GK y δt, el correspondiente punto Qt de intersección de las mediatrices da, db, dc, describe la parábola que contiene al baricentro, circuncentro y simediano y con punto en el infinito X542.

    ( Mostrar/Ocultar figura )
      Anopolis20c.png

    Cuando t varía y P queda fijo en la recta GK , el lugar geométrico que describe el punto Q de intersección de las mediatrices da, db, dc, es una recta δP que pasa por el circuncentro, X3.

    ( Mostrar/Ocultar figura )
      Anopolis20b.png

    Si P es un punto sobre GK de coordenadas (a^2(1-ξ)+ξ : b^2(1-ξ)+ξ : c^2(1-ξ)+ξ), la ecuación de la recta δP es:

    (b^2-c^2)((a^2-b^2-c^2+b^2c^2)ξ-b^2c^2)x + ... = 0.

    La recta δP contiene al punto P solamente si éste es el baricentro o el simediano.

    Cuando P varía en la recta GK, el lugar geométrico que describe el pie de la perpendicular, trazada por él a la recta δP, es una cúbica circunscrita al triángulo medial, que contiene a los centros X2, X3 (doble), X6, X542, X5108 y con asíntota pasando por X5099 (descrito por Seiichi Kirikami en Anopolis #846) y dirección la del punto del infinito X542.

    ( Mostrar/Ocultar figura )
      Anopolis20d.png


  • miércoles 06 de agosto del 2014

    Propiedad del centro del triángulo X427

    ( ADGEOM #1469, Tran Quang Hung)

    X(427) in ETC

    Let LA be the line tangent to the nine-point circle at the midpoint of segment BC, and define LB and LC cyclically. The triangle formed by the lines LA, LB, LC is homothetic to the orthic triangle, and the center of homothety is X(427). (Randy Hutson, 9/23/2011)

      Sean ABC un triángulo, A1 y A2 los puntos de intersección de la circunferencia circunscrita con la circunferencia A-exinscrita, Fa el punto de tangencia de la circunferencia de los nueve puntos y la circunferencia A-exinscrita.
      Sea Γa la circunferencia, distinta de la A-exinscrita, que pasa por A1 y A2 es tangente a la circunferencia de los nueve puntos y La el punto de contacto.
      Similarmente se definen los puntos Fb, Fc, Lb, Lc.
      Las rectas FaLa, FbLb, FcLc concurren en X427, sobre la recta de Euler.

    ( Mostrar/Ocultar figura )
      ADGEOM1469.png
    El centro radical Ea de las circunferencia que pasan por A1 y A2 es el punto de intersección de la recta A1A2 y la tangente en Fa a la circunferencia de los nueve puntos. La es el punto de contacto de la otra tangente a la circunfeencia de los nueve puntos desde Ea. Así, la recta FaLa es la polar de Ea respecto a la circunferencia de los nueve puntos.

    En coordenadas baricéntricas:

    Ea = ((b-c)(2a^3+a^2(b+c)+(b-c)^2(b+c)) : -(a+c)(a^3+2b^3+a^2c-b^2c-c^3+a(b^2-c^2)) : (a+b)(a^3+a^2b-b^3-bc^2+2c^3+a(-b^2+c^2))).

    La ecuación de la recta FaLa:

    (c-b)(-a^2+b^2+c^2)(a^3+b^3+c^3+a^2(b+c)+a(b^2+bc+c^2))x +
    (a+c)(a^2-b^2+c^2)(a^3+a^2b+b^3+ab(b-c)-b^2c+bc^2-c^3)y -
    (a+b)(a^2+b^2-c^2)(a^3-b^3+a^2c+b^2c-bc^2+c^3+ac(-b+c))z=0.

    Las ecuaciones de las rectas FbLb y FcLc se obtienen por permutacion cíclica. Estas tres rectas tienen punto común X_{427}:
    ADGEOM1469f.png


    Si consideramos las circunferencias Γb y Γc, definidas de forma similar a Γa, el centro radical de estas circunferencias tiene coordenadas baricéntricas:

    ((a^4(b+c) + 2a^3(b^2+c^2) + 2a^2(b^3+b^2c+bc^2+c^3) + 2a(b+c)^2(b^2+c^2) + (b+c)(b^2+c^2)^2)/(b^2+c^2-a^2) : ... : ...)

    con (6,9,13)-número de búsqueda en ETC: -23.546702367310941758314

  • martes 05 de agosto del 2014

    Homotecia con centro en el circuncentro

    ( BMO2 Jan 2014)

      Sea, ABC un triángulo, P un punto y DEF su triángulo circunceviano.
    Oa, Ob, Oc son los circuncentros de los triángulos BCP, CAP, ABP, respectivamente.
    Od, Oe, Of son los circuncentros de los triángulos EFP, FDP, DEP, respectivamente.
     Las rectas OaOd, ObOe, OcOf concurren en el punto P', imagen de P en la homotecia de centro el circuncentro y razón 1/2.



    ( Mostrar/Ocultar figura )
      BMO2Jan2014.png
    Si (u:v:w) son las coordenadas baricéntricas del punto P, los vértices de su triángulo circunceviano son

    D=(-a^2vw : v(c^2v+b^2w) : w(c^2v+b^2w)),
    E=(u(c^2u+a^2w) : -b^2uw : w(c^2u+a^2w)),
    F= (u(b^2u+a^2 v) : v(b^2u+a^2v) : -c^2uv).

    El circuncentro de BCP es:

    Oa =(-a^2(-u(b^2(u+v-w)+c^2(u-v+w))+ a^2(u^2+2vw+u(v+w))) : -(b^2-c^2)u(-c^2v+ b^2(u+v))+a^4vw+ a^2(c^2v(u-w)+b^2(u^2+uv+2uw+vw)) : a^4vw-(b^2-c^2)u(b^2w-c^2(u+w))+ a^2(b^2(u-v)w+c^2(u^2+2uv+uw+vw))).

    Y el circuncentro de DFP es:
    Od=(-(b^2-c^2)^2u^2-a^4(u^2-2vw)+ 2a^2u(c^2(u+v)+b^2(u+w)) : -a^4v(u+w)- b^2(b^2-c^2)u(v+w)+ a^2(c^2v(u+w)+b^2(2uv-uw-vw)):-a^4(u+v)w+ c^2(b^2-c^2)u(v+w)+ a^2(b^2(u+v)w-c^2(uv-2uw+vw))).

    Las coordenadas de los circunscentros Ob, Oc, Oe, Of se obtienen por permutación cíclica.

    El punto de intersección de las rectas OaOd, ObOe, OcOf es:

    P' = ( (3S^2-SBSC)u + a^2SAv + a^2SAw : ... : ... ).


    OP' : OP = 1 : 2.

      Los triángulos OaObOc y OdOeOf son simétricos, respecto a P'. Así, ellos están inscritos en una misma cónica. En el caso particular de que P sea el incentro, los vértices de su triángulo circunceviano coinciden los vértices de OaObOc; y tal cónica corta por cuarta vez a la circunferencia circunscrita en el cuarto punto de intersección con la hipérbola de Feuerbach (conjugada isogonal de la recta OI).
    ( Mostrar/Ocultar figura )
      BMO2Jan2014X1.png


  • lunes 04 de agosto del 2014

    Cuatro triángulos perspectivos

    ( Triángulos 208)

    Sean ABC un triángulo, Γa la circunferencia que pasa por B y C y es tangente internamente a la circunferencia inscrita y similarmente, las circunferencias Γb y Γc. Designamos por Pa el punto de contacto de Γa y la circunferencia inscrita; similarmente, sean Pb y Pc. Sea Qa el punto de concurrencia de las tangentes a la circunferencia inscrita en Pb y Pc; y similarmente, Qb y Qc. Finalmente, sea Ta el punto de intersección de las rectas BPc y CPb; similarmente se definen Tb y Tc.

    ( Mostrar/Ocultar figura )
      gtre2486.png
    •  El centro de perspectividad de los triángulos ABC y PaPbPc es el centro X479.
    •  El centro de perspectividad de los triángulos ABC y QaQbQc es el centro X57.
    •  El centro de perspectividad de los triángulos ABC y TaTbTc es el centro X55.
    •  El centro de perspectividad de los triángulos PaPbPc y QaQbQc es el centro X3598 (Primer punto de Liu).
    •  El centro de perspectividad de los triángulos PaPbPc y TaTbTc es el centro X3599 (Segundo punto de Liu).
    •  El centro de perspectividad de los triángulos QaQbQc y TaTbTc es el centro:

    (a(a^4-4a^3(b+c)+2a^2(3b^2-2bc+3c^2)-4a(b-c)^2(b+c)+(b-c)^2(b^2+6bc+c^2))) /(b+c-a)^2: ... : ... ).


    con (6,9,13)-número de búsqueda en ETC: -0.064037209873896799747666

  • lunes 07 de julio del 2014

    Propiedades del punto de De Longchamps

    Definition of De Longchamps point

    Let the given triangle have vértices A, B, and C, opposite the respective sides a, b, and c, as is the standard notation in triangle geometry. In the 1886 paper in which he introduced this point, de Longchamps initially defined it as the center of a circle Γ orthogonal to the three circles Γa, Γb, and Γc, where Γa is centered at A with radius a and the other two circles are defined symmetrically. De Longchamps then also showed that the same point, now known as the de Longchamps point, may be equivalently defined as the orthocenter of the anticomplementary triangle triangle of ABC, and that it is the reflection of the orthocenter of ABC around the circumcenter

    Reference:
    de Longchamps, G. (1886), "Sur un nouveau cercle remarquable du plan du triangle", Journal de Mathématiques spéciales, 2. Sér. (in French) 5: 57-60. See especially section 4, "détermination du centre de Γ", pp. 58-59.

      Sea ABC un triángulo y Φa la cónica circunscrita con tangentes en B y C perpendiculares a BC. Similarmente se definen las cónicas circunscritas Φb y Φc.
      Denotamos por A', B', C' los cuartos puntos de intersección de Φb y Φc, Φc y Φa, Φa y Φb, respectivamete.
      La tangente en B a Φb vuelve a cortar a Φa en Ba. La tangente en C a Φc vuelve a cortar a Φa en Ca. Similarmente se definen los puntos Cb, Ab y Ac, Bc.

    En coordenadas baricéntricas:
      Φa : a²yz+SCzx+SBxy=0,     Φb : SCyz+b²zx+SAxy=0,     Φc : SByz+SAzx+c²xy=0.

    Cuarto punto de intersección de la cónicas circunscritas Φb y Φc, tripolo de la recta determinada por sus perspectores E=(SC:b²:SA) y F=(SB:SA:c²):

    LongchampsA.png


      Las rectas AA', BB', CC' concurren en el punto de De Longchamps, X20.

    ( Mostrar/Ocultar figura )
      Longchamps.png
    Descargar fichero GeoGebra


      La tangente, SBy+SCz=0, en A a Φa vuelve a cortar a Φb en

    Ab=(SBSC^2 : -SC(SA(SB-SC)+SBSC) : SB(SA(SB-SC)+SBSC)),

    y a Φc en

    Ac=(SB^2SC : -SC(SA(SB-SC)-SBSC) : SB(SA(SB-SC)-SBSC).


      Los puntos Bc, Ba, Ca y Cb, se definen cíclicamente.

      Los puntos Ab, Ac, Bc, Ba, Ca y Cb están en una misma cónica, de centro:

    LongchampsCentro.png

    con (6,9,13)-número de búsqueda en ETC: 0.4464942066184852070174993, y está alineado (Peter Moses) con los centros del triángulo:

    X112, X376, X577, X1249, X1294, X3163, X3184.



      El triángulo delimitado por las rectas BaCa, CbAb, AcBc es perspectivo con ABC con centro de perspectividad el punto de De Longschamps.
      El triángulo delimitado por las rectas BcCb, CaAc, AbBa es perspectivo con ABC con centro de perspectividad el punto

    HG070714p.png
    con (6,9,13)-número de búsqueda en ETC: 7.114153709376457534791226568

  • viernes 27 de junio del 2014

    Centros de semejanza y puntos de Beltrami

      Sea ABC un triángulo y P un punto sobre su circunferencia circunscrita Γ. Se denotan por A', B', C' las reflexiones de P en las mediatrices de BC, CA, AB, respectivamente; A'B'C' está inscrito en Γ.

    •   Se designa por A+ el centro de la semejanza directa que transforma el segmento BC en el C'A', por B+ el centro de la semejanza directa que transforma el segmento CA en el A'B' y por C+ el centro de la semejanza directa que transforma el segmento AB en el B'C'.

      El lugar geométrico de los centros de semejanza A+, B+, C+ son tres circunferencias Γ‍+a, Γ‍+b, Γ‍+c, con un punto común B1 (primer punto de Beltrami, inverso del primer punto de Brocard Ω1, respecto a la circunferencia circunscrita, Γ).
      La circunferencia Γ+t circunscrita a A+B+C+ pasa por B1 y el circuncentro, O.

    ( Mostrar/Ocultar figura )
      beltrami1.png


    Podemos expresar un punto en la circunferencia circunscrita en la forma (Barry Wolk, Anopolis #317):
    circumcircle.png

    Entonces, el centro de la semejanza que transforma el segmento BC en C'A' y la ecuación de la circunferencia que describe, cuando P recorre la circunferencia circunscrita Γ, son:
    Bertrami centros semejanzas+.png

    Γ+a es la inversa, respecto a Γ, de la circunferencia que pasa por B y es tangente a AC en C. Es decir, Γ‍+a es la circunferencia que pasa por B y es tangente en C a la circunferencia circunscrita al triángulo ACO.

    Procediendo cíclicamente, sobre los lados de ABC, se obtienen otras dos circunferencias:
    Γ+b: c^2 x y + b^2 x z + a^2 y z - a^2 c^2 y (x + y + z)/(c^2-b^2) =0,
    Γ+c: c^2 x y + b^2 x z + a^2 y z - a^2 b^2 z (x + y + z)/(a^2 - c^2)=0.
    El punto común a estas tres circunferencias es el primer punto de Beltrami:

    B1 = ( a^2(a^2-b^2) : b^2(b^2-c^2) : c^2(c^2-a^2)).




    Cambiamos la orientación de los segmentos que determinan las semejanzas:

    •   Se designa por A- el centro de la semejanza directa que transforma el segmento CB en B'A', por B- el centro de la semejanza directa que transforma el segmento BA en A'C' y por C- el centro de la semejanza directa que transforma el segmento A'B' en el BC.

      El lugar geométrico de los centros de semejanza A‍-, B‍-, C‍- son tres circunferencias Γ‍-a, Γ‍-b, Γ‍-c, con un punto común B2 (segundo punto de Beltrami, inverso del segundo punto de Brocard Ω2, respecto a la circunferencia circunscrita, Γ).
      La circunferencia Γ‍-t circunscrita a A‍-B‍-C‍- pasa por B2 y el circuncentro O.

    ( Mostrar/Ocultar figura )
      beltrami2.png
    El centro de la semejanza que transforma el segmento CB en B'A' y la ecuación de la circunferencia que describe, cuando P recorre la circunferencia circunscrita Γ, son:
    Bertrami centros semejanzas+.png

    Γ-a es la inversa, respecto a Γ, de la circunferencia que pasa por C y es tangente a AB en B. Es decir, Γ‍-a es la circunferencia que pasa por C y es tangente en B a la circunferencia circunscrita al triángulo ABO.

    Procediendo cíclicamente, sobre los lados de ABC, se obtienen otras dos circunferencias:
    Γ-b: c^2 x y + b^2 x z + a^2 y z - a^2 c^2 y (x + y + z)/(a^2-b^2) =0,
    Γ-c: c^2 x y + b^2 x z + a^2 y z - a^2 b^2 z (x + y + z)/(b^2 - c^2)=0.
    El punto común a estas tres circunferencias es el segundo punto de Beltrami:

    B1 = ( a^2(a^2-c^2) : b^2(b^2-a^2) : c^2(c^2-b^2)).




    OTRAS CONSIDERACIONES:

    ♦   Los centros O‍+t y O‍-t de las circunferencias Γ‍+t y Γ‍-t están en las mediatrices de los segmentos OB1 y OB2, cuyo punto de intersección Q tiene primera coordenada baricéntrica:

    (a^2(-a^8 + a^6(b^2+c^2) + a^4(b^4+5b^2c^2+c^4) - a^2(2b^6+3b^4c^2+3b^2c^4+2c^6) + b^8 + c^8),

    con (6,9,13)-número de búsqueda en ETC: 1.99844087285917963875

      El segundo punto de intersección O' de las circunferencias Γ‍+t y Γ‍-t está en la recta de Lemoine, polar trilineal del simediano.
    ( Mostrar/Ocultar figura )
      beltrami semejanzas.png



    ♦   La envolvente de la recta determinada por los centros O‍+t y O‍-t de las circunferencias Γ‍+t y Γ‍-t es la parábola de foco el circuncentro y directriz la recta de Lemoine.
    El punto medio Mt de O‍+t y O‍-t queda en la tangente en el vértice V de esta parábola. Para t=0 (P(0) el punto de Steiner), M0 es el vértice (punto medio de X3 y X187, éste es el punto medio de los puntos de Beltrami):

    V= (a^2(4a^6 - 9a^4(b^2+c^2) + 2a^2(4b^4+b^2c^2+4c^4) - 3b^6+b^4c^2+ b^2c^4-3c^6 ) : ... : ...),

    con (6,9,13)-número de búsqueda en ETC: 2.47073957830213664904597
    ( Mostrar/Ocultar figura )
      BeltramiParabola.png


  • martes 24 de junio del 2014

    Construcción de triángulo

    ( Anopolis #1704, Antreas P. Hatzipolakis)

      Construir un triángulo ABC, dado A'B'C' el triángulo ceviano de algún punto, tal que A' es el pie de bisectriz en A y B' es el pie de la mediana por B.


    (Publicado en la matemática griega. revista Euclides, 12, 1973) [the translation of "Published in the Greek mathematical magazine Euclid, Dec. 1973"]

    Construcción analítica:

      Tomemos un punto arbitrario P en el plano del triángulo dado A'B'C', que será el de referencia para un sistema de coordenadas baricéntricas.
      Si P fuera el vértice del triángulo a construir (opuesto al lado que pasan por A'), otro vértice será la reflexión R de P en B'. El tercer vértice Q ha de ser RA'∩PC'.
    ( Mostrar/Ocultar figura )
      Anopolis1704.png

      Si (u:v:w) son las coordenadas baricéntricas de P respecto a A'B'C', R=(-u : 2u+v+2w : -w) y Q=(-u(2u+v+2w) : -v(2u+v+2w) : vw). Como A'B'C' debe ser el triángulo ceviano de un cierto punto en PQR, se tiene que verificar que las rectas PA', QB' RC' sean concurrentes; es decir,

    4 u w (u + w) (u + v + w)=0

      Luego, una condición necesaria para construir el triángulo es que el punto P esté sobre la recta δ: x+z=0, paralela a A'C' por B'.

      Debemos localizar la posición del punto P, en la recta δ, tal que la recta PR' (R' la reflexión de R en PA') pase por C'. Al variar P sobre δ, la recta PR' envuelve la circunferencia Γ de centro en A' y tangente a δ. (Nikos Dergiades, ADEGEOM #493):

    PR': w (c'^2 v (v + w) + w (-a'^2 v + b'^2 (v + w)))x -w (-w (a'^2 (u - v + w) + b'^2 (-u + v + w)) + c'^2 (w (v + w) + u (2 v + w)))y + c'^2 v (u v + w (v + w)) - w (a'^2 v (-v + w) + b'^2 (v^2 + u w + v w))z=0.

    Γ: a'^4 x^2 - 2 a'^2 b'^2 x^2 + b'^4 x^2 - 2 a'^2 c'^2 x^2 - 2 b^2 c'^2 x^2 + c'^4 x^2 + 2 a'^4 x y - 4 a'^2 b'^2 x y + 2 b'^4 x y - 4 a'^2 c'^2 x y - 4 b'^2 c'^2 x y + 2 c'^4 x y + a'^4 y^2 - 2 a'^2 b'^2 y^2 + b'^4 y^2 - 2 a'^2 c'^2 y^2 + 2 b'^2 c'^2 y^2 + c'^4 y^2 + 2 a'^4 x z - 4 a'^2 b'^2 x z + 2 b'^4 x z - 4 a'^2 c'^2 x z - 4 b'^2 c'^2 x z + 2 c'^4 x z + 2 a'^4 y z - 8 a'^2 b'^2 y z + 6 b'^4 y z - 4 a'^2 c'^2 y z + 2 c'^4 y z + a'^4 z^2 - 2 a'^2 b'^2 z^2 + 5 b'^4 z^2 - 2 a'^2 c'^2 z^2 - 2 b'^2 c'^2 z^2 + c'^4 z^2 =0.

      Por tanto, un lado del triángulo a construir es una tangente a Γ desde C'.

    Construcción:

    ( Mostrar/Ocultar figura )
      Anopolis1704c.png
      Trazamos la paralela δ a A'C' por B' y la circunferencia Γ con centro en A' y tangente a δ. Si C' es exterior a Γ, una tangente desde C' a Γ corta a δ en A, el vértice C es la reflexión de A en B' y el vértice C=AC'∩CA'.
      Una segunda tangente desde C' permite construir un segundo triángulo A1B1C1.
      En uno de estos triángulos A' será el pie de la bisectriz externa desde el vértice opuesto.
      Si C' es interior a la circunferencia Γ no hay solución.

  • sábado 14 de junio del 2014

    Propiedad de una hipérbola equilátera

    ( ADGEOM #1327, Dao Thanh Oai)

      Sean ABC un triángulo, P un punto en su plano y ℵP la hipérbola equilátera circunscrita a ABC y que pasa por P (pasa también por el ortocentro H). Una recta δ que pasa por P, vuelve a cortar a la hipérbola en un punto Q. Denotamos por A1, B1, C1 los puntos en los que las rectas por P y perpendiculares a AQ, BQ, CQ cortan a los lados BC, CA, AB, respectivamente.

    Los puntos A1, B1, C1 están alineados sobre una recta δ1 perpendicular a PQ.

    ( Mostrar/Ocultar figura )
      ADGEOM1327.png
      Si (u:v:w) son las coordenadas baricéntricas de P, respecto al triángulo ABC, la ecuación de la hipérbola equilátera circunscrita ℵP es (notación de Conway):

    u(SBv-SCw)yz + v(SCw-SAu)xz + w(SAu-SBv)xy = 0.


      A la recta δ, ((1-t)v - t w)x + (t-1)uy + tuz=0, le corresponde el segundo punto Q de intersección con la hipérbola ℵP:

    ((1-t)tu(SBv-SCw) : -t(SAu-SCw)((t-1)v+ tw) : (t-1)(SAu-SBv)((t-1)v+tw)) ).

      La envolvente de las rectas δ1, cuando δ gira alrededor de P, es una cónica (es fácil ver que la correspondencia B1↦ C1 entre las rectas AC y AB es una proyectividad) tangente a los lados de ABC y a la recta del infinito (párabola inscrita ℘p). Por tanto, su directriz pasa por H (es la recta PH) y su foco Fp está en la circunferencia circunscrita (es el conjugado isogonal del punto del infinito determinado por la dirección perpendicular a la recta PH).
      A toda hipérbola equilátera circunscrita a ABC, con uno de sus puntos sobre una recta que pasa por el ortocentro, le corresponde la misma parábola envolvente de las rectas δ1.

      Otra forma de construir el foco Fp es hallando la interseción de las rectas H1(PH∩BC), H2(PH∩CA), H3(PH∩AB), siendo H1H2H3 el triángulo circunceviano de H.

      Directriz PH:   SA(SBv-SCw)x + SB(SCw-SAu)y + SC(SAu-SBv)z=0.
      Foco Fp:   (a^2/(a^2(w-v)+(b^2-c^2)(v+w)) : ... : ... ).
      Ecuación de la parábola ℘p:

    El cuarto punto Qp de intersección de la hipérbola ℵp con la circunferencia circunscrita es el segundo punto de intersección de la hipérbola con la recta PFp.


    Las cordenadas de Qp son: (1/(b^2(SAu-SBv)w + c^2(SAuv-SCvw)) : ... : ... ).

    Este resultado nos da un método para construir el foco de una parábola, inscrita en un triángulo, de la que conocemos su directriz (que ha de pasar por el ortocentro):
    •  Construimos la hipérbola circunscrita al triángulo, que pasa por el ortocentro y por un punto P cualquiera de la directriz de la parábola.
    •  Construimos el cuarto punto Qp de intersección de tal hipérbola y la circunferencia circunscrita.
    •  El foco de la parábola es el segundo punto de intersección de la recta PQp con la circunferencia circunscrita.
    ( Mostrar/Ocultar figura )
       FocoParabolaInscrita.png


  • miércoles 11 de junio del 2014

    Circunferencias tangentes y cúbica de Lucas

    ( ADGEOM #1323, Emmanuel José García)

      Sean ABC un triángulo y P un punto en su plano. Las reflexiones de P en los vértices del triángulo ceviano XaXbXc de un punto X, forman un triángulo QaQbQc. Los tres puntos que resultan de reflejar este triángulo en los lados de ABC determinan una circunferencia Γ(X,P).

    ( Mostrar/Ocultar figura )
      ADGEOM1322.png


      Usando coordenadas baricéntricas, respecto al triángulo ABC, si ponemos X=(u:v:w) y P=(p:q:r), la reflexión del punto Qa en el lado BC es:

    ADGEOM1322pto.png

    Y la condición analítica para que el punto P esté en la circunferencia Γ(X,P) es:

    ADGEOM1322cond.png

      • Cuando P queda en el infinito (primera condición) la construcción de Γ(X,P) no es posible.
      • El último factor nos indica que si X=(u:v:w) está en la cúbica de Lucas (K007), P queda sobre Γ(X,P), para cualquier posición de P en el plano.
      • Los restantes tres factores que figuran en esta condición, mirados como ecuaciones en las variables p,q,r, reprentan las rectas perpendiculares a los lados de ABC en los pies de las cevianas del punto X. Por lo tanto, P está sobre Γ(X,P) si está sobre alguna de estas perpendiculares.

    P está sobre la circunferencia Γ(X,P) si y solo si X está sobre la cúbica de Lucas o sobre las rectas perpendiculares a los lados de ABC en los pies de las cevianas del punto X.


      Tomemos un punto L1 sobre la curva de Lucas y un punto P arbitrario del plano, entonces P está en la circunferencia Γ(L1,P) cuyo centro es el punto D1 (en la cúbica de Darboux, K004), tal que el triángulo pedal de D1 es el triángulo ceviano de L1.
      La recta D1P corta a la cúbica de Darboux en otros dos puntos D2 y D3 (reales o imaginarios). Si L2 y L3 son los puntos (en la cúbica de Lucas) tales que sus triángulos cevianos son los triángulos pedales de D2 y D3, respectivamente, se verifica que las tres circunferencias Γ(L1,P), Γ(L2,P) y Γ(L3,P) son tangentes en P.

    ( Mostrar/Ocultar figura )
      ADGEOM1322b.png


      Sean dos puntos X y X' conjugados isotómicos sobre la cúbica de Lucas y sean Y e Y' los puntos (sobre la cúbica de Darboux) tales que sus triángulos pedales son los triángulos cevianos de X y X', respectivamente. Entonces, todo punto P pertenece a las circunferencias Γ(X,P) y Γ(X',P), además, sus centros son Y e Y', respectivamente.
    Se tiene que la recta YY' pasa por el circuncentro de ABC y la recta XY pasa por el punto de De Longchamps L (simétrico del ortocentro H respecto el circuncentro O), para todo punto X sobre la cúbica de Lucas..

    Las circunferencias Γ(X,P) y Γ(X',P) son tangentes en P si y solo si P está en la recta YY'.


    En estas circunstancias, si G es el baricentro, también la circunferencia Γ(G,P), con centro en el circuncentro, es tangente en P a las circunferencias Γ(X,P) y Γ(X',P).

    En particular (Peter Moses):
    Si X=X7 = punto de Gergonne y X'=X8 = punto de Nagel, las circunferencias Γ(X7,P) y Γ(X8,P) son tangentes en P si y solo si P queda en la recta que pasa por O=X3, I=X1 and X40.

    ( Mostrar/Ocultar figura )
      ADGEOM1322Moses.png


  • martes 10 de junio del 2014

    Una caracterización de la cuártica de Euler-Morley

    ( Anopolis #1645, Antreas P. Hatzipolakis)

      Sean ABC un triángulo y P, Q dos puntos conjugados isogonales.
    Ap es el punto de intersección, distinto de P, de la recta AP con la circunferencia circunscrita a BPC; Aq es el punto de intersección, distinto de Q, de la recta AQ con la circunferencia circunscrita a BQC.
    Similarmente se definen los puntos Bp, Bq, Cp y Cq.
      Denotamos por R el centro radical de las circunferenecias cicunscritas a los triángulos AApAq, BBpBq y CCpCq.
      Si las coordenadas baricéntricas de P son (u:v:w),
    Rp =(a^2u(c^2v^2+b^2w^2) : b^2v(c^2 u^2+a^2w^2) : c^2(b^2u^2+a^2v^2)w).

    El lugar geométrico de los puntos P tales que el circuncentro, P y R están alineados es la cuártica de Euler-Morley.

    ( Mostrar/Ocultar figura )
      Anopolis1645.png


  • lunes 9 de junio del 2014

    Caracterización de la cúbica de Darboux en términos de la circunferencia pedal

    ( Anopolis #1648, Antreas P. Hatzipolakis)

      Sean ABC un triángulo, P, P* dos puntos conjugados isogonales y A'B'C', A"B"C" los triángulos pedales de P, P*, respectivamente.
    Se denota por A1 el punto de intersección (distinto de A) de la recta AA' con la circunferencia pedal de P, y por A* el punto de intersección de la recta B"C" con la tangente a la circunferencia pedal de P en A1.
    De forma similar se define los puntos B* y C*.

    El lugar geométrico de los puntos P tales que su circunferencia pedal está definida y los puntos A*, B* y C* están alineados es la cúbica de Darboux.

    ( Mostrar/Ocultar figura )
      Anopolis1648.png
    Si P=P*=I, incentro, la recta que contiene a los puntos A*, B*, C* es la tripolar de X7, punto de Gergonne.

    (Art of Problem Solving, http://www.artofproblemsolving.com/Forum/viewtopic.php?f=47&t=592844 ) Si P es el circuncentro (P* el ortocentro) , A*, B* y C* están en la recta de ecuación baricéntrica:
    (-a^2 + b^2 + c^2) (a^4 (b^2 + c^2) - b^2 c^2 (b^2 + c^2) + a^2 (b^4 - b^2 c^2 + c^4))x+... =0.

  • viernes 6 de junio del 2014

    Hipérbolas de asíntotas paralelas a los lados de un triángulo

    ( Anopolis #1605, Antreas P. Hatzipolakis)

      Sean p, q, r tres recta fijas y D un punto fijo sobre p. Una recta variable δ a través de D interseca a q en Q y δ', la reflexión de δ en p, corta a r en R. Sea M el punto medio de QR.
      El lugar geométrico de M, cuando δ gira alrededor de D, es una hipérbola de asíntotas paralelas a las rectas q y r, y que pasa por los puntos medios de B=p∩r y C=p∩q (cuando δ=p), de A=q∩r y q∩δ' (cuando δ pasa por A=q∩r) y de A=q∩r y r∩δ' (cuando δ pasa por A=q∩r).

    ( Mostrar/Ocultar figura )
      Anopolis1605.png
      Si hacemos variar el punto D sobre la recta p, se genera un haz de hipérbolas, cuyos cuatro puntos base son los puntos del infinito de las rectas q y r, el punto medio Ma de B=p∩r y C=p∩q, y H'a (conjugado armónico de la proyección ortogonal de A=q∩r sobre p).
      Las hipérbolas degeneradas son los pares de rectas que se obtienen al unir dos a dos los puntos base del haz.
    ( Mostrar/Ocultar figura )
      Anopolis1605haz.png


    Aplicación a la geometría del triángulo:

      Sean ABC un triángulo, P un punto y DEF el triángulo PEDAL de P. Consideremos las hipérbolas ℵa, ℵb y ℵc, según la construcción anterior, procediando cíclicamente sobre los lados del triángulo ABC y tomando, sucesivamente, los puntos D, E y F en sus lados opuestos.
    Si P tiene coordenadas baricéntricas (u:v:w), la ecuación de ℵa es:
    hipApedal.png

    Los centros de las hipérbolas ℵa, ℵb y ℵc están alineados si y sólo si P está sobre la hipérbola de Kiepert.

    ( Mostrar/Ocultar figura )
      Anopolis1605tr.png


      Sean ahora un triángulo ABC, un punto P y PaPbPc el triángulo CEVIANO de P. Consideremos las hipérbolas ℵa, ℵb y ℵc, según la construcción anterior, procediando cíclicamente sobre los lados del triángulo ABC y tomando, sucesivamente, los puntos Pa, Pb y Pc en sus lados opuestos.
    Si P tiene coordenadas baricéntricas (u:v:w), la ecuación de ℵa es:
    hipAceva.png

    Los centros de las hipérbolas ℵa, ℵb y ℵc están alineados si y sólo si P está sobre la ortopivotal cúbica de ortopivote X5, K060.

    ecuK060.png
    ( Mostrar/Ocultar figura )
      Anopolis1605ceva.png
      Para los centros en K060, P=X4, X30, X80, X265, la recta que contiene a los centros de las hipérbolas ℵa, ℵb y ℵc son las tripolares de X253, X69, X7, X264, respectivamente.

  • martes, 3 de junio del 2014

    Caracterizaciones de las cúbicas de Darboux, Lucas y Simson

      Dados un triángulo ABC y un punto P, sean MaMbMc el triángulo medial y DEF el triángulo pedal de P.
      Consideremos el lugar geométrico del punto medio de los puntos en que una recta variable, que pasa por D, corta a las rectas AB y AC.

    ( Mostrar/Ocultar figura )
      HG030614lg.png
    Se trata de una hipérbolaa, que pasa por A, D, Ma con asíntotas paralelas a los lados AB y AC. El centro de ℵa es el punto medio de AD. Denotamos por ta la tangente en A a la hipérbola ℵa

      Si P tiene coordenadas baricéntricas (u:v:w), la ecuación de ℵa y de ta son:
    HG030614hA.png

      Similarmente, se definen las tangentes tb y tb a las correspondientes hipérbolas ℵb y ℵc.
    ( Mostrar/Ocultar figura )
      HG030614conicas.png

    El triángulo A'B'C' delimitado por las tangentes ta, tb y tc es perspectivo con ABC si y solo si P está sobre la circunferencia circunscrita o sobre la cúbica de Darboux.
    El centro de perspectividad Q de ABC y A'B'C', cuando P recorre la cúbica de Darboux está sobre la cúbica de Lucas.

    Ocurre que, en este caso, los vértices D, E, F del triángulo pedal de P está en las rectas AA', BB', CC'.
    ( Mostrar/Ocultar figura )
      HG030614d.png
      Parejas de centros {P,Q}, P en la cúbica de Darboux y Q en la cúbica de Lucas:
    {X1,X7}, {X3,X2}, {X4,X4}, {X20,X69}, {X40,X8}, {X64,X253}, {X84,X189}, {X1490,X329}, {X1498,X20}, {X3345,X1034}, {X3346,X1032}.

    Cuando P queda sobre la circunferencia circunscrita a ABC, los vértices de su triángulo pedal quedan sobre la recta de Simson s(P) de P, y las tres tangentes ta, tb y tc concurren en el tripolo de s(P), sobre la cúbica de Simson.

    ( Mostrar/Ocultar figura )
      HG030614c.png


  • domingo 1 de junio del 2014

    Triángulos con su ortocentro en una recta paralela a la de Euler

    ( Anopolis #1584, Antreas P. Hatzipolakis)

    Dado un triángulo ABC y una recta d. Se consideran los triángulos ABaCa, con Ba en AB, Ca en AC y el ortocentro Ha sobre la recta d.
    1.1. El circuncentro Oa de ABaCa recorre la mediatriz ma de ASa. donde Sa es el punto "Anti-Steiner" (denominado así por Darij Grinberg en "Anti-Steiner point with respect to a triangle") de la recta d respecto al triángulo ABaCa.
    1.2. El centro de la circunferencia de los nueve puntos Na de ABaCa recorre una recta na.

    El punto Sa (en la circunferencia es circunscrita a ABaCa) es la intersección de las refleciones de d en AB y AC.
    Dos puntos particulares de la recta na son los puntos medios de EFa y FEa, siendo E y F las intersecciones de d con los lados AB y AC, respectivamente; y Ea y Fa las intersecciones de ma con los lados AB y AC, respectivamente.

    ( Mostrar/Ocultar figura )
      HG010614.png

    Denotamos por Ad el punto de intersección de ma y na. Similarmente, procediendo cíclicamente sobre los vértices de ABC, se definen los puntos Bd y Cd.

     Cuando la recta d es paralela a la recta de Euler de ABC, las rectas AAd, BBd, CCd concurren en el punto de Kosnita, X54.
      Existe una única recta d0, paralela a la recta de Euler, para la cual los puntos A0, B0 y C0 están alineados.

    ( Mostrar/Ocultar figura )
      HG010614d0.png

    La ecuación baricéntrica de la recta de d0 es:
    HG010614Ecd0.png
    Esta recta no contiene centros del triángulo, que figuren actualmente en ETC, salvo su punto en el infinito, X30.

  • sábado, 31 de mayo del 2014

    Recta de Euler e hipérbola asociada

    Dado un triángulo ABC, su recta de Euler corta a los lados BC, CA, AB en los puntos D, E, F, respectivamente. La circunferencia de centro en F y que pasa por A vuelve a cortar a AC en Ab. La circunferencia de centro en E y que pasa por A vuelve a cortar a AB en Ac. La recta AbAc (que pasa por el ortocentro) corta a BC en Aa.
    Similarmente, se definen los puntos Ba, Bb, Bc, Ca, Cb, Cc.

    Los puntos Aa, Bb y Cc están en la recta que corta a la circunferencia de Euler en los centros de las hipérbolas rectangulares de Jerabek y circunscrita que pasa por el centro X93.

    ( Mostrar/Ocultar figura )
      recta Euler Circunferencias.png
    Esta recta corta a la recta de Euler en el punto de coordenadas baricéntricas:
    HG310514P.png

    con número de búsqueda en ETC: 3.6723293499380232572548885

    Los seis puntos Ab, Ac, Ba, Bc, Ca y Cb están en una hipérbola que pasa por X107 (en la circunferencia circunscrita), X648 (tripolo de la recta de Euler).



    Las reflexiones de la recta AbAc en los lados de ABC se cortan en un punto Sa de la circunferencia circunscrita (Teorema de Collings), denomonado por Darij Grinberg "Anti-Steiner point" de la recta AbAc.
    Análogamente, se construyen los puntos Sb y Sc.

    Las rectas ASa, BSb, CSc son parelelas, con punto del infinito X523, conjugado isogonal del foco de la parábola de Kiepert.



  • martes, 27 de mayo del 2014

    Triángulo ceviano y ejes radicales

    ( Anopolis #1574, Antreas P. Hatzipolakis)

    Sean ABC un triángulo, P un punto y A'B'C' su triángulo ceviano.
    Conideremos los siguentes puntos: Ma el punto medio de AH, M'a el punto medio de A'H, y M1 el punto medio de MaM'a. Similarmente, se definen M2 y M3.
    Denotamos por:
    Ra el eje radical de las circunferencias M2(M2B') y M3(M3C').
    Rb el eje radical de las circunferencias M3(M3C') y M1(M1A').
    Rc el eje radical de las circunferencias M1(M1A') y M2(M2B').

      Las paralelas La, Lb, Lc a Ra, Rb, Rc a través de A, B, C, respectivamente, son concurrentes si y sólo si P está en la cúbica de Lucas. El punto de concurrencia Q queda sobre la cúbica de Darboux.

    ( Mostrar/Ocultar figura )
      Anopolis1574g.png
      Si P(u:v:w), en coordenadas baricéntricas,
    Q(a^4u(4u+v+w) - 2a^2(v-w)(b^2v-c^2w) - (b^2-c^2)(v+w)(b^2(u+2v)-c^2(u+2w)) : ... : ...).

      Parejas de centros {P,Q}, P en la cúbica de Lucas y Q en la cúbica de Darboux:
    {X2,X4}, {X4,X64}, {X7,X84}, {X8,X1}, {X20,X20}, {X69,X3}, {X189,X3345}, {X253,X3346}, {X329,X40}, {X1032,X3348}, {X1034,X3347}.

      La primera componente de las coordenadas baricéntricas del centro radical R de las circunferencias M1(M1A'), M2(M2B') y M3(M3C') es:
    a^4 v (u+v) w (u+w) (2 u^3+(v-w)^2 (v+w)+2 u (v+w)^2)-a^2 (c^2 v (2 u^5 (v+w)+v (v-w) w^2 (v+w)^2+u^4 v (v+3 w)+2 u v w^2 (v^2-w^2)-2 u^3 (v^3-v w^2)-u^2 (v^4+v^3 w-2 w^4))+b^2 w (2 u^5 (v+w)-v^2 (v-w) w (v+w)^2+u^4 w (3 v+w)+2 u^3 w (v^2-w^2)+u (-2 v^4 w+2 v^2 w^3)+u^2 (2 v^4-v w^3-w^4)))-(b^2-c^2) u (v+w) (b^2 w (u^3 (2 v-w)+2 u^2 v (v+w)+v (v^3+v^2 w+v w^2+w^3)+u (v^3+4 v^2 w+v w^2+w^3))-c^2 v (-u^3 (v-2 w)+2 u^2 w (v+w)+w (v^3+v^2 w+v w^2+w^3)+u (v^3+v^2 w+4 v w^2+w^3))).


      Los puntos P, Q y R están alineados si y sólo si P está en una curva algebraica de grado diez, que pasa por A, B, C, X2, X4, X20, los puntos medios de los lados (donde es tangente a los lados), pies de las cevianas de X69 y por los puntos del infinto de los lados.


      Si P es el baricentro, Q es el ortocentro, y R es X3830, que divide al segmento X2X4 en la razón 3:-1.


      Los puntos La∩Rb, La∩Rc, Lb∩Rc, Lb∩Ra, Lc∩Ra, Lc∩Rb quedan sobre una misma cónica de centro el punto medio de Q y R.



  • martes, 13 de mayo del 2014

    Construcción triángulo (AB AC IH)

    ( Art of Problem Solving, jrrbc)

    Construcción de un triángulo conocidas las rectas determinadas por dos de sus lados y la recta que pasa por el incentro y el ortocentro.

    Del triángulo ABC a determinar, admitamos que el vértice A sea el punto de intersección de las dos rectas lb y lc dadas, que coinciden con lados del triángulo. El incentro I de ABC ha de estar en una de las dos bisectrices de tales rectas y sobre la recta r dada, que ha de contener también al ortocentro H.

    Denotamos por Y y Z las proyecciones ortogonales de I sobre los lados conocidos lb y lc, estos son los puntos de contacto de la circunferencia inscrita Γ del triángulo buscado ABC con los lados AC y AB, respectivamente. Nos falta determinar el punto X de contacto Γ con el lado BC.
    ( Mostrar/Ocultar figura )
      jrrbc.png

    Esta es una construcción con regla y compás de posibles soluciones:
    (Ver detalles en el caso de construcción de triángulos: AB AC IH)

    Por el punto de intersección M de r con la perpendicular a lb por A, se traza la perpendicualar a lc, que corta a lb en Ba.
    Por el punto de intersección N de r con la perpendicular a lc por A, se traza la perpendicualar a lb, que corta a lc en Ca.
    Sean P el punto de interseccion de las rectas BaCa y YZ, y Q el punto de intersección de r con la reflexión de YZ respecto a I.
    Los puntos de corte de la recta PQ con Γ dan los puntos de tangencia del lado BC con la circunferencia inscrita.
    En consecuancia, al considerar la otra bisectriz de las rectas lb y lc, PUDIERAN EXISTIR HASTA CUATRO SOLUCIONES.

    Mostrar/Ocultar Un caso con tres soluciones
       gtr2531ec.png


  • sábado, 10 de mayo del 2014

    Conjugado isotómico del conjugado isogonal

    Sean ABC un triángulo, P un punto y DEF su triángulo circunceviano. Denotamos por (Ab) la circunferencia que pasa por D y es tangente a AB en B, y por (Ac) la circunferencia que pasa por D y es tangente a AC en C.
    Sea A' el otro punto de intersección de estas circunferencias (está sobre BC). Similarmente, se definen los puntos B' y C'. (ver Estrofoide de Jerabek)

    A'B'C' es el triángulo ceviano de tgP, conjugado isotómico del conjugado isogonal de P.

    ( Mostrar/Ocultar figura )
      tgP.png


  • viernes, 9 de mayo del 2014

    Proyectividad en la recta de Euler

     Sean ABC un triángulo y P un punto. El triángulo circunceviano de P es perspectivo con el triángulo ceviano del conjugado isotómico del conjugado isogonal de P si y solo si P está en la recta de Euler.
     El perspector P' también está en al recta de Euler y la correspondencia P ↦ P' es una proyectividad, con puntos dobles los de intersección de la recta de Euler con la circunferencia circunscrita.

    ( Mostrar/Ocultar figura )
      ProyectividadRectaEuler.png
    Si P es un punto del plano del triángulo ABC con coordenadas baricentricas (u:v:w), las coordenadas de los vértices de su triángulo circunceviano DEF (puntos en los que las cevianas de P vuelven a cortar a la circunferencia circunscrita) son:

    D(-a^2vw : v(c^2v+b^2w) : w(c^2v + b^2w)), E(u(c^2u+a^2w) : -b^2uw : w(c^2u+a^2w)), F(u(b^2u+a^2v) : v(b^2u+a^2v) : -c^2uv).

    Y las coordenadas de los vértices del triángulo ceviano de tgP:

    L(0 : c^2v : b^2w), M(c^2u : 0 : a^2w), N(b^2u : a^2v : 0).

    Los triángulos DEF y LMN son perspectivos si y sólo si
    ((-a^2b^2+b^4+a^2c^2-c^4)u - (a^4+a^2b^2-b^2c^2+c^4)v + (a^4-b^4-a^2c^2+b^2c^2)w) (c^2uv+b^2uw+a^2vw)=0.
    Es decir, si P está en la circunferencia circunscrita (DEF degenera en un punto) o en la recta de Euler.

    Si P(u:v:w) está en la recta de Euler el centro de prespectividad de DEF y LMN es:

    P' (2b^2c^2SAu^2 + a^2(b^2c^2+a^2(SA-a^2))vw + b^2(b^2c^2+a^2(SA-b^2))wu + c^2(b^2c^2+a^2(SA-c^2))uv : ... : ...)


    Pares de puntos homólogos (P↦P'), figurando ambos actualmemtes en ETC:
    (X2↦X22), (X3↦X20), (X4↦X3), (X22↦X1370), (X23↦X858), (X24↦X4), (X25↦X2), (X27↦X4184), (X28↦X21), (X29↦X4225), (X30↦X2071), (X186↦X30), (X199↦X3151), (X237↦X401), (X378↦X376), (X403↦X186), (X419↦X237), (X436↦X418), (X451↦X2915), (X468↦X23), (X1113↦X1113), (X1114↦X1114), (X1593↦X3522), (X1598↦X3523), (X2070↦X3153), (X2073↦X5196), (X2074↦X1325), (X2409↦X4230), (X3144↦X3145), (X3145↦X3152), (X3515↦X3146), (X3517↦X3091), (X3518↦X5), (X3520↦X550), (X3542↦X24), (X4183↦X1817), (X4185↦X4189), (X4186↦X4188), (X4213↦X199), (X4222↦X404), (X4227↦X4221), (X4230↦X4226), (X4231↦X4220), (X4232↦X1995), (X4233↦X4228), (X4238↦X4236), (X4241↦X4243), (X4244↦X4238), (X4246↦X3658), (X4247↦X4234), (X4249↦X4237), (X5136↦X4216), (X5200↦X1599), (X6240↦X3520), (X6353↦X25), (X6622↦X3515).

    Siendo los puntos dobles X1113 y X1114, y los puntos límites X186 y X2071.

    En términos de la referecia proyectiva sobre la recta de Euler {O,H;N} (de puntos base el circuncentro y el ortocentro y punto unidad el centro de la circunferencia de los nueve puntos), la correspondencia P↦P' se expresa por:
    proyectividadRectaEulerEc..png

    Los puntos dobles se obtienen para los valores de λ raíces del polinomio
    8a^2b^2c^2SASBSCC - 2a^2b^2c^2λ + λ²=0.
    λ12 = a^2b^2c^2(1 ± (a^2b^2c^2 - 8SASBSC)½).

    Obteniéndose los puntos de coordenadas homogéneas, en la referencia {O,H;N},
    (-abc : abc ± (a^2b^2c^2 - 8SASBSC)½).

    Que corresponde a los puntos X1113 y X1114, de interseccción de la recta de Euler con la circunferencia circunscrita.


    Adicionalmente podemos notar que:

    Cuando P varía en al recta de Euler, las rectas DL, EM, DN pasan por puntos fijos (sobre la circunferencia circunscrita) que son los vértices del triángulo circunceviano del punto de De Longchamps, respecto al triángulo A'B'C', antipodal de ABC.



  • jueves, 8 de mayo del 2014

    Estrofoide de Jerabek

    Sean ABC un triángulo, P un punto y DEF su triángulo circunceviano. Denotamos por Ab el centro de la circunferencia que pasa por D y es tangente a AB en B, y por Ac el centro de la circunferencia que pasa por D y es tangente a AC en C.
    Similarmente, se definen los puntos Bc, Ba y Ca, Cb.

    Las rectas AbAc, BcBa y CaCb no son concurrentes y el triángulo A'B'C' delimitado por ellas es perspectivo con ABC si y solo si P queda en la estrofoide de Jerabek (K039).

    ( Mostrar/Ocultar figura )
      estrofoideJerabekPropiedad.png

    Si (u:v:w) son las coordenadas baricentricas de P se tiene que:
    Ab = (-a^2b^2(a^2-b^2+c^2)w : -a^4c^2v-(b^2-c^2)^2(c^2v+b^2w)+ a^2(b^2+c^2)(2c^2v+b^2w) : 2a^2b^2c^2w),
    Ac = (-a^2c^2(a^2+b^2- c^2)v : 2a^2b^2c^2v : -a^4b^2w-(b^2-c^2)^2(c^2v+ b^2w)+a^2(b^2+c^2)(c^2v+2b^2w))

    La recta AbAc es:
    (a^4b^2c^2vw+(b^2-c^2)^2(c^2v+b^2w)^2- a^2(b^2+c^2)(c^2v+b^2w)^2)x + (-a^2b^2w(c^4v-b^4w+ b^2(a^2w+c^2(-v+w))))y + (-a^2c^2v(a^2c^2v+(b^2- c^2)(c^2v+b^2w)))z=0.

    Las rectas AbAc, BcBa y CaCb son concurrentes si y solo si P queda en una séxtica con puntos dobles en los vértices de ABC

    ( Mostrar/Ocultar figura )
      estrofoideJerabekPropiedad.png

    Notificación de Bernard Gibert:

    Los triángulos ABC y A'B'C' son ortológicos si y solo si P está sobre la estrofoide de Jerabek.   En este caso, el centro de ortología de ABC respecto a A'B'C' describe la estrofoide de Ehrmann (K025).

    ( Mostrar/Ocultar figura )
      estrofoideJerabekOrtologico.png



    Alguna información sobre la estrofoide de Jerabek

    •  Una estrofoide de foco F, de punto doble D y eje d0 (pasando por D) es el lugar geométrico de los puntos M de una recta variable d pasando por F y tal que PM=PD, donde P es el punto de intersección de la recta d0 con d
    ( Mostrar/Ocultar figura )
      estrofoide.png

    •  La estrofoide de Jerabek (K039) tiene punto doble el circuncentro, foco el inverso en la circunferencia circunscrita de X263 y eje la recta que une el circuncentro con X49.
    ( Mostrar/Ocultar figura )
      estrofoideJerabek.png

    •  La estrofoide de Jerabek es la imagen de la hipérbola de Jerabek en la inversión respecto a la circunferencia circunscrita.

    •  La coordenadas baricéntricas del foco de la estrofoide de Jerabek son:
    focoEstrofoideJerabek.png
    con número de búsqueda en ETC: -0.286262733091678965975303639

    •  La ecuación baricéntrica de su asíntota de la estrofoide de Jerabek es:
    (b^2-c^2)(a^4SA^4- 2a^2SBSCSA^3-((b^2-c^2)^4+2SBSC(SB^2+SC^2))SA^2- 2a^2SBSC((b^2-c^2)^2-SBSC)SA-(b^2-c^2)^2SB^2SC^2)x/(a^2SA) + ... =0.

    Cuyo punto del infinito:
    (a^2(a^6(b^2+c^2)-3a^4(b^4+c^4)+ a^2(3b^6-b^4c^2-b^2c^4+3c^6)-(b^4-c^4)^2): ... : ... ),
    con número de búsqueda en ETC: 1.022945264181160305475445435.

    La asíntota vuelve a cortar a la estrofoide de Jerabek en el punto S con primera coordenada baricéntrica:
    (a^2SA(a^2(SA^2-SBSC)-(b^2-c^2)^2SA) / (a^4SA^4- 2a^2SBSCSA^3-((b^2-c^2)^4+2SBSC(SB^2+SC^2))SA^2- 2a^2SBSC((b^2-c^2)^2-SBSC)SA-(b^2- c^2)^2SB^2SC^2), con número de búsqueda en ETC: 7.9946323139539709651682014

    •  La estrofoide de Jerabek es el lugar geométrico de los pies de las perpendiculares, trazadas desde el circuncentro a las tangentes a la parábola de directriz el eje d0 de la estrofoide y de foco F', la reflexión del circuncentro en el inverso (respecto a la circunferencia circunscrita) F del antipodal X265 del circuncentro en la hipérbola de Jerabek.

    Esta parábola pasa por X647 (centro de perspectividad del triángulo ABC y el triángulo tangencial de la hipérbola de Jerabek) y dirección del eje dada por el punto del infinito X924.
    Las coordenadas de su foco son:
    F' = (a^2(a^2-b^2-c^2) (a^8-a^6(b^2+c^2) + a^4b^2c^2 - a^2(b^2-c^2)^2(b^2+c^2) + (b^2-c^2)^2(b^4-b^2c^2+c^4)) : ... :...).
    con número de búsqueda en ETC: -7.354888360379703466465294356
    ( Mostrar/Ocultar figura )
      estrofoideJerabekParabola.png


  • viernes, 02 de mayo del 2014

    Lugares geométricos y triángulos circuncevianos

    ( Anopolis #1416, Antreas P. Hatzipolakis)

    Sean ABC un triángulo, P un punto y A'B'C' su triángulo circunceviano.
    Denotamos:
    Ab el centro de la circunferencia pasando por C' y tangente a AB en B,
    Ac el centro de la circunferencia pasando por B' y tangente a AC en C.
    Similarmente se definen los puntos Bc,Ba y Ca,Cb.
    Sean además Ma, Mb, Mc los puntos medios de los segmentos AbAc, BcBa, CaCb.

    •  El lugar geométrico de los puntos P tales que el triángulo delimitado por las rectas AbAc, BcBa, CaCb es nodegenerado y perspectivo con ABC es la cúbica de Neuberg, K001 = pK(X6, X30),

    ( Mostrar/Ocultar figura )
      Anopolis1416a2.png


    •  El lugar geométrico de los puntos P tales que las rectas AbAc, BcBa, CaCb son concurrentes es una séxtica con puntos dobles en los vértices de ABC, pasa por los centros X2, X3 y X1139 y por los puntos, Oa, Ob, Oc, donde las cevianas del circuncentro vuelven a corta a al cúbica de Neuberg.

    ( Mostrar/Ocultar figura )
      Anopolis1416a2.png
    Cuando P=X1139 (conjugado isogonal del punto de reflexión de Parry, X399), las rectas AbAc, BcBa, CaCb son paralelas, con punto del infinito X523, isogonal conjugado del foco de la parábola de Kiepert.
    Otros puntos comunes con la cúbica de Neuberg son Oa, Ob, Oc:
    Oc=(a^2SA:b^2SB:2SASB), ...


    •  El lugar geométrico de los puntos P tales que los triángulos ABC y MaMbMc son ortológicos es es la cúbica de Thomson, K002 = pK(X6, X2),

    Si P recorre la cúbica de Thomson, el centro ortológico Q de MaMbMc con respecto a ABC recorre la cúbica imagen de la cúbica de Darboux mediente la homotecia de centro el circuncentro y razón 1/2.
    ( Mostrar/Ocultar figura )
      Anopolis1416b1.png
    Algunos centros ortológicos R de ABC con respecto a MaMbMc de puntos P en la cúbica de Thomson: {X1, X80}, {X3, X1294}, {X4, X265}, {X6, X671}, {X57, X3254}
    ( Mostrar/Ocultar figura )
      Anopolis1416b2.png


    •  El lugar geométrico de los puntos P tales que las mediatrices de los segmentos AbAc, BcBa, CaCb son concurrentes es una nónica con puntos cuadruples en los vértices de ABC, pasa por los centros X1, X2, X3, X13, X14, X1113, X1114, y X1138 y por los pies de las cevianas del circuncentro.

    ( Mostrar/Ocultar figura )
      Anopolis1416c.png


  • martes, 22 de abril del 2014

    Punto de reflexión de Parry y otros asociados

    ( Anopolis #1382, Antreas P. Hatzipolakis)

    Sean ABC un triángulo, da, db, dc tres rectas paralelas a través de A, B, C, respectivamente, y d'a, d'b, d'c las reflexiones de da, db, dc en BC, CA, AB, respectivamente.

    Si las recta da, db, dc son paralelas a la recta de Euler de ABC, las rectas d'a, d'b, d'c concurren en X399 (punto de reflexión de Parry)

    ( a^2(a^8 - 4a^6(b^2+c^2) + a^4(6b^4+b^2c^2+6c^4) + a^2(-4b^6+b^4c^2+b^2c^4-4c^6) + (b^2-c^2)^2(b^4+4b^2c^2+c^4) : ... : ...)


    Cuando las rectas da, db, dc giran alrededor de A, B, C, permaneciendo paralelas, la recta de Euler de A'B'C' pasa por un punto fijo Q, cuyas coordenadas baricéntricas han sido calculadas por J. F. García Capitán, Hyacinthos message 15827 (November 19, 2007).

    El eje radical de las circunferencias circunscrita y de Euler de A'B'C' pasa por un punto fijo Y, cuando las rectas da, db, dc giran alrededor de A, B, C, permaneciendo paralelas.

    Y = (a^10(b^2+c^2)
    -a^8(3b^4+4b^2c^2+3c^4)
    +a^6(2b^6+5b^4c^2+5b^2c^4+2c^6)
    +2a^4(b^8-3b^6c^2+b^4c^4-3b^2c^6+c^8)
    -a^2(b^2-c^2)^2(3b^6-4b^4c^2-4b^2c^4+3c^6)
    +(b^2-c^2)^6: ... : ...)

    con número de búsqueda en ETC: -12.444306554956166449865828
    ( Mostrar/Ocultar figura )
      Anopolis1372.png
    Los puntos Y y Q son antipodales en la circunferencia que describe el punto de intersección la recta de Euler de A'B'C' y el eje radical de las circunferencias circunscrita y de Euler de A'B'C'. Esta circunferencia pasa por X399.

  • jueves, 17 de abril del 2014

    Perpendiculares a cevianas y lugares geométricos

    ( Anopolis #1354, Antreas P. Hatzipolakis)

    •  Sean ABC un triángulo, I su incentro y P un punto. La perpendicular por P a AI corta a AB y AC en Ab y Ac, respectivamente. Se denota por r1 el eje radical de las circunferencias circunscritas a los triángulos ABC y AAbAc.
    Los ejes radicales r2 y r3 se definen de forma cíclica.

    El lugar geométrico de los puntos P tales que el triángulo ABC es perspectivo con el triángulo A'B'C', delimitado por los ejes radicales r1, r2 y r3, está formado por la hipérbola equilátera que pasa por el incentro (hipérbola de Feuerbach) y por la cúbica circunscrita a ABC de ecuación baricéntrica:
    x (c (a - b) (a - b + c) y^2 + b (a - c) (a + b - c) z^2) +
    y (a (b - c) (a + b - c) z^2 + c (b - a) (-a + b + c) x^2) +
    z (b (c - a) (-a + b + c) x^2 + a (c - b) (a - b + c) y^2 ) - 2 a b c x y z = 0.


    Cuando P se mueve sobre la hipérbola de Feuerbach el centro de perspectividad Q de los triángulos ABC y A'B'C' recorre la cúbica generalizada de Lemoine K(X1)=K360.
    ( Mostrar/Ocultar figura )
      Anopolis1352Feuerbach.png




    •  Sean ABC un triángulo y P un punto. La perpendicular por P a AP corta a AB y AC en Ab y Ac, respectivamente. Se denota por r1 el eje radical de las circunferencias circunscritas a los triángulos ABC y AAbAc.
    Los ejes radicales r2 y r3 se definen de forma cíclica.

    El lugar geométrico de los puntos P tales que el triángulo ABC es perspectivo con el triángulo A'B'C', delimitado por los ejes radicales r1, r2 y r3, está formado por la circunferencia circunscrita a ABC y por la cúbica de Darboux K004



    Cuando P se mueve sobre la circunferencia circunscrita los ejes radicales r1, r2 y r3 son concurrentes en un punto Q, que describe la isocúbica no pivotal nK(X(577),X(394),X(3))=cK(#X(3),X(394)) (conico-pivotal isocubic, Jean-Pierre Ehrmann and Bernard Gibert.- Special Isocubics in the Triangle Plane, §8), que tiene al circuncentro como punto aislado y pasa por X878 (que corresponde a P=X98, punto de Tarry).
    La "cónica de contacto" de cK(#X(3),X(394)) es la cónica circunscrita de Johnson.
    (Otros ejemplos de "conico-pivotal isocubics" )

    ( Mostrar/Ocultar figura )
      Anopolis1352POR.png


  • miércoles, 16 de abril del 2014

    Triángulos con misma área que el triángulo órtico

    ( TriangulosCabri #707, Ricardo Barroso)

    Dado un triángulo ABC, el lugar geométrico de los puntos P para los cuales su triángulo pedal tiene la misma área que el triángulo órtico es la circunferencia de centro el circuncentro y que pasa por el ortocentro.

    Las coordenadas baricentricas de los vértices del triángulo pedal de un punto P=(x:y:z) son:
    D = (0 : (b^2-c^2)x+a^2(x+2y) : (-b^2+c^2)x+a^2(x+2z)),
    E = ((a^2-c^2)y+b^2(2x+y) : 0 :-a^2y+c^2y+b^2(y+2 z)),
    F = {(a^2-b^2)z+c^2(2x+z) : (-a^2+b^2)z+c^2(2y+z) : 0).
    El área de DEF coincide con el área de triángulo órtico (pedal del ortocentro) si y sólo si las coordenadas de P satisfacen a la ecuación:
    a^6 x^2 - a^4 b^2 x^2 - a^2 b^4 x^2 + b^6 x^2 - a^4 c^2 x^2 + 2 a^2 b^2 c^2 x^2 - b^4 c^2 x^2 - a^2 c^4 x^2 - b^2 c^4 x^2 + c^6 x^2 + 2 a^6 x y - 2 a^4 b^2 x y - 2 a^2 b^4 x y + 2 b^6 x y - 3 a^4 c^2 x y + 6 a^2 b^2 c^2 x y - 3 b^4 c^2 x y + c^6 x y + a^6 y^2 - a^4 b^2 y^2 - a^2 b^4 y^2 + b^6 y^2 - a^4 c^2 y^2 + 2 a^2 b^2 c^2 y^2 - b^4 c^2 y^2 - a^2 c^4 y^2 - b^2 c^4 y^2 + c^6 y^2 + 2 a^6 x z - 3 a^4 b^2 x z + b^6 x z - 2 a^4 c^2 x z + 6 a^2 b^2 c^2 x z - 2 a^2 c^4 x z - 3 b^2 c^4 x z + 2 c^6 x z + a^6 y z - 3 a^2 b^4 y z + 2 b^6 y z + 6 a^2 b^2 c^2 y z - 2 b^4 c^2 y z - 3 a^2 c^4 y z - 2 b^2 c^4 y z + 2 c^6 y z + a^6 z^2 - a^4 b^2 z^2 - a^2 b^4 z^2 + b^6 z^2 - a^4 c^2 z^2 + 2 a^2 b^2 c^2 z^2 - b^4 c^2 z^2 - a^2 c^4 z^2 - b^2 c^4 z^2 + c^6 z^2 = 0.

    Esta ecuación se puede poner en la forma:
    a^2yz+b^2zx+c^2xy-(-a^2+b^2+c^2)(a^2-b^2+c^2)(a^2+b^2-c^2)/((a+b+c)(-a+b+c)(a-b+c)(a+b-c)) (x+y+z)^2=0.
    Se trata de la circunferencia de centro en el circuncentro y que pasa por el ortocentro.
    ( Mostrar/Ocultar figura )
      RB707.png



  • martes, 8 de abril del 2014

    Una propiedad del cuadrado baricéntrico del simediano

    Sean ABC un triángulo y L un punto variable sobre la paralela a BC por A. El lugar geométrico de los ortocentros de los triángulos LBC es una parábola; denotamos por ta la tangente en su vértice. Similarmente y de forma cíclica se consideran la tangentes tb y tc.

    El cento de homotecia de los triángulos ABC y el delimitado por ta, tb y tc es el X32.

    ( Mostrar/Ocultar figura )
      HG080414.png

    Sea L=(t:1:-1) un punto variable sobre la recta paralela a BC por A. Las coordenadas baricéntricas del ortocentro del triángulo LBC son:
    HL = ( (a^2(t-2)+(c^2-b^2)t)((b^2-c^2)t+a^2(2+t)) : -(a^2(t-1)-c^2(t-1)-b^2(1+t))((b^2-c^2)t+a^2(2+t)) : -(a^2(t-2)+(c^2-b^2)t)(-c^2(t-1)+a^2(1+t)-b^2(1+t)) ).

    Este punto describe la parábola ℘a de ecuación:
    a: -a^2SAx^2 + 4a^4yz+(3a^4+b^4-4a^2c^2-2b^2c^2+c^4)zx + (3a^4-4a^2b^2+b^4-2b^2c^2+c^4)xy = 0.

    Cuya tangente ta en su vértice tiene por ecuación:
    (2a^4-2a^2b^2+b^4-2a^2c^2-2b^2c^2+c^4)x + a^4y + a^4z = 0.
    Permutando cíclicamente, se obtienen las tangentes tb y tc en los vértices de las correpondientes parábolas ℘b y ℘c. El punto de interseccion de estas últimas es:
    A' = (-a^4+2a^2(b^2+c^2)-2(b^4-b^2c^2+c^4) : b^4 : c^4).
    Y silarmente resultan las coordenadas de B'=tc∩ta y C' = ta∩tb. En consecuencia, las rectas AA', BB' y CC' concurren en el punto X32 = (a^4:b^4:c^4).

    Adicionalmente tenemos:

    •  El centro de homotecia de los triángulos ABC y el delimitado por las directrices de las tres parábolas ℘a, ℘b y ℘c tiene coordenadas baricentricas:

    (a^4+S^2 : b^4+S^2 : c^4+S^2)

    con número de búsqueda en ETC: 0.64658408249618255603004536878
    ( Mostrar/Ocultar figura )
      HG080414d.png
    La directriz de la parábola ℘a es (a^4-3S^2)x+(a^4+S^2)y+(a^4+S^2)z=0.

    •  El centro de homotecia de los triángulos ABC y el delimitado por las perpendiculares por los focos a los ejes de las tres parábolas ℘a, ℘b y ℘c tiene coordenadas baricentricas:

    (a^4-S^2 : b^4-S^2 : c^4-S^2)

    con número de búsqueda en ETC: -0.2507364947244949174662
    ( Mostrar/Ocultar figura )
      HG080414f.png
    La perpendicular al eje de la parábola ℘a por su foco es (a^4-5S^2)x+(a^4-S^2)y+(a^4-S^2)z=0.

    •  El ortocentro X4 de ABC es común a las tres parábolas. La tangente en X4 a la parábola ℘a corta a los lados AB y AC en Ac y Ab, respectivamente. Similarmente se determina los puntos Ba, Bc, Cb y Ca.
    Estos seis puntos quedan en una misma cónica, cuyo centro tiene coordenadas baricéntricas (notación de Conway):

    (SASBSC + 2(b^2SB^2+c^2 SC^2) : SASBSC + 2(c^2SC^2+a^2SA^2 ) : SASBSC + 2(a^2SA^2+b^2SB^2)),

    con número de búsqueda en ETC: 1.8894135371622742397888827245

    Los puntos Aa, Bb y Cc de intersección de las tangentes en el ortocentro a las parábolas ℘a, ℘b y ℘c con los lados BC, CA u AB, respectivamente, están en la recta SA^2x+SB^2y+SC^2z=0, polar trilineal de X393.
    ( Mostrar/Ocultar figura )
      HG080414a.png


  • lunes, 7 de abril del 2014

    Conjugado isogonal respecto al triángulo antipedal

    ( Anopolis #1309, Antreas P. Hatzipolakis)

    Sean ABC un triángulo, P un punto y A'B'C' el triángulo antipedal de P. El lugar geométrico de los puntos P tales que P, el conjugado isogonal de P respecto a ABC y el conjugado isogonal de P respecto a A'B'C' están alineados es cúbica de McKay, K003 = pK(X6, X3).

    ( Mostrar/Ocultar figura )
      Anopolis1309.png

    Si P(u:v:w), su conjugado isogonal es P*(a^2vw:b^2wu:c^2uv) (en coordenadas baricéntricas).
    El conjugado isogonal Q de P respecto a A'B'C' es la intersección de las rectas AD', BE', CF', siendo D', E' , F' las reflexiones de P respecto a los puntos medios D, F, F de los lados de ABC (ver §4.2Cojugado isogonal).
    Q=(SASC(v+w) + SB(SA(v+w)-SC u) : ... : ... ).


    Si P está sobre la cúbica de McCay, su conjugado isogonal Q respecto a A'B'C' es la reflexión de P respecto al circuncentro.

  • miércoles, 2 de abril del 2014

    El centro de congruencia de Yff

    Sean ABC un triángulo, P un punto, A1 y A2 los centros de semejanza interno y externo de las circunferencias con centro en B y C que pasan por P. Similarmente, y de forma cíclica, se definen los puntos B1, B2, C1, A2. Entonces, las rectas AA1, BB2 y CC1 concurren en un punto Q, cuya polar trilineal pasa por A2, B2 y C2.

    ( Mostrar/Ocultar figura )
      HG020414.png

    Esta construcción de los puntos A1 y A2 conincide con la generalización dada por Peter Moses en ETC, ya que PA1 es la bisectriz del ángulo ∠BPC:
    "Let ABC be a triangle and P a point. Let D be the point on side BC such that ∠BPD = ∠DPC), and likewise for point E on side CA and point F on side AB. If P = (p : q : r) (trilinears), then the lines AD, BE, CF concur in the point K(P) = f(p,q,r,A) : f(q,r,p,B) : f(r,p,q,C), where f(p,q,r,A) = (q^2 + r^2 + 2qr cos A)-1/2. Moreover, if P* is the inverse of P in the circumcircle, then K(P*) = K(P). [Peter Moses, Feb. 1, 2010, based on Seiichi Kirikami's construction of X(174)]"

    Si P=(u:v:w), en coordenadas baricéntricas, el punto de intersección de las rectas AA1, BB2 y CC1 es:
    Yff.png

    Al punto P', inverso de P en la circunferencia circunscrita, le corresponde el mismo punto Q que a P.

    Si P=X1 es el incentro, Q=X174 es centro de congruencia de Yff:
    Iff
    ( Mostrar/Ocultar figura )
      HG020414X1.png

    Parejas de puntos {P,Q}:
    {X1,X174}, {X3,X2}, {X13,X13}, {X15,X1}, {X16,X1}, {X36,X174}.

    Cuando P es el baricentro, Q=(1/Sqrt[-a^2+2b^2+2c^2]:1/Sqrt[2a^2-b^2+2c^2]:1/Sqrt[2a^2+2b^2-c^2]), con número de búsqueda en ETC: 1.62327927239083782493133901

  • martes, 1 de abril del 2014

    Triángulos ortológicos asociados al baricentro

    ( Anopolis #1286, Antreas P. Hatzipolakis)

    Sean ABC un triángulo y A'B'C' el triángulo medial.
    Adoptamos las siguientes notaciones:

    Ab y Ac los circuncentros de los triángulos AGB' y AGC', resp.
    Bc y Ba los circuncentros de los triángulos BGC' y BGA', resp.
    Ca y Cb los circuncentros de los triángulos CGA' y CGB', resp.
    Ma, Mb y Mc los puntos medios de AbAc, BcBa, CaCb, resp.


    En coordenadas baricéntricas:
    Ma
    Mb
    Mc

    Los triángulos ABC y MaMbMc son ortológicos.
    El centro de ortología de MaMbMc con respecto a ABC es el centro de la circunferencia de los nueve puntos, X5.
    El centro de ortología de ABC con respecto a MaMbMc es:

    Ortologico1.png
    ( Mostrar/Ocultar figura )
      Anopolis1283.png


    Sean M1, M2 y M3 los simétricos de Ma, Mb y Mc respecto a las medianas AA', BB' y CC'. Los triángulos ABC y M1M2M3 son ortológicos


    En coordenadas baricéntricas:
    M1
    M2
    M3

    El centro de ortología de M1M2M3 y ABC es X2845:
    (a^2SA +10SBSC: b^2SB + SCSA : c^2SC + SASB).

    El centro de ortología de ABC y M1M2M3 es:
    Ortologico2.png
    ( Mostrar/Ocultar figura )
      Anopolis1283.png


  • lunes, 31 de marzo del 2014

    Ejemplos de triángulos perspectivos y ortológicos

    ( Anopolis #1273, Antreas P. Hatzipolakis)

    Sean ABC un triángulo, P un punto y A'B'C' el triángulo pedal de P.
    Adoptamos las siguientes notaciones:

    Ab y Ac son las proyecciones ortogonales de A' sobre PB' y PC', respectivamente.
    A3+ es la intersección de la paralela por B' a AC con la paralela por Ac a A'B'.
    A3- es la intersección de la paralela por B' a AC con la paralela por A' a B'Ac.
    Es decir, la circunferencia de centro en B' y radio A'Ac interseca a la paralela por B' a A'Ac en los dos puntos A3+ y A3-.

    A2+ es la intersección de la paralela por C' a AB con la paralela por Ab a A'C'.
    A2- es la intersección de la paralela por C' a A'B con la paralela por A' a C'Ab.

    M1+ y M1- son los puntos medios de A2+A3+ y A2-A3-, respectivamente.

    ( Mostrar/Ocultar figura )
      Anopolis1273b.png

    Similarmente y de forma cíclica, se definen los puntos:
    B3+, B1+, C1+, C2+, B3-, B1-, C1-, C2- y M2+, M2-, M3+, M3-.

    1. El lugar geométrico de los puntos P tales que las mediatrices de los segmentos A2+A3+, B3+B1+, C1+C2+ son concurrentes es una quártica circunscrita a ABC.

    ( Mostrar/Ocultar figura )

    Información sobre esta cuártica:
      Pasa por el circuncentro y ortocentro. Tres de sus asíntotas son perpendiculares a los lados de ABC y delimitan un triángulo perspectivo con ABC, con centro de perspectividad el foco de la parábola de Kiepert. Las ecuaciones baricéntricas de estas asíntotas (notación de Conway) son:
    (b^2-c^2)SBSBx + a^2SBy + a^2SCz =0   y cíclicamente.
      La cuarta asíntota tiene como punto del infinito a X523, conjugado isogonal del foco de la parábola de Kiepert, X110.

    NOTA: Si P es el circuncentro o está en el infinito, la configuración no tiene sentido. Cuando P es el ortocentro, las mediatrices de los segmentos A2+A3+, B3+B1+, C1+C2+ concurren en el punto de primera coordenada baricéntrica:

    a^2(a^6(b^2+c^2) - 3a^4(b^4+c^4) + a^2(3b^6+b^4c^2+b^2c^4+3c^6) - (b^2-c^2)^2(b^4+4b^2c^2+c^4)),

    con número de búsqueda en ETC: -1.91350459625470323717507806


    2. El lugar geométrico de los puntos P tales que las mediatrices de los segmentos A2-A3-, B3-B1-, C1-C2- son concurrentes es una quártica circunscrita a ABC.

    ( Mostrar/Ocultar figura )

      Cuando P es el ortocentro, las mediatrices de los segmentos A2-A3-, B3-B1-, C1-C2- concurren en el centro de la circunferencia de Taylor, X389. Además, en este caso el triángulo M1-M2-M3- es perspectivo con ABC (centro de perspectividad el semidiano) y su circunferencia circunscrita es concéntrica con la circunferencia de Taylor.
    ( Mostrar/Ocultar figura )



    3. El lugar geométrico de los puntos P tales que los triángulos ABC y M1+M2+M3+ son perspectivos es una cúbica circunscrita a ABC.
    Estos triángulos son ortológicos si solo si P está sobre la recta de Euler.

    ( Mostrar/Ocultar figura )


    Los centros de ortología y de perspectividad de los triángulos ABC y M1+M2+M3+ correspondientes a los puntos X4 y X361 son colineales:
    ( Sondat's Theorem. If two triangles are mutually orthologic and perspective, the centres of orthology and perspectivity are collinear. Further, it is perpendicular to the perspectrix.
    An Application of Sondat's Theorem, Ion Patrascu and Florentin Smarandache
    ):

    •  P=X4
    ( Mostrar/Ocultar figura )
      El centro de perspectividad de los triángulos ABC y M1+M2+M3+ es el simediano, X6.
      El centro de ortología de ABC respecto a M1+M2+M3+ coordenadas baricéntricas:

    O31 = ( SA/(a^4 - 2a^2(b^2+c^2) + b^4+6b^2c^2+c^4) : ... : ...),

    con número de búsqueda en ETC: 10.2893367499920063626210307
      El centro de ortología de M1+M2+M3+ respecto a ABC coordenadas baricéntricas:

    O32 = ( 2a^10 - a^8(b^2+c^2) - 8a^6(b^4-b^2c^2+c^4) + 10a^4(b^2-c^2)^2(b^2+c^2) - 2a^2(b^4-c^4)^2 - (b^2-c^2)^4(b^2+c^2): ... : ...),

    con número de búsqueda en ETC: 7.3799576190510080677803629

    •  P=X631
    ( Mostrar/Ocultar figura )
    En este caso los triángulos ABC y M1+M2+M3+ son homotéticos.
      El centro de homotecia de los triángulos ABC y M1+M2+M3+ es el punto de coordenadas baricéntricas:

    Q = ( a^2(a^4 - 2a^2(b^2+c^2) + b^4+6b^2c^2+c^4 ) : ... : ...),

    con número de búsqueda en ETC: 3.08200205691896287503357243
      El centro de ortología de ABC respecto a M1+M2+M3+ es el ortocentro.
      El centro de ortología de M1+M2+M3+ respecto a ABC es el punto de coordenadas baricéntricas:

    Y = ( 2a^10 - 5a^8(b^2+c^2) + 4a^6(b^2-c^2)^2 - 2a^4(b^6+7b^4c^2+7b^2c^4+c^6) + 2a^2(b^2-c^2)^2(b^4+14b^2c^2+c^4) - (b^2-c^2)^4(b^2+c^2): ... : ...),

    con número de búsqueda en ETC:-1.5600580790622942195564665


    4. El lugar geométrico de los puntos P tales que los triángulos ABC y M1-M2-M3- son perspectivos es una cúbica circunscrita a ABC.
    Estos triángulos son ortológicos si solo si P está sobre la recta de Euler.

    ( Mostrar/Ocultar figura )
    Esta cúbica pasa por el ortocentro, donde es tangente a la recta de Euler, por X1498 y el tercer punto E común con la recta de Euler tiene por coordenadas:

    E = ( a^2(a^8 - 2a^6(b^2+c^2) - 12a^4b^2c^2+ 2a^2(b^2+c^2)^3 - (b^2-c^2)^2(b^4-6b^2c^2+c^4)): ... : ...),



    Casos en que los triángulos ABC y M1-M2-M3- son simultáneamente perspectivos y ortológicos:

    •  P=X4
    ( Mostrar/Ocultar figura )
    En este caso los triángulos ABC y M1-M2-M3- son homotéticos.
      El centro de homotecia de los triángulos ABC y M1-M2-M3- es el simediano X6.
      El centro de ortología de ABC respecto a M1-M2-M3- es el ortocentro.
      El centro de ortología de M1-M2-M3- respecto a ABC es el punto de coordenadas baricéntricas:

    O42 = ( 2a^10 - 5a^8(b^2+c^2) + 4a^6(b^2+c^2)^2 - 2a^4(b^2-c^2)^2(b^2+c^2) + 2a^2(b^2-c^2)^4- (b^2-c^2)^4(b^2+c^2): ... : ...),

    con número de búsqueda en ETC:-1.34146146000198321561337218

    •  P=E
    ( Mostrar/Ocultar figura )

      El centro de perspectividad de los triángulos ABC y M1-M2-M3- es el punto de coordenadas baricéntricas:

    Z = ( a^2SA /(b^2+c^2-3a^2) : b^2SB /(c^2+a^2-3b^2) : c^2SC /(a^2+b^2-3c^2) ),

    con número de búsqueda en ETC: -2.28208807333641855014440139
      El centro de ortología de ABC respecto a M1-M2-M3-es el punto de coordenadas baricéntricas:

    U = ( a^2/(5a^10 - 11a^8(b^2+c^2) + 2a^6(b^4+22b^2c^2+c^4) + 2a^4(5b^6-13b^4c^2-13b^2c^4+5c^6) - a^2(b^2-c^2)^2(7b^4+18b^2c^2+7c^4)+ (b^2-c^2)^4(b^2 + c^2)) : ... : ...),

    con número de búsqueda en ETC:-16.11015588490167262757688
      El centro de ortología de M1-M2-M3- respecto a ABC es el punto de coordenadas baricéntricas:

    V = ( a^2(a^12(b^2+c^2) - 2a^10(2b^4+7b^2c^2+2c^4) + 5a^8(b^6+5b^4c^2+5b^2c^4+c^6) - 12a^6b^2c^2(b^4+6b^2c^2+c^4) + a^4(-5b^10+11b^8c^2+26b^6c^4+26b^4c^6+11b^2c^8-5c^10) + 2a^2(b^2-c^2)^4(2b^4-3b^2c^2+2c^4) - (b^2-c^2)^4(b^6-7b^4c^2-7b^2c^4+c^6)) : ... : ... ),

    con número de búsqueda en ETC: -12.933974089689188258369107


    5. El lugar geométrico de los puntos P tales que los triángulos M1+M2+M3+ y M1-M2-M3- son perspectivos es la recta de Euler. Estos triángulos son ortológicos para todo punto P del plano.

    ( Mostrar/Ocultar figura )

    Los centros de ortología O1O2 están alineados con Q, por el Teorema de Sondat.

    Cuando P recorre la recta de Euler, el centro de perspectividad Q de los triángulos M1+M2+M3+ y M1-M2-M3- recorre la recta X2X6, que pasa por el baricentro y semidiano. La envolvente de las rectas PQ es una parábola (t_P_PCp) tangente a la recta de Euler en X376 (baricentro del triángulo antipedal de X2) y a la recta X2X6 en X597 (punto medios del baricentro y simediano).
    el punto del infinito de esta parábola es:

    ( 10a^6 - a^4(b^2+c^2) - 4a^2(b^4+c^4) - 5(b^2-c^2)^2(b^2+c^2): ... : ... ),

    con número de búsqueda en ETC: 1.0956343251291829068665975668

  • jueves, 27 de marzo del 2014

    Recta de Euler del triángulo antipedal conteniendo al circuncentro

    ( Anopolis #1268, Antreas P. Hatzipolakis)

    Sean ABC un triángulo, P un punto y A'B'C' el triángulo antipedal de P. El lugar geométrico de P tal que la recta de Euler de su triángulo antipedal pasa por el circuncentro es una séptica que pasa por: X(1), X(4), X(6), X(13), X(14), X(74), los excentros, por los pies de las alturas.

    ( Mostrar/Ocultar figura )


  • miércoles, 26 de marzo del 2014

    Cuártica lugar de centros de homotecia

    ( Anopolis #1265, Antreas P. Hatzipolakis)

    Sean ABC un triángulo, P un punto y A'B'C' el triángulo pedal de P.
    Se denota por Ab, Ac las proyecciones ortogonales de A' sobre PB',PC', respectivamente.
    l1 es el eje radical de las circunferencias Ab(AbB') y Ac(AcC'), y m1 es el eje radical de las circunferencias Ab(AbC') y Ac(AcB'). l1 y m1 son paralelos, ya que son perpendiculares a AbAc.
    Similarmente, se consideran los ejes radicales l2, l3, m2 y m3.
    Consideremos el triángulo LaLbLc delimitado por l1, l2 y L3 y el triángulo MaMbMc delimitado por m1, m2 y m3, ellos son homotéticos.

    El lugar geométrico de los puntos P que coinciden con el centro de homotecia de los triángulos LaLbLc y MaMbMc es la cuártica:

    ( Mostrar/Ocultar figura )

    Esta cuártica pasa por el baricentro (X2), por los puntos isodinámicos (X15, X16), por los puntos del infinito de la hipérbola de Kiepert (X3412, X3413) y por los pies Va, Vb, Vc y Wa, Wb, Wc de las bisectrices interiores y exteriores.

  • sábado, 22 de marzo del 2014

    Una caracterización de la cúbica de McCay

    ( Anopolis #1248, Antreas P. Hatzipolakis)

    Sean ABC un triángulo, P un punto y PaPbPc el triángulo antipedal de P.
    Se denota por Ha, Hb y Hc los ortocentros de los triángulos PBC, PCA y PAB, respectivamente.
    Y por H1, H2 y H3 los ortocentros de los triángulos PaBC, PbCA y PcAB, respectivamente.

    El lugar geométrico de los puntos P tales que los triángulos HaHbHc y H1H2H3 sean perspectivos es la cúbica de McKay,

    ( Mostrar/Ocultar figura )

    En coordenadas baricéntricas, si P=(u:v:w):
    Ha = ((SBv+SC(u+v))(SCw+SB(u+w)) : (SBv+SC(u+v))(SAu-SCw) : (SAu-SBv)(SCw+SB(u + w))),
    H1 = (-u:u+w:u+v).
    Las coordenadas de los ortocentros Hb, Hc, H2, H3 se obtienen por permutación cíclica.

    Cuando el punto P se mueve sobre la cúbica de McKay el centro de perspectividad Q de los triángulos HaHbHc y H1H2H3 está sobre la tercera cúbica de Musselman (K028).

  • viernes, 21 de marzo del 2014

    Puntos asociados a los triángulos duales de los triángulos cevianos de los puntos de Fermat

    ( DC(X(13)) y DC(X(14)), Encyclopedia of Triangle Centers)

    En ETC, justo antes del centro X2979, se da el concepto de triángulo dual:
      Sea UVW un triángulo en plano del triángulo ABC. U' el conjugado isogonal del punto del infinito de la recta VW; se definen V' y W' cíclicamente. Al triángulo U'V'W' se le denomina dual de UVW.
    Y se define una aplicación entre puntos del plano (en coordenadas baricéntricas) por:

    P=(u:v:w)   →   DC(P) = ( a^2vw/(v+w) : b^2wu/(w+u) : c^2uv/(u+ v) )
    DC(P) es el punto tal que su triángulo circunceviano es el triángulo dual del triángulo ceviano de P.

    En particular:

    DC(X13) = X2981, conjugado isogonal de X396 = punto medio de X13 y X15.
    DC(X14) (no incluido actualmente en ETC), conjugado isogonal de X395 = punto medio de X14 y X16.



    Otra interpretación geométrica de los centros DC(X13) y DC(X14):

     Dado un triángulo ABC, sea DDbDc el triángulo equilátero INSCRITO en ABC, con D, Db y Dc sobre BC, CA y AB, respectivamente, y DbDc paralelo a BC. Cíclicamente se define los triángulos equiláteros inscritos EEcEa y FFaFb.
     Las rectas DbDc, EcEa y FaFb delimitan un triángulo A'B'C' homotético a ABC y el centro de homotecia es DC(X13)= X2981:

    DC(X(13))
    ( Mostrar/Ocultar figura )

    El triángulo DEF es el ceviano de X13. Las coordenadas de los puntos Db y Dc son, respectivamente:
    (a^2(3SC+√3 S) : 0 : 2(a^2SA+SC(SB+√3 S))),   (a^2(3SB+√3 S) : 2(a^2SA+SB(SC+√3 S)) : 0).

    Los centros Oa, Ob, Oc de los triángulos equiláteros inscritos DDbDc, EEbEc, FFbFc, respectivamente, están alineados y las rectas AOa, BOb, COc concurren en X17.

    Los seis puntos Db, Dc, Ec, Ea, Fa y Fb están en una misma cónica.



     Dado un triángulo ABC, sea DDbDc el triángulo equilátero EXINSCRITO en ABC, con D, Db y Dc sobre BC, CA y AB, respectivamente, y DbDc paralelo a BC. Cíclicamente se define los triángulos equiláteros exinscritos EEcEa y FFaFb.
     Las rectas DbDc, EcEa y FaFb delimitan un triángulo A'B'C' homotético a ABC y el centro de homotecia es DC(X14):

    DC(X(14))
    ( Mostrar/Ocultar figura )

    El triángulo DEF es el ceviano de X14. Las coordenadas de los puntos Db y Dc son, respectivamente:
    (a^2(3SC-√3 S) : 0 : 2(a^2SA+SC(SB-√3 S))),   (a^2(3SB-√3 S) : 2a^2SA+2SB(SC-√3 S), 0) : 0).

    Los centros Oa, Ob, Oc son los centros de los triángulos equiláteros exinscritos DDbDc, EEbEc, FFbFc, respectivamente, están alineados y las rectas AOa, BOb, COc concurren en X18.

    Los seis puntos Db, Dc, Ec, Ea, Fa y Fb están en una misma cónica.

  • lunes, 17 de marzo del 2014

    Una interpretación de las circunferencias de Tucker

    ( ADGEOM #1165, Emmanuel José García)

    Dados un triángulo ABC y un número real t, las paralelas al lado BC a la distancia |ta| cortan a AB en Cb y C'b (hacia el exterior e interior del triángulo, respectivamente), y al lado AC en Bc y B'c (hacia el exterior e interior del triángulo, respectivamente).
    Procediendo cíclicamente, tomando las distancias tb y tc, se definen los puntos Ca, Ac, Ab y Ba, y los puntos C'a, A'c, A'b y B'a.
    Los puntos Cb, Bc, Ca, Ac, Ab y Ba están en una misma circunferencia Γt; así mismo, los puntos C'b, B'c, C'a, A'c, A'b y B'a están en una misma circunferencia Γ‍′t. Estas son circunferencias de Tucker, cambiando t por (-t), se pasa de una circunferencia a otra.

    Para obtener las coordenadas baricéntricas de los puntos anteriores, tomamos la circunferencia de centro en el punto medio de BC y radio ta:
    c^2xy+b^2xz+a^2yz +(1/4) (x+y+z) ((a^2-2b^2-2c^2+4a^2t^2)x + (-a^2+4a^2t^2)y + (-a^2+4a^2t^2)z) = 0.
    La ecuación conjunta de las tangentes a esta circunferencia paralelas a BC es:
    (a^4-2a^2b^2+b^4-2a^2c^2-2b^2c^2+c^4+4a^4t^2)x^2 + 4a^4t^2y^2 + 4a^4t^2z^2 + 8a^4t^2yz + 8a^4t^2zx + 8a^4t^2xy = 0.
    Y sus intersecciones con los lados AC y AB son:
    Ba=(2at : 0 : -bc-2at),   Ca=(2at : -bc-2at : 0),   B'a=(2at : 0 : bc-2at),   C'a=(2at : bc-2at : 0).
    Por permutación cíclica obtenemos las coordenadas del resto de los puntos.
    ( Mostrar/Ocultar figura )
    La ecuación baricéntrica de la circunferencia Γt es:
    Tucker Circle
    Se trata de una circunferencia de Tucker; en efecto, la antiparalela por Ac a BC, respecto a AB y AC es c(bc+2at)x+2actya(ab+2ctz=0, que corta a AC en Ca= (2at : -bc-2at : 0 ), y sucesivamente.

    La ecuación de la circunferencia Γ‍′t, resulta de la anterior, cambiando t por -t.
    El eje radical de las circunferencias Γt y Γ‍′t, para todo t, es la polar del simediano respecto a la circunferencia circunscrita, que corta perpendicularmente al eje de Brocard en X187, punto medio de los puntos isodinámicos.

    Casos particulares de circunferencia de Tucker:

    Circunferencia de Apolonio.   [t = (a+b+c)/4=s/2]

    Tucker Circle
    La circunferencia que toca las tres circunferencias exinscritas de un triángulo y los abarca a menudo se conoce como circunferencia de Apolonio (Clark Kimberling,- "Triangle Centers and Central Triangles.". 1998, p. 102).
    Darij Grinberg; Paul Yiu.- "The Apollonius Circle as a Tucker Circle." Forum Geom. 2(2002), 175-182.

    Circunferencia circunscrita   [t=0]

    a²yz + b²zx + c²xy = 0.


    Primera circunferencia de Lemoine  [t= -(a b c)/(2(a^2+b^2+c^2))]

    Tucker Circle
    Las paralelas a los lados de un triángulo por su simediano, cortan a sus lados es seis puntos cocíclicos situados en la denominada primera circunferencia de Lemoine.

    Circunferencia coseno o Segunda circunferencia de Lemoine [t=-(a b c)/(a^2 + b^2 + c^2)]

    Tucker Circle
    Cada antiparalela por el simediano a un lado, respecto a los otros dos, corta a estos en dos puntos. Los seis puntos así determinados son cocíclicos estando en la que se conoce como circunferencia coseno.

    Circunferencia de Gallatly  [t=-abc(a^2+b^2+c^2)/(4(a^2b^2+b^2c^2+c^2a^2))]

    Tucker Circle
    Se llama circunferencia de Gallatly de un triángulo ABC a la circunferencia pedal de sus puntos de Brocard.

    Circunferencia de Kenmotu  [t=(abc Sqrt[(-a+b+c)(a+b- c)(a-b+c)(a+b+c)] - abc(a^2+b^2+c^2)/(2(a^4+b^4+c^4))]
    Si en un triángulo ABC se inscriben tres cuadrados iguales con un vértice común, las diagonales de los cuadrados, que une vértices que están en los lados de ABC, son antiparalelas a los lados de ABC. Por eso estos seis vértices pertenecen a una circunferencia de Tucker, circunferencia de Kenmotu.

    Circunferencia de Taylor  [t=-S^2/(2abc)]

    Tucker Circle
    La circunferencia de Taylor de un triángulo es la que pasa por las proyecciones de los pies de las alturas sobre los lados.

    Circunferencia de Tucker biceviana con centro en AH  [t=-bc/(2a)]

    Tucker Circle
    Una de las seis circunferencias de Tucker que se obtiene cuando el centro está sobre la altura desde el vértice A, pasa por este vértice.
    Bernard Gibert.- "Bicevian Tucker Circles" Forum Geom. 7 (2007) 87–97.

    Circunferencia de Tucker biceviana con centro en la mediatriz de los puntos medios de AB y AC  [t=-bc/(4a)]

    Tucker Circle
    Una de las seis circunferencias de Tucker que se obtiene cuando el centro está sobre la mediatriz del lado del triángulo medial con vértices en los puntos medios de AB y AC, pasa por estos puntos medios.

  • jueves, 13 de marzo del 2014

    Triple reflexión en los lados de un triángulo

    ( Anopolis #1197, Seiichi Kirikami)

    Dados un triángulo ABC y un punto P, se denotan las reflexiones de P en BC, CA y AB por P1, P2 y P3, respectivamente.
    Sean P12 y P13 las reflexiones de P1 en CA y AB, respectivamente. Se definen P23, P21 y P31, P32 cíclicamente.
    Finalmente, se denotan las reflexiones de P21 y P31 en CA y AB por P212 P213 y P312, P313 respectivamente. Y se definen cíclicamente los puntos P323, P321, P123, P121 y P131, P132, P231, P232.

    Los puntos P1, P21, P31, P231 y P321 están en una misma circunferencia Γa, cuyo centro Oa no depende de la posición del punto P y tiene coordenadas baricéntricas (-a^2:2SC:2SB).

    De forma similar se consideran las circunferencias Γb y Γc y sus respectivos centros Ob y Oc.
    ( Mostrar/Ocultar figura )


    Si D = P321P123∩P132P231 ∈ Γa, E = P132P231∩P213P312 ∈ Γb y F = P213P312∩P321P123 ∈ Γc, los triángulos DEF son todos iguales, para cualquier P, y se tiene que:
    Area(DEF) = |SASBSC|/(a^2b^2c^2) Area(ABC)

    Los triángulos ABC y DEF son perspectivos si y solo si P queda en la hipérbola
    ( a^4 b^2 x^2-2 a^2 b^4 x^2+b^6 x^2-a^4 c^2 x^2+b^4 c^2 x^2+2 a^2 c^4 x^2-b^2 c^4 x^2-c^6 x^2-a^6 x y+3 a^4 b^2 x y-3 a^2 b^4 x y+b^6 x y-a^4 c^2 x y+b^4 c^2 x y+2 a^2 c^4 x y-2 b^2 c^4 x y-a^6 y^2+2 a^4 b^2 y^2-a^2 b^4 y^2-a^4 c^2 y^2+b^4 c^2 y^2+a^2 c^4 y^2-2 b^2 c^4 y^2+c^6 y^2+a^6 x z+a^4 b^2 x z-2 a^2 b^4 x z-3 a^4 c^2 x z+2 b^4 c^2 x z+3 a^2 c^4 x z-b^2 c^4 x z-c^6 x z+2 a^4 b^2 y z-a^2 b^4 y z-b^6 y z-2 a^4 c^2 y z+3 b^4 c^2 y z+a^2 c^4 y z-3 b^2 c^4 y z+c^6 y z+a^6 z^2+a^4 b^2 z^2-a^2 b^4 z^2-b^6 z^2-2 a^4 c^2 z^2+2 b^4 c^2 z^2+a^2 c^4 z^2-b^2 c^4 z^2 = 0)
    (H) equilátera que pasa por el ortocentro, por el simediano, por la reflexión del circuncentro en el ortocentro (X382), por el punto de reflexión de Parry (X399) y de puntos del infinito X2574 y X2575, conjugados isogonales de los puntos en los que la recta de Euler corta a la circunferencia circunscrita.

    ( Mostrar/Ocultar figura )

    El centro de perspectividad de los triángulos ABC y DEF, cuando P recorre (H), queda sobre la hipérbola de Jerabek (hipérbola equilátera circunscrita a ABC que pasa por el circuncentro).
    El centro de la hipérbola (H) es el punto de coordenadas baricéntricas:

    (a^2 (a^8(b^2+c^2) - 2a^6(b^4+c^4) + a^4b^2c^2(b^2+c^2) + a^2(b^2-c^2)^2(2b^4+b^2c^2+2c^4) - b^10 + b^8c^2 + b^2c^8 - c^10) : ... : ...),

    que tiene (6-9-13)-número de búsqueda en ETC: -6.618666655241975536408988331.

    Peter Moses obtiene que este punto está en las rectas determinadas por los pares de centros del triángulo con número de orden: {6,1205}, {23,110}, {51,125}, {52,3627}, {74,389}, {113,5562}, {185,1986}, {265,5446}, {1204,2935}, {1614,2914}, {3060,3448}. Y que se puede expresar como:
    3 X(51) - 2 X(125),   3 X(51) - 4 X(1112),   3 X(3060) - X(3448).


    Los triángulos OaObOc y DEF son perspectivos si y solo si P queda en la hipérbola de Jerabek. El centro de perspectividad queda sobre la hipérbola equilátera (H).

    ( Mostrar/Ocultar figura )


    Las tangentes en D, E y F a las circunferencias Γa, Γa y Γc son concurrentes si y solo si P recorre la cónica circunscrita a ABC con centro X5 (centro de la circunferencia de los nueve puntos).

    ( Mostrar/Ocultar figura )


    Las tangentes en D, E y F a las circunferencias Γa, Γa y Γc delimitan un triángulo no degenerado perspectivo con DEF si y solo si P recorre la cúbica de Darboux del triángulo órtico, K044 = pK(X216,X4).

    ( Mostrar/Ocultar figura )


    Las tangentes en D, E y F a las circunferencias Γa, Γa y Γc delimitan un triángulo no degenerado perspectivo con ABC si y solo si P recorre una cuártica circunscrita a ABC, que pasa por el ortocentro (doble), por X265 (reflexión del circuncentro en el centro de la hipérbola de Jerabek) y por X1986 (punto de reflexión de Hatzipolakis) y sus dos asíntotas (ortogonales) tienen la dirección de los puntos X2574 y X2575 (conjugados isogonales de los puntos de intersección de la recta de Euler con la circunferencia circunscrita).

    ( Mostrar/Ocultar figura )


    Las tangentes en D, E y F a las circunferencias Γa, Γa y Γc delimitan un triángulo no degenerado perspectivo con OaObOc si y solo si P recorre una cuártica circunscrita a ABC, que pasa por el circuncentro, ortocentro y X54 (punto de Kosnita) y sus dos asíntotas (ortogonales) tienen la dirección de los puntos X2574 y X2575 (conjugados isogonales de los puntos de intersección de la recta de Euler con la circunferencia circunscrita).

    ( Mostrar/Ocultar figura )


  • miércoles, 12 de marzo del 2014

    Triángulos ceviano y pedal de misma área

    ( Problema 700, Ricardo Barroso)

    Lugar geométrico de los puntos del plano del triángulo ABC tales que sus triángulos ceviano y pedal tiene la misma área

    Si P=(u:v:w) son las coordenadas baricéntricas de un punto del plano, las áreas de sus triángulos ceviano PaPbPc y pedal DEF son, respectivamente:
    2uvw/((u+v)(u+w)(v+w)) Area(ABC),
    (-a+b+c)(a+b-c)(a-b+c)(a+b+c)(c^2uv+b^2uw + a^2vw)/(4a^2b^2c^2(u+v+w)^2) Area(ABC)
    Estas dos cantidades son iguales si P está en la quíntica, con puntos dobles en los vértices de ABC (pudiendo ser aislados) y asíntotas los lados del triángulo antimedial, de ecuación:
    a^4 c^2 x^3 y^2 - 2 a^2 b^2 c^2 x^3 y^2 + b^4 c^2 x^3 y^2 - 2 a^2 c^4 x^3 y^2 - 2 b^2 c^4 x^3 y^2 + c^6 x^3 y^2 + a^4 c^2 x^2 y^3 - 2 a^2 b^2 c^2 x^2 y^3 + b^4 c^2 x^2 y^3 - 2 a^2 c^4 x^2 y^3 - 2 b^2 c^4 x^2 y^3 + c^6 x^2 y^3 + a^4 b^2 x^3 y z - 2 a^2 b^4 x^3 y z + b^6 x^3 y z + a^4 c^2 x^3 y z + 4 a^2 b^2 c^2 x^3 y z - b^4 c^2 x^3 y z - 2 a^2 c^4 x^3 y z - b^2 c^4 x^3 y z + c^6 x^3 y z + a^6 x^2 y^2 z - a^4 b^2 x^2 y^2 z - a^2 b^4 x^2 y^2 z + b^6 x^2 y^2 z + 8 a^2 b^2 c^2 x^2 y^2 z - 3 a^2 c^4 x^2 y^2 z - 3 b^2 c^4 x^2 y^2 z + 2 c^6 x^2 y^2 z + a^6 x y^3 z - 2 a^4 b^2 x y^3 z + a^2 b^4 x y^3 z - a^4 c^2 x y^3 z + 4 a^2 b^2 c^2 x y^3 z + b^4 c^2 x y^3 z - a^2 c^4 x y^3 z - 2 b^2 c^4 x y^3 z + c^6 x y^3 z + a^4 b^2 x^3 z^2 - 2 a^2 b^4 x^3 z^2 + b^6 x^3 z^2 - 2 a^2 b^2 c^2 x^3 z^2 - 2 b^4 c^2 x^3 z^2 + b^2 c^4 x^3 z^2 + a^6 x^2 y z^2 - 3 a^2 b^4 x^2 y z^2 + 2 b^6 x^2 y z^2 - a^4 c^2 x^2 y z^2 + 8 a^2 b^2 c^2 x^2 y z^2 - 3 b^4 c^2 x^2 y z^2 - a^2 c^4 x^2 y z^2 + c^6 x^2 y z^2 + 2 a^6 x y^2 z^2 - 3 a^4 b^2 x y^2 z^2 + b^6 x y^2 z^2 - 3 a^4 c^2 x y^2 z^2 + 8 a^2 b^2 c^2 x y^2 z^2 - b^4 c^2 x y^2 z^2 - b^2 c^4 x y^2 z^2 + c^6 x y^2 z^2 + a^6 y^3 z^2 - 2 a^4 b^2 y^3 z^2 + a^2 b^4 y^3 z^2 - 2 a^4 c^2 y^3 z^2 - 2 a^2 b^2 c^2 y^3 z^2 + a^2 c^4 y^3 z^2 + a^4 b^2 x^2 z^3 - 2 a^2 b^4 x^2 z^3 + b^6 x^2 z^3 - 2 a^2 b^2 c^2 x^2 z^3 - 2 b^4 c^2 x^2 z^3 + b^2 c^4 x^2 z^3 + a^6 x y z^3 - a^4 b^2 x y z^3 - a^2 b^4 x y z^3 + b^6 x y z^3 - 2 a^4 c^2 x y z^3 + 4 a^2 b^2 c^2 x y z^3 - 2 b^4 c^2 x y z^3 + a^2 c^4 x y z^3 + b^2 c^4 x y z^3 + a^6 y^2 z^3 - 2 a^4 b^2 y^2 z^3 + a^2 b^4 y^2 z^3 - 2 a^4 c^2 y^2 z^3 - 2 a^2 b^2 c^2 y^2 z^3 + a^2 c^4 y^2 z^3 = 0.
    Esta curva pasa por el ortocentro H, ya que sus triángulos medial y pedal coinciden.
    La tangente en el ortocentro (punto ordinario de la curva) tiene la dirección determinada por el punto del infinito X924 (conjugado isogonal de X925).
    Si el triángulo es acutángulo, el ortocentro es interior al triángulo ABC. Así existen infinitos puntos, por continuidad, en el interior de un triángulo acutángulo tales que las áreas de sus triángulos ceviano y pedal coinciden.
    ( Mostrar/Ocultar figura )


  • martes, 25 de febrero del 2014

    Centros de ortología de triángulos cevianos y antipedales

    ( r2040, Quim Castellsaguer)

    Dados un triángulo ABC y un punto Q, el lugar geométrico de los puntos P tales que los triángulos antipedal de P y ceviano de Q son ortológicos es la cónica circunscrita que pasa por Q y por su complemento.

    ( Mostrar/Ocultar figura )
    El centro de ortología X del triángulo ceviano QaQbQc de Q, respecto al triángulo antipedal DEF de P, describe la cónica circunscrita a QaQbQc, que pasa por Q y por el "crosspoint" C(X2,Q) de X2 y Q.

    Las ecuaciones de las cónicas citadas son, si (p:q:r) son las coordenadas baricéntricas de Q:
    p(q^2-r^2)yz + q(r^2-p^2)zx + r(p^2-q^2) = 0,
    qr(q-r)x^2 + rp(r-p)y^2 + pq(p-q)z^2 + p^2(q-r)yz + q^2(r-p)zx + r^2(p-q)xy = 0.
    Estas cónicas son homotéticas (¿centros de homotecia?), tienen en común los puntos Q y C(X2,Q) , y se verifica que P, X y C(X2,Q) están alineados.
    El "crosspoint" C(X2,Q) de X2 y Q, tiene esta otra interpretación geométrica: "The lines tangent to the conic {A,B,C,P,U} at P and U intersect in X. (Randy Hutson, September 10, 2012)".

    Un caso particular:
    Cuando Q es el incentro (resultado r2040 en Todo Triángulo Web de Joaquín Castellsaguer), la cónica que pasa por A, B, C, Q, cQ tiene de ecuación baricéntrica:
    a(b2-c2)yz + b(c2-a2)zx +c(a2-b2)xy = 0,

    que contiene a los centros del triángulo de números de orden: 1, 10, 19, 37, 65, 75, 82, 91, 158, 225, 267, 596, 759, 775, 876, 897, 921, 969, 994, 1247, 1581, 1910, 2153, 2154, 2166, 2168, 2186, 2190, 2214, 2216, 2217, 2218, 2219, 2363, 2588, 2589, 2652, 2962, 3668, 4674.


    Dados un triángulo ABC y un punto P, el lugar geométrico de los puntos Q tales que los triángulos antipedal de P y ceviano de Q son ortológicos es la isocúbica pivotal pK(G,P), de polo el baricentro y pivote el conjugado isotómico de P.

    Si P=(u:v:w),   pK(G,P) : vwx(y^2-z^2) + wuy(z^2-x^2) + uvz(x^2-y^2 = 0.
    ( Mostrar/Ocultar figura )
    El centro de ortología Y del triángulo ceviano QaQbQc de Q, respecto al triángulo antipedal DEF de P, describe (cuando Q recorre pK(G,P)) la isocúbica pivotal de polo el "crosspoint" C(X2,Q) de X2 y Q, y pivote el anticomplemento aP de P.

    Un caso particular:

    Dados un triángulo ABC, el lugar geométrico de los puntos Q tales que los triángulos antipedal (excentral) del incentro y ceviano de Q son ortológicos es la isocúbica pivotal K034 = pK(X2,X75), de polo el baricentro y pivote el conjugado isotómico del incentro.

    ( Mostrar/Ocultar figura )

    El centro de ortología Y, del triángulo ceviano QaQbQc de Q respecto al triángulo excentral IaIbIc, describe (cuando Q recurre pK(X2,X75)) la isocúbica pivotal K033 = pK(X37,X8).

  • jueves, 20 de febrero del 2014

    El punto X275 como centro de homotecia

    Dado un triángulo ABC, sean A' el punto diametralmente opuesto a A, D el punto de intersección de AA' con BC y A" la reflexión de A' en D.
    Los puntos Ba y Ca son las proyecciones ortogonales de A" sobre AC y AB, respectivamente. Se definen similarmente los puntos Cb, Ab, Ac y Bc.

    El triángulo UVW, delimitado por las rectas BaCa, CbAb y AcBc, es homotético a ABC y su centro de homotecia es X275.

    X275 = (1/(SA (SB SC + S^2)) : 1/(SB (SC SA + S^2)) : 1/(SC (SA SB + S^2))).


    El centro de homotecia de UVW y A'B'C' (que ha de estar en la recta X3X275) es el punto de coordenadas baricéntricas:

    Y = (SB SC (3 a^4 SA^4 - SA^3 (SB^3 - 3 a^2 SB SC + SC^3) - SA^2 SB SC (7 SB^2 + 16 SB SC + 7 SC^2) - 8 a^2 SA SB^2 SC^2 - 2 SB^3 SC^3):...:...),

    con número de búsqueda en ETC: 25.2481990527062715026144
    ( Mostrar/Ocultar figura )


    Además, los seis puntos Ba, Ca, Cb, Ab, Ac y Bc están en una misma cónica de ecuación baricéntrica:


  • miércoles, 19 de febrero del 2014

    Dos triángulos de misma área

    Dado un triángulo ABC, los triángulos A1B1C1 y A2B2C2 tales que

    A1B1/B1A = b/c,   B1C1/C1B = c/a,   C1A1/A1C = a/b,  
    A2B2/B2A = c/b,   B2C2/C2B = a/c,   C2A2/A2C = b/a,  
    tienen la misma área igual a:
    abc/(ab(a+b)+bc(b+c)+ca(c+a)+abc)  área(ABC)

    ( Mostrar/Ocultar figura )



  • miércoles, 12 de febrero del 2014

    Construcción del triángulo: O Ma Hb

    ( Construcción de triángulos #176) 4th Irish Mathematical Olympiad 4 May 1991, Paper 1

    Construir un triángulo conociendo su circuncentro y los pies de la mediana y la altura desde dos vértices distintos.

    ( Mostrar/Ocultar figura )

    Utilizando un sistema de coordenadas respecto al que O(0,0) y Ma(0,-m), sean Hb(α,β) y un punto variable B'(t,-m) en la perpendicular por Ma a OMa.
    La perpendicular por Hb a HbB' corta a la circunferencia de centro en O y radio OB' en los puntos A' y B', que describen un lugar geométrico que corta a la perpendicular por Ma a OMa en los vértices B y C del triángulo ABC a construir. Las coordenadas de estos puntos son (±(α2+(m+β)2)½,-m). Así, pueden ser construidos con regla y compás. Una vez construidos B y C, la construcción del vértice A es inmediata. Existen DOS SOLUCIONES.

  • jueves, 30 de enero del 2014

    Perspectividad con los triángulos medial y órtico

    (ADGEOM #1085 Emmanuel Garcia)

      Dado un triángulo ABC, con longitud de lados a=BC, b=CA y c=AB, sean |at|, |bt| y |ct| las distancias de los lados de ABC a los lados del triángulo A'B'C', tal que at=bt=ct (t número real). El circuncentro y el ortocentro son los únicos puntos del plano tales que sus triángulo pedales son perspectivos con A'B'C', para todo número real t.

    Relativo al triángulo ABC, sean I el incentro, r el radio de la circunferencia inscrita y LMN el triángulo de contacto interior. La recta pa situada a una distancia |at| del lado BC es la paralela a dicho lado por el punto que resulta de aplicar una homotecia de centro en I y razón (r+at)/r al punto L. Esta recta pa tiene ecuación baricéntrica (similarmente, las rectas pb y pc):

    pa:  (a+b+c)r+a^2t)x + a^2ty + a^2tz = 0
    pa:  b^2tx + (a+b+c)r+b^2t)y + b^2tz = 0
    pa:  c^2tx + c^2ty + (a+b+c)r+c^2t)y = 0.

    Los vértices del triángulo A'B'C' delimitado por las rectas pa, pb y pc son:

    A' = ((a+b+c)r + (b^2+c^2)t : -b^2t : -c^2t),
    B' = (-a^2 t : (a+b+c)r + (c^2+a^2)t : -c^2t},
    C' = (-a^2t : -b^2 t : (a+b+c)r + (a^2+b^2)t).

      Nótese que el centro de homotecia de ABC y A'B'C' es el simediano (a^2:b^2:c^2). La razón de homotecia es:
    1 + (a^2+b^2+c^2)t / ((a+b+c)r).
    lo que nos da otra vía para obtener los triángulos A'B'C'


    Dado un punto P(u:v:w) los vértices de su triángulo pedal son:

    D = (0 : (b^2-c^2)u + a^2(u+2v) : (c^2-b^2)u + a^2(u+2 w)),
    E = ((a^2-c^2)v + b^2(v+2u) : 0 : (c^2-a^2)v + b^2(v+2w)),
    F = ((a^2-b^2)w + c^2(w+2u), (b^2-a^2)w + c^2(w+2v) : 0).

    Imponiendo que los triángulos A'B'C' y DEF sean perspectivos se llega a que las coordenadas de P deben satisfacer a un polinomio de tercer grado en la variable t. La anulación de los coeficientes de este polinomio nos permite afirmar que P ha de estar en la cúbica de Darboux (para que el término independiente se anule), sobre la recta de Euler (para que el coeficiente del término de segundo grado se anule).
    Los único puntos comunes de la recta de Euler y la cúbica de Darboux son el circuncentro y el ortocentro.
    Como los puntos P que anulan al coeficiente del término de primer grado del polinomio están en una cónica que pasa por el circuncentro y el ortocentro, queda establecida la afirmación inicial. Es decir:

      Los únicos triángulos pedales que son perspectivos a todos los triángulos homotéticos a ABC, mediante las homotecias de centro en el simediano, son los triángulos medial y órtico.



    • El lugar geométrico de los centros de perspectividad del triángulo órtico y cada triángulo imagen de ABC en una homotecia de centro en el simediano, es la hipérbola equilátera circunscrita al triángulo órtico y que pasa por el ortocentro.
    Esta hipérbola, con centro en X(1112), pasa por los puntos X(4), X(6), X(52), X(113), X(155), X(185), X(193), X(1162), X(1163), X(1829), X(1839), X(1843), X(1858), X(1986), X(2574), X(2575), X(2904), X(2905), X(2906), X(2907), X(2914), X(3574), X(5095).
    ( Mostrar/Ocultar figura )


    • El lugar geométrico de los centros de perspectividad del triángulo medial y cada triángulo imagen de ABC en una homotecia de centro en el simediano, es la recta que pasa por el baricentro y simediano.
    ( Mostrar/Ocultar figura )


  • miércoles, 29 de enero del 2014

    El centro X(468)

      Dado un triángulo ABC y un punto variable P sobre la recta de Euler, el lugar geométrico de los centros de la hipérbola equilátera circunscrita al triángulo pedal de P y que pasa por P es una recta, que corta a la recta de Euler en X468.

    ( Mostrar/Ocultar figura )
    La ecuación baricéntrica de la recta lugar geométrico de los centros de las hipérbolas que se tratan es:

    (b^2-c^2)(-a^10 + 2a^8(b^2+c^2) - 5a^6b^2c^2 + a^4(-2b^6+3b^4c^2+3b^2c^4-2c^6) + a^2(b^2-c^2)^2(b^4-b^2c^2+c^4) + 3b^2c^2(b^2-c^2)^2(b^2 + c^2))x+ ... = 0.

    Esta recta pasa por los centros X(468) (sobre la recta de Euler), por X(1112) y cuyo punto del infinito es X(511) (conjugado isogonal del punto de Tarry, cuarto punto de intersección de la circunferencia circunscrita y la hipérbola de Kiepert).

    X(1112) es el centro de la hipérbola cuando P es el ortocentro.
    X(468) es el centro de la hipérbola cuando P es X(186), el inverso del ortocentro en la circunferencia circunscrita



  • martes, 28 de enero del 2014

    El centro X(2165)

      Dado un triángulo ABC, sean TaTbTc su triángulo tangencial y DEF el triángulo circunceviano del ortocentro. La tangente en D a la circunferencia circunscrita corta a TaTb en Db y a TaTc en Dc.
      Los puntos Tb, Tc, Db y Dc están en una circunferencia Γa y denotamos por da el eje radical de esta circunferencia y la circunferencia circunscrita.
      Similarmente se definen los ejes radicales db y dc.

    ABC y el triángulo A'B'C', delimitado por da, db y dc, son perspectivos, con centro de perspectividad en X2165.

    ( Mostrar/Ocultar figura )


  • jueves, 23 de enero del 2014

    La isocúbica pK(X2, X6)

      Dado un triángulo ABC y un punto P, sean Pr y Pl sus brocardianos (Paul Yiu.- Introduction to the Geometry of the Triangle, 8.4 pag, 106), PcbPacPba y PbcPcaPab los triángulos cevianos de Pr y Pl, respectivamente.

    La isocúbica pivotal pK(X2, X6) es el lugar geométrico de los puntos P tales que las mediatrices de PcbPbc, PacPca y PbaPab son concurrentes.

    Esta caracterización geométrica de pK(X2, X6) le ha motivado a Bernard Gibert a asignarle el número K659 es su catálogo de cúbicas asociadas a un triángulo.
    ( Mostrar/Ocultar figura )
    Esta cúbica, de ecuación baricéntrica:

    a^2 x (y^2 - z^2) + b^2 y (z^2 - x^2) + c^2 z (x^2 - y^2) = 0,

    pasa por los vértices de ABC y de su triángulo antimedial y por los centros X(2), X(6), X(76), X(194), X(2998).

    NOTAS sobre cónicas bicevianas de los brocardianos de un punto.

    Construcción de los brocardianos:
    Si PaPbPc es el triángulo ceviano de P, el punto Pcb es la intersección de BC con la paralela a AC por Pc y el punto Pbc es la intersección de BC con la paralela a AB por Pb. Los puntos Pac, Pca, Pba y Pab se definen similarmente.

    •   La cónica biceviana de los brocardianos de un punto P es una circunferencia solo cuando P= X194.

    Este hecho geométrico está descrito en el párrafo 2.3 de Bernard Gibert.- Tucker cubics and Bicentric Cubics.
    Si (u:v:w) son las coordenadas baricéntricas de P, la ecuación de la cónica biceviana de los brocardianos de P es:

    vwx^2 + wuy^2 + uvz^2 -(u^2 + vw)yz -(v^2 + wu)zx -(w^2 + uv)xy = 0.

    Una cónica de ecuación baricéntrica f x^2 + g y^2 + h z^2 + 2 p y z + 2 q z x + 2 r x y = 0 es una circunferencia cuando

    (g + h -2p)/a^2 = (h + f -2q)/b^2 = (f + g -2r)/c^2.

    Así, la cónica biceviana que nos ocupa es circunferencia cuando:

    (u + v)(u + w)/a^2 = (v + w)(v + u)/b^2 = (w + u)(w + v)/c^2.

    Resolviendo estas ecuaciones obtenemos que las coordenadas (u:v:u) son proporcionales a (a^2(b^2+c^2)-b^2c^2 : b^2(c^2+a^2)-c^2a^2 : c^2(a^2+b^2)-a^2b^2), que son las el centro X194.
    La circunferencia biceviana de los brocardianos de X194 tiene centro con coordenadas:

    (a^2 (a^4 (b^4-4 b^2 c^2+c^4)-a^2 (b^6-4 b^4 c^2-4 b^2 c^4+c^6)+b^2 c^2 (b^4-4 b^2 c^2+c^4)) : ... : ...),

    que tiene (6-9-13)-número de búsqueda en ETC: 0.1830166211743119241902625514

    •   La cónica biceviana de los brocardianos de un punto P tienen su centro en P si y sólo si P está en la elipse circunscrita de Steiner.
    ( Mostrar/Ocultar figura )

    Si P está sobre la elipse circunscrita de Steiner sus brocardianos están en la recta del infinito.

    •   Los puntos A' = BPac∩CPab, B' = CPba∩APbc, C' = APca∩BPca están alineados si y solo si P está sobre la elipse circunscrita de Steiner
    ( Mostrar/Ocultar figura )

    Cuando P está sobre la elipse circunscrita de Steiner, la recta p que contiene a los puntos A', B', C' también pasa por el baricentro. Si tP es la tangente en P a la elipse y Q = p∩tP, el área de ABC es cuatro veces el área del triángulo ceviano de Q. Por tanto, Q describe la cúbica T(1/4) = K015 (ver párrafo 3.1 de Bernard Gibert.- Tucker cubics and Bicentric Cubics.)

  • lunes, 20 de enero del 2014

    Conjeturas

      Dado un triángulo ABC y un punto P, existen DOS triángulos isósceles con baricentro A, el ángulo desigual de amplitud la del ángulo en el vértice A, y los vértices iguales sobre las cevianas BP y CP.
      Sean A1B1C1 y A2B2C2 tales triángulos, con ∠B1A1C1 = ∠B2A2C2 = ∠BAC, B1 y B2 sobre BP, C1 y C2 sobre CP, y A el baricentro de A1B1C1 y de A2B2C2.
      Los triángulos isósceles A1B1C1 y A2B2C2 son inversamente semejantes. Si A'1 es la reflexión de A1 en B1C1, los triángulos A'1B1C1 y A2B2C2 son directamente semejantes; denotamos por A' su centro de semejanza directa. Similarmente se consideran los centros de semejanza B' y C'.

    1)   Para cualquier P, las circunferencias (BB'∩CC',B,C), (CC'∩AA',C,A) y (AA'∩BB',A,B) son concurrentes en un punto Q.

    ( Mostrar/Ocultar figura )

    2)   Si P recorre la circunferencia circunscrita a ABC, las circunferencias (BB'∩CC',B,C), (CC'∩AA',C,A) y (AA'∩BB',A,B) concurren en un punto fijo Q0, con (6-9-13)-número de búsqueda en ETC: 19.52051544033555

    ( Mostrar/Ocultar figura )

    3)   Existe un único punto P0 sobre la circunferencia circunscrita tal que AA', BB' y CC' son concurrentes (en el punto Q0).

    ( Mostrar/Ocultar figura )


  • jueves, 02 de enero del 2014

    Rectas perpendiculares por un punto que cortan a dos rectas dadas

    (ADGEOM #1014 Dao Thanh Oai)

    Sean un punto D y dos rectas p y q, entonces la recta que pasa por los puntos medios de los dos segmentos que pares de rectas perpendiculares por D determinan sobre p y q, pasa por un mismo punto al variar las rectas perpendiculares.

    ( Mostrar/Ocultar figura )

    El punto fijo X asociado a D, p, q es el punto de intersección de las mediatrices de los segmentos que las rectas perpendiculares por D a p y q, determinan sobre éstas.

    Damos dos aplicaciones de este resultado a la geometría del triángulo:

    1a) Dado un triángulo ABC y un punto P, los puntos medios de los segmentos que dos rectas perpendiculares variables que pasan por A determinan al cortar cada una a las rectas PB y PC, definen un recta que pasa por un punto fijo Pa.

    Pa es el punto de intersección de la mediatriz del segmento que la perpendicular por A a PB determina sobre las rectas PB y PC con la mediatriz del segmento que la perpendicular por A a PC determina sobre las rectas PB y PC.
    Se define de forma similar los puntos Pb y Pc, procediendo cíclicamente sobre los vértices de ABC.

    El lugar geométrico de los puntos P tales que ABC y PaPbPc son perspectivos es la hipérbola de Kiepert, y el centro de perspectividad Q es el punto donde la paralela por P a la recta de Euler vuelve a cortar a la hipérbola de Kiepert.

    ( Mostrar/Ocultar figura )
    Si P=(1/(SA+t) : 1/(SB+t) : 1/(SC+t)) es un punto genérico de la hipérbola de Kiepert, entonces el centro de perspectividad Q tiene coordenadas baricéntricas

    Q=(1/(S²+SAt) : 1/(S²+SBt) : 1/(S²+SCt)).

    Pares de centros {P,Q} sobre la hipérbola de Kiepert que figuran actualmente en ETC, expresados por sus correspondientes índices {i,j}, P=Xi, Q=Xj:
    {2, 4}, {4, 2}, {13, 13}, {14, 14}, {98, 671}, {262, 598}, {485, 1327}, {486, 1328}, {598, 262}, {671, 98}, {1327, 485}, {1328, 486}, {3424, 5485}, {3429, 4052}, {4052, 3429}, {5485, 3424}

    Algún par {P,Q}, con P y Q centros en la hipérbola de Kiepert, tales que Q no figura actualmente en la ETC:

    {X10, (1/(2 S^2-3(a^2+b^2+c^2+2(ab+ac+bc))SA):...:...)}, (6-9-13)-número de búsqueda en ETC: -7.75319920991533803152915369

    {X17, (1/(S+3Sqrt[3]SA):...:...)}, (6-9-13)-número de búsqueda en ETC: -1.860007742164222715329129330

    {X18, (1/(S-3Sqrt[3]SA):...:...)}, (6-9-13)-número de búsqueda en ETC: 126.1499312579877710386653603

    {X76, (1/(3(SA²-SBSC)+2S²):...:...)}, (6-9-13)-número de búsqueda en ETC: -16.87874108766941076512556987

    {X83, (1/(3(SA²-SBSC)+4S²):...:...)}, (6-9-13)-número de búsqueda en ETC: -2.887934280286545741662150280

    {X94, (1/((S² - 3 SA²) (S² + 3 SBSC)):...:...)}, (6-9-13)-número de búsqueda en ETC: 0.3900113547977998375532972499

    { (1/(3(SA²-SBSC)-2S²):...:...)} (1/(3 a^2 - 2 b^2 - 2 c^2):...:...), }, (6-9-13)-números de búsqueda del segundo en ETC: 4.63650295237392362290626430 y 4.1268565215227781921162084412, respectivamente,

    { (1/(3(SA²-SBSC)-4S²):...:...), (1/(a^2-6SA):...:...) }, el primero con (6-9-13)-números de búsqueda en ETC: 2.98578607000337615936536037 y el segundo es el conjugado isotómico de X3630.

    { (1/(a^2 + 6 SA):...:...), (1/(5 S^2 + 3 (SA+SB+SC)SA):...:...)}, el primero es el conjugados isotómico de X3631 y (6-9-13)-número de búsqueda del segundo en ETC: 0.0063415316860299433902901912.

    { (1/(5 S^2 - 3 (SA+SB+SC)SA):...:...), (1/(3 a^2 - 2 b^2 - 2 c^2)):...:...), }, (6-9-13)-números de búsqueda del segundo en ETC: 4.63650295237392362290626430 y 4.1268565215227781921162084412, respectivamente,

    2a) Dado un triángulo ABC y un punto P, los puntos medios de los segmentos que dos rectas perpendiculares variables que pasan por P determinan al cortar cada una a las rectas AB y AC, definen un recta que pasa por un punto fijo Pa.

    Pa es el punto de intersección de la mediatriz del segmento que la perpendicular por P a AB determina sobre las rectas AB y AC con la mediatriz del segmento que la perpendicular por P a AC determina sobre las rectas AB y AC.
    Se define de forma similar los puntos Pb y Pc, procediendo cíclicamente sobre los vértices de ABC.

    El lugar geométrico de los puntos P tales que ABC y PaPbPc son perspectivos es la cúbica de Darboux (K004), y el centro de perspectividad Q queda sobre la cúbica de Thomson (K002).

    ( Mostrar/Ocultar figura )


    OTRA FORMA DE ENUNCIAR ESTA PROPIEDAD GEOMÉTRICA DE LA CÚBICA DE DARBOUX:

    Sean ABC un triángulo, un punto P y DEF su triángulo pedal. La recta PE corta a AB en E' y PF corta a AC en F'. Los puntos E, F, E' y F' están en una circunferencia de centro Pa. Se definen similarmente los puntos Pb y Pc. El lugar geométrico de los puntos P tales que ABC y PaPbPc son perspectivos es la cúbica de Darboux (K004), y el centro de perspectividad Q queda sobre la cúbica de Thomson (K002).

    Si P=(p:q:r) es un punto de la cúbica de Darboux, entonces el centro de perspectividad Q, en la cúbica de Thomson, tiene coordenadas baricéntricas

    Q=(a^4(p^2-2(q-r)^2+p(q+r)) + 2a^2(b^2(p(3r-q)+(q+r)^2)+c^2(p(3q-r)+(q+r)^2)) - (b^2-c^2)^2p(p-q-r) : ... : ... ).

    Pares de centros {P,Q} sobre las cúbicas de Darboux y Thomson que figuran actualmente en ETC, expresados por sus correspondientes índices {i,j}, P=Xi, Q=Xj:
    {1, 1}, {3, 6}, {4, 2}, {20, 4}, {40, 57}, {64, 3}, {84, 9}, {1490, 282}, {1498, 1073}, {2130, 3349}, {2131, 3350}, {3182, 3342}, {3183, 3344}, {3345, 223}, {3346, 1249}, {3347, 3341}, {3348, 3343}, {3353, 3352}, {3354, 3351}, {3355, 3356}, {3472, X?}, {3473, X?}, {3637, X?}.

  • sábado, 28 de diciembre del 2013

    Construcción del triángulo: ha ma A=B

    ( Construcción de triángulos #174)

    Construir un triángulo sabiendo que es isósceles y conociendo la altura y la mediana desde uno de los vértices de ángulos iguales.

    ( Mostrar/Ocultar figura )

    Construimos el triángulo AHaMa tal que los lados AHa y AMa sean de las longitudes dadas ha y ma, respectivamente, y que el ángulo en Ha es recto.
    Reflejamos el punto A en Ma, obteniéndose el punto A'.
    Las tangentes desde A' a la parábola de foco A y directriz HaMa, cortan a la recta HaMa en los puntos C1 y C2. Las reflexiones de estos puntos en Ma dan los puntos B1 y B2.
    Los triángulos AB1C1 y AB2C2 son dos soluciones al problema propuesto.

  • jueves, 19 de diciembre del 2013

    Cónica pasando por las reflexiones de un punto respecto a los lados de un cuadrivértice

    ( Quadri-Figures-Group #391 Randy Hutson)

    Sean ABC un triángulo, Q un punto, y el cuadrivértice ℘ = P1P2P3P4=GPaPbPc, formado por el baricentro y los vértices de triángulo antimedial.
    Consideremos las reflexiones de Q respecto de los lados del cuadrivértice ℘:

    Qij es la reflexión de Q en las rectas PiPj para todo i,j en {1,2,3,4}, i<j.

    El lugar geométrico de los puntos Q tales que sus reflexiones Qij, en los lados de ℘, estén en una misma cónica es una séptica que tiene a los vértices de ℘ como puntos singulares aislados, contiene a los vértices del triángulos diagonal y a los vértices(1) del triángulo de Miquel (QA-Tr2 ) de ℘, pasa por X316, Droussent pivot (QA-P4 "Isogonal Center") y tiene como punto del infinito al conjugado isogonal del foco X110 de la parábola de Kiepert: X523 ("Involutory Conjugate" de QA-P2=X99).

    ( Mostrar/Ocultar figura )

    Si Q=X316 la cónica que pasa por sus seis reflexiones respecto a los lados del cuadrivértice ℘ = GPaPbPc contiene a X316 y tiene por ecuación: Con centro es QA-P2=X99, punto de Steiner.

    (1) The vértices of QA-Tr2 can be constructed as the 2nd intersection point of the circles (QA-P4,Pi,Pj) and (QA-P4,Pk,Pl), where (i,j,k,l) = (1,2,3,4) / (1,3,2,4) / (1,4,2,3).

  • viernes, 6 de diciembre del 2013

    Una descripción geométrica del centro X691

    Sean ABC un triángulo y P un punto. Consideremos los dos triángulos equiláteros AAbAc y AA'bA'c tales que Ab y A'b están en la ceviana BP, Ac y A'c están en la ceviana CP (ver HG03113). Denotamos por A' el punto de intersección de AbAc y A'bA'c. Se definen cíclicamente los puntos B' y C'.
    Las rectas AA', BB', CC' son paralelas.


    Si P varía sobre la circunferencia circunscrita, el lugar geométrico del punto A' es una circunferencia Γa que pasa por A y su centro Oa tiene coordenadas baricéntricas:

    Oa = ( -2 (a^2 - b^2) (a^2 - c^2) (b^2 - c^2), b^2 (a^2 - b^2) (a^2 + b^2 - 2 c^2), c^2 (-a^2 + c^2) (a^2 - 2 b^2 + c^2) : ... : ... ).

    Similarmente, se obtienen sendas circunferencias Γb y Γc, con centros Ob y Oc , al proceder cíclicamente. Las rectas AOa, BOb, COc concurren en el centro X691.
    (a^2/((b^2-c^2)(2a^2-b^2-c^2)) : b^2/((c^2-a^2)(2b^2-c^2-a^2)) : c^2/((a^2-b^2)(2c^2-a^2-b^2)) ).

    Si P es un punto sobre la circunferencia circunscrita, sea Q el conjugado isogonal del punto del infinito determinado por las rectas paralelas AA', BB', CC'. Las rectas PQ son paralelas, cuando P varía; el punto del infinito que determina sus direcciones es X542 y el punto del infinito determinado por la dirección perpendicular a las de las rectas PQ es X690, cuyo conjugado isogonal es X691.
    ( Mostrar/Ocultar figura )

    Las tangentes en A, B, C a las circunferencias Γa, Γb, Γc, respectivamente concurren en X842, antipodal de X691.

  • miércoles, 4 de diciembre del 2013

    Puntos conjugados isogonales y centros ortológicos

    ( Anopolis #1135 Antreas P. Hatzipolakis)

    Sean ABC un triángulo, P un punto, P* su conjugado isogonal, A'B'C' y A"B"C" los triángulos pedales de P y P*, respectivamente.
    Denotamos por:
    Ab la intersección de la paralela a AP* por A' con AB.
    Ac la intersección de la paralela a AP* por A' con AC.
    A2 la intersección de la paralela a AP por A" con AB.
    A3 la intersección de la paralela a AP por A" con AC.
    Los puntos Ab, Ac, A2 y A3 están en una circunferencia, por construcción. Sea Oa su centro. Similarmente se definen Ob y Oc.

    El lugar geométrico de los puntos P tales que los triángulos ABC y OaObOb son ortológicos es una curva algebraica de grado seis, invariante por isoconjugación, que pasa por el circuncentro, ortocentro, por los puntos X1113 y X1114 (en los que la recta de Euler corta a la circunferencia circunscrita) y por los conjugados de éstos, X2574, X2575, en la recta del infinito, de ecuación baricéntrica:

    a^4 c^6 x^3 y^3 - b^4 c^6 x^3 y^3 - a^2 c^8 x^3 y^3 + b^2 c^8 x^3 y^3 - a^4 b^4 c^2 x^4 y z + a^2 b^6 c^2 x^4 y z + a^4 b^2 c^4 x^4 y z - a^2 b^2 c^6 x^4 y z - a^6 b^2 c^2 x^3 y^2 z - a^4 b^4 c^2 x^3 y^2 z + 2 a^2 b^6 c^2 x^3 y^2 z + 3 a^4 b^2 c^4 x^3 y^2 z - 2 b^6 c^4 x^3 y^2 z - 3 a^2 b^2 c^6 x^3 y^2 z + b^4 c^6 x^3 y^2 z + b^2 c^8 x^3 y^2 z - 2 a^6 b^2 c^2 x^2 y^3 z + a^4 b^4 c^2 x^2 y^3 z + a^2 b^6 c^2 x^2 y^3 z + 2 a^6 c^4 x^2 y^3 z - 3 a^2 b^4 c^4 x^2 y^3 z - a^4 c^6 x^2 y^3 z + 3 a^2 b^2 c^6 x^2 y^3 z - a^2 c^8 x^2 y^3 z - a^6 b^2 c^2 x y^4 z + a^4 b^4 c^2 x y^4 z - a^2 b^4 c^4 x y^4 z + a^2 b^2 c^6 x y^4 z + a^6 b^2 c^2 x^3 y z^2 - 3 a^4 b^4 c^2 x^3 y z^2 + 3 a^2 b^6 c^2 x^3 y z^2 - b^8 c^2 x^3 y z^2 + a^4 b^2 c^4 x^3 y z^2 - b^6 c^4 x^3 y z^2 - 2 a^2 b^2 c^6 x^3 y z^2 + 2 b^4 c^6 x^3 y z^2 + a^8 c^2 x y^3 z^2 - 3 a^6 b^2 c^2 x y^3 z^2 + 3 a^4 b^4 c^2 x y^3 z^2 - a^2 b^6 c^2 x y^3 z^2 + a^6 c^4 x y^3 z^2 - a^2 b^4 c^4 x y^3 z^2 - 2 a^4 c^6 x y^3 z^2 + 2 a^2 b^2 c^6 x y^3 z^2 - a^4 b^6 x^3 z^3 + a^2 b^8 x^3 z^3 - b^8 c^2 x^3 z^3 + b^6 c^4 x^3 z^3 - 2 a^6 b^4 x^2 y z^3 + a^4 b^6 x^2 y z^3 + a^2 b^8 x^2 y z^3 + 2 a^6 b^2 c^2 x^2 y z^3 - 3 a^2 b^6 c^2 x^2 y z^3 - a^4 b^2 c^4 x^2 y z^3 + 3 a^2 b^4 c^4 x^2 y z^3 - a^2 b^2 c^6 x^2 y z^3 - a^8 b^2 x y^2 z^3 - a^6 b^4 x y^2 z^3 + 2 a^4 b^6 x y^2 z^3 + 3 a^6 b^2 c^2 x y^2 z^3 - 2 a^2 b^6 c^2 x y^2 z^3 - 3 a^4 b^2 c^4 x y^2 z^3 + a^2 b^4 c^4 x y^2 z^3 + a^2 b^2 c^6 x y^2 z^3 - a^8 b^2 y^3 z^3 + a^6 b^4 y^3 z^3 + a^8 c^2 y^3 z^3 - a^6 c^4 y^3 z^3 + a^6 b^2 c^2 x y z^4 - a^2 b^6 c^2 x y z^4 - a^4 b^2 c^4 x y z^4 + a^2 b^4 c^4 x y z^4=0.
    ( Mostrar/Ocultar figura )
    Si P=X3 o P=X4 el centro ortológico de OaObOb respecto a ABC es

    ( a^10 - 2 a^8 (b^2 + c^2) + a^6 (b^4 + c^4) - a^4 (b^2 - c^2)^2 (b^2 + c^2) + 2 a^2 (b^8 - b^6 c^2 - b^2 c^6 + c^8) - (b^2 - c^2)^4 (b^2 + c^2) : ... : ... ),

    que tiene (6-9-13)-número de búsqueda en ETC: 6.74586117499516838898883187
    Si P=X3 o P=X4 el centro ortológico de ABC respecto a OaObOb es

    ( a^2/(2 a^10 - 5 a^8 (b^2 + c^2) + 2 a^6 (2 b^4 + b^2 c^2 + 2 c^4)- 2 a^4 (b^2 - c^2)^2 (b^2 + c^2) + 2 a^2 (b^8 - b^6 c^2 - b^2 c^6 + c^8)- (b^2 - c^2)^4 (b^2 + c^2) ) : ... : ... ),

    que tiene (6-9-13)-número de búsqueda en ETC: 146.543260333090931926435953

  • sábado, 30 de noviembre del 2013

    Caracterización de la cuártica de Stammler

    ( Anopolis #1124 Antreas P. Hatzipolakis)

    Sean ABC un triángulo y Q un punto. El lugar geométrico de los puntos P tales que la tripolar de P y la recta QP* (P* conjugado isogonal de P) son perpendiculares es una cuártica C(Q) con puntos nodales en los vértices de ABC, que pasa por el baricentro y por los puntos de intersección, X1113 y X1114, de la recta de Euler con la circunferencia circunscrita.

    Si Q(p:q:r) (coordenadas baricéntricas) la ecuación de la cuártica C(Q) es:

    Esta cuártica es la transformada isogonal de la hipérbola rectangular H(Q) de ecuación:

    Las direcciones de sus asíntotas son las mismas para todo Q, dadas por los conjugados isogonales de X1113 y X1114.

    Casos particulares:

    Q el incentro. El lugar geométrico es la cuártica C(X1) que pasa por los centros X1, X2, X29, X280, X1113, X1114 y X1222, de ecuación:

    ( Mostrar/Ocultar figura )

    Q el baricentro. El lugar geométrico es la cuártica C(X2) que pasa por los centros X2, X4, X6, X253, X523, X1113, X1114, X1222, X2346, de ecuación:

    ( Mostrar/Ocultar figura )

    Q el circuncentro. El lugar geométrico es la cuártica de Stammler (Q066), que pasa por los centros X1, X2, X4, X254, X1113, X1114, X1138, X2184, X3223, X3346 y X3459, de ecuación:

    ( Mostrar/Ocultar figura )

    Q el ortocentro. El lugar geométrico es la recta de Euler (junto con los lados de ABC).



    Una propiedad geométrica de la quíntica de Euler-Morley:

    "La quíntica de Euler-Morley (Q003, Bernard Gibert) es el lugar geométrico de los puntos Q tales la cuártica C(Q) pasa por Q".

    ( Mostrar/Ocultar figura )

    En este caso, la hipérbola H(Q) conjugada isogonal de la cuártica C(Q) pasa, además de por Q y X6, por Q*, que ahora también está en C(Q).
    De esta hipérbola H(Q) conocemos tres puntos (X6, Q, Q*) y las direcciones de las asíntotas (dadas por los conjugados isogonales de los puntos, X1113 y X1114, en los que la recta de Euler corta a la circunferencia circunscrita), por tanto puede ser construida ( IIPPP )

  • viernes, 29 de noviembre del 2013

    Ejes radicales de circunferencias centradas en los vértices y una centrada en el centro radical de las anteriores

    (ADGEOM #893, Dao Thanh Oai )

    a Lolilla, por su "cumple"

    Sean ABC un triángulo, A(Ra), B(Rb), C(Rc) circunferencias centradas en los vértices A, B, C, de radios Ra, Ra, Ra, respectivamente. Si P es el centro radical de estas circunferencias, consideremos la circunferencia P(Rp) centrada en P y de radio Rp y los ejes radicales ea, eb, ec de P(Rp) con las circunferencias A(Ra), B(Rb), C(Rc), respectivamente. Entonces:

    El triángulo A1B1C1, delimitado por ea, eb y ec es perspectivo con ABC

    ( Mostrar/Ocultar figura )
    El centro radical P de las circunferencias A(Ra), B(Rb), C(Rc) tiene coordenadas baricéntricas:

    P = ( a4 - a²(b²+c²-2Ra²+Rb²+Rc²) - (b²-c²)(Rb²-Rc²): ... : ... ).


    y el centro de perspectividad D de A1B1C1 y ABC es:

    D = ( 1/(-a^2 b^4 c^2 + b^6 c^2 - a^2 b^2 c^4 - 2 b^4 c^4 + b^2 c^6 + a^6 Ra^2 - 3 a^4 b^2 Ra^2 + 3 a^2 b^4 Ra^2 - b^6 Ra^2 - 3 a^4 c^2 Ra^2 - 2 a^2 b^2 c^2 Ra^2 + b^4 c^2 Ra^2 + 3 a^2 c^4 Ra^2 + b^2 c^4 Ra^2 - c^6 Ra^2 - a^2 b^2 Ra^4 + b^4 Ra^4 - a^2 c^2 Ra^4 - 2 b^2 c^2 Ra^4 + c^4 Ra^4 + 2 a^2 b^2 c^2 Rb^2 + 2 b^4 c^2 Rb^2 - 2 b^2 c^4 Rb^2 + 2 a^2 b^2 Ra^2 Rb^2 - 2 b^4 Ra^2 Rb^2 + 2 b^2 c^2 Ra^2 Rb^2 - a^2 b^2 Rb^4 + b^4 Rb^4 + b^2 c^2 Rb^4 + 2 a^2 b^2 c^2 Rc^2 - 2 b^4 c^2 Rc^2 + 2 b^2 c^4 Rc^2 + 2 a^2 c^2 Ra^2 Rc^2 + 2 b^2 c^2 Ra^2 Rc^2 - 2 c^4 Ra^2 Rc^2 - 4 b^2 c^2 Rb^2 Rc^2 - a^2 c^2 Rc^4 + b^2 c^2 Rc^4 + c^4 Rc^4 - a^6 Rp^2 + 3 a^4 b^2 Rp^2 - 3 a^2 b^4 Rp^2 + b^6 Rp^2 + 3 a^4 c^2 Rp^2 - 2 a^2 b^2 c^2 Rp^2 - b^4 c^2 Rp^2 - 3 a^2 c^4 Rp^2 - b^2 c^4 Rp^2 + c^6 Rp^2) : ... : ... ).


    Si se dejan fijas las circunferencias A(Ra), B(Rb), C(Rc) y se varía el radio Rp, el punto D queda en la hipérbola rectangular circunscrita a ABC que pasa por P. La ecuación de tal hipérbola es:

    (b^2 - c^2 + Rb^2 - Rc^2) (a^4 - a^2 b^2 - a^2 c^2 + 2 a^2 Ra^2 - a^2 Rb^2 - b^2 Rb^2 + c^2 Rb^2 - a^2 Rc^2 + b^2 Rc^2 - c^2 Rc^2) y z + ... = 0.



    Algunos casos particulares

    Hipérbola de Feuerbach   Ra² = bc(b+c-a)/(a+b+c), Rb² = ca(a -b+c)/(a+b+c), Rc² = ab(a+b-c)/(a+b+c).

    Hipérbola de Jerabek   Ra=Rb=Rc = R (radio de la circunferencia circunscrita).

    Hipérbola de Kiepert   Ra² = 1/9 (-a^2+2b^2+2c^2), Rb² = 1/9 (2a^2-b^2+2c^2), Rc² = 1/9 (2a^2+2b^2-c^2).

    Estos valores de los radios, tomados para las circunferencias centradas en los vértices, no son los únicos posibles para describir las citadas hipérbolas. Para la hipérbola de Feuerbach, por ejemplo, se ha tomado las distancias de los vértices al incentro, que está en dicha hipérbola; pero podemos tomar la distancia de los vértices a cualquier otro punto que esté en la hipérbola, como radios de tales circunferencias.

    Así, en general, si tomamos como radios de las circunferencia centradas en los vértices las distancias a un punto de coordenadas baricéntricas (u:v:w), la ecuación de la hipérbola donde está el punto D es:

    u((a^2-b^2+c^2)v - (a^2+b^2-c^2)w)yz + v((a^2+b^2-c^2)w - (-a^2+b^2+c^2)w)zx + w((-a^2+b^2+c^2)u - (a^2-b^2+c^2)v)xy = 0.



    Otros casos de circunferencias fijas centradas en los vértices

    Circunferencias de radios las longitudes de los lados opuestos. Le corresponde la hipérbola que pasa por los centros X4, P=X20, X253, X1249, X1294, X3346 y X3668, Su ecuación es:
    (b^2-c^2) (3 a^4 - 2 a^2 b^2 - b^4 - 2 a^2 c^2 + 2 b^2 c^2 - c^4)yz+ ... = 0.

    ( Mostrar/Ocultar figura )

    Circunferencias tangentes a los lados opuestos del vértice en el que está centrada. La correspondiente hipérbola rectangular contiene a los centros X4, X389, X577 y tiene por ecuación:
    a^4 (b^2 - c^2)(a^2 - b^2 - c^2)^2 (a^6 b^2 - 3 a^4 b^4 + 3 a^2 b^6 - b^8 + a^6 c^2 - 3 a^2 b^4 c^2 + 2 b^6 c^2 - 3 a^4 c^4 - 3 a^2 b^2 c^4 - 2 b^4 c^4 + 3 a^2 c^6 + 2 b^2 c^6 - c^8)yz+ ... = 0.

    Circunferencias de radios los ex-inradios El centro de perspectividad D está en la hipérbola que contiene a los centros X4, X282, X1034, X1490, X3176, X3341, X3347. Tiene en común con la cúbica de Thomson los seis puntos A, B, C, X4, X282 y X3341. Su ecuación es:
    a (a - b - c) (b - c) (a^6 - 2 a^5 b - a^4 b^2 + 4 a^3 b^3 - a^2 b^4 - 2 a b^5 + b^6 - 2 a^5 c - 2 a^4 b c + 2 a b^4 c + 2 b^5 c - a^4 c^2 + 2 a^2 b^2 c^2 - b^4 c^2 + 4 a^3 c^3 - 4 b^3 c^3 - a^2 c^4 + 2 a b c^4 - b^2 c^4 - 2 a c^5 + 2 b c^5 + c^6)yz+ ... = 0.



  • miércoles, 27 de noviembre del 2013

    Una caracterización de la cúbica de Thomson

    ( K002 Bernard Gibert)

    Sean ABC un triángulo, P un punto y A'B'C' el triángulo circunceviano de P. Las rectas BC, CA, AB intersecan a las rectas B'C', C'A', A'B' en 9 puntos, de los cuales 6 no están en la tripolar p del producto ceviano(1) de P y K (simediano); estos seis puntos quedan en una cónica C(P) cuya ecuación baricéntrica, si (u:v:w) son las coordenadas de P, es:

    Σ
    abc uvw xyz
    a^2v^2w^2(c^2v+b^2w)x^2 - uvw(2b^2c^2u^2 + a^2u(c^2v+b^2w) + a^4vw)yz = 0.
    ( Mostrar/Ocultar figura )
    El centro de la cónica C(P) es el punto Q, de coordenadas:

    Q = ( u(a^2v(a^2w-c^2(2v+w)) - b^2(2c^2u(v+w)+a^2w(v+2w))) : ... : ... ).

    El lugar geométrico de los puntos P tales que P, Q, K son distintos y están alineados es la cúbica de Thomson.

    ( Mostrar/Ocultar figura )

    Nota: Si el punto P está sobre la circunferencia circunscrita, su triángulo circunceviano no está definido y si P está sobre la tripolar del simediano (eje de Lemoine) la cónica C(P) degenera en el producto de dos rectas que se cortan en el simediano.
    ( Mostrar/Ocultar figura )


    Los puntos P (no situados en el eje de Lemoine) que son centros del triángulo y figuran actualmente en ETC, a los que corresponde centros Q de la cónica C(P), que también figuran en ETC, son los siguientes pares {P,Q}:

    {X1, X1001}, {X3, X3}, {X4, X6}, {X6, X5024}, {X513, X999}, {X523, X381}, {X2574, X3}, {X2575, X3}.

    El centro de la cónica C(X2) es:

    ( a^4-3a^2(b^2+c^2)-4b^2c^2 : b^4-3b^2(c^2+a^2)-4c^2a^2 : c^4-3c^2(a^2+b^2)-4a^2b^2 ),

    que tiene (6-9-13)-número de búsqueda en ETC: 3.332127762248135206469312965

    (1) Producto ceviano de P y U (o Cevapunto de P y U) es el tripolo de la polar de P (resp, U) en la cónica circunscrita de perspector U (resp. P).

  • martes, 26 de noviembre del 2013

    Incentro, exincentros y circunferencias concurrentes

    ( Anopolis #1120 Antreas P. Hatzipolakis)

    Sean ABC un triángulo, I el incentro y Ia, Ib, Ic los exincentros. Denotemos por N1, N2, N3, los centros de las circunferencias de los nueve puntos de los triángulos IBC, ICA, IAB y por Na, Nb, Nc, los centros de las circunferencias de los nueve puntos de los triángulos IaBC, IbCA, IcAB.

    Las circunferencias circunscritas a los triángulos N1BC, N2CA, N3AB, concurren en X502.

    Las circunferencias circunscritas a los triángulos NaBC, NbCA, NcAB, concurren en X5620, de coordenadas baricéntricas:

    ( (b+c)(a^6 - a^4(b^2+c^2) - a^2(b^4-3b^2c^2+c^4)- 2abc(b-c)^2(b+c) + (b-c)^4(b+c)^2) : ... : ... ).

    ( Mostrar/Ocultar figura )

    •  Información complementaria:
    Las rectas IaNa, IbNb y IcNc concurren en X191.
    Las rectas IaN1, IbN2 y IcN3 concurren en X5506.


    •  Randy Hutson da una construcción alternativa del punto X5620 en Anopolis #1132:

    Sea DEF el triángulo incentral (triángulo ceviano del incentro), Se considera el triángulo BCA' semejante a DEF, construido hacia el interior de ABC. El punto A' es el segundo puntos de intersección de la recta IaNa con la circunferencia circunscrita al triángulo NaBC.
    Se definen B' y C' cíclicamente. Entonces la rectas AA', BB', CC' concurren en X5620.

    Las coordenadas baricéntricas de A' son:
    A' = (2a^2(a+b)(a+c) : -(a+c)(a^3+a^2(b-c)+(b-c)^2(b+c)+a(b^2-b c-c^2)) : -(a+b)(a^3+a^2(c-b)+(b-c)^2(b+c)-a(b^2+b c-c^2)) ).

    ( Mostrar/Ocultar figura )


    •  Caso en que los triángulos semejantes al triángulo incentral son construidos externamente:
    Sea DEF el triángulo incentral (triángulo ceviano del incentro), Se considera el triángulo BCA" semejante a DEF, construido hacia el exterior de ABC.
    Se definen B" y C" cíclicamente. Entonces la rectas AA", BB", CC" concurren en X502. (Randy Hutson, 9/23/2011)

    ( Mostrar/Ocultar figura )
    Los triángulos A"B"C" y IaIbIc son perspectivos, con centro de perspectividad:

    Z = ( a(a^6+2a^5(b+c) + a^4(b^2+4b c+c^2) - a^2(b^4+2b^3c+b^2c^2+2b c^3+c^4) - 2a(b^5+b^4c-b^3c^2-b^2c^3+b c^4+c^5) - (b-c)^2(b+c)^4) : ... : ... )

    que tiene (6-9-13)-número de búsqueda en ETC: -10.786517909750622287908

    Randy Hutson ( Anopolis #1134) nota que este punto es el isogonal conjugado de X399 en el triángulo excentral, IaIbIc.

  • lunes, 25 de noviembre del 2013

    "Central line" y cuártica asociada

    (§ 1.3 Remark 5 Bernard Gibert.- Bicevian Conics)

    Sean ABC un triángulo, δ una recta, D su tripolo, M un punto en δ, C(M) la cónica biceviana de D y M (que pasa por los vértices de los triángulos cevianos DaDbDc y MaMbMc de D y M) y, finalmente, Mδ el centro de C(M).
    El lugar geométrico de Mδ, cuando M varía, es una cuártica que pasa por los puntos de intersección de δ con los lados de ABC.

    Si px+qy+rz=0 es la ecuación baricéntrica de la recta δ, su cuarto punto de intersección con la cuártica tiene de coordenadas:

    Q(δ) = ( (q - r) (2 q r + p (q + r)) : (r - p) (2 r p + q (r + p)) : (p - q) (2 p q + r (p + q)) )

    Este punto es el centro de la cónica C(M), donde M es el punto del infinito de la recta δ. La ecuación de la cónica C(M) es (Bernard Gibert.- Bicevian Conics § 1.3 (1) ):

    p(p-q)(r-p)x^2 + q(q-r)(p-q)y^2 + r(r-p)(q-r)z^2 + p(q-r)^2yz + q(r-p)^2zx + r(p-q)^2xy = 0.

    ( Mostrar/Ocultar figura )

    EJEMPLOS: Alguna "central line" y su correspondiente cuarto punto de intersección con la cuártica asociada. La "central line" la denotamos por Li si sus coeficientes con las coordenadas baricéntricas del centro del triángulo Xi.

    "central line" δ ecuación / nombre centro asociado Q(δ) nº de búsqueda
    L1 ax+by+cz=0 ( (b-c)(2bc+a(b+c)) :...:... ) 137.478425435838696608231248
    L3 a^2SAx+...=0 ( (b^2-c^2)SA(a^6(b^2+c^2)-2a^4(b^4+b^2c^2+c^4)+ a^2(b^2-c^2)^2(b^2+ c^2)+2b^2c^2(b^2-c^2)^2):...:... ) 34.10580608877537813269670609
    L4 SBSCx+ ... =0 X2525
    L7 (a-b+c)(a+b-c)x+ ... =0 X3700
    L75 "Antiorthic Axis" X4979
    L76 "Lemoine axis" ( a^2(b^2-c^2)(2a^2+b^2+c^2):..:... ) 15.27801656267128914852474955
    L99 (c^2-a^2)(a^2-b^2)x+ ... =0 X1648
    L190 (c-a)(a-b)x+ ... =0 X1647
    L514 "Nagel Line" ( (2a-b-c) (2a^2-2a(b+c)+b^2+c^2):..:... ) -41.5918336266397249053218231
    L525 "Euler Line" ( (2a^4-a^2(b^2+c^2)-(b^2-c^2)^2) (2a^8-2a^6(b^2+c^2))- a^4(b^4-4 b^2c^2+c^4)+(b^2-c^2)^2(b^4+c^4):..:... ) -2.926469376504842758944837077
    L648 x/((b^2-c^2)SA)+ ... =0 X1650
    L668 x/(a(b-c))+ ... =0 X1646
    L670 x/(a^2(b^2-c^2))+ ... =0 X1645
    L671 x/(2a^2-b^2-c^2)+ ... =0 X1649
    L850 "Brocard axis" ( a^2(a^2(b^2+c^2)-b^4-c^4) (2a^6- 2a^4(b^2+c^2)-a^2(b^4-4b^2c^2+c^4)+(b^2-c^2)^2(b^2+c^2)):..:... ) -3.75510528847191427972334279
    L3239 "Soddy line" ( (2a^3-a^2(b+c)-(b-c)^2(b+c)) (-2(a-b)(a-c)(a+b-c)^2(a-b+c)^2 + (b-c)(-a+b+ c)^2((a-b)(a+b-c)^2-(a-c)(a-b+c)^2)):..:... ) -3.2612053200015515644411874
    L3265 "VanAubel line" ( (2 a^6 - a^4 (b^2 + c^2) - (b^2 - c^2)^2 (b^2 + c^2)) (-2 (a - b) (a + b) (a - c) (a + c) (a^4 - (b^2 - c^2)^2)^2 + (b - c) (b + c) (-a^2 + b^2 + c^2)^2 (3 a^4 (b^2 - c^2) - (b^2 - c^2)^3 - 2 a^2 (b^4 - c^4))):..:... ) -2.740416687177399065884132982
    L3268 "Fermat axis" ( (2 a^6 - 2 a^4 (b^2 + c^2) - (b^2 - c^2)^2 (b^2 + c^2) + a^2 (b^4 + c^4)) (-2 (a - b) (a + b) (a - c) (a + c) (a^2 - a b + b^2 - c^2) (a^2 + a b + b^2 - c^2) (a^2 - b^2 - a c + c^2) (a^2 - b^2 + a c + c^2) - (b - c) (b + c) (b^2 - c^2) (-a^2 + b^2 - b c + c^2) (-a^2 + b^2 + b c + c^2) (-2 a^4 + (b^2 - c^2)^2 + a^2 (b^2 + c^2))):..:... ) -2.39457958479639858720527989
    L4373 x/(b+c-3a)+ ... =0 X4394


  • viernes, 22 de noviembre del 2013

    Punto en la circunferencia circunscrita

    ( r1958, Quim Castellsaguer)

    Sean un triángulo ABC, I el incentro; Na, Nb , Nc los centros de las circunferencias de los nueve puntos de IBC, ICA, IAB. Las circunferencias {ANbNc}, {BNcNa}, {CNaNb} concurren en un punto de la circunferencia circunscrita.


    El centro de la circunferencia de los nueve puntos del triángulo IBC tiene por coordenadas baricéntricas:

    Na = (-a (b + c) : a^2 + a b - (b + c)^2 : a^2 + a c - (b + c)^2).

    Permutando cíclicamente estas coordenadas se obtienen las de los puntos Nb y Nc.
    La ecuación de la circunferencia circunscrita al triángulo ANbNc es:

    c^2 x y + b^2 x z + a^2 y z + (x + y + z) (-(((a^4 c - a^3 b c - a^2 b^2 c + a b^3 c + 2 a^3 c^2 - b^3 c^2 + b^2 c^3 - 2 a c^4 + b c^4 - c^5) y)/( 2 (a^3 + a^2 b - a b^2 - b^3 + a^2 c - a b c + b^2 c - a c^2 + b c^2 - c^3))) - ((-a^4 b - 2 a^3 b^2 + 2 a b^4 + b^5 + a^3 b c - b^4 c + a^2 b c^2 - b^3 c^2 - a b c^3 + b^2 c^3) z)/( 2 (-a^3 - a^2 b + a b^2 + b^3 - a^2 c + a b c - b^2 c + a c^2 - b c^2 + c^3)))=0.

    Por permutación cíclica obtenemos las ecuaciones de las circunferencias circunscritas a los triángulos BNcNa y CNaNb.

    El punto de concurrencia de estas tres circunferencias es:

    (a/((b - c) (a^3 - a^2 (b + c) - a (b^2 + b c + c^2) + (b + c)^3)) : ... : ... ),

    que tiene (6-9-13)-número de búsqueda en ETC: 0.2921043592404663185080644011
    ( Mostrar/Ocultar figura )


  • jueves, 21 de noviembre del 2013

    Parábolas con focos los vértices del triángulo circunceviano del simediano y directrices las medianas

    Sean ABC un triángulo. Se construyen triángulos semejantes variables ABBa, rectángulo en B, y ACCa, rectángulo en C, tales que θ = ∠BABa = - ∠CACa (ángulos orientados). La envolvente de las rectas BaCa es una parábola ℘a.

    En efecto, la correspondencia Ba (θ) ↦ Ca (-θ), entre puntos de la recta perpendicular a AB por B y los de la recta perpendicular a AC por C, es una proyectividad; por lo que, las rectas BaCa envuelven un cónica, tangente a estas rectas y también al lado BC en un punto Ta, ya que BC. Finalmente, como los puntos del infinito de cada recta se corresponden, la recta del infinito es también tangente: se trata de una parábola.

    La ecuación baricéntrica de ℘a es:

    4 b²c²x² + SB²y² + SC²z² + 2 SB SC yz + 4 b²SB zx + 4 c² SCxy = 0,

    su directriz es la mediana por A y su foco es el punto D donde la simediana por A vuelve a cortar a la circunferencia circunscrita a ABC.

    Los puntos de tangencia de la parábola ℘a con las rectas perpendiculares a AB y a AC por B y C, respectivamente, están en la altura por A; el punto Ta de tangencia con el lado BC es:

    Ta = (0 : a^2 + b^2 - c^2 : -a^2 + b^2 - c^2).


    ( Mostrar/Ocultar figura )
      HG211113.png
    Descargar fichero GeoGebra

    Procediendo cíclicamente, se obtienen las parábolas ℘b y ℘c. Se verifica:

    Las parábolas ℘a, ℘b y ℘c son tangentes a los lados de ABC en puntos alineados, sobre la tripolar del ortocentro X4.

    Gracias a Viktor Kitaysky, por una corrección



  • miércoles, 20 de noviembre del 2013

    Bisectrices de un triángulo y los centros X(171) y X(238)

    Sean ABC un triángulo y Wa el pie de la bisectriz exterior en A. Denotamos por Ωa la única cónica circunscrita que tiene las mismas tangentes desde Wa que la circunferencia circunscrita, su ecuación baricéntrica es:

    b c x (y + z) + a^2 y z = 0.

    Es tangente en A al lado (y+z=0) del triángulo antimedial y pasa por el punto Ta de intersección de la bisectriz interior en A con la mediatriz de BC (cuarto punto de intersección de Ωa con la circunferencia circunscrita).

    Para la construcción de la cónica Ωa ver: Cónica: dado tres puntos y dos tangentes (PPPtt) o bien (PPPtP), (PPPtP(1)).

    ( Mostrar/Ocultar figura )

    Análogamente, procediendo cíclicamente sobre los vértices de ABC, se consideran la cónicas Ωb y Ωc. Tomamos el cuarto punto Qa de intersección de las cónicas Ωb y Ωc y similarmente definimos los puntos Qb y Qc, entonces:

    Los triángulos ABC y QaQbQc son perspectivos con centro de perspectividad el punto X238.

    (a (a^2 - b c) : b (b^2 - c a) : c (c^2 - a b))

    ( Mostrar/Ocultar figura )

    La tangente ta en Ta a la cónica Ωa es:
    b(b^2 c + 2 b c^2 + c^3) x + a^2 c^2 y + a^2 b^2 z =0.

    Al tomar, de forma análoga, las tangentes tb y tc, se forma un triángulo perspectivo con ABC y el centro de perspectividad es el punto X985:

    (a/(b^2+b c+c^2) : b/(c^2+c a+a^2) : c/(a^2+a b+b^2))

    ( Mostrar/Ocultar figura )


    •   La cónica Ωa puede ser descrita como un lugar geométrico de la siguiente forma:

    Sean M un punto variable sobre la circunferencia circunscrita y Wa el pie de la bisectriz exterior en A, entonces el punto de intersección de la recta MWa con la recta simétrica de AM, respecto a la bisectriz en A, describe la cónica Ωa.
    ( Mostrar/Ocultar figura )


    •   Un estudio similar se puede hacer si se considera el pie Va de la bisectriz interior en A:

    Sean M un punto variable sobre la circunferencia circunscrita y Va el pie de la bisectriz interior en A, entonces el punto de intersección de la recta MVa con la recta simétrica de AM, respecto a la bisectriz en A, describe la cónica Ψa.

    La cónica Ψa está circunscrita al triángulo ABC, es tangente en A al lado del triángulo antimedial y pasa por el punto T'a de intersección de la bisectriz exterior en A con la circunferencia circunscrita. Su ecuación baricéntrica es:

    b c x (y + z) - a^2 y z = = 0.

    Similarmente, se definen la cónicas Ψb y Ψc.
    Tomamos el cuarto punto Pa de intersección de las cónicas Ψb y Ψc y similarmente definimos los puntos Pb y Pc, entonces:

    Los triángulos ABC y PaPbPc son perspectivos con centro de perspectividad el punto X171.

    (a (a^2 + b c) : b (b^2 + c a) : c (c^2 + a b))

    ( Mostrar/Ocultar figura )


    La tangente t'a en T'a a la cónica Ψa es:
    b(-b^2 c + 2 b c^2 - c^3) x + a^2 c^2 y + a^2 b^2 z = 0.

    Al tomar, de forma análoga, las tangentes t'b y t'c, se forma un triángulo perspectivo con ABC y el centro de perspectividad es el punto X983:

    (a/(b^2-b c+c^2) : b/(c^2-c a+a^2) : c/(a^2-a b+b^2))

    ( Mostrar/Ocultar figura )


  • martes, 12 de noviembre del 2013

    Curva de Durán Loriga

    en recuerdo de Ricardo Mariño Caruncho, mi profesor.
    1912-2003, La Coruña
    Escritor, periodista, Licenciado en ciencias exactas, estudioso y admirador de la obra de Juan Jacobo Durán Loríga.
    Era hijo de Pedro Mariño Ortega el gran Arquitecto Municipal entre cuyas obras destaca el Ayuntamiento de La Coruña.
    Otras facetas de su vida además de las matemáticas son sus conocimientos lingüísticos:inglés,francés en este idioma escribe un libro "Geometrie Euclidianne Abstraite" publicación muy valorada en el campo matemático europeo.
    Obras suyas son además: Elementos de la geometría vectorial, Notas sobre Juan Jacobo Durán-Loríga y Estudio genealógico sobre Juan Jacobo Durán Loríga.

    Sea ABC un triángulo, para todo punto P situado en una recta d que pasa por el baricentro, el lugar geométrico de los puntos del plano del triángulo igualmente iluminados por tres focos de igual intensidad, colocados en sus vértices, que por otro de intensidad suma, situado en P, tiene una asíntota perpendicular a la recta d.


    El lugar geométrico descrito es el de los puntos Q del plano del triángulo, tales que el cuadrado de la distancia de Q a P sea media armónica de los cuadrados de las distancias de Q a sus vértices.

    3/QP² = 1/QA² + 1/QB² + 1/QC².

    El lugar geométrico es un curva algebraica de grado cinco, cuya ecuación consta de gran número de sumandos (junto con la recta del infinito).
    La curva corta a la recta del infinito en los puntos cíclicos (dobles) y en los puntos cuyas coordenadas baricéntricas, si (u:v:w) son las P, tienen la expresión:

    ( 3a^2(v-w) + (b^2-c^2)(2u-v-w) : 3b^2(w-u) + (c^2-a^2)(2v-w-u) : 3c^2(u-v) + (a^2-b^2)(2w-u-v) ).

    Este punto está en la dirección perpendicular a la de la recta GP.

    En el caso particular de P=G (baricentro), la quíntica se descompone en la recta del infinito y la cuártica de ecuación:

    a^2 b^2 c^2 x^4 - 2 b^4 c^2 x^4 - 2 b^2 c^4 x^4 + a^4 c^2 x^3 y - 4 a^2 b^2 c^2 x^3 y + b^4 c^2 x^3 y - 3 a^2 c^4 x^3 y + 9 b^2 c^4 x^3 y + 2 c^6 x^3 y - 2 a^4 c^2 x^2 y^2 + 2 a^2 b^2 c^2 x^2 y^2 - 2 b^4 c^2 x^2 y^2 + 11 a^2 c^4 x^2 y^2 + 11 b^2 c^4 x^2 y^2 - 11 c^6 x^2 y^2 + a^4 c^2 x y^3 - 4 a^2 b^2 c^2 x y^3 + b^4 c^2 x y^3 + 9 a^2 c^4 x y^3 - 3 b^2 c^4 x y^3 + 2 c^6 x y^3 - 2 a^4 c^2 y^4 + a^2 b^2 c^2 y^4 - 2 a^2 c^4 y^4 + a^4 b^2 x^3 z - 3 a^2 b^4 x^3 z + 2 b^6 x^3 z - 4 a^2 b^2 c^2 x^3 z + 9 b^4 c^2 x^3 z + b^2 c^4 x^3 z + a^6 x^2 y z - 6 a^4 b^2 x^2 y z + 8 a^2 b^4 x^2 y z - 3 b^6 x^2 y z - 6 a^4 c^2 x^2 y z + 8 a^2 b^2 c^2 x^2 y z + b^4 c^2 x^2 y z + 8 a^2 c^4 x^2 y z + b^2 c^4 x^2 y z - 3 c^6 x^2 y z - 3 a^6 x y^2 z + 8 a^4 b^2 x y^2 z - 6 a^2 b^4 x y^2 z + b^6 x y^2 z + a^4 c^2 x y^2 z + 8 a^2 b^2 c^2 x y^2 z - 6 b^4 c^2 x y^2 z + a^2 c^4 x y^2 z + 8 b^2 c^4 x y^2 z - 3 c^6 x y^2 z + 2 a^6 y^3 z - 3 a^4 b^2 y^3 z + a^2 b^4 y^3 z + 9 a^4 c^2 y^3 z - 4 a^2 b^2 c^2 y^3 z + a^2 c^4 y^3 z - 2 a^4 b^2 x^2 z^2 + 11 a^2 b^4 x^2 z^2 - 11 b^6 x^2 z^2 + 2 a^2 b^2 c^2 x^2 z^2 + 11 b^4 c^2 x^2 z^2 - 2 b^2 c^4 x^2 z^2 - 3 a^6 x y z^2 + a^4 b^2 x y z^2 + a^2 b^4 x y z^2 - 3 b^6 x y z^2 + 8 a^4 c^2 x y z^2 + 8 a^2 b^2 c^2 x y z^2 + 8 b^4 c^2 x y z^2 - 6 a^2 c^4 x y z^2 - 6 b^2 c^4 x y z^2 + c^6 x y z^2 - 11 a^6 y^2 z^2 + 11 a^4 b^2 y^2 z^2 - 2 a^2 b^4 y^2 z^2 + 11 a^4 c^2 y^2 z^2 + 2 a^2 b^2 c^2 y^2 z^2 - 2 a^2 c^4 y^2 z^2 + a^4 b^2 x z^3 + 9 a^2 b^4 x z^3 + 2 b^6 x z^3 - 4 a^2 b^2 c^2 x z^3 - 3 b^4 c^2 x z^3 + b^2 c^4 x z^3 + 2 a^6 y z^3 + 9 a^4 b^2 y z^3 + a^2 b^4 y z^3 - 3 a^4 c^2 y z^3 - 4 a^2 b^2 c^2 y z^3 + a^2 c^4 y z^3 - 2 a^4 b^2 z^4 - 2 a^2 b^4 z^4 + a^2 b^2 c^2 z^4=0.


    Si el triángulo ABC es equilátero (que suponemos de lado unidad) y P es su centro, la ecuación de la cuártica queda:

    SumaCiclica[x^4 - 2 x^3 (y + z) - 3 x^2 y z - 3 y^2 z^2]=0

    ( Mostrar/Ocultar figura )
    A esta cuártica se le ha dado por llamar curva de Durán-Loriga, en honor al matemático gallego que la estudió:
    DURÁN LORIGA, J. J. (1909): Sobre un problema de física, Revista de la Real Academia de Ciencias Exactas, Físicas y Naturales de Madrid, VIII: 242-271. (PDF)

    En este artículo la ecuación de esta cuártica viene expresada de la forma (α, β y γ las coordenadas baricéntricas de un punto cualquiera):

    S4 - 6ωS2 + 9Sαβγ + 3 ω2=0,
    S = α+β+γ      ω = αβ+βγ+γα.

    Durán-Loriga
    Juan Jacobo Durán Loriga
    (Fuente: http://www.culturagalega.org/albumdaciencia/detalle.php?id=246)


    En la gráfica que se expone a continuación se muestran de las quínticas DL(X1) y DL(X8) , que corresponde a los caso en que P es el incentro y el punto de Nagel, respectivamente. Sus asíntotas, que tienen la dirección del punto X3667, son perpendiculares a la recta X1X8, que pasa por el baricentro, X2.
    ( Mostrar/Ocultar figura )


  • lunes, 11 de noviembre del 2013

    Circunferencias con diámetros sobre los lados de un triángulo y la cúbica de Lucas

    Sean ABC un triángulo, P un punto no situado en sus lados y DEF el triángulo ceviano de P. Las circunferencias de diámetros AE y AF se vuelven a cortar en A'. Los puntos B' y C' se definen cíclicamente. Los triángulos ABC y A'B'C' son perspectivos si y solo si P está en cúbica de Lucas (K007). El centro de perspectividad queda en la cúbica de Darboux.

    ( Mostrar/Ocultar figura )

    Si P=(u:v:w), en coordenadas baricéntricas respecto a ABC, las coordenadas de A' son:

    ( u(SAu(v-w)² + SC(u+v)w² + SBv²(u+w)) : vw(SC(u+v)w + SAu(-v+w)) : vw(SAu(v-w) + SBv(u+w)) ).


    Si Ha, Hb y Hc son los ortocentros de los triángulos AEF, BFD y CDE, respectivamente, el centro de perspectividad de ABC y HaHbHc es también el punto Q, cuando P varía en la cúbica de Lucas. (Ver el mensaje ADGEOM #437 de Nikos Dergiades, y K645, en catálogo de cúbicas de Bernard Gibert).

  • domingo, 03 de noviembre del 2013

    Triángulos equiláteros con vértices en tres cevianas

    ( ADGEOM #693)

    Sean ABC un triángulo y P un punto.
    Consideremos los tres triángulos equiláteros AAbAc, BBcBa y CCaCb, tales que Ba y Ca están en la ceviana AP, Cb y Ab están en la ceviana BP, y Ac y Bc están en la ceviana CP.
    El punto Ab es la intersección de la recta BP con la recta pac que resulta de girar PC alrededor de A un ángulo de 60° en el sentido antihorario. El punto Ac es la intersección de la recta CP con la recta pab que resulta de girar BP alrededor de A un ángulo de 60° en el sentido horario. Procediendo cíclicamente, se construyen los puntos Bc, Ba, Ca y Cb.

    Los centros Oa, Ob y Oc de estos tres triángulos equiláteros están alineados con P

    ( Mostrar/Ocultar figura )
    Si (u:v:w) son las coordenadas baricéntricas de P, la recta L(P) que contiene a los centros de los triángulos equiláteros tiene ecuación:

    (a^2vw(v-w)-b^2w(v^2+uw+vw)+c^2v(w^2+uv+vw))x + (b^2wu(w-u)-c^2u(w^2+vu+wu)+a^2w(u^2+vw+wu))y + (c^2uv(u-v)-a^2v(u^2+wv+uv)+b^2u(v^2+wu+uv))z = 0

    Ocurre además que el triángulo A'B'C' delimitado por las rectas AbAc, BcBa y CaCb es equilátero.

    Cuando el punto P recorre la circunferencia circunscrita, la recta L(P) pasa por el punto fijo X110, foco de la parábola de Kiepert.
    ( Mostrar/Ocultar figura )

    Existe un único punto Ω, sobre la circunferencia circunscrita, para el cual las rectas AbAc, BcBa y CaCb son concurrentes, y sus coordenadas baricéntricas son:

    Ω = (1 / ((b²-c²)(3√3 b²c²SA+S(S²+9SA²))) : ... : ... ).

    Este centro tiene primera coordenada trilineal exacta en el triángulo de ETC: 0.00000387325998676449736 y ha sido incluido en ETC con el nombre X(5618) = "1st Montesdeoca Equilateral Triangles Point".

    [Peter Moses, December 4, 2013]. X5618 es el punto medio de X13 (punto de Fermat) y X5623 (cociente ceviano del punto del infinito de la recta de Euler y X13).
    X5623 es el punto de intersección de las rectas AbAc, BcBa, CaCb, cuando P=X5618.
    ( Mostrar/Ocultar figura )

    El lugar geométrico de los puntos P para los cuales las rectas AbAc, BcBa y CaCb son concurrentes es una curva algebraica de grado ocho cuya ecuación es bastante complicada.
    ( Mostrar/Ocultar figura )

    Invirtiendo los sentidos de los giros para construir el triángulo equilátero con un vértice en A y los otros dos vértices en las cevianas que no parten de A (así como para los que tienen uno de sus vértices en B o en C), obtenemos la descripción del punto X5619 = "2nd montesdeoca equilateral triangles point"

    X5619 = (1 / ((b²-c²)(3√3 b²c²SA-S(S²+9SA²))) : ... : ... ).

    Sea el punto A'b de intersección de BP con la recta qac que resulta de girar CP alrededor de A un ángulo de 60° en el sentido de las agujas del reloj. Y el punto A'c de intersección de CP con la recta qab que resulta de girar BP alrededor de A un ángulo de 60° en el sentido antihorario. El triángulo AA'bA'c resulta ser equilátero.
    Se definen los puntos B'c, B'a, C'a y C'b cíclicamente.

    X5619 es el único punto sobre la circunferencia circunscrita para el que las rectas A'bA'c, B'cB'a, C'aC'b son concurrentes.

    [Peter Moses, December 4, 2013]:   X5619 es el punto medio de X14 (segundo punto de Fermat) y X5624 (cociente ceviano del punto del infinito de la recta de Euler y X14).
    X5624 es el punto de intersección de las rectas A'bA'c, B'cB'a, C'aC'b, cuando P=X5619.
    ( Mostrar/Ocultar figura )

    La figura siguiente expone los triángulos AAbAc y AA'bA'c simultáneamente, para un punto P arbitrario en el plano.
    ( Mostrar/Ocultar figura )


  • sábado, 2 de noviembre del 2013

    In Memoriam (José María Pedret)

    ( TriangulosCabri #693, Ricardo Barroso )

    Sean ABC un triángulo y Q un punto. Encontrar los puntos P sobre la ceviana AQ tales que si Pb y Pc son los pies de las cevianas BP y CP, respectivamente, se cumpla que APb=APc.


    Los cuadrados de las distancias de A a Pb y a Pc son, respectivamente,

    (b^2 w^2)/(u + w)^2,      (c^2 v^2)/(u + v)^2.

    Por lo que, APb = APc si P está en una de las dos cónicas circunscritas a ABC con centro en el punto medio de BC y tangentes en A a las bisectrices interior y exterior, de ecuaciones respectivas:

    cy(x+z) + bz(x+y) = 0,    cy(x+z) - bz(x+y) = 0.

    Así, las solución del problema planteado son los dos puntos A1 y A2: segundos puntos de intersección de la ceviana AQ con cada una de las cónicas consideradas:

    A1 = (-(b+c)qr : q(cq+br) : r(cq+br)),    A2 = ((b-c)qr : q(cq-br) : r(cq-br)).

    En el caso de que Q=H (ortocentro) :

    A1 = (-a^4+(b^2-c^2)^2 : (a+b-c)(a-b+c)(a^2+b^2-c^2) : (a+b-c)(a-b+c)(a^2-b^2+c^2))
    A2 = (a^4-(b^2-c^2)^2 : (b+c-a)(a+b+c)(a^2+b^2-c^2) : (b+c-a)(a+b+c)(a^2-b^2+c^2))

    ( Mostrar/Ocultar figura )

    Construcción geométrica de los puntos A1 y A2:
    Aparte de la construcción descrita anteriormente, ya que tales cónicas pueden ser construidas (PPPtP o PPPtP(2)), podemos proceder como sigue:

    Si Qa es el pie de la ceviana AQ, sea L el conjugado armónico de Qa respecto a B y C, es decir. L es el punto donde la tripolar de Q (respecto a ABC) corta a la recta BC.
    La perpendicular por L a la bisectriz interior en A, corta a AB y AC en los pies de las cevianas BA1 y CA1.
    La perpendicular por L a la bisectriz exterior en A, corta a AB y AC en los pies de las cevianas BA2 y CA2.

    (Ver otra solución en el caso de Q el ortocentro de Ercole Suppa)



  • domingo, 27 de octubre del 2013

    Una caracterización geométrica de la cúbica de Lucas

    Sean ABC un triángulo, P un punto y DEF el triángulo ceviano de P.
    La transformación afín que lleva los puntos A, B y C en D, E y F, respectivamente, tiene sus rectas fijas (propias) perpendiculares si y solo si P está en la cúbica de Lucas (K007).

    Si las coordenadas baricéntricas de P, respecto a ABC, son (u:v:w) la matriz asociada a esta transformación afín es:

    0u(u+v)(v+w)u(u+w)(v+w)
    v(u+v)(u+w)0v(u+w)(v+w)
    w(u+v)(u+w)w(u+v)(v+w)0
    El punto fijo propio, correspondiente a la raíz λ1=-(v+w)(u^2+uv+uw+vw) del polinomio característico, es Q=(u(v+w) : v(w+u) : w(u+v)), complemento del conjugado isotómico de P, es decir el centro de la cónica inscrita de perspector P (§10.7.3 The perspector of a conic. Paul Yiu).

    Las otras dos raíces son:

    λ2,3 = 1/2 ((u+v)(u+w)(v+ w) ± (v+w)½ ((u+v)(u+ w)(u(v^2-w^2)+v(w^2-u^2)+w(u^2-v^2)))½)

    La condición necesaria y suficiente para que los puntos fijos (en la recta del infinito) correspondientes a las raíces λ2 y λ3, determinen dos direcciones perpendiculares es:

    (b^2+c^2-a^2)u(v^2+w^2) + (c^2+a^2-b^2)v(w^2+u^2) + (a^2+b^2-c^2)w(u^2+v^2)=0.

    Es decir, el punto P debe de estar en la cúbica de Lucas. En este caso, el punto fijo Q está sobre la cúbica de Thomson (K002).
    Las rectas fijas de la transformación afín coinciden con los ejes de la cónica inscrita de perspector P, ellas son paralelas a las asíntotas de la hipérbola (rectangular) circunscrita que pasa por P y Q.

    Comparar esta propiedad de la cúbica de Lucas con la nº 7 en la web de Bernard Gibert:
    "Let P be the perspector of an inscribed conic (C) with center Q. The circum-conic (C') passing through P and Q is a rectangular hyperbola if and only if P lies on the Lucas cubic".

    ( Mostrar/Ocultar figura )


    Nota:
    Cuando el punto P está en la recta del infinito, la transformación afín que lleva ABC en el triángulo ceviano DEF de P, es una homología, con centro en P y eje el de perspectividad de ABC y DEF.

  • miércoles, 23 de octubre del 2013

    La cúbica nodal de Tucker como lugar geométrico

    Sean ABC un triángulo, P un punto y DEF el triángulo ceviano de P.
    Consideremos los puntos U, V y W tales que:

    AU : UD = BV : VE = CW : WF = m : n

    Si (u:v:w) son las coordenadas baricéntricas de P, existen dos pares (m1,m1) y (m2,m2) reales o imaginarios:

    ( u^2(v+w) + v^2(w+u) + w^2(u+v) + 2uvw ∓ √[(u+v)(v+w)(w+u)] √[u(v-w)^2+v(w-u)^2+w(u-v)^2], 4uvw )

    para los cuales los puntos U, V y W están alineados. Las rectas que los contienen se cortan en el "crosspoint" de P y el baricentro.

    Existe un solo par (m,n), proporcional a (2,1), si P está en la cúbica nodal de Tucker. En este caso, el punto U coincide con la intersección de la recta AD y la paralela por el baricentro a BC, similarmente ocurre con los puntos V y W. Esta propiedad es la número 1 en K015.

    Al variar P sobre la cúbica K015 las rectas UVW son tangentes a la elipse inscrita de Steiner. En particular, los puntos de tangencia son los centros X(3163), X(115) y X(2482) si P coincide con los centros X(4240), X(5466) y X(5468), respectivamente.
    ( Mostrar/Ocultar figura )


  • martes, 22 de octubre del 2013

    Construcción del homólogo de un punto mediante una homografía

    ( AdvancedPlaneGeometry #730, Bernard Gibert )

    Se considera la aplicación definida en términos de coordenadas baricéntricas, respecto a un triángulo ABC, por las ecuaciones:

    x' = (2SB SC - S²)x + (2SB² + S²)y + (2SC² + S²)z
    y' = (2SA² + S²)x + (2SA SC - S²)y + (2SC² + S²)z
    z' = (2SA² + S²)x + (2SB² + S²)y + (2SA SB - S²)z
    Se trata de una transformación lineal (homografía), ya que el determinante de la matriz asociada es proporcional a (a^2+b^2+c^2)^2S^4>0.

    Pares de puntos homólogos {P,P'}, centros del triángulo, son los siguientes:
    {X(2),X(5304)}, {X(4),X(2)}, {X(5),X(6)}, {X(10),X(614)}, {X(30),X(230)}, {X(64),X(1249)}, {X(140),X(5306)}, {X(511),X(3291)}, {X(512),X(647)}, {X(516),X(3011)}, {X(517),X(3290)}, {X(541),X(3018)}, {X(546),X(3815)}, {X(690),X(1637)}, {X(1499),X(523)}, {X(1503),X(468)}, {X(1513),X(385)}, {X(1514),X(858)}, {X(1538),X(44)}, {X(2780),X(2492)}, {X(2821),X(3310)}, {X(2883),X(25)}, {X(3309),X(650)}, {3566),X(2501)}, {X(3835),X(657)}, {X(3853),X(3054)}, {X(3861),X(3055)}.

    Los cuatro centros X(4), X(5), X(10) y X(511) son tales que no hay tres de ellos alineados, en un triángulo escaleno, ya que:
    Det[X(4), X(5), X(10)] = -(1/8) (a-b)(a-c)(b-c)(a-b-c)(a+b-c)(a-b+c)(a+b+c)^3,
    Det[X(4), X(5), X(511)] = -(1/8) (a^2-b^2)(a^2-c^2)(b^2-c^2)(a-b-c)^2(a+b-c)^2(a-b+c)^2(a+b+c)^2,
    Det[X(4), X(10), X(511)] = 1/4 (a-b)(a-c)(b-c)(a-b-c)(a+b-c)(a-b+c)(a+b+c)^3(ab+ac+bc),
    Det[X(5), X(10), X(511)] = abc(a-b)(a-c)(b-c)(a-b-c)(a+b-c)(a-b+c)(a+b+c)^2.

    En consecuencia, los cuatro pares de puntos homólogos {X(4),X(2)}, {X(5),X(6)}, {X(10),X(614)} y {X(511),X(3291)} determinan completamente la homografía.
    La construcción del homólogo de un punto se expone en un Applet GeoGebra:
    ( Mostrar/Ocultar figura )

    Bernard Gibert (en comunicación personal) obtiene que los puntos dobles de esta homografía son los de intersección (distintos de X(2)) de la hipérbola de Kiepert y la hipérbola equilátera que pasa por X(2), X(5) y X(6) y de asíntotas paralelas a las de la hipérbola de Jerabek.
    (Construcción de una hipérbola dado tres puntos y las direcciones de sus asíntotas (IIPPP)
    ( Mostrar/Ocultar figura )

    (Ver otro ejemplo de de construcción de puntos homólogos en una homografía, con Cabri)

  • sábado, 19 de octubre del 2013

    La isocúbica pivotal pK(X3926,X3926)

    (CL007: pK(W,W) cubics, parallel tripolars cubics, Bernard Gibert )

    a Kake, por su "cumple"

    Sean ABC un triángulo, A'B'C' su triángulo antipodal, P un punto y DEF el triángulo pedal de P.
    Designamos por D' el conjugado armónico de D respecto a B y C. Definimos los conjugados armónicos E' y F' similarmente.
    El punto A1 es la intersección del lado BC con la perpendicular a AD' por A'. De forma similar se definen B1 y C1.

    Los triángulos ABC y A1B1C1 son perspectivos si y solo si P queda en al cúbica de Darboux.

    El lugar geométrico de los centros de perspectividad de los triángulos ABC y A1B1C1, cuando P varía sobre la cúbica de Darboux, es la isocúbica pivotal pK(X3926,X3926), de ecuación baricéntrica:
    pK(X3926,X3926)
    Esta cúbica contiene a los centros X2, X69, X345, X348, X3926 y al cociente ceviano X3926/X2.
    ( Mostrar/Ocultar figura )

    Otra caracterización de la cúbica de Darboux:
    Ya que si P está en la cúbica de Darboux el triángulo pedal DEF de P es el triángulo ceviano de un punto P'; entonces, se tiene que los puntos D', E' y F' están alineados sobre la tripolar de P'. El recíproco también es cierto, es decir:

    "La cúbica de Darboux es el lugar geométrico de los puntos P, en el plano del triángulo ABC, tal que los conjugados armónicos (respecto a los vértices de ABC) de los vértices de su triángulo pedal están alineados."



    NOTA:

    Si se reemplaza el triángulo pedal DEF de P por el triángulo ceviano y hacemos la misma construcción anterior para este caso, resulta que los triángulos ABC y A1B1C1 son siempre perspectivos y el centro de perspectividad es el producto baricéntrico de P y X69.
    ( Mostrar/Ocultar figura )


  • lunes, 14 de octubre del 2013

    Baricentros de los triángulos antimediales

    ( AdvancedPlaneGeometry #709 Francisco Javier García Capitán )

    Resultado r1896 (Quim Castellsaguer):
    Los baricentros Ga, Gb, Gc de los triángulos antimediales de ABC y el baricentro G de ABC son concíclicos en una circunferencia de centro el mismo de la circunferencia de los nueve puntos.
    Otros centros sobre esta circunferencia son los de su intersección con el eje de Fermat: X381 (punto medio de X2X4) y X5465 (proyección ortogonal de X2 sobre el eje de Fermat).

    ( Mostrar/Ocultar figura )

    Los triángulos GaGbGc y ABC son inversamente semejantes. La matriz de coeficientes asociada a las ecuaciones de la semejanza, en coordenadas baricéntricas respecto a ABC, es:

    \left(
\begin{array}{ccc}
 2 a^2 b^2 c^2 & a^2 c^2 \left(a^2+2
   b^2-c^2\right) & a^2 b^2 \left(a^2-b^2+2
   c^2\right) \\
 b^2 c^2 \left(2 a^2+b^2-c^2\right) & 2 a^2
   b^2 c^2 & a^2 b^2 \left(-a^2+b^2+2
   c^2\right) \\
 -b^2 c^2 \left(-2 a^2+b^2-c^2\right) & a^2
   c^2 \left(-a^2+2 b^2+c^2\right) & 2 a^2
   b^2 c^2
\end{array}
\right)

    Esta transformación tiene tres puntos fijos correspondientes a las tres raíces del polinomio característico:

    (6a^2b^2c^2 - λ) (a^8b^2c^2 - a^6b^4c^2 - a^4b^6c^2 + a^2b^8c^2 - a^6b^2c^4 + 3a^4b^4c^4 - a^2b^6c^4 - a^4b^2c^6 - a^2b^4c^6 + a^2b^2c^8 - λ^2)=0.

    El punto correspondiente a la raíz λ=6a^2b^2c^2 es el centro de la semejanza inversa:

    S= (a^2(a^4 - 3a^2(b^2 + c^2) + 2b^4 - 11b^2c^2 +2c^4): ... : ... )


    Los puntos correspondientes a las otras dos raíces del polinomio característico son los puntos del infinito de la hipérbola equilátera de Jerabek. Por lo que las rectas fijas de la semejanza son las paralelas por S a las asíntotas de esta hipérbola.

    (Ver otro ejemplo de semejanza invesa)

  • viernes, 11 de octubre del 2013

    Una propiedad de la cúbica K172

    ( Anopolis #1072 Antreas P. Hatzipolakis)

    Sean ABC un triángulo, P un punto y DEF el triángulo pedal de P, Denotamos por E' el punto sobre BC tal que las circunferencias de los nueve puntos de ABC y A'B'C' son tangentes. Los puntos E' y F' se definen de forma similar, procediendo cíclicamente sobre los lados de ABC.
    Los triángulos ABC y D'E'F son perspectivos si y solo si P está sobre la cúbica de Darboux ( K008). El lugar de los centros de perspectividad están sobre la cúbica K172.

    ( Mostrar/Ocultar figura )


  • jueves, 3 de octubre del 2013

    Los puntos X(74), X(262) y X(3426) como centros de perspectividad de triángulos

    (ETC Clark Kimberling )

    a Silvia, por su "cumple"

    Sean ABC un triángulo, Ab el punto de intersección de la altura por B con la perpendicular a AB por A, y Ac el punto de intersección de la altura por C con la perpendicular a AC por A. Similarmente y cíclicamente se define los puntos Bc, Ba, Ca y Cb , entonces las rectas AbAc, BcBa y CaCb determinan un triángulo A'B'C' perspectivo con ABC, con centro de perspectividad en X262, de coordenadas baricéntricas:

    ( 1/(a^4 - a^2(b^2+c^2) - 2b^2c^2) : 1/(b^4 - b^2(c^2+a^2) - 2c^2a^2) : 1/(c^4 - c^2(a^2+b^2) - 2a^2b^2) ).

    ( Mostrar/Ocultar figura )

    El triángulo A"B"C" delimitado por las rectas BcCb, CaAc y AbBa es perspectivo con ABC, con centro de perspectividad en X3426, de coordenadas baricéntricas:

    (a^2/(5a^4 - 4a^2(b^2+c^2) - (b^2-c^2)^2) : b^2/(5b^4 - 4b^2(c^2+a^2) - (c^2-a^2)^2) : c^2/(5c^4 - 4c^2(a^2+b^2) - (a^2-b^2)^2) ).

    (Otra propiedad de X3426 en "El punto X(3426) como centro ortológico" )
    ( Mostrar/Ocultar figura )

    Si ahora consideramos el punto Mab de intersección de la mediatriz de AC con la perpendicular a AB por A y el punto Mac de intersección de la mediatriz de AB con la perpendicular a AC por A, las rectas AbAc y MabMac se cortan en un punto A'''.
    Similarmente, se definen los puntos B''' y C'''; entonces, los triángulos A"B"C" y A'''B'''C''' son perspectivos, con centro de perspectividad en X74, que es el cuarto punto de intersección de la circunferencia circunscrita a ABC y la hipérbola de Jerabek.
    ( Mostrar/Ocultar figura )


  • miércoles, 2 de octubre del 2013

    Un centro del triángulo asociado a polares de cónicas

    Con las notaciones dadas en "A question from Mathesis and Bicentric Pairs" :

    En el plano del triángulo ABC el lugar geométrico de los puntos P tales que las perpendiculares trazadas desde los vértices A, B, C respectivamente a las rectas PC, PA, PB son concurrentes en un punto Q es la cónica Φ1 circunscrita a ABC, que pasa por el punto de Tarry (X98) y por el conjugado isogonal P(40) del primer punto de Beltrami, P(2) (inverso del primer punto de Brocard, Ω1, respecto a la circunferencia circunscrita).
    El lugar de los puntos Q, cuando P varía en la cónica Φ1 es la cónica Φ2 circunscrita a ABC, que pasa por el punto de Tarry (X98) y por el conjugado isogonal U(40) del segundo punto de Beltrami, U(2) (inverso del segundo punto de Brocard Ω2, respecto a la circunferencia circunscrita).

    Cuando P=A, el punto Q (que denotamos por Ac) es la intersección de la perpendicular por A a AC con la altura por C.
    Cuando Q=A, el punto P (que denotamos por Ab) es la intersección de la perpendicular por A a AB con la altura por B.

    El lugar geométrico del punto D=PC ∩ QB, cuando P varía sobre la cónica Φ1, es una cónica, Γa, tangente en B y C a las perpendiculares a AB y AC, respectivamente; además pasa por Db=AC∩BAc y por Dc=AB∩CAb. Su ecuación baricéntrica respecto a ABC es:

    SA² x² + SBSC yz + b² SB zx + c² SC xy =0.

    Procediendo cíclicamente, resultan las cónicas Γb y Γc, lugares geométricos de los puntos E=PA ∩ QC y F=PB ∩ QA. Sus ecuaciones son:

    SB² y² + a² SA yz + SCSA zx + c² SC xy=0,    SC² z² + a² SA yz + b² SB zx + SASB xy =0


    Las polares de A, B y C respecto a las cónicas Γa, Γb y Γc, respectivamente, determinan un triángulo A'B'C perspectivo con ABC, con centro de perspectividad:

    ( 1/(2a²SA³ - b²c²SBSC) : 1/(2b²SB³ - c²a²SCSA) : 1/(2c²SC³ - a²b²SASB) ).

    Este centro tiene primera coordenada trilineal exacta en el triángulo de ETC: -5.73512334607375866920459161
    ( Mostrar/Ocultar figura )


  • martes, 1 de octubre del 2013

    El punto X(3426) como centro ortológico

    ( Resultat #1622 Quim Castellsaguer )

    Sean A'B'C' el triángulo antipodal de ABC; A"B"C" el triángulo ceviano del retrocentro R (X69 conjugado isotómico del ortocentro); D, E, F los puntos medios de A'A", B'B", C'C". Los triángulos ABC y DEF son ortológicos, y un centro de ortología es el punto de De Longchamps, X20.
    El otro centro de ortología es el centro X3426 = X(3)-VERTEX CONJUGATE OF X(6).

    Los triángulos circuncevianos del circuncentro y simediano son, respectivamente, A'B'C' y LMN; entonces las rectas A'L, B'M, C'N delimitan un triángulo A1B1C1 perspectivo con ABC, con centro de perspectividad en X3426.
    (Otra propiedad de X3426 en "Los puntos X(74), X(262) y X(3426) como centros de perspectividad de triángulos" )

    ( Mostrar/Ocultar figura )
    Nota justo después del X(3414) en ETC: ( http://faculty.evansville.edu/ck6/encyclopedia/ETCPart3.html#X3414 ) Let T be the vertex triangle of the circumcevian triangles, AUBUCU and AXBXCX, of U and X; viz., the sidelines of T are AUAX, BUBX, CUCX. Then T is perspective to ABC, and the perspector is the U-vertex conjugate of X.

  • lunes, 30 de septiembre del 2013

    "A question from Mathesis and Bicentric Pairs"

    ( AdvancedPlaneGeometry #674 Ercole Suppa )
    (J.Neuberg, Mathesis 1885, vol. 5, pag. 208, Questions 258, 259)

    En el plano del triángulo ABC el lugar geométrico de los puntos P tales que las perpendiculares trazadas desde los vértices A, B, C respectivamente a las rectas PC, PA, PB son concurrentes en un punto Q es la cónica Φ1 circunscrita a ABC, que pasa por el punto de Tarry (X98) y por el conjugado isogonal P(40) del primer punto de Beltrami, P(2) (inverso del primer punto de Brocard, Ω1, respecto a la circunferencia circunscrita).
    El lugar de los puntos Q, cuando P varía en la cónica Φ1 es la cónica Φ2 circunscrita a ABC, que pasa por el punto de Tarry (X98) y por el conjugado isogonal U(40) del segundo punto de Beltrami, U(2) (inverso del segundo punto de Brocard Ω2, respecto a la circunferencia circunscrita).

    El lugar geométrico de los puntos P tales que las perpendiculares trazadas desde los vértices A, B, C respectivamente a las rectas PB, PC, PA son concurrentes en un punto Q es la cónica Φ2 circunscrita a ABC, que pasa por el punto de Tarry (X98) y por el conjugado isogonal del segundo punto de Beltrami (inverso del segundo punto de Brocard respecto a la circunferencia circunscrita). El lugar de los puntos Q, cuando P varía en la cónica Φ2 es la cónica Φ1 circunscrita a ABC, que pasa por el punto de Tarry (X98) y por el conjugado isogonal del primer punto de Beltrami (inverso del primer punto de Brocard respecto a la circunferencia circunscrita).

    Las ecuaciones baricéntricas de las cónicas Φ1 y Φ2 son respectivamente:
    (a^2+b^2-c^2)y z+ (-a^2+b^2+c^2)z x + (a^2-b^2+c^2)x y =0,    (a^2-b^2+c^2)y z+ (a^2+b^2-c^2)z x + (-a^2+b^2+c^2)x y =0.

    ( Mostrar/Ocultar figura )


    •  Cuando P=X98, Q=U(40) y cuando P=P(40), Q=X98.

    •  Las cónicas Φ1 y Φ2 son congruentes, con ejes respectivos perpendiculares y el centro de giro que transforma una en otra es el punto de Vecten (X485).

    •  Los perspectores de las cónicas Φ1 y Φ2, (a^2+b^2-c^2 : -a^2+b^2+c^2 : a^2-b^2+c^2) y (a^2-b^2+c^2 : a^2+b^2-c^2 : -a^2+b^2+c^2) son el par bicéntrico P(45) y U(45).

    •  Los centros de las cónicas Φ1 y Φ2, son:

    (a^4 + 2a^2(b^2-2c^2) + b^4-4b^2c^2+3c^4 : ... : ...) y
    (a^4 + 2a^2(c^2-2b^2) + c^4-4b^2c^2+3b^4 : ... : ...)

    que forman un par bicéntrico, que no figura en la lista confeccionada por Clark Kimberling. La recta que pasa por los centro de las cónicas solamente contiene un centro de ETC: su punto del infinito, X3566.

    •  La recta P(40)U(40) vuelve a cortar a las cónicas Φ1 y Φ2, respectivamente, en los puntos de coordenadas baricéntricas:

    ((a^2-b^2)(b^2-c^2)(a^2+b^2-c^2) : (b^2-c^2)(c^2-a^2)(-a^2+b^2+c^2) : (c^2-a^2)(a^2-b^2)(a^2-b^2+c^2)),
    ((a^2-c^2)(c^2-b^2)(a^2-b^2+c^2) : (b^2-a^2)(a^2-c^2)(a^2+b^2-c^2) : (c^2-b^2)(b^2-a^2)(-a^2+b^2+c^2)),

    que forman un par bicéntrico, que no figura en la lista confeccionada por Clark Kimberling.

    •  Los polos de la recta P(40)U(40) respecto a las cónicas Φ1 y Φ2, respectivamente, tienen coordenadas baricéntricas:

    (a^8 - a^6(b^2+c^2) + a^4b^2c^2 + a^2(b^2-c^2)^3 - (b^2-c^2)^2(b^4+b^2c^2-c^4) : ... : ...),
    (a^8 - a^6(b^2+c^2) + a^4b^2c^2 - a^2(b^2-c^2)^3 - (b^2-c^2)^2(c^4+b^2c^2-b^4) : ... : ...),

    que forman un par bicéntrico, que no figura en la lista confeccionada por Clark Kimberling.

  • sábado, 14 de septiembre del 2013

    Pares bicéntricos deducidos de los triángulos pedal y simétrico de un punto respecto a un triángulo

    ( Anopolis #999 Antreas P. Hatzipolakis)

    Sean ABC un triángulo, P un punto, A'B'C' el triángulo pedal de P y A", B", C" las reflexiones de P en los lados BC, CA, AB, respectivamente.

    El centro radical Rp de las circunferencias de diámetros A'B", B'C", C'A" y el centro radical Sp de las circunferencias de diámetros A'C", B'A", C'B" forman un par bicéntrico

    Si (u:v:w) son las coordenadas baricéntricas de P:

    Rp = (a^2 (3 b^2 c^2 u^2 (c^2 v + b^2 w) + u (c^4 (a^2 + b^2 - c^2) v^2 + 2 b^4 (a^2 - b^2 + c^2) w^2 + b^2 c^2 (3 a^2 - 2 b^2 + 2 c^2) v w) + a^2 (a^2 - b^2 - c^2)v w (2 c^2 v + b^2 w)) : ... : ... ),

    Sp = (a^2 (3 b^2 c^2 u^2 (c^2 v + b^2 w) + u (2 c^4 (a^2 + b^2 - c^2) v^2 + b^2 c^2 (3 a^2 + 2 b^2 - 2 c^2) v w + b^4 (a^2 - b^2 + c^2) w^2) + a^2 (a^2 - b^2 - c^2) v w (c^2 v + 2 b^2 w) ) : ... : ... ).


    ( Mostrar/Ocultar figura )
    Algunos casos particulares (coordenadas baricéntricas):
    P=X1
    f(a,b,c)=a (a^2 (b + 2 c) + a (b^2 - c^2)-2 b^3 + 3 b c^2 - c^3)
    La recta que pasa por este par bicéntrico, sólo contiene al centro X65, su punto medio.

    P=X6
    f(a,b,c)=5 a^2 - 2 b^2
    La recta que pasa por este par bicéntrico, sólo contiene a los centros X523, su punto del infinito X1992, su punto medio.

  • viernes, 13 de septiembre del 2013

    Circunferencias asociadas a los puntos isodinámicos

    ( Anopolis #1000 Antreas P. Hatzipolakis)

    Sean ABC un triángulo, P un punto, A'B'C' el triángulo pedal de P y A", B", C" las reflexiones de P en los lados BC, CA, AB, respectivamente.

    Los seis puntos medios de los segmentos A'B", B'C", C'A", A'C", B'A", C'B" están sobre una cónica, ya que los lados opuestos del hexágono que ellos forman son paralelos.

    ( Mostrar/Ocultar figura )
    La cónica es una circunferencia si y sólo si P es uno de los dos puntos isodinámicos (X15 y X16).

    Si P=X6 es el simediano, la cónica tiene centro en este punto.
    Cuando P se mueve sobre el eje de Brocard, los centros de las cónicas están sobre la recta X5X6. Sobre la que está los centros de las dos cónicas que son circunferencias.

  • jueves, 12 de septiembre del 2013

    Rectas de Euler paralelas

    ( AdvancedPlaneGeometry #613 Dominik Burek)

    El lugar geométrico de los puntos P tales que las rectas de Euler de los triángulos ABC y IPP* son paralelas es una séxtica que pasa por A, B, C, (puntos dobles), por I, por los exincentros, por los conjugados isogonales de sus puntos.
    Ecuación baricéntrica: (Mostrar)

      -a^4 b c^4 x^4 y^2 + a^3 b^2 c^4 x^4 y^2 + a^2 b^3 c^4 x^4 y^2 - a b^4 c^4 x^4 y^2 + 2 a^3 b c^5 x^4 y^2 + b^4 c^5 x^4 y^2 - b^3 c^6 x^4 y^2 - 2 a b c^7 x^4 y^2 - b^2 c^7 x^4 y^2 + b c^8 x^4 y^2 + a^5 c^4 x^3 y^3 - a^4 b c^4 x^3 y^3 - a b^4 c^4 x^3 y^3 + b^5 c^4 x^3 y^3 + a^4 c^5 x^3 y^3 + 2 a^3 b c^5 x^3 y^3 + 2 a b^3 c^5 x^3 y^3 + b^4 c^5 x^3 y^3 - 3 a^3 c^6 x^3 y^3 + a^2 b c^6 x^3 y^3 + a b^2 c^6 x^3 y^3 - 3 b^3 c^6 x^3 y^3 - a^2 c^7 x^3 y^3 - 4 a b c^7 x^3 y^3 - b^2 c^7 x^3 y^3 + 2 a c^8 x^3 y^3 + 2 b c^8 x^3 y^3 - a^4 b c^4 x^2 y^4 + a^3 b^2 c^4 x^2 y^4 + a^2 b^3 c^4 x^2 y^4 - a b^4 c^4 x^2 y^4 + a^4 c^5 x^2 y^4 + 2 a b^3 c^5 x^2 y^4 - a^3 c^6 x^2 y^4 - a^2 c^7 x^2 y^4 - 2 a b c^7 x^2 y^4 + a c^8 x^2 y^4 - 2 a^4 b^3 c^2 x^4 y z + a^3 b^4 c^2 x^4 y z + 3 a^2 b^5 c^2 x^4 y z - a b^6 c^2 x^4 y z - b^7 c^2 x^4 y z - 2 a^4 b^2 c^3 x^4 y z + 4 a^3 b^3 c^3 x^4 y z - a^2 b^4 c^3 x^4 y z - 2 a b^5 c^3 x^4 y z + b^6 c^3 x^4 y z + a^3 b^2 c^4 x^4 y z - a^2 b^3 c^4 x^4 y z + 3 a^2 b^2 c^5 x^4 y z - 2 a b^3 c^5 x^4 y z - a b^2 c^6 x^4 y z + b^3 c^6 x^4 y z - b^2 c^7 x^4 y z + 3 a^5 b^2 c^2 x^3 y^2 z - 3 a^4 b^3 c^2 x^3 y^2 z - 3 a^3 b^4 c^2 x^3 y^2 z + 3 a^2 b^5 c^2 x^3 y^2 z + 2 a^5 b c^3 x^3 y^2 z - 5 a^4 b^2 c^3 x^3 y^2 z + 4 a^3 b^3 c^3 x^3 y^2 z - a^2 b^4 c^3 x^3 y^2 z - 4 a^4 b c^4 x^3 y^2 z - 6 a^3 b^2 c^4 x^3 y^2 z + a b^4 c^4 x^3 y^2 z - 3 b^5 c^4 x^3 y^2 z + 6 a^2 b^2 c^5 x^3 y^2 z - 4 a b^3 c^5 x^3 y^2 z + 3 b^4 c^5 x^3 y^2 z + 4 a^2 b c^6 x^3 y^2 z + 5 a b^2 c^6 x^3 y^2 z + 3 b^3 c^6 x^3 y^2 z - 2 a b c^7 x^3 y^2 z - 3 b^2 c^7 x^3 y^2 z + 3 a^5 b^2 c^2 x^2 y^3 z - 3 a^4 b^3 c^2 x^2 y^3 z - 3 a^3 b^4 c^2 x^2 y^3 z + 3 a^2 b^5 c^2 x^2 y^3 z - a^4 b^2 c^3 x^2 y^3 z + 4 a^3 b^3 c^3 x^2 y^3 z - 5 a^2 b^4 c^3 x^2 y^3 z + 2 a b^5 c^3 x^2 y^3 z - 3 a^5 c^4 x^2 y^3 z + a^4 b c^4 x^2 y^3 z - 6 a^2 b^3 c^4 x^2 y^3 z - 4 a b^4 c^4 x^2 y^3 z + 3 a^4 c^5 x^2 y^3 z - 4 a^3 b c^5 x^2 y^3 z + 6 a^2 b^2 c^5 x^2 y^3 z + 3 a^3 c^6 x^2 y^3 z + 5 a^2 b c^6 x^2 y^3 z + 4 a b^2 c^6 x^2 y^3 z - 3 a^2 c^7 x^2 y^3 z - 2 a b c^7 x^2 y^3 z - a^7 c^2 x y^4 z - a^6 b c^2 x y^4 z + 3 a^5 b^2 c^2 x y^4 z + a^4 b^3 c^2 x y^4 z - 2 a^3 b^4 c^2 x y^4 z + a^6 c^3 x y^4 z - 2 a^5 b c^3 x y^4 z - a^4 b^2 c^3 x y^4 z + 4 a^3 b^3 c^3 x y^4 z - 2 a^2 b^4 c^3 x y^4 z - a^3 b^2 c^4 x y^4 z + a^2 b^3 c^4 x y^4 z - 2 a^3 b c^5 x y^4 z + 3 a^2 b^2 c^5 x y^4 z + a^3 c^6 x y^4 z - a^2 b c^6 x y^4 z - a^2 c^7 x y^4 z - a^4 b^4 c x^4 z^2 + 2 a^3 b^5 c x^4 z^2 - 2 a b^7 c x^4 z^2 + b^8 c x^4 z^2 + a^3 b^4 c^2 x^4 z^2 - b^7 c^2 x^4 z^2 + a^2 b^4 c^3 x^4 z^2 - b^6 c^3 x^4 z^2 - a b^4 c^4 x^4 z^2 + b^5 c^4 x^4 z^2 + 2 a^5 b^3 c x^3 y z^2 - 4 a^4 b^4 c x^3 y z^2 + 4 a^2 b^6 c x^3 y z^2 - 2 a b^7 c x^3 y z^2 + 3 a^5 b^2 c^2 x^3 y z^2 - 5 a^4 b^3 c^2 x^3 y z^2 - 6 a^3 b^4 c^2 x^3 y z^2 + 6 a^2 b^5 c^2 x^3 y z^2 + 5 a b^6 c^2 x^3 y z^2 - 3 b^7 c^2 x^3 y z^2 - 3 a^4 b^2 c^3 x^3 y z^2 + 4 a^3 b^3 c^3 x^3 y z^2 - 4 a b^5 c^3 x^3 y z^2 + 3 b^6 c^3 x^3 y z^2 - 3 a^3 b^2 c^4 x^3 y z^2 - a^2 b^3 c^4 x^3 y z^2 + a b^4 c^4 x^3 y z^2 + 3 b^5 c^4 x^3 y z^2 + 3 a^2 b^2 c^5 x^3 y z^2 - 3 b^4 c^5 x^3 y z^2 - 2 a^7 b c x y^3 z^2 + 4 a^6 b^2 c x y^3 z^2 - 4 a^4 b^4 c x y^3 z^2 + 2 a^3 b^5 c x y^3 z^2 - 3 a^7 c^2 x y^3 z^2 + 5 a^6 b c^2 x y^3 z^2 + 6 a^5 b^2 c^2 x y^3 z^2 - 6 a^4 b^3 c^2 x y^3 z^2 - 5 a^3 b^4 c^2 x y^3 z^2 + 3 a^2 b^5 c^2 x y^3 z^2 + 3 a^6 c^3 x y^3 z^2 - 4 a^5 b c^3 x y^3 z^2 + 4 a^3 b^3 c^3 x y^3 z^2 - 3 a^2 b^4 c^3 x y^3 z^2 + 3 a^5 c^4 x y^3 z^2 + a^4 b c^4 x y^3 z^2 - a^3 b^2 c^4 x y^3 z^2 - 3 a^2 b^3 c^4 x y^3 z^2 - 3 a^4 c^5 x y^3 z^2 + 3 a^2 b^2 c^5 x y^3 z^2 + a^8 c y^4 z^2 - 2 a^7 b c y^4 z^2 + 2 a^5 b^3 c y^4 z^2 - a^4 b^4 c y^4 z^2 - a^7 c^2 y^4 z^2 + a^4 b^3 c^2 y^4 z^2 - a^6 c^3 y^4 z^2 + a^4 b^2 c^3 y^4 z^2 + a^5 c^4 y^4 z^2 - a^4 b c^4 y^4 z^2 + a^5 b^4 x^3 z^3 + a^4 b^5 x^3 z^3 - 3 a^3 b^6 x^3 z^3 - a^2 b^7 x^3 z^3 + 2 a b^8 x^3 z^3 - a^4 b^4 c x^3 z^3 + 2 a^3 b^5 c x^3 z^3 + a^2 b^6 c x^3 z^3 - 4 a b^7 c x^3 z^3 + 2 b^8 c x^3 z^3 + a b^6 c^2 x^3 z^3 - b^7 c^2 x^3 z^3 + 2 a b^5 c^3 x^3 z^3 - 3 b^6 c^3 x^3 z^3 - a b^4 c^4 x^3 z^3 + b^5 c^4 x^3 z^3 + b^4 c^5 x^3 z^3 - 3 a^5 b^4 x^2 y z^3 + 3 a^4 b^5 x^2 y z^3 + 3 a^3 b^6 x^2 y z^3 - 3 a^2 b^7 x^2 y z^3 + a^4 b^4 c x^2 y z^3 - 4 a^3 b^5 c x^2 y z^3 + 5 a^2 b^6 c x^2 y z^3 - 2 a b^7 c x^2 y z^3 + 3 a^5 b^2 c^2 x^2 y z^3 - a^4 b^3 c^2 x^2 y z^3 + 6 a^2 b^5 c^2 x^2 y z^3 + 4 a b^6 c^2 x^2 y z^3 - 3 a^4 b^2 c^3 x^2 y z^3 + 4 a^3 b^3 c^3 x^2 y z^3 - 6 a^2 b^4 c^3 x^2 y z^3 - 3 a^3 b^2 c^4 x^2 y z^3 - 5 a^2 b^3 c^4 x^2 y z^3 - 4 a b^4 c^4 x^2 y z^3 + 3 a^2 b^2 c^5 x^2 y z^3 + 2 a b^3 c^5 x^2 y z^3 - 3 a^7 b^2 x y^2 z^3 + 3 a^6 b^3 x y^2 z^3 + 3 a^5 b^4 x y^2 z^3 - 3 a^4 b^5 x y^2 z^3 - 2 a^7 b c x y^2 z^3 + 5 a^6 b^2 c x y^2 z^3 - 4 a^5 b^3 c x y^2 z^3 + a^4 b^4 c x y^2 z^3 + 4 a^6 b c^2 x y^2 z^3 + 6 a^5 b^2 c^2 x y^2 z^3 - a^3 b^4 c^2 x y^2 z^3 + 3 a^2 b^5 c^2 x y^2 z^3 - 6 a^4 b^2 c^3 x y^2 z^3 + 4 a^3 b^3 c^3 x y^2 z^3 - 3 a^2 b^4 c^3 x y^2 z^3 - 4 a^4 b c^4 x y^2 z^3 - 5 a^3 b^2 c^4 x y^2 z^3 - 3 a^2 b^3 c^4 x y^2 z^3 + 2 a^3 b c^5 x y^2 z^3 + 3 a^2 b^2 c^5 x y^2 z^3 + 2 a^8 b y^3 z^3 - a^7 b^2 y^3 z^3 - 3 a^6 b^3 y^3 z^3 + a^5 b^4 y^3 z^3 + a^4 b^5 y^3 z^3 + 2 a^8 c y^3 z^3 - 4 a^7 b c y^3 z^3 + a^6 b^2 c y^3 z^3 + 2 a^5 b^3 c y^3 z^3 - a^4 b^4 c y^3 z^3 - a^7 c^2 y^3 z^3 + a^6 b c^2 y^3 z^3 - 3 a^6 c^3 y^3 z^3 + 2 a^5 b c^3 y^3 z^3 + a^5 c^4 y^3 z^3 - a^4 b c^4 y^3 z^3 + a^4 c^5 y^3 z^3 + a^4 b^5 x^2 z^4 - a^3 b^6 x^2 z^4 - a^2 b^7 x^2 z^4 + a b^8 x^2 z^4 - a^4 b^4 c x^2 z^4 - 2 a b^7 c x^2 z^4 + a^3 b^4 c^2 x^2 z^4 + a^2 b^4 c^3 x^2 z^4 + 2 a b^5 c^3 x^2 z^4 - a b^4 c^4 x^2 z^4 - a^7 b^2 x y z^4 + a^6 b^3 x y z^4 + a^3 b^6 x y z^4 - a^2 b^7 x y z^4 - a^6 b^2 c x y z^4 - 2 a^5 b^3 c x y z^4 - 2 a^3 b^5 c x y z^4 - a^2 b^6 c x y z^4 + 3 a^5 b^2 c^2 x y z^4 - a^4 b^3 c^2 x y z^4 - a^3 b^4 c^2 x y z^4 + 3 a^2 b^5 c^2 x y z^4 + a^4 b^2 c^3 x y z^4 + 4 a^3 b^3 c^3 x y z^4 + a^2 b^4 c^3 x y z^4 - 2 a^3 b^2 c^4 x y z^4 - 2 a^2 b^3 c^4 x y z^4 + a^8 b y^2 z^4 - a^7 b^2 y^2 z^4 - a^6 b^3 y^2 z^4 + a^5 b^4 y^2 z^4 - 2 a^7 b c y^2 z^4 - a^4 b^4 c y^2 z^4 + a^4 b^3 c^2 y^2 z^4 + 2 a^5 b c^3 y^2 z^4 + a^4 b^2 c^3 y^2 z^4 - a^4 b c^4 y^2 z^4=0
    ( Mostrar/Ocultar figura )


  • martes, 10 de septiembre del 2013

    Centros ortológicos sobre una cónica

    ( Anopolis #991 Antreas P. Hatzipolakis)

    Sean ABC un triángulo, P un punto, A'B'C' el triángulo tangencial, P un punto y L una recta por P.
    Consideremos las proyecciones ortogonales A*, B*, C* de los vértices A, B, C sobre la recta L, respectivamente.
    Los puntos medios Ma, Mb, Mc de AA*, BB*, CC*, respectivamente.
    Los puntos medios M1, M2, M3 de A'Ma, B'Mb, C'Mc, respectivamente.

    Entonces, los triángulos ABC y M1M2M3 son ortológicos.
    Si (u:v:w) son las coordenadas baricéntricas de P, las perpendiculares por los vértices M1, M2 y M3 a los lados BC, AC y AB, respectivamente, concurren sobre una cónica. cuyo centro tiene coordenadas baricéntricas:

    Q = ( a^4(2u+v+w) - 2a^2(b^2+c^2)(3u+2(v+w)) + (b^2-c^2)^2(4u+3(v+w)) : ... : ... ).

    ( Mostrar/Ocultar figura )
    Algunos casos particulares para centros del triángulo:

    P=X1, el centro de la cónica es el punto de coordenadas:

    (2a^4 - a^3(b+c) - a^2(5b^2-2bc+5c^2) + a(b-c)^2(b+c) + 3(b^2-c^2)^2 :...:...)

    que tiene (6-9-13)-número de búsqueda en ETC: 1.473737731682850782032396958

    P=X2, Q=X547, punto medio de X2 y X5.

    P=X3, Q=X3628, baricentro del conjunto de los vértices del triángulo medial y X5.
    En este caso, la cónica lugar de los centros ortológicos es la circunferencia de ecuación:

    a^2yz+b^2zx+c^2xy + (3/64)(x+y+z)((3a^2-5b^2-5c^2)x + (3b^2-5c^2-5a^2)y + (3c^2-5a^2-5b^2)z) = 0.



    P=X4, Q=X5.

    P=X5, en centro de la cónica es el punto de coordenadas:

    (2a^4 - 9a^2(b^2+c^2) + 7(b^2-c^2)^2:...:..)

    que tiene (6-9-13)-número de búsqueda en ETC: 1.3315581357050847119617684585

    P=X6, en centro de la cónica es el punto de coordenadas:

    (2a^6 - 5a^4(b^2+c^2) - 16a^2b^2c^2 + 3(b^2-c^2)^2(b^2+c^2) : ..:...)

    que tiene (6-9-13)-número de búsqueda en ETC: 1.3865627298839303658207405225

    P=X20, Q=X140, punto medio de X3 y X5.


    P=X40, Q=X3634, baricentro del conjunto de los vértices del triángulo medial y X10.

  • lunes, 9 de septiembre del 2013

    Par bicéntrico sobre la recta X(140)X(1499)

    Cresta de gallo

    ( Anopolis #980 Antreas P. Hatzipolakis)

    Sean ABC un triángulo, O el circuncentro, DEF el triángulo medial y A', B', C' los puntos medios de AO, BO, CO, respectivamente.

    Los centros radicales:
    P, de las circunferencias de diámetros A'E, B'F, C'D, y U, de las circunferencias de diámetros A'F, B'D, C'E, forman un par bicéntrico (que podría recibir el nombre de una flor: CRESTA DE GALLO).

    Sus coordenadas baricéntricas son:

    P = (3a^4-a^2(2b^2+7c^2)+b^4-3b^2c^2+2c^4 : ... : ... )

    U = (3a^4-a^2(7b^2+2c^2)+2b^4-3b^2c^2+c^4 : ... : ... ).

    ( Mostrar/Ocultar figura )
    El punto medio de PU es X140 (punto medio de ON). El punto del infinito de PU es X1499 (punto Biham).

  • domingo, 8 de septiembre del 2013

    Cúbica de Brocard (segunda), hipérbola de Jerabek y tripolar de X(2966)

    ( AdvancedPlaneGeometry #573 Dominik Burek)

    Sean ABC un triángulo y P un punto. La circunferencia circunscrita al triángulo BCP interseca a las rectas AB y AC en los puntos Ca y Ba. Similarmente, tenemos los puntos Ab, Cb, Bc y Ac. El lugar geométrico de los puntos P tales que Ca, Ba, Ab, Cb, Bc y Ac estén en una cónica es la (segunda) cúbica de Brocard ( K018 del catálogo de Bernard Gibert):

    Locus property:
    6. Locus of point M such that the three circles MBC, MCA, MAB meet the sidelines of triangle ABC again at six points lying on a same conic.

    ( Mostrar/Ocultar figura )

    Cuando P está en K018 las rectas BcCb, CaAc y AbBa delimitan un triángulo A'B'C' perspectivo con ABC, con centro de perspectividad Q en la hipérbola de Jerabek.
    El centro de perspectividad correspondiente al conjugado isogonal P* de P es el punto Q', diametralmente opuesto a Q en la hipérbola de Jerabek.

    En particular (para los puntos X2, X6, X13, X14, X15, X16, X111, X368, X534 sobre K018):
    Si P=X2 (P*=X6), Q=X67 y Q'=X6.
    Si P=X13 (P*=X15), Q=X4 y Q'=X74.
    Si P=X14 (P*=X16), Q=X4 y Q'=X74.
    Si P=X111 (P*=X524), Q=X6 y Q'=X76.
    Del punto X368 no se disponen coordenadas.

    La cúbica K018 y la hipérbola de Jerabek tienen seis puntos comunes: A, B, C, X6 y otros dos reales o imaginarios. La recta determinada por estos dos últimos es la tripolar de X2996.

    X2996 =(1/((b^2 - c^2) (b^4 + c^4 - a^2 (b^2 + c^2))) : ... : .... ).
    Barycentric product X(98)*X(99).
    Let P(2) and U(2) be the 1st and 2nd Beltrami points (as indexed at Bicentric Pairs), and let P(40) and U(40) be the isogonal conjugates of P(2) and U(2), respectively. Then X(2966) is the point of intersection of the lines P(2)U(40) and P(40)U(2). (Peter Moses, July 1, 2009).
  • sábado, 07 de septiembre del 2013

    Caracterización de la cúbica de Thomson

    (Anopolis #959 Antreas P. Hatzipolakis)

    Sean ABC un triángulo, O su circuncentro, P un punto y Oa, Ob, Oc los circuncentros de los triángulos PBC, PCA, PAB, respectivamente.

    La cúbica de Thomson es el lugar geométrico de los puntos P tales que el baricentro de OaObOc está sobre la recta OP.

    ( Mostrar/Ocultar figura )

  • viernes, 06 de septiembre del 2013

    Pares bicéntricos con punto medio en la recta de Euler

    (Anopolis #954 Antreas P. Hatzipolakis)

    Sea ABC un triángulo y A'B'C' el triángulo homotético a ABC por la homotecia de centro en el baricentro y razón t.
    Se denota por:
    Ab = B'C' ∩ AC, Ac = B'C' ∩ AB, Bc = C'A' ∩ BA, Ba = C'A' ∩ BC, Ca = A'B' ∩ CB, Cb = A'B' ∩ CA.

    Rt el centro radical de las circunferencias A(AAb), B(BBc), C(CCa):
    ( c^2 (b^2 - c^2) (2 + t)^2 + a^4 (-5 + 4 t + t^2) + a^2 (-3 b^2 (1 + 4 t + t^2) + c^2 (17 + 8 t + 2 t^2)) : -a^4 (2 + t)^2 + b^2 (b^2 (-5 + 4 t + t^2) - 3 c^2 (1 + 4 t + t^2)) + a^2 (c^2 (2 + t)^2 + b^2 (17 + 8 t + 2 t^2)) : -b^4 (2 + t)^2 + c^4 (-5 + 4 t + t^2) + b^2 c^2 (17 + 8 t + 2 t^2) + a^2 (b^2 (2 + t)^2 - 3 c^2 (1 + 4 t + t^2)) )
    St el centro radical de las circunferencias A(AAc), B(BBa), C(CCb):
    ( -b^2 (b^2 - c^2) (2 + t)^2 + a^4 (-5 + 4 t + t^2) + a^2 (-3 c^2 (1 + 4 t + t^2) + b^2 (17 + 8 t + 2 t^2)) : -c^4 (2 + t)^2 + b^4 (-5 + 4 t + t^2) + b^2 c^2 (17 + 8 t + 2 t^2) + a^2 (c^2 (2 + t)^2 - 3 b^2 (1 + 4 t + t^2)) : -a^4 (2 + t)^2 + c^2 (c^2 (-5 + 4 t + t^2) - 3 b^2 (1 + 4 t + t^2)) + a^2 (b^2 (2 + t)^2 + c^2 (17 + 8 t + 2 t^2)) )

    Rt y St forman un par bicéntrico de punto medio sobre la recta de Euler, si t=m/n:
    Mt = ( (b^2-c^2)^2(m+2 n)^2 + a^2(b^2+c^2)(m^2+4mn-14n^2) - 2a^4(m^2+ 4mn-5n^2) : ... : .... ).

    ( Mostrar/Ocultar figura )


    Situaciones particulares del punto medio del par bicéntrico:
    t=1,-5 X(5) ABC=A'B'C'
    t=-1/2 X(3530) A'B'C'= triángulo medial
    t=-2 X(3) A'B'C'= triángulo anticomplementario
    t=-1 (16a^4 - 17a^2(b^2+c^2) + (b^2-c^2)^2: ... : ...) A'B'C'= triángulo reflexión de ABC en el baricentro
    t↦0 X(5054) A'B'C' degenera en el baricentro
    t= -2 ± Sqrt[6] X(2)
    t= -2 ± 3Sqrt[2] X(4)

    Para todo t, el punto del infinito de la recta RtSt, determinada por cada par bicéntrico es el punto de Biham, X1499.

  • jueves, 5 de septiembre del 2013

    Propiedades de los puntos X(960), X(2883), X(3743), como centros de cónicas

    ( AdvancedPlaneGeometry #458 Dominik Burek)

    Sean ABC un triángulo, P un punto y DEF el triángulo ceviano de P. Las rectas que pasan por E y F, paralelas a AD intersecan a BC en A1 y A2. Similarmente tenemos los puntos B1, B2, C1, C2. Entonces los seis puntos A1, A2, B1, B2, C1, C2 están en una cónica.

    Su ecuación baricéntrica, si P(u:v:w), es

    Σ v^2w^2(u+v+w)x^2 - uvw(2u^2+(v+w)(2u+v+w))yz=0.

    Esta cónica es homotética a la cónica circunscrita de centro en el complemento P' de P (PG=2GP'). ( Francisco Javier García Capitán )
    ( Mostrar/Ocultar figura )

    Cuando P=X4 esta cónica es la circunferencia de Taylor.

    Otros casos particulares:
    P X(1) X(2) X(4) X(8) X(20)
    Centro cónica X(3743) X(2) X(389) X(960) X(2883)

  • miércoles, 04 de septiembre del 2013

    El centro del triángulo X84 como centro ortológico

    (Anopolis #947 Antreas P. Hatzipolakis)

    Sea ABC un triángulo. Se denota por: Ab y Ac las proyecciones ortogonales de A sobre las bisectrices por B y C, respectivamente. Ha es el ortocentro del triángulo AAbAc. Similarmente, se consideran los ortocentros Hb y Hc.

    Entonces los triángulos ABC y HaHbHb son ortológicos y sus centros ortológicos son el ortocentro de ABC y el centro del triángulo X84.

    ( Mostrar/Ocultar figura )

  • martes, 03 de septiembre del 2013

    El centro del triángulo X3521 como centro ortológico

    (Anopolis #943 Antreas P. Hatzipolakis)

    Sean ABC un triángulo y O su circuncentro. Se denota por: Na, Nb, Nc los centros de las circunferencias de los nueve puntos de los triángulos OBC, OCA, OAB, respectivamente, y por Ma, Mb, Mc los puntos medios de ANa, BNb, CNc, respectivamente.
    Entonces, los triángulos ABC y MaMbMc son ortológicos. Los centro ortológicos son X5 y X3521.

    ( Mostrar/Ocultar figura )

  • miércoles, 28 de agosto del 2013

    Un centro del cuadrivértice

    (Quadri-Figures-Group #206 Chris van Tienhoven)


    Sean ABC un triángulo, P4 un punto y P1P2P3 su triángulo anticeviano; entonces, P1P2P3P4 es un cuadrivértice con puntos diagonales A, B y C (ABC es el triángulo diagonal del cuadrivértice).
    Un sistema de QA-DT-coordenadas relativas al cuadrivértice P1P2P3P4, se define tomando como triángulo de referencia el triángulo diagonal y uno de los vértices del cuadrivértices, por ejemplo P4, con coordenadas (p:q:r); los otros tres vértices forman el triángulo anticeviano de P4: P1(-p:q:r), P2(p:-q:r), P3(p:q:-r).

    Denotamos por Ci el centro de la cónica que pasa por los cinco puntos A, B, C, Pi y QA-P10 (i=1,2,3,4), donde QA-P10 es el baricentro del triángulo diagonal ABC. Se verifica que las rectas PiCi son concurrentes en el punto de DT-coordenadas:

    ( p^2(q^2+r^2) : q^2(r^2+p^2) : r^2(p^2+q^2 ).

    ( Mostrar/Ocultar figura )

    GENERALIZACIÓN:
    Sean X un punto y Ci el centro de la cónica que pasa por los cinco puntos A, B, C, Pi y X (i=1,2,3,4).
    Las rectas PiCi son concurrentes si solo si X queda en la cónica de los nueve puntos (QA-Co1: Nine-point Conic) circunscrita al triángulo diagonal y centro en el baricentro del cuadrivértice (QA-P1), junto con una cuártica que pasa P1, P2, P3, P4 A, B, C, QA-P10 de ecuación ((Eckart Schmidt Quadri-Figures-Group #209 ):

    p^2(q^2-r^2)y^2z^2 + q^2(r^2-p^2)z^2x^2 + r^2(p^2-q^2)x^2y^2 = 0.

    Esta cuártica es conjugada isogonal de la cónica de ecuación:

    p^2x^2(q^2-r^2)+q^2y^2(r^2-p^2)+r^2z^2(p^2-q^2)=0.

    Esta cónica contiene a las vértices del triángulo anticeviano de sus puntos. Como pasa por los centros QA-P10 y QA-P19 también pasa por los vértices de sus triángulos anticevianos, GaGbGc (triángulo antimedial del triángulo diagonal) y LMN, respectivamente. Así, cónica y cuártica pueden construirse.

    La cónica y la cuártica tienen tangentes comunes en los vértices del triángulo antimedial, las cuales determinan un triángulo perspectivo con el triángulo diagonal; su centro de proyectividad tiene DT-coordenadas:

    ( 1/(p^2(q^2-r^2)) : 1/(q^2(r^2-p^2)) : 1/(r^2(p^2-q^2)) ).



    • Si X recorre la cuártica el punto Y de intersección de las rectas PiCi describe la cúbica QA-Cu6 (QA-P1-InvolutionCenter Cubic), de ecuación:

    q^2 r^2 (q^2 - r^2) x^3 + q^2 r^2 (2 p^2 (y - z) - (q^2 - r^2) (y + z)) x^2 + p^2 r^2 (r^2 - p^2) y^3 + p^2 r^2 (2 q^2 (z - x) - (r^2 - p^2) (x + z)) y^2 + p^2 q^2 (p^2 - q^2) z^3 + p^2 q^2 (2 r^2 (x - y) - (p^2 - q^2) (x + y)) z^2=0.

    ( Mostrar/Ocultar figura )

    • Si X recorre la cónica de los nueve puntos (QA-Co1: Nine-point Conic) el punto Y de intersección de las rectas PiCi describe la misma cúbica que cuando X recorre la cuártica.
    ( Mostrar/Ocultar figura )



  • sábado, 24 de agosto del 2013

    Otro par bicéntrico

    Morgallana

    (Anopolis #871 Antreas P. Hatzipolakis)

    Sean ABC un triángulo y A'B'C' el triángulo ceviano del incentro. Se denota por:
    (Oab), (Oac) las circunferencias circunscritas a los triángulos AAbA', AAcA', resp.
    (Obc), (Oba) las circunferencias circunscritas a los triángulos BAcB', BBaB', resp.
    (Oca), (Ocb) las circunferencias circunscritas a los triángulos CCaC', CCbC', resp.

    Las coordenadas baricéntricas del centro radical R de las circunferencias (Obc), (Oca), (Oab) son:
    R = (a (a^2(b-2c) - a(b^2+b c-c^2) - (b-c)c^2 ) : ... : ... ).

    Las coordenadas baricéntricas del centro radical S de las circunferencias (Oba), (Ocb), (Oac) son:
    S = (a (a^2(2b-c) - a(b^2-b c-c^2) - b^2(b-c)) : ... : ... ).

    Los puntos R y S forman un par bicéntrico P(110) (que podría recibir el nombre de una flor: MORGALLANA o botón de oro), por tanto el punto del infinito de la recta RS es un centro: X1938, que coincide con el punto del infinito del par bicéntrico PU(15).

    La suma de las coordenadas baricéntricas de este par baricéntrico es el centro X942, inverso en la circunferencia inscrita del inverso en la circunferencia circunscrita del incentro.

    La diferencia de las coordenadas baricéntricas de este par baricéntrico es el centro de coordenadas baricéntricas:

    ( a(b-c)(3a^2-2a b-b^2-2a c-c^2): ... : ...),

    con número de búsqueda en ETC: -8.16982446237566024925964383. El punto medio de RS es un centro, de coordenadas baricéntricas:

    M = (a (3a^4(b-c)^2 + a^3(-5b^3+4 b^2c+4b c^2-5c^3) + a^2(b^4+5b^3c-14b^2c^2+5b c^3+c^4) + a(b-c)^2(b^3+c^3) - b c(b-c)^2(b^2+c^2)) : ... : ...),

    con número de búsqueda en ETC: 1.2448168635877587605351158452.
    ( Mostrar/Ocultar figura )

  • viernes, 23 de agosto del 2013

    Baricentro de un triángulo asociado al triángulo medial

    (Anopolis #872 Antreas P. Hatzipolakis)

    Sean ABC un triángulo y A'B'C' su triángulo medial. Se denota por:
    Obc y Ocb los circuncentros de los triángulos BCB' y BCC', respectivamente, y por Ma el punto medio de ObcOcb. Similarmente se definen Mb y Mc.
    El baricentro G' de MaMbMc queda en la recta de Euler de ABC y sus coordenadas baricéntricas son:

    G' = (2 SB SC + 11 a^2 SA : 2 SC SA + 11 b^2 SB : 2 SA SB + 11 c^2 SC),

    es el complemento de X3845 y divide al segmento GO en la razón GG':G'O = 3:1.
    ( Mostrar/Ocultar figura )

  • miércoles, 21 de agosto del 2013

    Un par bicéntrico

    Canarina

    (Anopolis #860 Antreas P. Hatzipolakis)

    Sean ABC un triángulo y A'B'C' el triángulo ceviano del incentro. Se denota por:
    (Nab), (Nac) las circunferencias de los nueve puntos de los triángulos AIB', AIC', resp.
    (Nbc), (Nba) las circunferencias de los nueve puntos de los triángulos BIC', BIA', resp.
    (Nca), (Ncb) las circunferencias de los nueve puntos de los triángulos CIA', CIB', resp.

    Las coordenadas baricéntricas del centro radical R de las circunferencias (Nbc), (Nca), (Nab) son:
    R = ( a(a + b - c) (a - b + c) (a^3 b - a b^3 + 2 a^3 c - 2 a b^2 c - b^3 c + a^2 c^2 - 3 a b c^2 - 3 b^2 c^2 - 2 a c^3 - 3 b c^3 - c^4) : ... : ... ).

    Las coordenadas baricéntricas del centro radical S de las circunferencias (Nba), (Ncb), (Nac) son:
    S = ( a (a + b - c) (a - b + c) (2 a^3 b + a^2 b^2 - 2 a b^3 - b^4 + a^3 c - 3 a b^2 c - 3 b^3 c - 2 a b c^2 - 3 b^2 c^2 - a c^3 - b c^3) : ... : ... ).

    Los puntos R y S forman un par bicéntrico P(111) (que podría recibir el nombre de una flor: CANARINA o bicácaro), por tanto el punto del infinito de la recta RS es un centro: X513.
    También el punto medio de RS es un centro, de coordenadas baricéntricas:

    M = (a(3a^5(b+c)+ a^4(b^2+c^2) - 2a^3(3b^3+b^2c+b c^2+3c^3) - 2a^2(b^4+ b^3c+4b^2c^2+b c^3+c^4) + a(b-c)^2(3b^3+5b^2c+5b c^2+3c^3)+ (b-c)^2(b+c)^4) : ... : ...),

    con número de búsqueda en ETC: 2.235165076122804495064988816.
    ( Mostrar/Ocultar figura )
    Los ejes radicales de los tres pares de circunferencias (Nab), (Nac); (Nbc), (Nba) y (Nca), (Ncb) concurren en el centro X3649
  • martes, 20 de agosto del 2013

    La cúbica pivotal pK(X1073,X253)

    (Anopolis #859 Antreas P. Hatzipolakis)

    Sean ABC un triángulo, P un punto y A'B'C' el triángulo ceviano de P. Denotamos por Ab la reflexión, respecto a C, de la proyección ortogonal de A' sobre AC y por Ac la reflexión, respecto a B, de la proyección ortogonal de A' sobre AB . Sea Oa el circuncentro del triángulo AAbAc; similarmente, se consideran los circuncentros Ob y Oc.

    • Los triángulos ABC y OaObOc son perspectivos si y solo si P está en la cúbica pivotal pK(X1073, X253). Esta cúbica pasa por los centros X(2), X(3), X(64), X(69), X(253), X(1073), ...

    ( Mostrar/Ocultar figura )

    • Los triángulos A'B'C' y OaObOc son perspectivos si y solo si P está en la cubica pivotal pK(Ω, X), donde polo y pivote son, respectivamente::

    Ω = ( a^2SA/(a^4 + 2a^2(b^2+c^2) - 3(b^2-c^2)^2) : ... : ...),

    con número de búsqueda en ETC: 5.40902452312896952998608493.

    X = ( 1/(a^4 + 2a^2(b^2+c^2) - 3(b^2-c^2)^2) : ... : ...),

    con número de búsqueda en ETC: 6.399682552061115067504709200.
    ( Mostrar/Ocultar figura )
    Esta cúbica contiene a su pivote (como toda isocúbica pivotal), a su polo y a los centros X(2), X(3), X(69), ...

  • sábado, 17 de agosto del 2013

    Propiedad de las isocúbicas pK(X2,X264) y pK(X216,X4)

    (Anopolis #842 Antreas P. Hatzipolakis)

    a Clara, por su "cumple"

    Sean ABC un triángulo, P un punto y A'B'C' el triángulo ceviano de P. Denotamos por Ab la reflexión de C respecto a la perpendicular por A' a AC y por Ac la reflexión de B respecto a la perpendicular por A' a AB. Sea Oa el circuncentro del triángulo AAbAc; similarmente, se consideran los circuncentros Ob y Oc.
    El lugar geométrico de P tal que los triángulos A'B'C' y OaObOc son perspectivos es la isocúbica pK(X2,X264) (K045 del catálogo de Bernard Gibert).
    El lugar geométrico de los centros de perspectividad es la isocúbica pK(X216,X4) (K044 del catálogo de Bernard Gibert).
    ( Mostrar/Ocultar figura )

    • Los triángulos ABC y OaObOc son perspectivos si y solo si P está sobre la cúbica que pasa a través de los puntos X(2), X(3), X(54), X(69), X(95), X(96), X(97),... los pies de las cevianas de X(95); de ecuación baricéntrica:

    Σ[ x(c^2(a^2+b^2-c^2)(a^4-a^2 b^2-2a^2c^2-b^2c^2+ c^4) y^2 - b^2(a^2-b^2+ c^2)(a^4-2a^2b^2+b^4-a^2c^2-b^2c^2) z^2)] =0.


    Esta cúbica ha sido incluida en el Catálogo de Bernard Gibert con el número K646 = Montesdeoca cubic, pK(X97, X95)
    ( Mostrar/Ocultar figura )
    En este caso el centro de perspectividad de los triángulos ABC y OaObOc queda sobre la cúbica K044 = pK(X216,X4).

  • domingo, 11 de agosto del 2013

    Un centro ortológico relativo al triángulo medial

    Sean ABC un triángulo, A'B'C' el triángulo medial y A"B"C" el triángulo ceviano del incentro. Se denota por Ab y Ac las proyecciones ortogonales de A sobre BB" y CC". Similarmente, se definen los puntos Bc, Ba, Ca y Ca.
    Si Ma, Mb y Mc son los puntos medios de AbAc, BcBa y CaCb, entonces:
    • Los triángulos ABC y MaMbMc son ortológicos, con centros ortológicos X84 y X946.
    • Los triángulos A'B'C' y MaMbMc son ortológicos, con centros ortológicos X946 y el punto X de coordenadas baricéntricas:

    X = ( (a^3+a^2(b+c) - a(b+c)^2 - (b-c)^2(b+c)) (a^3(b+c) - a^2(b-c)^2 - a(b-c)^2(b+c) + (b^2-c^2)^2) : ... : ... ).

    que tiene (6-9-13)-número de búsqueda en ETC: 30.3292407999481595887008929
    ( Mostrar/Ocultar figura )


  • sábado, 10 de agosto del 2013

    Cónicas con centro y perspector coincidentes

    En una cónica (no autoadjunta) expresada en coordenadas baricéntricas respecto a un triángulo ABC por la ecuación:

    fx^2 + gy^2 + hz^2 + 2pyz + 2qzx + 2rxy = 0,

    su centro y perspector (centro de perspectividad de ABC y el triángulo formado por las polares de sus vértices respecto a la cónica) coinciden si y sólo si p=q=r≠0.
    (Paul Yiu).

    UN EJEMPLO:
    (Anopolis #774 Antreas P. Hatzipolakis)

    Sean ABC un triángulo, A'B'C' su triángulo antimedial y A"B"C" el triángulo pedal del incentro.
    Se denota por Ab = A"B" ∩ B'C' y por Ac = A"C" ∩ B'C'.
    Similarmente y de forma cíclica de definen Bc, Ba y Ca, Cb.

    Los seis puntos Ab, Ac, Bc, Ba, Ca y Cb están en una misma cónica de ecuación:

    (b+c-a)^2(a+b-3c)(a-3b+c)x^2 + (c+a-b)^2(b+c-3a)(b-3c+a)y^2 + (a+b-c)^2(c+a-3b)(c-3a+b)z^2 + 8S^2(yz+zx+xy) =0.

    Su centro y perspector coinciden en el centro de coordenadas:

    ( 1/((b+c-a)(a^3-a^2(b+c) - a(b^2-8b c+c^2) + (b+c)(b^2-4b c+c^2)) : ... : ...),

    que tiene (6-9-13)-número de búsqueda en ETC: -1.411602064280868133943591227.
    ( Mostrar/Ocultar figura )
    Los triángulos ABC y el triángulo DEF formado por las polares de sus vértices respecto a la cónica son homotéticos. En el caso general que la cónica tenga por ecuación fx^2 + gy^2 + hz^2 + 2k(yz + zx + xy) = 0, el centro de homotecia (perspector de la cónica) es:

    (1/(f-k) : 1/(g-k) : 1/(h-k)),

    y la razón de homotecia es:

    (k^2(f+g+h-2k)-fgh)/((f-k)(g-k)(h-k)).

    El centro de perspectividad del triángulo antimedial A'B'C' y el triángulo DEF formado por las polares de sus vértices respecto a la cónica son homotéticos, con centro de homotecia:

    (f g h - 2 g h k - f k^2 + g k^2 + h k^2, f g h - 2 f h k + f k^2 - g k^2 + h k^2, f g h - 2 f g k + f k^2 + g k^2 - h k^2).



  • viernes, 9 de agosto del 2013

    Construir un triángulo dado a, A, (b-c)/(b+c)

    (Anopolis #777 Antreas P. Hatzipolakis)

    Trazamos un segmento BC de longitud a dada.
    Sea un punto V sobre el arco capaz de ángulo A sobre el segmento BC.
    La circunferencia de centro V y radio VB corta a la recta CV en los punto S y D, tales que CS=VC+VB y SD=VC-CB.
    El punto Cv que divide al segmento DS en la relación DCv : CvS = m:n, describe una circunferencia que pasa por B y C.
    Deberemos localizar la ubicación del punto V tal que Cv coincida con C.
    El punto V solución es la intersección de la tangente en C a tal circunferencia con el arco capaz considerado.

    ( Mostrar/Ocultar figura )

    Más construcciones de triángulos en: http://amontes.webs.ull.es/pdf/trresolu.pdf

  • jueves, 8 de agosto del 2013

    Propiedad del centro X3663

    (Anopolis #786 Antreas P. Hatzipolakis)

    Sean ABC un triángulo, I el incentro y A'B'C' el triángulo medial y A"B"C" el triángulo ceviano del incentro.
    Se toman los puntos

    Ab=BB" ∩ B'C',   Ac=CC" ∩ B'C',   Bc=CC" ∩ C'A',   Ba=AA" ∩ C'A',   Ca=AA" ∩ A'B',   Cb=BB" ∩ A'B'.

    La tangente en Ab a la cónica que pasa por los puntos Ab, Bc, Ba, Ca, Cb, de ecuación baricéntrica:

    (-a^3-a(b+c)^2+c(b+c)^2+ a^2(2b+c))x+ (a-c)(a-b+c)^2y + (-a^3+3a^2c-(b-c)^2c+ a(b^2-4b c-c^2))z=0,

    y la tangente en Ac a la cónica que pasa por los puntos Ac, Bc, Ba, Ca, Cb, de ecuación baricéntrica:

    (a^3+a(b+c)^2-b(b+c)^2-a^2(b+2c))x + (a^3-3a^2b+b(b-c)^2+ a(b^2+4b c-c^2))y -(a-b)(a+b-c)^2z=0,

    se cortan en el centro X3663:

    ( (b-c)^2+a(b+c) : (c-a)^2+b(c+a) : (a-b)^2+c(a+b) ).

    ( Mostrar/Ocultar figura )

    Un proceso cíclico en la construcción anterior nos conduce al mismo centro del triángulo.

  • miércoles, 7 de agosto del 2013

    La cúbica de McCay como lugar geométrico

    ( AdvancedPlaneGeometry #458 Dominik Burek - Bernard Gibert)

    Sean ABC un triángulo, P un punto, O1, O2 y O3 los circuncentros de los triángulos BCP, CAP y ABP. Entonces el triángulo O1O2O3 y el triángulo pedal del conjugado isogonal P* de P, con respecto a ABC, son homotéticos con centro de homotecia, si P(u:v:w), de coordenadas baricéntricas:

    Q = ( a^2v w(a^2(u^2+2v w + u(v+w)) - u(b^2(u+v-w) + c^2(u-v+w)): ... : ... )


    Los puntos P, P*, QP, Q, O ) están alineados si y solo si P está sobre la cúbica de McCay (K003 = pK(X6,X3) del catálogo de Bernard Gibert):
    Σ a^2(b^2+c^2-a^2)x(c^2y^2-b^2z)=0.
    ( Mostrar/Ocultar figura )


  • martes, 6 de agosto del 2013

    Lugares geométricos y circunferencias asociadas al incentro

    (Anopolis #759 Antreas P. Hatzipolakis)

    Sean ABC un triángulo, I el incentro y Oa, Ob, Ob los circuncentros de los triángulos IBC, ICA, IAB, respectivamente.
    Sean P un punto y Aa, Ab, Ac las proyecciones ortogonales de Oa sobre AP, BP, CP, respectivamente, y O1 el circuncentro del triángulo AaAbAc. Similarmente, se definen O2 y O3.

    El circuncentro OP del triángulo O1O2O3 es el punto medio de P y el circuncentro O de ABC.

    ( Mostrar/Ocultar figura )

    Los ejes radicales de la circunferencia circunscrita a O1O2O3 y las circunscritas a AaAbAc, BaBbBc, CaCbCc delimitan un triángulo A'B'C' homotético a ABC.
    Si P=(u:v:w) el centro de homotecia Q tiene coordenadas baricéntricas:

    Q = ( a(2b c u^2 - a^2v w- b(b-2c)u w + c(2b-c)u v):
    b(2c a v^2 - b^2w u- c(c-2a)v u + a(2c-a)v w):
    c(2a b w^2 - c^2u v- a(a-2b)w v + b(2a-b)w u) ).

    • El punto Q coincide con P, cuando éste está en la circunferencia circunscrita o coincide con el incentro.

    • Cuando el punto P varía sobre una recta, el punto Q describe un cónica que pasa por el incentro y por los puntos de intersección de la recta con la circunferencia circunscrita, si existen.

    Si P recorre una recta tangente a la circunferencia circunscrita en un punto D, la cónica descrita por el punto Q es tangente a dicha recta en Q y a la paralela por el incentro en éste (por lo que el centro es el punto medio de ID.
    ( Mostrar/Ocultar figura )


  • lunes, 5 de agosto del 2013

    Las cúbicas K007 y K279 como lugares geométricos

    (Anopolis #758 Antreas P. Hatzipolakis)

    Sean ABC un triángulo, A'B'C' el triángulo ceviano de un punto P y Oa, Ob, Oc los centros de las circunferencias circunscritas a los triángulos AB'C', BC'A', CA'B', respectivamente, los cuales se cortan en un punto M. Se denota por O* el circuncentro del triángulo OaObOc.

    • El lugar geométrico de los puntos P tales que O* está en la recta de Euler es la cúbica (K279 = pK(X2,X3260) del catálogo de Bernard Gibert):
    Σ b^2c^2 (2a^4-a^2(b^2+c^2)-(b^2-c^2)^2)x(y^2-z^2)=0.

    ( Mostrar/Ocultar figura )
    • El lugar geométrico de los puntos P tales que M, O*, O están alineados es la cúbica de Lucas (K007 = pK(X2,X69) del catálogo de Bernard Gibert):
    Σ (b^2+c^2-a^2)x(y^2-z^2)=0.
    ( Mostrar/Ocultar figura )
    Cuando P recorre la cúbica de Lucas, el punto M recorre la cúbica de Darboux.

  • domingo, 4 de agosto del 2013

    Cónica conteniendo ortocentros

    ( AdvancedPlaneGeometry #436 Barry Wolk)

    Sean ABC un triángulo, D, E, F puntos sobre los lados BC, CA, AB, respectivamente. Se denota por Ha, Hb, Hc los ortocentros de los triángulos AEF, BFD, CDE, respectivamente, entonces los puntos D, E, F, Ha, Hb, Hc quedan sobre una cónica.

    ( Mostrar/Ocultar figura )
    Considérese el hexágono DHcEHaFHbD. Las rectas DHc y FHa son ambas perpendiculares a AC, y cíclicamente; así, los lados opuestos de este hexágono son paralelos (se cortan dos a dos en la recta del infinito). Por el recíproco del teorema de Pascal, estos seis puntos están sobre una cónica.

  • sábado, 3 de agosto del 2013

    Centros de circunferencias de nueve puntos sobre rectas por el circuncentro

    (Anopolis #728 Antreas P. Hatzipolakis)

    Sean ABC un triángulo, A'B'C' el triángulo ceviano de un punto P y A", B", C" los puntos simétricos de P respecto a A', B', C', respectivamente.

    El centro N" de la circunferencia de los nueve puntos de A"B"C" está en la recta OP si P queda en una séptica

      c^4 x^4 y^3 - c^4 x^3 y^4 + a^2 c^2 x^4 y^2 z + 2 b^2 c^2 x^4 y^2 z - c^4 x^4 y^2 z + 2 a^2 c^2 x^3 y^3 z - 2 b^2 c^2 x^3 y^3 z - 2 a^2 c^2 x^2 y^4 z - b^2 c^2 x^2 y^4 z + c^4 x^2 y^4 z - a^2 b^2 x^4 y z^2 + b^4 x^4 y z^2 - 2 b^2 c^2 x^4 y z^2 + a^2 b^2 x^3 y^2 z^2 - b^4 x^3 y^2 z^2 - a^2 c^2 x^3 y^2 z^2 + c^4 x^3 y^2 z^2 + a^4 x^2 y^3 z^2 - a^2 b^2 x^2 y^3 z^2 + b^2 c^2 x^2 y^3 z^2 - c^4 x^2 y^3 z^2 - a^4 x y^4 z^2 + a^2 b^2 x y^4 z^2 + 2 a^2 c^2 x y^4 z^2 - b^4 x^4 z^3 - 2 a^2 b^2 x^3 y z^3 + 2 b^2 c^2 x^3 y z^3 - a^4 x^2 y^2 z^3 + b^4 x^2 y^2 z^3 + a^2 c^2 x^2 y^2 z^3 - b^2 c^2 x^2 y^2 z^3 + 2 a^2 b^2 x y^3 z^3 - 2 a^2 c^2 x y^3 z^3 + a^4 y^4 z^3 + b^4 x^3 z^4 + 2 a^2 b^2 x^2 y z^4 - b^4 x^2 y z^4 + b^2 c^2 x^2 y z^4 + a^4 x y^2 z^4 - 2 a^2 b^2 x y^2 z^4 - a^2 c^2 x y^2 z^4 - a^4 y^3 z^4=0
    circunscrita a los triángulos ABC, medial y excentral, y que pasa por el incentro, baricentro y circuncentro.
    ( Mostrar/Ocultar figura )
    Las tangentes en los vértices del triángulo medial son las mediatrices de ABC.

    • Más en general, si en vez de tomar las reflexiones de P en los vértices de su triángulo ceviano, se consideran los puntos At, Bt, Ct sobre las rectas AA', BB', CC', respectivamente, tales que AtA' / AtP = BAtB'/BtP = CtC'/CtP = t.
    El centro Nt de la circunferencia de los nueve puntos de AtBtCt está en la recta OP, cuando t varía, si P queda en la misma séptica anterior.


  • viernes, 2 de agosto del 2013

    Propiedad del centro del triángulo X3579

    (Anopolis #741 Antreas P. Hatzipolakis)

    Sean ABC un triángulo, entonces el centro de la circunferencia de los nueve puntos del triángulo medial DEF del triángulo excentral IaIbIc es el centro X3579.

    ( Mostrar/Ocultar figura )


  • jueves, 1 de agosto del 2013

    Cuárticas asociadas a triángulos circuncevianos

    Sean ABC un triángulo, P un punto, DEF el triángulo circunceviano de P y D1, E1, F1 los puntos de intersección de BC, CA, AB con EF, FD, DE, respectivamente. Los puntos D1, E1, F1 están en una recta que denotamos por dP y, si P=(u:v:w), el tripolo de dP (respeto a ABC) es el punto:

    U = ( 1/(c^2v+b^2w) : 1/(a^2w+c^2u) : 1/(b^2u+a^2v) ),

    que es el "cevapoint" de P y el simediano.
    ( Mostrar/Ocultar figura )

    Las rectas AD1, BE1, CF1 vuelven a cortar a la circunferencia circunscrita en A', B', C', respectivamente. Los triángulos DEF y A'B'C' son perspectivos con centro de perspectividad:

    V = ( (c^2uv+b^2uw-a^2vw)/(c^2v+b^2w) : (a^2wu+c^2vu-b^2wu)/(a^2w+c^2u) : (b^2wu+a^2wv-c^2uv)/(b^2u+a^2v) ).

    que es el producto baricéntrico de U y el anticomplemento del conjugado isogonal P* de P (ADGEOM #422).

    Los puntos P, U, V están alineados.

    El triángulo A"B"C" determinado por las rectas AA', BB', CC' es perspectivo con ABC y con A'B'C', con centros de perspectividad U y V, respectivamente.

    • El punto U está en la recta del infinito (la recta dP es tangente a la elipse inscrita de Steiner) si y solo si P está sobre la elipse circunscrita de Steiner del triángulo tangencial TaTbTc.
    ( Mostrar/Ocultar figura )


    • El punto V está en la recta del infinito si y solo si P está sobre una cuártica
      b^2*c^4*x^3*y + a^2*c^4*x^2*y^2 + b^2*c^4*x^2*y^2 - c^6*x^2*y^2 + a^2*c^4*x*y^3 + b^4*c^2*x^3*z + a^2*b^2*c^2*x^2*y*z + a^2*b^2*c^2*x*y^2* z + a^4*c^2*y^3*z + a^2*b^4*x^2*z^2 - b^6*x^2*z^2 + b^4*c^2*x^2*z^2 + a^2*b^2*c^2*x*y*z^2 - a^6*y^2*z^2 + a^4*b^2*y^2*z^2 + a^4*c^2*y^2*z^2 + a^2*b^4*x*z^3 + a^4*b^2*y*z^3=0
    tangente en A, B,C a los lados del triángulo tangencial, y que pasa por los vértices de éste.
    ( Mostrar/Ocultar figura )

    • Las rectas paralelas a BC, CA, AB que pasan respectivamente por los vértices Ta, Tb, Tc del triángulo tangencial, determinan un triángulo T'aT'bT'c, que es perspectivo con el triángulo DEF circunceviano de P si y sólo si P está sobre una cuártica
      a^2*b^4*c^4*x^3*y - b^6*c^4*x^3*y + a^2*b^2*c^6*x^3*y + b^2*c^8*x^3*y + 2*a^4*b^2*c^4*x^2*y^2 - 2*a^2*b^4*c^4*x^2*y^2 + a^6*c^4*x*y^3 - a^4*b^2*c^4*x*y^3 - a^2*b^2*c^6*x*y^3 - a^2*c^8*x*y^3 - a^2*b^6*c^2*x^3*z - b^8*c^2*x^3*z - a^2*b^4*c^4*x^3*z + b^4*c^6*x^3*z - a^4*b^4*c^2*x^2*y*z - a^2*b^6*c^2*x^2*y*z + a^4*b^2*c^4*x^2*y* z + a^2*b^2*c^6*x^2*y*z + a^6*b^2*c^2*x*y^2*z + a^4*b^4*c^2*x*y^2* z - a^2*b^4*c^4*x*y^2*z - a^2*b^2*c^6*x*y^2*z + a^8*c^2*y^3*z + a^6*b^2*c^2*y^3*z + a^4*b^2*c^4*y^3*z - a^4*c^6*y^3*z - 2*a^4*b^4*c^2*x^2*z^2 + 2*a^2*b^4*c^4*x^2*z^2 - a^6*b^2*c^2*x*y*z^2 + a^2*b^6*c^2*x*y* z^2 - a^4*b^2*c^4*x*y*z^2 + a^2*b^4*c^4*x*y*z^2 + 2*a^4*b^4*c^2*y^2* z^2 - 2*a^4*b^2*c^4*y^2*z^2 - a^6*b^4*x*z^3 + a^2*b^8*x*z^3 + a^4*b^4*c^2*x*z^3 + a^2*b^6*c^2*x*z^3 - a^8*b^2*y*z^3 + a^4*b^6*y*z^3 - a^6*b^2*c^2*y*z^3 - a^4*b^4*c^2*y*z^3=0
    circunscrita a ABC y a >T'aT'bT'c, que pasa por los centros X1, X3, X25, X56, X251, X5004, X5005.

    Para los puntos P = X1, X3, X25, X56, X251 los centros de perspectividad Q son, respectivamente:

    -- X3,
    -- (a^2 (a^8 - 8a^4b^2c^2 - 2a^6(b^2+c^2) + 2a^2(b^2+c^2)^3- (b^2-c^2)^4):...:...), está en la recta de Euler.
    -- X25,
    -- ( a^2 (a+b-c)(a-b+c)(a^6 - a^4(b-c)^2 + 4a^3b c(b+c) - a^2(b^4-4b^3c+22 b^2c^2-4b c^3+c^4) + 4a b c(b+c)^3 + (b-c)^2(b+c)^4 ) : ... : ...),
    -- X1180.

    ( Mostrar/Ocultar figura )


  • miércoles, 31 de julio del 2013

    Circunferencias concurrentes y conjugados isogonales de ortocentros

    (Anopolis #532 Antreas P. Hatzipolakis)

    Sean ABC un triángulo, P un punto y Ha, Hb, Hc los ortocentros de los triángulos PBC, PCA, PAB, respectivamente.
    Denotamos por Da, Db, Dc los conjugados isogonales de Ha, Hb, Hc, con respecto al triángulo ABC.

    • Los puntos Da, Db, Dc están alineados con el circuncentro O en la recta dP de ecuación baricéntrica, si P=(u:v:w),
    dP:   b^2c^2u(a^2(w-v)+(b^2-c^2)(v+w))x + a^2c^2v(b^2(u-w)-a^2(u+w)+c^2(u+w))y + a^2b^2w(c^2(v-u) + a^2(u+v)- b^2(u+v))z =0.

    ( Mostrar/Ocultar figura )

    • Las circunferencias circunscritas a los triángulos DaBC, DbCA, DcAB son concurrentes en el punto D, sobre la recta dP, de coordenadas
    D = (a^2 v w (-a^2 (u + v) (u + w) + u (b^2 (u + v) + c^2 (u + w))) : b^2 u w (a^2 v (u + v) - (-c^2 v + b^2 (u + v)) (v + w)) : c^2 u v (-c^2 (u + w) (v + w) + w (a^2 (u + w) + b^2 (v + w))).

    El punto D es el QA-P4 in EQF (Encyclopedia of Quadri-Figures, Chris van Tienhoven)

    • Las circunferencias circunscritas a los triángulos ADbDc, BDcDa, CDaDb son concurrentes en el punto D', sobre la sobre la circunferencia circunscrita, de coordenadas baricéntricas:
    D' = ( a^2 (a^2 u - b^2 u - c^2 u + a^2 v - b^2 v + c^2 v) (a^2 u - b^2 u - c^2 u + a^2 w + b^2 w - c^2 w) (a^2 c^2 u^2 v^2 - b^2 c^2 u^2 v^2 - c^4 u^2 v^2 + a^2 c^2 u v^3 - b^2 c^2 u v^3 + c^4 u v^3 - a^4 u^2 v w + 3 a^2 b^2 u^2 v w - 2 b^4 u^2 v w + 2 a^2 c^2 u^2 v w - b^2 c^2 u^2 v w - c^4 u^2 v w + 2 a^2 b^2 u v^2 w - 2 b^4 u v^2 w + 2 b^2 c^2 u v^2 w + a^4 v^3 w - a^2 b^2 v^3 w + a^2 c^2 v^3 w - 2 b^4 u^2 w^2 - a^4 u v w^2 - a^2 b^2 u v w^2 - 2 b^4 u v w^2 + 2 a^2 c^2 u v w^2 + 3 b^2 c^2 u v w^2 - c^4 u v w^2 - a^4 v^2 w^2 - a^2 b^2 v^2 w^2 + a^2 c^2 v^2 w^2) (2 c^4 u^2 v^2 + a^4 u^2 v w - 2 a^2 b^2 u^2 v w + b^4 u^2 v w - 3 a^2 c^2 u^2 v w + b^2 c^2 u^2 v w + 2 c^4 u^2 v w + a^4 u v^2 w - 2 a^2 b^2 u v^2 w + b^4 u v^2 w + a^2 c^2 u v^2 w - 3 b^2 c^2 u v^2 w + 2 c^4 u v^2 w - a^2 b^2 u^2 w^2 + b^4 u^2 w^2 + b^2 c^2 u^2 w^2 - 2 a^2 c^2 u v w^2 - 2 b^2 c^2 u v w^2 + 2 c^4 u v w^2 + a^4 v^2 w^2 - a^2 b^2 v^2 w^2 + a^2 c^2 v^2 w^2 - a^2 b^2 u w^3 - b^4 u w^3 + b^2 c^2 u w^3 - a^4 v w^3 - a^2 b^2 v w^3 + a^2 c^2 v w^3) : ... : ... ).

    • Si P está en la circunferencia circunscrita, el ortocentro de HaHbHc es P y los triángulos ABC y HaHbHc son simétricos respecto al punto medio (sobre la circunferencia de los nueve puntos) de P y el ortocentro H.

    Cuando P varía sobre la circunferencia circunscrita, el punto D coincide con el circuncentro y el punto Q=PH∩dP describe la cúbica de Lemoine (K009 del catálogo de Bernard Gibert).
    ( Mostrar/Ocultar figura )
    La circunferencia circunscrita a OBC (donde queda Da) interseca a la cúbica de Lemoine además en dos puntos A1, A2. Procediendo cíclicamente, de definen los dos pares de puntos B1, B2 y C1, C2.
    Las rectas A1A2, B1BA2 y C1, C2 concurren en X1147, sobre la cúbica.

  • martes, 30 de julio del 2013

    Circunferencias concurrentes asociadas al triángulo ceviano de un punto

    (Anopolis #729 Antreas P. Hatzipolakis)

    Sean ABC un triángulo, P un punto y A'B'C' el triángulo ceviano de P. Se denota por Pa, Pb, Pc los conjugados isogonales de P con respecto a los triángulos AB'C', BC'A', CA'B', respectivamente.
    Entonces:
    1. Las circunferencias circunscritas a los triángulos PaB'C', PbC'A', PcA'B' son concurrentes en un punto R1.
    2. Las circunferencias circunscritas a los triángulos A'PbPc, B'PcPa, C'PaPb son concurrentes en un punto R2.

    ( Mostrar/Ocultar figura )
    Si P(u:v:w) se tiene que:
    Pa = (-a^2(u+v)(u+w) + u(b^2(u+v-w) + c^2(u-v+w)) : b^2u*w : c^2u*v),
    Pb = (a^2v*w : -b^2(v+w)(v+u) + v(c^2(v+w-u) + a^2(v-w+u)) : c^2u*v),
    Pc = (a^2v*w : b^2u*w : -c^2(w+u)(w+v) + w(a^2(w+u-v) + b^2(w-u+v))).


    Las coordenadas baricéntricas de R1=( f1(u,v,w)
      a^2*(-(a^2*c^2*u^4*v^2) + b^2*c^2*u^4*v^2 + c^4*u^4*v^2 - a^2*c^2*u^3*v^3 + b^2*c^2*u^3*v^3 + a^4*u^4*v*w - 2*a^2*b^2*u^4*v*w + b^4*u^4*v*w - 2*a^2*c^2*u^4*v*w + 2*b^2*c^2*u^4*v*w + c^4*u^4*v*w + 2*a^4*u^3*v^2*w - 4*a^2*b^2*u^3*v^2*w + 2*b^4*u^3*v^2*w - 4*a^2*c^2*u^3*v^2*w + b^2*c^2*u^3*v^2*w + 2*c^4*u^3*v^2*w + a^4*u^2*v^3*w - 2*a^2*b^2*u^2*v^3*w + b^4*u^2*v^3*w - 2*a^2*c^2*u^2*v^3*w - b^2*c^2*u^2*v^3*w - a^2*b^2*u^4*w^2 + b^4*u^4*w^2 + b^2*c^2*u^4*w^2 + 2*a^4*u^3*v*w^2 - 4*a^2*b^2*u^3*v*w^2 + 2*b^4*u^3*v*w^2 - 4*a^2*c^2*u^3*v*w^2 + b^2*c^2*u^3*v*w^2 + 2*c^4*u^3*v*w^2 + 4*a^4*u^2*v^2*w^2 - 5*a^2*b^2*u^2*v^2* w^2 + b^4*u^2*v^2*w^2 - 5*a^2*c^2*u^2*v^2*w^2 - 2*b^2*c^2*u^2* v^2*w^2 + c^4*u^2*v^2*w^2 + 2*a^4*u*v^3*w^2 - 2*a^2*b^2*u*v^3*w^2 - a^2*c^2*u*v^3*w^2 - a^2*b^2*u^3*w^3 + b^2*c^2*u^3*w^3 + a^4*u^2*v*w^3 - 2*a^2*b^2*u^2*v*w^3 - 2*a^2*c^2*u^2*v* w^3 - b^2*c^2*u^2*v*w^3 + c^4*u^2*v*w^3 + 2*a^4*u*v^2*w^3 - a^2*b^2*u*v^2*w^3 - 2*a^2*c^2*u*v^2* w^3 + a^4*v^3*w^3)* (-(a^2*c^2*u^4*v^2) + b^2*c^2*u^4*v^2 + c^4*u^4*v^2 - 2*a^2*c^2*u^3*v^3 + 2*b^2*c^2*u^3*v^3 - a^2*c^2*u^2*v^4 + b^2*c^2*u^2*v^4 - c^4*u^2*v^4 + a^4*u^4*v*w - 2*a^2*b^2*u^4*v*w + b^4*u^4*v*w - 2*a^2*c^2*u^4*v*w + 2*b^2*c^2*u^4*v*w + c^4*u^4*v*w + 2*a^4*u^3*v^2*w - 4*a^2*b^2*u^3*v^2*w + 2*b^4*u^3*v^2*w - 4*a^2*c^2*u^3*v^2*w + 4*b^2*c^2*u^3*v^2*w + 2*c^4*u^3*v^2*w + a^4*u^2*v^3*w - 2*a^2*b^2*u^2*v^3*w + b^4*u^2*v^3*w - 4*a^2*c^2*u^2*v^3*w + 4*b^2*c^2*u^2*v^3*w - 2*c^4*u^2*v^3*w - 2*a^2*c^2*u*v^4*w + 2*b^2*c^2*u*v^4*w - 2*c^4*u*v^4*w - a^2*b^2*u^4*w^2 + b^4*u^4*w^2 + b^2*c^2*u^4*w^2 + 2*a^4*u^3*v*w^2 - 4*a^2*b^2*u^3*v*w^2 + 2*b^4*u^3*v*w^2 - 4*a^2*c^2*u^3*v*w^2 + 4*b^2*c^2*u^3*v*w^2 + 2*c^4*u^3*v*w^2 + 4*a^4*u^2*v^2*w^2 - 4*a^2*b^2*u^2*v^2* w^2 - 4*a^2*c^2*u^2*v^2*w^2 + 6*b^2*c^2*u^2*v^2*w^2 + 2*a^4*u*v^3*w^2 - 2*b^4*u*v^3*w^2 - 2*a^2*c^2*u*v^3*w^2 + 6*b^2*c^2*u*v^3* w^2 - 4*c^4*u*v^3*w^2 + a^2*b^2*v^4*w^2 - b^4*v^4*w^2 - a^2*c^2*v^4*w^2 + 2*b^2*c^2*v^4*w^2 - c^4*v^4*w^2 - 2*a^2*b^2*u^3*w^3 + 2*b^2*c^2*u^3*w^3 + a^4*u^2*v*w^3 - 4*a^2*b^2*u^2*v*w^3 - 2*b^4*u^2*v*w^3 - 2*a^2*c^2*u^2*v*w^3 + 4*b^2*c^2*u^2*v* w^3 + c^4*u^2*v*w^3 + 2*a^4*u*v^2*w^3 - 2*a^2*b^2*u*v^2*w^3 - 4*b^4*u*v^2*w^3 + 6*b^2*c^2*u*v^2*w^3 - 2*c^4*u*v^2*w^3 + a^4*v^3*w^3 - 2*b^4*v^3*w^3 + 4*b^2*c^2*v^3*w^3 - 2*c^4*v^3*w^3 - a^2*b^2*u^2*w^4 - b^4*u^2*w^4 + b^2*c^2*u^2*w^4 - 2*a^2*b^2*u*v*w^4 - 2*b^4*u*v*w^4 + 2*b^2*c^2*u*v*w^4 - a^2*b^2*v^2*w^4 - b^4*v^2*w^4 + a^2*c^2*v^2*w^4 + 2*b^2*c^2*v^2*w^4 - c^4*v^2*w^4)
    :g1(u,v,w):h1(u,v,w)) son polinomios de grado 12.
    Las coordenadas baricéntricas de R2=( f2(u,v,w)
      a^10*v^3*(u + v)^3*(v - w)^2*w^3*(u + w)^3* (u + v + w) - u^3*(-(c^2*v) + b^2*(u + v))*(v + w)^5* (-(b^2*w) + c^2*(u + w))* (b^2*(u + v)*w - c^2*v*(u + w))^2* (b^2*(u + v - w) + c^2*(u - v + w)) + a^8*v^2*(u + v)^2*(v - w)*w^2*(u + w)^2* (-(b^2*(u + v)*w*(u^3*w + 2*v*w*(v + w)^2 + u^2*(3*v^2 + 2*v*w + 2*w^2) + u*(3*v^3 + 4*v^2*w + 2*v*w^2 + w^3))) + c^2*v*(u + w)* (u^3*v + 2*v*w*(v + w)^2 + u^2*(2*v^2 + 2*v*w + 3*w^2) + u*(v^3 + 2*v^2*w + 4*v*w^2 + 3*w^3))) - a^2*u^2*(v + w)^3* (-(b^8*(u + v)^3*w^3*(u^3*w - v*w*(v + w)^2 - u^2*(3*v^2 + v*w + w^2) + u*(-3*v^3 - 2*v^2*w + v*w^2 - 2*w^3))) + c^8*v^3*(u + w)^3*(-(u^3*v) + v*w*(v + w)^2 + u^2*(v^2 + v*w + 3*w^2) + u*(2*v^3 - v^2*w + 2*v*w^2 + 3*w^3)) - 2*b^6*c^2*(u + v)^2*w^2* (2*v^2*w^2*(v + w)^2 + u^4*(v^2 + w^2) + u^3*(5*v^3 + 3*v^2*w + v*w^2 + 2*w^3) + u*v*w*(7*v^3 + 6*v^2*w + 2*v*w^2 + 5*w^3) + u^2*(4*v^4 + 8*v^3*w + 3*v^2*w^2 + 6*v*w^3 + w^4)) - 2*b^2*c^6*v^2* (u + w)^2*(2*v^2*w^2*(v + w)^2 + u^4*(v^2 + w^2) + u^3*(2*v^3 + v^2*w + 3*v*w^2 + 5*w^3) + u*v*w*(5*v^3 + 2*v^2*w + 6*v*w^2 + 7*w^3) + u^2*(v^4 + 6*v^3*w + 3*v^2*w^2 + 8*v*w^3 + 4*w^4)) + b^4*c^4*v*(u + v)*w*(u + w)* (6*v^2*w^2*(v + w)^2 + 2*u^4*(2*v^2 + v*w + 2*w^2) + 6*u*v*w*(3*v^3 + 2*v^2*w + 2*v*w^2 + 3*w^3) + u^3*(11*v^3 + 7*v^2*w + 7*v*w^2 + 11*w^3) + u^2*(7*v^4 + 20*v^3*w + 12*v^2*w^2 + 20*v*w^3 + 7*w^4))) + a^6*v*(u + v)*w*(u + w)* (b^4*(u + v)^2*w^2*(v^2*w^2*(v + w)^3 + u^4*v*w*(3*v + w) + 2*u*v*w*(v + w)^2* (2*v^2 - v*w + w^2) + u^3*(2*v^4 + 4*v^3*w + 5*v^2*w^2 - w^4) + u^2*(2*v^5 + 3*v^4*w + 5*v^3*w^2 + 7*v^2*w^3 + 2*v*w^4 - w^5)) + c^4*v^2*(u + w)^2* (v^2*w^2*(v + w)^3 + u^4*v*w* (v + 3*w) + 2*u*v*w*(v + w)^2* (v^2 - v*w + 2*w^2) + u^3*(-v^4 + 5*v^2*w^2 + 4*v*w^3 + 2*w^4) + u^2*(-v^5 + 2*v^4*w + 7*v^3*w^2 + 5*v^2*w^3 + 3*v*w^4 + 2*w^5)) - 2*b^2*c^2*v*w* (v^3*w^3*(v + w)^3 + 4*u*v^2*w^2* (v + w)^2*(v^2 + w^2) + 2*u^5*(v^2 + v*w + w^2)^2 + u^6*(v^3 + v^2*w + v*w^2 + w^3) + u^3*v*w*(6*v^4 + 15*v^3*w + 20*v^2*w^2 + 15*v*w^3 + 6*w^4) + u^4*(v^5 + 6*v^4*w + 14*v^3*w^2 + 14*v^2*w^3 + 6*v*w^4 + w^5) + u^2*v*w*(3*v^5 + 10*v^4*w + 16*v^3*w^2 + 16*v^2*w^3 + 10*v*w^4 + 3*w^5))) - a^4*u*(v + w)*(-(b^2*c^4*v^2*w*(u + w)^2* (-3*v^3*(v - w)*w^2*(v + w)^2 + 2*u^5*v*(v^2 + v*w + w^2) + u^4*(9*v^4 + 10*v^3*w + 5*v^2*w^2 - 6*v*w^3 - 6*w^4) + u*v^2*w* (4*v^4 + 6*v^3*w - 5*v^2*w^2 - 6*v*w^3 + w^4) + u^2*v*(5*v^5 + 11*v^4*w + 16*v^3*w^2 - v^2*w^3 - 19*v*w^4 - 8*w^5) + u^3*(12*v^5 + 15*v^4*w + 10*v^3*w^2 - 5*v^2*w^3 - 16*v*w^4 - 6*w^5))) + c^6*v^3*(u + w)^3* (-(v^2*(v - w)*w^2*(v + w)^2) + u^4*v*(v^2 + 2*v*w + 3*w^2) + u*v^2*w*(2*v^3 + 3*v^2*w - 2*v*w^2 - 3*w^3) + 2*u^3*(v^4 + v^3*w + 2*v^2*w^2 - w^4) + u^2*(v^5 + 2*v^4*w + 6*v^3*w^2 + 2*v^2*w^3 - 5*v*w^4 - 2*w^5)) + b^6*(u + v)^3*w^3*(v^2*(v - w)*w^2* (v + w)^2 + u^4*w*(3*v^2 + 2*v*w + w^2) + u*v*w^2*(-3*v^3 - 2*v^2*w + 3*v*w^2 + 2*w^3) + 2*u^3*(-v^4 + 2*v^2*w^2 + v*w^3 + w^4) + u^2*(-2*v^5 - 5*v^4*w + 2*v^3*w^2 + 6*v^2*w^3 + 2*v*w^4 + w^5)) - b^4*c^2*v*(u + v)^2*w^2* (3*v^2*(v - w)*w^3*(v + w)^2 + 2*u^5*w*(v^2 + v*w + w^2) + u*v*w^2*(v^4 - 6*v^3*w - 5*v^2*w^2 + 6*v*w^3 + 4*w^4) + u^4*(-6*v^4 - 6*v^3*w + 5*v^2*w^2 + 10*v*w^3 + 9*w^4) + u^2*w*(-8*v^5 - 19*v^4*w - v^3*w^2 + 16*v^2*w^3 + 11*v*w^4 + 5*w^5) + u^3*(-6*v^5 - 16*v^4*w - 5*v^3*w^2 + 10*v^2*w^3 + 15*v*w^4 + 12*w^5)))
    :g2(u,v,w):h2(u,v,w)) son polinomios de grado 15.

    Si P=(a:b:c) es el incentro,

    R1 = (a^2(a^2-b^2-b*c-c^2)(a^3+a^2b-a*b^2-b^3+a^2c+a*b*c+b^2c-a*c^2+b*c^2-c^3)
    (a^6-2a^4b^2+a^2b^4+a^2b^3c-b^5c-2a^4c^2-a^2b^2c^2+a^2b*c^3+2b^3c^3+a^2c^4-b*c^5) :...:...),

    con número de búsqueda en ETC: 2.60524307195861030629200757

    R2 = ( (a-b-c) (b-c)^2 (a^3+a^2b-a*b^2-b^3+a^2c+a*b*c+b^2c-a*c^2+b*c^2-c^3)
    (a^6-a^4b^2-a^2b^4+b^6-2a^2b^3c+2b^5c-a^4c^2-a^2b^2c^2-b^4c^2-2a^2b*c^3 -4b^3c^3-a^2c^4-b^2c^4+2b*c^5+c^6):...:...),

    con número de búsqueda en ETC: -0.96947306546315331344307981

    Si P=(1:1:1) es el baricentro.

    R1 = (a^2(4a^4-7a^2b^2-2b^4-7a^2c^2+14b^2c^2-2c^4) (4a^4-6a^2b^2+2b^4-6a^2c^2+b^2c^2+2c^4):...:...),

    con número de búsqueda en ETC: 5.184059393723475028771865549

    R2 = ((b-c)^2(b+c)^2(-7a^2+2b^2+2c^2)(-2a^4+2b^4-5b^2c^2+2c^4):...:...),

    con número de búsqueda en ETC: -0.196430427865079067265962063

  • jueves, 25 de julio del 2013

    Propiedad del centro X3649

    ( Art of Problem Solving) Sean ABC un triángulo y DEF el triángulo pedal del incentro, X1. Las paralelas por D, E, F a las rectas de Euler de los triángulos BCX1, CAX1, ABX1, concurren en el centro X3649 ( KS(INTOUCH TRIANGLE), Peter Moses and Seiichi Kirikami):

    ( (b+c)(2a+b+c)(a-b+c)(a+b-c) : (c+a)(2b+c+a)(b-c+a)(b+c-a) : (a+b)(2c+a+b)(c-a+b)(c+a-b) ).


    Las paralelas citadas anteriormente son las rectas DO1, EO2, FO3, siendo O1, O2, O3 los circuncentros de los triángulos AEF, BFD, CDE, respectivamente.

    ( Mostrar/Ocultar figura )


  • miércoles, 24 de julio del 2013

    Rectángulos de diagonal mínima inscritos en un triángulo

    ( AdvancedPlaneGeometry #387 Angel Montesdeoca)

    Sea ABC un triángulo, de los rectángulos inscritos en ABC, con uno de sus lados sobre los lados del triángulo, tomemos los tres que tienen diagonal de longitud mínima. Los lados de estos rectángulos paralelos a los de ABC, delimitan un triángulo A'B'C' homotético a ABC, con centro de homotecia en el punto de coordenadas baricéntricas:

    U = ( a^4 / (3a^4+ 2a^2(b^2 + c^2) - (b^2 - c^2)^2 ) : b^4 / (3b^4+ 2b^2(c^2 + a^2) - (c^2 - a^2)^2 ) : c^4 / (3c^4+ 2c^2(a^2 + b^2) - (a^2 -b^2)^2 ) ) =
    = ( a^4/(a^4+S^2) : b^4/(b^4+S^2) : c^4/(c^4+S^2) ),

    que tiene (6-9-13)-número de búsqueda en ETC: 1.417717949831766615928603129,

    ( Mostrar/Ocultar figura )

    En consecuencia, los seis puntos de intersección de los lados de los triángulos ABC y A'B'C' están en una cónica.

    CONSTRUCCIÓN DE LOS RECTÁNGULOS INSCRITOS

    Sea K un punto variable sobre el lado BC y KLcMbNc un rectángulo, con Lc, Mb, Nc sobre los lados AB, AC, BC, respectivamente.
    El lugar geométrico de los puntos X y X' sobre la perpendicular por K a BC tales que KX=KX'=KMb, es una hipérbola de eje secundario BC y tangente en A a AB.
    Esta hipérbola pasa por A, cuando K es el pie de la altura por A (el rectángulo KLcMbNc degenera en el segmento KA); además, como KLc es menor que la diagonal KMb=KX, la recta AB es tangente a la hipérbola. El punto A', simétrico de A respecto a BC también es de la hipérbola.
    Otros puntos sobre la hipérbola son Xb, X'b sobre la perpendicular por B a BC, tales que KXb=KX'b=BC.

    Ya podemos construir la hipérbola, dado cuatro puntos y la tangente en uno de ellos (PPPtP1). Su ecuación baricéntrica puede obtenerse teniendo en cuenta que A(1:0:0), A'(2a^2:SB:SC), Xb(-a^2:SC-S:SB), X'b(-a^2 :SC+S:SB) y la tangente en A es z=0.
    La ecuación del haz de cónicas es:

    z (SB x + a^2 z) + t (SB y + (-SC + S) z) (-SB y + (SC + S) z)=0,

    e imponiendo que pase por A', t= -2 a^2/(4 S^2).

    La posición del punto K para el cual los puntos X y X' coinciden con los vértices V y V' de la hipérbola, corresponde a un vértice del rectángulo solución de diagonal mínima.

    A la misma solución se llega si se toma el rectángulo variable KLbMcNb el rectángulo con Lb, Mc, Nb sobre los lados AC, AB, BC, respectivamente.
    Los puntos Y y Y' sobre la perpendicular por K a BC tales que KY=KY'=KMc, describen una hipérbola de eje secundario BC y tangente en A a AC. La posición del punto K para el cual los puntos Y y Y' coinciden con los vértices W y W' de la hipérbola, corresponde a un vértice del rectángulo solución.
    ( Mostrar/Ocultar figura )


    Otra construcción, más sencilla, de los rectángulos de diagonal mínima es dada por Paul Yiu :
    ( Mostrar/Ocultar figura )
    (1) Let BB'C'C be the rectangle with the line B'C' containing the vertex A.
    (2) Construct the diagonal B'C and drop the perpendicular from B to intersect at X.
    (3) Construct the parallel through X to BC to intersect AC and AB at Ba and Ca respectively.

    BaCa is the side (parallel to BC) of the inscribed rectangle with minimum diagonal.

  • sábado, 20 de julio del 2013

    Cúbicas de Darboux y central de Spieker

    (Anopolis #638 Antreas P. Hatzipolakis)

    Sean ABC un triángulo, P un punto y A'B'C' el triángulo antipedal de P. Si P está sobre la cúbica de Darboux, las paralelas a IA', IB' y I'C (I el incentro) por A,B y C, concurren en un punto R sobre la cúbica central de Spieker (K033 del catálogo de Bernard Gibert).

    ( Mostrar/Ocultar figura )


    Cuando P coincide con los antipodales de A, B y C, en la circunferencia circunscrita, el triángulo antipedal A'B'C' de P degenera en los vértices de ABC y las rectas IA', IB', I'C son paralelas a las bisectrices (direcciones de las asíntotas de la cúbica K003).

  • viernes, 19 de julio del 2013

    Cúbica de Darboux y circunferencias coaxiales

    (Anopolis #626 Antreas P. Hatzipolakis)

    Sean ABC un triángulo, A1B1C1 el triángulo órtico y P un punto. Las rectas AP,BP,CP vuelven a cortas a las circunferencias circunscritas a los triángulos PBC,PCA,PAB, resp. en A',B',C', resp.

    Las circunferencias circunscritas a los triángulos AA1A', BB1B', CC1C' son coaxiales si P está sobre la cúbica de Darboux (K004 del catálogo de Bernard Gibert).
    El eje radical de las circunferencias coaxiales pasa por el ortocentro.

    ( Mostrar/Ocultar figura )


    Si P=X1, X3, X20, X40, X64, el eje radical de las tres circunferencia coaxiales pasa por X1 . (X6 (Art of Problem Solving)), X64, X57, X2 (recta de Euler), respectivamente.

  • jueves, 18 de julio del 2013

    Triángulos a mitad de camino

    (Anopolis #616 Antreas P. Hatzipolakis)

    Sean ABC un triángulo, P un punto y P* su conjugado isogonal. Denotamos por A',B',C' los puntos medios de AP, BP, CP, respectivamente, y por A",B",C" los puntos medios de AP*, BP*, CP*, respectivamente.
    Sean D,E,F los puntos medios de A'A", B'B", C'C", respectivamente, entonces los triángulos ABC y DEF son ortológicos (las perpendiculares desde los vértices de uno de ellos a los correspondientes lados del otro son concurrentes) y los centros ortológicos son el circuncentro y el punto de coordenadas, P(u:v:w):

    (a^2vw(4u+3v+3w)+ b^2uw(3u+2v+2w) + c^2uv(3u+2v+2w)) /
    3b^2c^4u^4v^2 + a^2c^4u^3v^3 + 5b^2c^4u^3v^3 + 3c^6u^3v^3 + a^2c^4u^2v^4 + 2b^2c^4u^2v^4 + 2c^6u^2v^4 + a^2b^2c^2u^4vw + 3b^4c^2u^4vw + 3b^2c^4u^4vw + 7a^2b^2c^2u^3v^2*w + 5b^4c^2u^3v^2w + 19b^2c^4u^3v^2w + a^4c^2u^2v^3w + 9a^2b^2c^2u^2v^3w + 2b^4c^2u^2v^3w + 9a^2c^4u^2v^3w + 12b^2c^4u^2v^3w + 2c^6u^2v^3w + a^4c^2uv^4w + 3a^2b^2c^2uv^4w + 5a^2c^4uv^4w + 3b^4c^2u^4w^2 + 7a^2b^2c^2u^3vw^2 + 19b^4c^2u^3vw^2 + 5b^2c^4u^3vw^2 + 34a^2b^2c^2u^2v^2w^2 + 12b^4c^2u^2v^2w^2 + 12b^2c^4u^2v^2w^2 + 7a^4c^2uv^3w^2 + 19a^2b^2c^2uv^3w^2 + 5a^2c^4uv^3w^2 + 3a^4c^2v^4w^2 + a^2b^4u^3w^3 + 3b^6u^3w^3 + 5b^4c^2u^3w^3 + a^4b^2u^2vw^3 + 9a^2b^4u^2vw^3 + 2b^6u^2vw^3 + 9a^2b^2c^2u^2vw^3 + 12b^4c^2u^2vw^3 + 2b^2c^4u^2vw^3 + 7a^4b^2uv^2w^3 + 5a^2b^4uv^2w^3 + 19a^2b^2c^2uv^2w^3 + a^6v^3w^3 + 3a^4b^2v^3w^3 + 3a^4c^2v^3w^3 + a^2b^4u^2w^4 + 2b^6u^2w^4 + 2b^4c^2u^2w^4 + a^4b^2uvw^4 + 5a^2b^4uvw^4 + 3a^2b^2c^2uvw^4 + 3a^4b^2v^2w^4 : ... : ...).

    ( Mostrar/Ocultar figura )

    Cuando P el baricentro o el circuncentro, estos puntos son, respectivamente:

    ( (10a^2 + 7(b^2+c^2))/ (a^6+15a^4(b^2+c^2) + 3a^2(7b^4+37b^2c^2+7c^4)+7(b^6+9b^4c^2+9b^2c^4+c^6)) : ... : ...)

    ( (2a^4 - 5a^2(b^2+c^2)+ 3(b^2-c^2)^2 )/ (a^6+a^4(b^2+c^2) - 5a^2(b^4+b^2c^2+c^4) + 3(b^2-c^2)^2 (b^2+c^2)) : ... : ...)



  • miércoles, 17 de julio del 2013

    Cúbica de Neuberg y circunferencias coaxiales

    (Anopolis #616 Antreas P. Hatzipolakis)

    Sean ABC un triángulo, A'B'C' el triángulo órtico, P un punto y A", B", C" los puntos medios de AP, BP, CP, respectivamente.
    Si P está sobre la cúbica de Neuberg (K001 del catálogo de Bernard Gibert) las circunferencias circunscritas a los triángulos AA'A", BB'B", CC'C", son coaxiales, con eje la recta de Euler.

    ( Mostrar/Ocultar figura )

    Cuando P=O los puntos de concurrencia de las circunferencias son el centro de la circunferencia de los nueve puntos, X5, y el inverso del ortocentro en la circunferencia circunscrita a ABC, X186.

  • martes, 16 de julio del 2013

    Puntos de Kosnita y circunferencias concurrentes

    (Anopolis #603 Antreas P. Hatzipolakis)

    Sean ABC un triángulo, I el incentro y Na, Nb, Nc los centros de las circunferencias de los nueve puntos de los triángulos IBC, ICA, IAB, resp.
    Se denota por Ka, Kb, Kc los conjugados isogonales de Na, Nb, Nc con respecto a los triángulos IBC, ICA, IAB, resp. (es decir, Ka, Kb, Kc son los puntos de Kosnita de los triángulos IBC, ICA, IAB).
    Las coordenadas baricéntricas del punto de Kosnita de IBC son:
    Ka = (a^2(a-b)(a-c) : b(a-b)(a^2- b^2+c^2-a(b+2c)) : c(a-c)(a^2+b^2-c^2-a(2b+c)).

    Las circunferencias circunscritas a ABC, AKbKc, BKcKa, CKaKb son concurrentes en el punto X de coordenadas baricéntricas:

    X = ( a^2/((b-c)(2a^4-a^3(b+c)-a^2(3b^2+4b*c+3c^2)+a(b+c)(b^2+3b*c+c^2)+(b^2-c^2)^2)) : ... : ... ),

    que tiene (6-9-13)-número de búsqueda en ETC: -0.4151794812633478152167512428

    ( Mostrar/Ocultar figura )


    Las circunferencias circunscritas a los triángulos KaBC, KbCA, KcAB concurren en X484.

  • domingo, 14 de julio del 2013

    Triángulos medial y órtico. Conjugados isogonales

    (Anopolis #588 Antreas P. Hatzipolakis)

    Sean ABC un triángulo, HaHbHc el triángulo órtico y MaMbMc triángulo medial.

    A'=(2a^4+a^2(b^2+c^2)-(b^2-c^2)^2: b^2(a^2+b^2-c^2):c^2(a^2-b^2+c^2)) es el conjugado isogonal de Ha respecto al triángulo GMbMc, B' el conjugado isogonal de Hb respecto al triángulo GMcMa y C' el conjugado isogonal de Hc respecto al triángulo GMaMb.

    Entonces, los triángulos ABC y A'B'C' son perspectivos, con centro de perspectividad X25, el centro de homotecia de los triángulo órtico y tangencial.

    ( Mostrar/Ocultar figura )


    A"=(-a^6(b^2+c^2)+3a^4(b^4+c^4)-3a^2(b^2-c^2)^2(b^2+c^2)+(b^2-c^2)^2(b^4+c^4) : b^2(a^2-b^2+c^2)(-a^2+b^2+c^2)^2 : c^2(a^2+b^2-c^2)(-a^2+b^2+c^2)^2) es el conjugado isogonal de Ma respecto al triángulo HHbHc, B" el conjugado isogonal de Mb respecto al triángulo HHcHa y C" el conjugado isogonal de Hc respecto al triángulo HHaHb.

    Entonces, los triángulos ABC y A"B"C" son perspectivos, con centro de perspectividad el circuncentro, X3.

    El centro radical R de las circunferencias circunscritas a los triángulos AA'A", BB'B", CC'C" es el punto de coordenadas baricéntricas:

    ( a^2( 3a^16(b^2+c^2) -
    a^14(9b^4+26b^2c^2+9c^4) +
    3a^12(b^6+16b^4c^2+16b^2c^4+c^6) +
    a^10(15b^8-24b^6c^2-62b^4c^4-24b^2c^6+15c^8) -
    a^8(15b^10+12b^8c^2-11b^6c^4-11b^4c^6+12b^2c^8+15c^10) +
    a^6(-3b^12+30b^10c^2+51b^8c^4+116b^6c^6+51b^4c^8+30b^2c^10-3c^12) +
    a^4(9b^14-36b^12c^2-26b^10c^4+85b^8c^6+85b^6c^8-26b^4c^10-36b^2c^12+9c^14) -
    a^2(b^2-c^2)^2(3b^12-14b^10c^2-3b^8c^4+76b^6c^6-3b^4c^8-14b^2c^10+3c^12)-
    3b^2c^2(b^2-c^2)^6(b^2+c^2)) : ... : ... ),

    que tiene (6-9-13)-número de búsqueda en ETC: 0.3197478385508268447724636294

    Otras perspectividades:
    • Los triángulos HaHbHc y A"B"C" son simétricos respecto al centro de la circunferencia de Taylor, X389.
    • Los triángulos MaMbMc y A'B'C' son perspectivos con centro de perspectividad el punto de coordenadas baricéntricas:

    ( (a^2+2SA)(a^4+SBSC+S^2) : (b^2+2SB)(b^4+SCSA+S^2) : (c^2+2SC)(c^4+SASB+S^2) ),

    que tiene (6-9-13)-número de búsqueda en ETC: 0.6225282852339818799280226572

  • sábado, 13 de julio del 2013

    Triángulos circuncevianos ortológicos con el triángulo de referencia

    (Anopolis #579 Antreas P. Hatzipolakis)

    Sean ABC un triángulo y P un punto, denotamos por A'B'C' el triángulo circunceviano de P.
    Ab y Ac son las proyecciones ortogonales de A' sobre BB' y CC', respectivamente y Ha el ortocentro de A'AbAc. Similarmente se consideran los correspondientes ortocentros Hb y Hc.
    Los triángulos ABC y HaHbHc son ortológicos (las perpendiculares desde los vértices de uno de ellos a los correspondientes lados del otro son concurrentes) en los siguientes casos:

    1. Cuando P está sobre la circunferencia circunscrita: el triángulo circunceviano degenera en el punto P.

    2. Cuando P está sobre la cúbica de Darboux (K004 del catálogo de Bernard Gibert).

    ( Mostrar/Ocultar figura )


    3. Cuando P está en sobre una de tres circunferencias de diámetros BC, CA, AB. Ocurre en este caso que los ortocentros Ha, Hb y Hc están alineados.


  • martes, 9 de julio del 2013

    Triángulos homotéticos y la quíntica Stothers

    ( AdvancedPlaneGeometry #292 Angel Montesdeoca)

    Sean ABC un triángulo, P un punto y k un número real. A'B'C' el triángulo homotético a ABC, mediante la homotecia de centro P y razón k.
    Sea Ab=BC∩C'A', Ac=BC∩A'B', y se definen Bc, Ba, Ca, Cb cíclicamente.
    Esta seis intersecciones están en una cónica, c(P,k).
    Cuando k varía los centros Z(P,k) de las cónicas c(P,k) quedan sobre una recta d(P) que pasa por P.

    d(P) es un eje de las cónicas c(P,k), para todo k, si y solo si P está en la quíntica Stothers o en la recta del infinito".

    ( Mostrar/Ocultar figura )

    Si P(u:v:w), d(P): vw(v-w)x + wu(w-u)y + uv(u-v)z=0 es un diámetro de las cónicas c(P,k). El punto del infinito del diámetro conjugado de d(P) es (u(v-w) : v(w-u) : w(u-v)).
    Estos diámetros son perpendiculares (ejes) si sólo si (u+v+w)(c^2u^3v^2 - c^2u^2v^3 - a^2u^2v^2w + b^2u^2v^2w - b^2u^3w^2 + a^2u^2v*w^2 - c^2u^2v*w^2 - b^2u*v^2w^2 + c^2u*v^2w^2 + a^2v^3w^2 + b^2u^2w^3 - a^2v^2w^3)=0.

  • lunes, 8 de julio del 2013

    Circunferencias concurrentes en la recta de Euler del triángulo excentral

    (Anopolis #529 Antreas P. Hatzipolakis)

    Sean ABC un triángulo, A'B'C' el triángulo excentral, y A",B",C" los puntos medios de las alturas A'A, B'B, C'C del triángulo excentral
    Las circunferencias circunscritas a los triángulos A"BC, B"CA, C"AB concurren en el punto (sobre la recta de Euler del triángulo excentral) X1319 = Bevan-Schröder point.

    ( Mostrar/Ocultar figura )


    Además, Oa, Ob y Oc son los circuncentros de los triángulos A"BC, B"CA y C"AB, las rectas A"Oa, B"Ob y C"Oc concurren en el punto de coordenadas baricéntricas:

    ( a (2 a^3 - a^2 (b + c) - 2 a (b^2 - b*c + c^2) + b^3 + b^2 c + c^2 b + c^3): ... : ... ),

    que tiene (6-9-13)-número de búsqueda en ETC: 2.763124242396552353038

  • domingo, 7 de julio del 2013

    Circunferencias concurrentes en la recta de Euler

    (Anopolis #527 Antreas P. Hatzipolakis)

    Sean ABC un triángulo, A'B'C' el triángulo órtico, y A",B",C" los puntos medios de las alturas.
    Las circunferencias circunscritas a los triángulos A"B'C', B"C'A', C"A'B' concurren en el punto (sobre la recta de Euler) de coordenadas baricéntricas:

    ( SB SC(SA(c^2-b^2)^2 - SB SC(2SA-a^2)): ... : ... ),

    que tiene (6-9-13)-número de búsqueda en ETC: -3.381534618985148558065036
    ( Mostrar/Ocultar figura )


  • sábado, 6 de julio del 2013

    Hipérbola circunscrita a un triángulo dada una asíntota

    ( AdvancedPlaneGeometry #269 Paul Yiu)

    Sean ABC un triángulo y una recta h. La hipérbola circunscrita a ABC y con asíntota h, puede ser construida como en el caso (PPPtP), cuando se dan cuatro puntos y la tangente en uno de ellos.
    También podemos hacer uso del hecho de que la otra asíntota es la conjugada isotómica de la asíntota dada , y acudir a cualquiera de los casos de construcción de una hipérbola dado un punto y dos asíntotas: (hhP) o (hhP(2)).

    ( Mostrar/Ocultar figura )


  • miércoles, 3 de julio del 2013

    Rectángulos sobre los lados de un triángulo

    (Anopolis #510 Antreas P. Hatzipolakis)

    Sean ABC un triángulo, DEF su triángulo medial y t un número real. Denotamos por D', E', F' las imágenes de D, E, F mediante las homotecias de centro en el circuncentro O y razón t. Consideremos los rectángulos BCCaBa, CAAbCb, ABBcAc tales que D', E', F' son, respectivamente, los puntos medios de los lados CaBa, AbCb, BcAc.
    Entonces, las mediatrices de AbAc, BcBa, CaCb concurren en el punto, sobre la recta de Euler, P(t)=3(1+t) X(2) - (1+3t) X(3)


    Los seis puntos Ab, Ac, Bc, Ba, Ca, Cb están en una cónica con centro Z(t) = t X(2) + (1+t) X(3).

  • martes, 2 de julio del 2013

    Triángulos con la recta HI que biseca un lado del triángulo medial

    ( Art of Problem Solving)

    Enunciado:
    Si ABC es un triángulo con B', C' los puntos medios de AC, AB, respectivamente, H y I el ortocentro e incentro de ABC. Probar que si IH biseca a B'C' entonces ABC es isósceles.

    Las coordenadas baricéntricas del punto de intersección de las rectas IH y B'C' son:
    (2a^4 - a^2(b-c)^2 - a^3(b+c) + a(b-c)^2(b+c) - (b^2-c^2)^2 : (a-c)(a^3 + a^2(b-c) - a(b-c)^2 - (b-c)(b+c)^2) : (a-b)(a^3 - a(b-c)^2 - a^2(b-c) + (b-c)(b+c)^2) ).
    Para que este punto coincida con el punto medio de B'C' de coordenadas (2:1:1), ha de ocurrir que b=c (solución trivial) o bien que:
    c = a - b + (2*2^(2/3)a(a - b))/ (3^(1/3)(a(a - b)(-9b + Sqrt[-48a^2 + 48a*b + 81b^2]))^(1/3)) + (2^(1/3)(a(a - b)(-9b + Sqrt[-48a^2 + 48a*b + 81b^2]))^(1/3))/3^(2/3).

    Así, existen triángulos NO isósceles para los cuales IH y B'C'.

    ( Mostrar/Ocultar figura )


  • viernes, 21 de junio del 2013

    Triángulos equiláteros de mismo baricentro y puntos en la hipérbola de Kiepert

    ( AdvancedPlaneGeometry #214 Dao Thanh Oai)

    Sean ABC un triángulo y los dos triángulos equiláteros A1A2A3, A'1A'2A'3, con el mismo baricentro G que ABC y tales que los segmentos A2A3, A'2A'3 son iguales y paralelos al segmento BC.

    ( Mostrar/Ocultar figura )

    Similarmente, se consideran la pareja de triángulos equiláteros B1B2B3, B'1B'2B'3, con el mismo baricentro G que ABC y tales que los segmentos B2B3, B'2B'3 son iguales y paralelos al segmento CA.
    Y se consideran la pareja de triángulos equiláteros C1C2C3, C'1C'2C'3, con el mismo baricentro G que ABC y tales que los segmentos C2C3, C'2C'3 son iguales y paralelos al segmento AB.

    Las coordenadas baricéntricas de los vértices de A1A2A3 son (S es el doble del área de ABC):
    A1 = ( a^4 + (b^2 - c^2)^2 - 2a^2(b^2 + c^2 - 2Sqrt[3]S) :
    a^4 + (b^2 - c^2)(b^2 - c^2 - 2Sqrt[3] S) - 2a^2(b^2 + c^2 + Sqrt[3] S) :
    a^4 + (b^2 - c^2)(b^2 - c^2 + 2Sqrt[3] S) - 2a^2(b^2 + c^2 + Sqrt[3] S))

    A2 = ( b^4 - 2b^2(a^2 + c^2 + Sqrt[3] S) + (-a^2 + c^2)(-a^2 + c^2 + 2Sqrt[3] S) :
    b^4 + (-a^2 + c^2)^2 + 2b^2(-a^2 - c^2 + 2Sqrt[3] S) :
    b^4 + (-a^2 + c^2)(-a^2 + c^2 - 2Sqrt[3] S) - 2b^2(a^2 + c^2 + Sqrt[3] S) )

    A3 = (c^4 + (a^2 - b^2)(a^2 - b^2 - 2Sqrt[3] S) - 2c^2(a^2 + b^2 + Sqrt[3] S) :
    c^4 - 2c^2(a^2 + b^2 + Sqrt[3] S) + (a^2 - b^2) (a^2 - b^2 + 2Sqrt[3] S) :
    (a^2 - b^2)^2 + c^4 + 2c^2(-a^2 - b^2 + 2Sqrt[3] S) )

    Las coordenadas baricéntricas de los vértices de A'1A'2A'3 (triángulo simétrico de A1A2A3 respecto a G) son:
    A'1 = (a^4 + (b^2 - c^2)^2 - 2a^2(b^2 + c^2 + 2 Sqrt[3] S) :
    a^4 + 2a^2(-b^2 - c^2 + Sqrt[3] S) + (b^2 - c^2) (b^2 - c^2 + 2 Sqrt[3] S) :
    a^4 + (b^2 - c^2)(b^2 - c^2 - 2 Sqrt[3] S) + 2a^2(-b^2 - c^2 + Sqrt[3] S))

    A'2 = (2(a^4 + (b^2 - c^2)^2 + 2a^2(-b^2 - c^2 + Sqrt[3] S)) :
    -a^4 + 2a^2(b^2 + c^2 - Sqrt[3] S) - (b^2 - c^2) (b^2 - c^2 + 2 Sqrt[3] S) :
    5a^4 + (b^2 - c^2)(5b^2 - 5c^2 + 2 Sqrt[3] S) - 2a^2(5b^2 + 5c^2 + Sqrt[3] S))

    A'3 = (2(a^4 + (b^2 - c^2)^2 + 2a^2(-b^2 - c^2 + Sqrt[3] S)) :
    5a^4 + (b^2 - c^2)(5b^2 - 5c^2 - 2 Sqrt[3] S) - 2a^2(5b^2 + 5c^2 + Sqrt[3] S) :
    -a^4 + 2a^2(b^2 + c^2 - Sqrt[3] S) + (b^2 - c^2)(-b^2 + c^2 + 2 Sqrt[3] S)).

    • Las rectas AA1, BB1 y CC1 se cortan en X13, primer punto isogonal (Fermat - Torricelli).

    ( Mostrar/Ocultar figura )

    • Las rectas AA'1, BB'1 y CC'1 se cortan en X14, segundo punto isogonal.

    ( Mostrar/Ocultar figura )

    • Denotamos por M1, N1 y P1 los puntos medios de A2A3, B2B3 y C2C3, respectivamente. Las rectas AM1, BN1 y CP1 se cortan en el centro de perspectividad de Kiepert K1 (sobre la hipérbola de Kiepert) de coordenadas baricéntricas:

    K1 = ( 1 / (SA-2S/√3) : 1 / (SB-2S/√3) : 1 / (SC-2S/√3) ),

    que tiene (6-9-13)-número de búsqueda en ETC: 1.05544963972706758475865636
    ( Mostrar/Ocultar figura )

    • Denotamos por M'1, N'1 y P'1 los puntos medios de A'2A'3, B'2B'3 y C'2C'3, respectivamente. Las rectas AM'1, BN'1 y CP'1 se cortan en el punto K2 (sobre la hipérbola de Kiepert) de coordenadas baricéntricas:

    K2 = ( 1 / (SA+2S/√3) : 1 / (SB+2S/√3) : 1 / (SC+2S/√3) ),

    que tiene (6-9-13)-número de búsqueda en ETC: 0.982582066301352428967254767
    ( Mostrar/Ocultar figura )

    • Denotamos por M el punto medio de M1 y el punto medio Ma de BC, por N el punto medio de N1 y el punto medio Mb de CA y por P el punto medio Mc de P1 y el punto medio de AB. Las rectas AM, BN y CP se cortan en el punto K3 (sobre la hipérbola de Kiepert) de coordenadas baricéntricas:

    K3 = ( 1 / (SA-5S/√3) : 1 / (SB-5S/√3) : 1 / (SC -5S/√3) ),

    que tiene (6-9-13)-número de búsqueda en ETC: 4.995109899468789806283186502
    ( Mostrar/Ocultar figura )

    • Denotamos por M' el punto medio de M'1 y el punto medio Ma de BC, por N' el punto medio de N'1 y el punto medio Mb de CA y por P' el punto medio Mc de P'1 y el punto medio de AB. Las rectas AM', BN' y CP' se cortan en el punto K4 (sobre la hipérbola de Kiepert) de coordenadas baricéntricas:

    K4 = ( 1 / (SA+5S/√3) : 1 / (SB+5S/√3) : 1 / (SC+5S/√3) ),

    que tiene (6-9-13)-número de búsqueda en ETC: 1.781091236364735925107070176
    ( Mostrar/Ocultar figura )

    • Denotamos por Ga, Gb y Gb los baricentros de BCA1, CAB1 y ABC1, respectivamente. Las rectas AGa, BGb y CGc se cortan en el punto K5 (sobre la hipérbola de Kiepert) de coordenadas baricéntricas:

    K5 = ( 1 / (SA+4S/√3) : 1 / (SB+4S/√3) : 1 / (SC+4S/√3) ),

    que tiene (6-9-13)-número de búsqueda en ETC: 1.623294634697676359144692801
    ( Mostrar/Ocultar figura )

    • Denotamos por G'a, G'b y G'b los baricentros de BCA'1, CAB'1 y ABC'1, respectivamente. Las rectas AG'a, BG'b y CG'c se cortan en el punto K6 (sobre la hipérbola de Kiepert) de coordenadas baricéntricas:

    K6 = ( 1 / (SA-4S/√3) : 1 / (SB-4S/√3) : 1 / (SC-4S/√3) ),

    que tiene (6-9-13)-número de búsqueda en ETC: 7.402174494117207095178203633
    ( Mostrar/Ocultar figura )


    Otros centros del triángulo en esta configuración

    • Las rectas MaM1, MbN1 y McP1 concurren en X618, primer punto isogónico del triángulo medial.

    ( Mostrar/Ocultar figura )

    • Las rectas MaM'1, MbN'1 y McP'1 concurren en X619, segundo punto isogónico del triángulo medial.

    ( Mostrar/Ocultar figura )

    • Las rectas MM', NN' y PP' concurren en X549, punto medio del baricentro y el circuncentro de ABC.

    ( Mostrar/Ocultar figura )

    • Las rectas GaG'a, GbG'b y GcG'c concurren en X3524, en la recta de Euler de ABC.

    ( Mostrar/Ocultar figura )

    • Las rectas MGa, NGb y PGc concurren en el punto de coordenadas baricéntricas:

    ( 53a^4 + 11(b^2-c^2)^2 - 64a^2(b^2+c^2) - 2√3(a^2+3b^2+3c^2)S : ... : ... ),

    que tiene (6-9-13)-número de búsqueda en ETC: 4.3452957082099683645807850740
    ( Mostrar/Ocultar figura )

    • Las rectas M'G'a, N'G'b y P'G'c concurren en el punto de coordenadas baricéntricas:

    ( 53a^4 + 11(b^2-c^2)^2 - 64a^2(b^2+c^2) + 2√3(a^2+3b^2+3c^2)S : ... : ... ),

    que tiene (6-9-13)-número de búsqueda en ETC: 6.729170058363700358702487375
    ( Mostrar/Ocultar figura )


  • jueves, 20 de junio del 2013

    El centro del triángulo X(3162) como centro de una cónica

    Sean ABC un triángulo, P un punto, D, E y F los pies de las cevianas de P en los lados BC, CA y AB. Denotamos por A', B' y C' los puntos donde las mediatrices de AD, BE y CF cortan a BC, CA y AB, respectivamente. El lugar geométrico de los puntos P tales que A', B' y C' estén alineados es una séxtica ( Mostrar/Ocultar fórmula )

      a^4*c^2*x^4*y^2 - b^4*c^2*x^4*y^2 - a^2*c^4*x^4*y^2 + b^2*c^4*x^4*y^2 + 2*a^4*c^2*x^3*y^3 - 2*b^4*c^2*x^3*y^3 + a^4*c^2*x^2*y^4 - b^4*c^2*x^2*y^4 - a^2*c^4*x^2*y^4 + b^2*c^4*x^2*y^4 - 2*b^4*c^2*x^4*y*z + 2*b^2*c^4*x^4*y*z + 2*a^4*c^2*x^3*y^2*z - 4*b^4*c^2*x^3*y^2*z - 2*a^2*c^4*x^3*y^2*z + 2*b^2*c^4*x^3*y^2*z + 4*a^4*c^2*x^2*y^3*z - 2*b^4*c^2*x^2*y^3*z - 2*a^2*c^4*x^2*y^3*z + 2*b^2*c^4*x^2*y^3*z + 2*a^4*c^2*x*y^4*z - 2*a^2*c^4*x*y^4*z - a^4*b^2*x^4*z^2 + a^2*b^4*x^4*z^2 - b^4*c^2*x^4*z^2 + b^2*c^4*x^4*z^2 - 2*a^4*b^2*x^3*y*z^2 + 2*a^2*b^4*x^3*y*z^2 - 2*b^4*c^2*x^3*y*z^2 + 4*b^2*c^4*x^3*y*z^2 - 2*a^4*b^2*x^2*y^2*z^2 + 2*a^2*b^4*x^2*y^2*z^2 + 2*a^4*c^2*x^2*y^2*z^2 - 2*b^4*c^2*x^2*y^2*z^2 - 2*a^2*c^4*x^2*y^2*z^2 + 2*b^2*c^4*x^2*y^2*z^2 - 2*a^4*b^2*x*y^3*z^2 + 2*a^2*b^4*x*y^3*z^2 + 2*a^4*c^2*x*y^3*z^2 - 4*a^2*c^4*x*y^3*z^2 - a^4*b^2*y^4*z^2 + a^2*b^4*y^4*z^2 + a^4*c^2*y^4*z^2 - a^2*c^4*y^4*z^2 - 2*a^4*b^2*x^3*z^3 + 2*b^2*c^4*x^3*z^3 - 4*a^4*b^2*x^2*y*z^3 + 2*a^2*b^4*x^2*y*z^3 - 2*b^4*c^2*x^2*y*z^3 + 2*b^2*c^4*x^2*y*z^3 - 2*a^4*b^2*x*y^2*z^3 + 4*a^2*b^4*x*y^2*z^3 + 2*a^4*c^2*x*y^2*z^3 - 2*a^2*c^4*x*y^2*z^3 + 2*a^2*b^4*y^3*z^3 - 2*a^2*c^4*y^3*z^3 - a^4*b^2*x^2*z^4 + a^2*b^4*x^2*z^4 - b^4*c^2*x^2*z^4 + b^2*c^4*x^2*z^4 - 2*a^4*b^2*x*y*z^4 + 2*a^2*b^4*x*y*z^4 - a^4*b^2*y^2*z^4 + a^2*b^4*y^2*z^4 + a^4*c^2*y^2*z^4 - a^2*c^4*y^2*z^4=0.
    que pasa por los vértices de ABC (puntos dobles) y por los centros X1, X4 y X8.
    Además, ocurre que los seis puntos en los que la séxtica vuelve a cortar a los lados de ABC están en una cónica ( Mostrar/Ocultar fórmula )
      Σ(a^6 - a^4 (b^2 + c^2) - a^2 (b^2 - c^2)^2 + (b^2 - c^2)^2 (b^2 + c^2)) x^2 - 2 (b^2 + c^2) (a^4 - (b^2 - c^2)^2) y*z=0.
    de centro en X3162 y perspector de coordenadas baricéntricas:

    ( 1 / (a^6 + a^4(b^2+c^2) - a^2((b^2+c^2)^2-b^2 c^2) - (b^2+c^2)(b^4-b^2c^2+c^4)) : ... : ... ),

    que tiene (6-9-13)-número de búsqueda en ETC: 8.11040473813735146364958223
    ( Mostrar/Ocultar figura )


  • miércoles, 19 de junio del 2013

    Rectas de Euler concurrentes

    (Anopolis #443 Antreas P. Hatzipolakis)

    Sean ABC un triángulo y A'B'C' su triángulo de reflexión (ie. A', B' y C' son los simétricos de A, B y C respecto a BC, CA y AB, resp.). Denotamos por:
    Ab y Ac las proyecciones ortogonales de A' sobre BB' y CC', resp.
    Bc y Ba las proyecciones ortogonales de B' sobre CC' y AA', resp.
    Ca y Cb las proyecciones ortogonales de C' sobre AA' y BB', resp.
    Entonces, las rectas de Euler La, Lb y Lc de los triángulos A'AbAc, B'BcBa y C'CaCb, respectivamente, son concurrentes en el punto X de coordenadas baricéntricas:

    ( a^2 (a^8(b^2+c^2) - 2a^6(b^4+c^4) + a^4b^2c^2(b^2+c^2) + a^2(b^2-c^2)^2(2b^4+b^2c^2+2c^4) - b^10+b^8c^2+b^2c^8-c^10) : ... : ... ),

    que tiene (6-9-13)-número de búsqueda en ETC: -6.61866665524197553640898833
    ( Mostrar/Ocultar figura )


    Generalización (Anopolis #442)
    Cuando A', B' y C' son puntos sobre AHa, BHb, CHc tal que:
    AA' / AHa = BB' / BHb = C'C / CHc = t,
    el lugar geométrico del punto de concurrencia de las rectas de Euler La, Lb y Lc de los triángulos A'AbAc, B'BcBa y C'CaCb, respectivamente, es la recta que pasa por los centros del triángulo X51, X125, X132, X427, X1112, X2781 y X5480.


  • martes, 18 de junio del 2013

    Rectas de Euler de dos triángulos ortológicos

    (Anopolis #436 Antreas P. Hatzipolakis)

    Sean ABC un triángulo, A'B'C' su triángulo de reflexión (ie. A', B' y C' son los simétricos de A, B y C respecto a BC, CA y AB, resp.) y AtBtCt el triángulo antipedal del circuncentro O (triángulo tangencial).
    Sean Oa, Ob y Oc los circuncentros de los triángulos AtB'C', BtC'A' y CtA'B', resp. Ellos quedan en las alturas de ABC (sobre las rectas AA', BB' y CC').
    Por tanto, ABC y OaObOc son ortológicos con un centro de ortología en el ortocentro H de ABC.
    El otro centro de ortología es el punto de intersección de las perpendiculares por A, B y C a las rectas ObOc, OcOa y OaOb, resp., de ecuaciones baricéntricas:

    -SC(4SB²-a²c²)y + SB(4SC²-a²b²)z=0,    SC(4SA²-b^2c²)x - SA(4SC²-b^2a²)z=0,    -SB(4SA²-c²b^2)x + SA(4SB²-c²a²)y =0.
    Que se cortan en X265.
    ( Mostrar/Ocultar figura )

    Las rectas de Euler de los triángulos ABC y OaObOc se cortan en el punto de coordenadas baricéntricas:

    ( a^8(b^2+c^2) - 2a^6(b^4-b^2c^2+c^4) - a^4b^2c^2(b^2+c^2) + a^2(b^2-c^2)^2(2b^4-b^2c^2+2c^4) - (b^2-c^2)^4(b^2+c^2) : ... : ... ),

    que tiene (6-9-13)-número de búsqueda en ETC: -1.67191299053365041718858431

  • lunes, 17 de junio del 2013

    Punto sobre la recta de Euler

    (Anopolis #440 Randy Hutson)

    Dado un triángulo ABC, sea DEF el triángulo acotado por las rectas La, Lb y Lc, donde La es la polar del ortocentro H con respecto a la circunferencia centrada en A y pasando por N, centro de la circunferencia de los nueve puntos; Lb y Lc se definen cíclicamente.

    El triángulo DEF es homotético a ABC con centro de homotecia X (en la recta de Euler), de coordenadas baricéntricas:

    ((a^6-3a^4(b^2+c^2)+a^2(3b^4+b^2c^2+3c^4)-(b^2-c^2)^2(b^2+c^2)) /(b^2+c^2-a^2) : ... : ... ),

    que tiene (6-9-13)-número de búsqueda en ETC: 1.7457649287207846078851083739
    ( Mostrar/Ocultar figura )


  • domingo, 16 de junio del 2013

    Circunferencias inscritas de igual radio

    (TrianguloCabri #685 #686 Ricardo Barroso)

    Dado un triángulo ABC, construir:
    1. Tres circunferencias de igual radio, tangentes a los lados AB y AC, BC y BA, CA y CB, respectivamente, y que tengan un punto común.
    2. Tres circunferencias de igual radio, tangentes a los lados AB y AC, BC y BA, CA y CB, respectivamente, y tangentes exteriormente a otra circunferencia, también de mismo radio.

    1.   La circunferencia inscrita a ABC tiene por ecuación baricéntrica:
    4(c2x y + b2x z + a2y z) - (x+y+z)((b+c-a)2x + (c+a-b)2y + (a+b-c)2z)=0.
    La ecuación de la circunferencia Γa imagen mediante la homotecia de centro A y razón k es:
    4(c2xy + b2xz + a2yz)- (x + y + z)((b + c - a)2k2x + ((b + c - a)k - 2c)2y + ((b + c - a)k - 2b)2z)=0.
    Similarmente, se obtienen las circunferencias Γb y Γc imágenes de la circunferencia inscrita, mediante las homotecias de razón k y centros en B y C.
    El centro radical E (sobre la recta IO) de las tres circunferencias Γa, Γb y Γc tiene por coordenadas :
    E = ( a(a3 - a2(b+c)k - a(b2+c2-2bck)+ (b-c)2(b+c)k) :
    b(b3 - b2(c+a)k - b(c2+a2-2cak)+ (c-a)2(c+a)k) :
    c(c3 - c2(a+b)k - c(a2+b2-2abk)+ (a-b)2(a+b)k) ).
    Para que este punto esté en las tres circunferencias, se debe anular la expresión siguiente:
    (2abc + (a3 - a2b - ab2 + b3 - a2c - b2c - ac2 - bc2 + c3)k)
    (-2abc + (a3 - a2b - ab2 + b3 - a2c + 4abc - b2c - ac2 - bc2 + c3)k)=0.
    Los dos valores de k que la anulan, dan lugar a sendos centros radicales donde concurren las tres circunferencias. Estos puntos son:
    ( a2(b+c-a) : b2(c+a-b) : c2(a+b-c) )    y    (a2(a2-(b-c)2) : b2(b2-(c-a)2) : c2(c2-(a-b)2) ).
    Que son los centros de homotecia interior X55 y exterior X56, respectivamente, de las circunferencia inscrita y circunscrita a ABC.
    ( Mostrar/Ocultar figura )
    Los otros puntos de intersección de las circunferencias de igual radio, que se cortan en X55, forman un triángulo perspectivo con ABC, cuyo centro de perspectividad es X12: conjugado armónico del punto de Feuerback respecto al incentro y al centro de la circunferencia de los nueve puntos.
    ( Mostrar/Ocultar figura )
    Los otros puntos de intersección de las circunferencias de igual radio, que se cortan en X56, forman un triángulo perspectivo con ABC, cuyo centro de perspectividad es X11: punto de Feuerback, único punto común de las circunferencias inscrita y la de los nueve puntos.

    2.   El eje radical de la circunferencia Γa y la circunferencia Γ con el mismo radio, |kr|, y centro en E es:
    -4bc(k-1)(a3(b+c)k-a(b-c)2(b+c)k+(b2-c2)2k-a2(b2k+c2k+bc(1+k))) x +
    4c(k-1)(a^4c - a3b(b+c)k + ab(b-c)2(b+c)k + (b2-c2)2(c - bk) + a2(-2c3 + b2c(k-1) + b3k + bc2k)) y+
    4b(k-1) (a^4b - a3c(b+c)k + a(b - c)2c(b + c)k + (b2 - c2)2(b - ck) + a2(-2b3+ bc2(k-1) + b2ck+ c3k)) z=0.
    Esta recta es tangente a Γa si:
    (abc + (a3 - a2b - ab2 + b3 - a2c + abc - b2c - ac2 - bc2 + c3)k)
    (-abc + (a3 - a2b - ab2 + b3 - a2c + 3abc - b2c - ac2 - bc2 + c3)k) =0

    Resuelta esta ecuación en k, nos dan dos valores para la razón de homotecia que transforma la circunferencia inscrita en las circunferencias Γa, Γb y Γc. Los correspondientes centros radicales de cada terna son:
    ( a2(a2-b2+bc-c2) : b2(b2-c2+ca-a2) : c2(c2-a2+ab-b2) ),
    ( a2(a2-b2-bc-c2) : b2(b2-c2-ca-a2) : c2(c2-a2-ab-b2) ),
    que son, respectivamente, el inverso del incentro en la circunferencia circunscrita, X36, y el conjugado armónico de éste respecto al incentro y circuncentro, X35.

    Los puntos de tangencia de cada pareja de circunferencias Γa y Γ que se obtienen son:
    A1 =( 2a2b2c2(2a^4+ a2(-3b2+2bc-3c2) + (b2-c2)2) : -2a2b^4c2 (a2-b2-2ac+c2) : -2a2b2c^4(a2-2ab+b2-c2) ),
    A2=( 2a2b2c2(2a^4+(b2-c2)2- a2(3b2+2bc+3c2)) : -2a2b^4c2(a2-b2+2ac+c2) : -2a2b2c^4(a2+2ab+b2-c2) ).
    Permutando cíclicamente las coordenadas de estos puntos, se obtienen las de los puntos B1, C1 y B2, C2 de contacto de la circunferencia Γ con Γb y Γc. Se verifica que las rectas AA1, BB1 y CC1 concurren en el punto X56. ( Mostrar/Ocultar figura)
    Y las rectas AA2, BB2 y CC2 se cortan en el punto X55. ( Mostrar/Ocultar figura)


  • sábado, 15 de junio del 2013

    Reflexiones en la recta que pasa por dos puntos conjugados isogonales

    (Anopolis #420 Antreas P. Hatzipolakis)

    ◊  Sean ABC un triángulo, P y P* dos puntos conjugados isogonales y A'B'C' el triángulo antipedal de P.
    Las perpendiculares por A', B' y C' a las reflexiones de AP, BP y CP, respectivamente, respecto a PP* son concurrentes, en un punto Q sobre las circunferencia circunscrita a A'B'C'.

    En particular, si P=O (A'B'C' es el triángulo tangencial), el punto de concurrencia Q tiene coordenadas baricéntricas (Anopolis #419):

    ( a^2(a^16 - 2a^14(b^2+c^2)- 2a^12(b^4-5b^2c^2+c^4)+ 2a^10(b^2+c^2)(3b^4-7b^2c^2+3c^4) - a^8b^2c^2(3b^2-4c^2)(4b^2-3c^2)- 6a^6(b-c)^2(b+c)^2(b^2+c^2)(b^4-b^2c^2+c^4)+ a^4(b-c)^2(b+c)^2(2b^8+12b^6c^2-b^4c^4+12b^2c^6+2c^8)+ 2a^2(b-c)^2(b+c)^2(b^2+c^2)(b^8-4b^6c^2+3b^4c^4-4b^2c^6+c^8)- (b-c)^4(b+c)^4(b^8+2b^6c^2+2b^2c^6+c^8)) : ... : ...),

    y tiene (6-9-13)-número de búsqueda en ETC: -6.980189770294538.
    ( Mostrar/Ocultar figura )


    ◊  Sean ABC un triángulo, P y P* dos puntos conjugados isogonales y A'B'C' el triángulo antipedal de P.
    Para que las perpendiculares por A', B' y C' a las reflexiones de AP*, BP* y CP*, respectivamente, respecto a PP* sean concurrentes, el punto P ha de estar en la circunferencia circunscrita a ABC o sobre la curva algebraica de grado nueve (invariante por isoconjugación) de ecuación baricéntrica:
    ( Mostrar/Ocultar fórmula )
      -a^2*b^2*c^6*x^6*y^3 + b^2*c^8*x^6*y^3 - 3*a^2*b^2*c^6*x^5*y^4 - 3*a^2*b^2*c^6*x^4*y^5 - a^2*b^2*c^6*x^3*y^6 + a^2*c^8*x^3*y^6 + 3*b^4*c^6*x^6*y^2*z - 6*a^2*b^2*c^6*x^4*y^4*z + 3*a^4*c^6*x^2*y^6*z + 3*b^6*c^4*x^6*y*z^2 + 6*a^2*b^4*c^4*x^5*y^2*z^2 + 3*a^4*b^2*c^4*x^4*y^3*z^2 - 3*a^2*b^2*c^6*x^4*y^3*z^2 + 3*a^2*b^4*c^4*x^3*y^4*z^2 - 3*a^2*b^2*c^6*x^3*y^4*z^2 + 6*a^4*b^2*c^4*x^2*y^5*z^2 + 3*a^6*c^4*x*y^6*z^2 - a^2*b^6*c^2*x^6*z^3 + b^8*c^2*x^6*z^3 + 3*a^4*b^4*c^2*x^4*y^2*z^3 - 3*a^2*b^6*c^2*x^4*y^2*z^3 - 3*a^6*b^2*c^2*x^2*y^4*z^3 + 3*a^4*b^4*c^2*x^2*y^4*z^3 + a^8*c^2*y^6*z^3 - a^6*b^2*c^2*y^6*z^3 - 3*a^2*b^6*c^2*x^5*z^4 - 6*a^2*b^6*c^2*x^4*y*z^4 - 3*a^2*b^6*c^2*x^3*y^2*z^4 + 3*a^2*b^4*c^4*x^3*y^2*z^4 - 3*a^6*b^2*c^2*x^2*y^3*z^4 + 3*a^4*b^2*c^4*x^2*y^3*z^4 - 6*a^6*b^2*c^2*x*y^4*z^4 - 3*a^6*b^2*c^2*y^5*z^4 - 3*a^2*b^6*c^2*x^4*z^5 + 6*a^4*b^4*c^2*x^2*y^2*z^5 - 3*a^6*b^2*c^2*y^4*z^5 + a^2*b^8*x^3*z^6 - a^2*b^6*c^2*x^3*z^6 + 3*a^4*b^6*x^2*y*z^6 + 3*a^6*b^4*x*y^2*z^6 + a^8*b^2*y^3*z^6 - a^6*b^2*c^2*y^3*z^6=0.
    ( Mostrar/Ocultar figura )


  • viernes, 14 de junio del 2013

    Reflexiones de la recta que pasa por dos puntos conjugados isogonales

    (Anopolis #424 Antreas P. Hatzipolakis)

    Sean ABC un triángulo, P y P* dos puntos conjugados isogonales y A'B'C' el triángulo pedal de P.
    Las perpendiculares por A', B' y C' a las reflexiones de PP*, respecto a AP, BP y CP son concurrentes en un punto Q, si P está sobre la nónica que pasa por los vértices de ABC (puntos triples), por el incentro I, por el ortocentro H y por los exincentros Ia, Ib, Ic. ( Mostrar/Ocultar fórmula )

      -a^4*b^2*c^4*x^6*y^3 + 2*a^2*b^4*c^4*x^6*y^3 - b^6*c^4*x^6*y^3 + a^2*b^2*c^6*x^6*y^3 - b^4*c^6*x^6*y^3 - a^6*c^4*x^5*y^4 + 3*a^2*b^4*c^4*x^5*y^4 - 2*b^6*c^4*x^5*y^4 + 2*a^4*c^6*x^5*y^4 - 3*a^2*b^2*c^6*x^5*y^4 + b^4*c^6*x^5*y^4 - a^2*c^8*x^5*y^4 + b^2*c^8*x^5*y^4 - 2*a^6*c^4*x^4*y^5 + 3*a^4*b^2*c^4*x^4*y^5 - b^6*c^4*x^4*y^5 + a^4*c^6*x^4*y^5 - 3*a^2*b^2*c^6*x^4*y^5 + 2*b^4*c^6*x^4*y^5 + a^2*c^8*x^4*y^5 - b^2*c^8*x^4*y^5 - a^6*c^4*x^3*y^6 + 2*a^4*b^2*c^4*x^3*y^6 - a^2*b^4*c^4*x^3*y^6 - a^4*c^6*x^3*y^6 + a^2*b^2*c^6*x^3*y^6 + 3*a^2*b^4*c^4*x^6*y^2*z - 3*b^6*c^4*x^6*y^2*z - 3*b^4*c^6*x^6*y^2*z + 2*a^4*b^2*c^4*x^5*y^3*z + 2*a^2*b^4*c^4*x^5*y^3*z - 4*b^6*c^4*x^5*y^3*z - 4*a^2*b^2*c^6*x^5*y^3*z + 2*b^4*c^6*x^5*y^3*z + 2*b^2*c^8*x^5*y^3*z - a^6*c^4*x^4*y^4*z + a^4*b^2*c^4*x^4*y^4*z + a^2*b^4*c^4*x^4*y^4*z - b^6*c^4*x^4*y^4*z + 2*a^4*c^6*x^4*y^4*z - 2*a^2*b^2*c^6*x^4*y^4*z + 2*b^4*c^6*x^4*y^4*z - a^2*c^8*x^4*y^4*z - b^2*c^8*x^4*y^4*z - 4*a^6*c^4*x^3*y^5*z + 2*a^4*b^2*c^4*x^3*y^5*z + 2*a^2*b^4*c^4*x^3*y^5*z + 2*a^4*c^6*x^3*y^5*z - 4*a^2*b^2*c^6*x^3*y^5*z + 2*a^2*c^8*x^3*y^5*z - 3*a^6*c^4*x^2*y^6*z + 3*a^4*b^2*c^4*x^2*y^6*z - 3*a^4*c^6*x^2*y^6*z + 3*a^2*b^4*c^4*x^6*y*z^2 - 3*b^6*c^4*x^6*y*z^2 - 3*b^4*c^6*x^6*y*z^2 + a^4*b^4*c^2*x^5*y^2*z^2 - 2*a^2*b^6*c^2*x^5*y^2*z^2 + b^8*c^2*x^5*y^2*z^2 + a^4*b^2*c^4*x^5*y^2*z^2 + 2*a^2*b^4*c^4*x^5*y^2*z^2 - b^6*c^4*x^5*y^2*z^2 - 2*a^2*b^2*c^6*x^5*y^2*z^2 - b^4*c^6*x^5*y^2*z^2 + b^2*c^8*x^5*y^2*z^2 + 2*a^6*b^2*c^2*x^4*y^3*z^2 - 3*a^4*b^4*c^2*x^4*y^3*z^2 + b^8*c^2*x^4*y^3*z^2 + 7*a^2*b^4*c^4*x^4*y^3*z^2 - b^6*c^4*x^4*y^3*z^2 - 3*a^2*b^2*c^6*x^4*y^3*z^2 - b^4*c^6*x^4*y^3*z^2 + b^2*c^8*x^4*y^3*z^2 + a^8*c^2*x^3*y^4*z^2 - 3*a^4*b^4*c^2*x^3*y^4*z^2 + 2*a^2*b^6*c^2*x^3*y^4*z^2 - a^6*c^4*x^3*y^4*z^2 + 7*a^4*b^2*c^4*x^3*y^4*z^2 - a^4*c^6*x^3*y^4*z^2 - 3*a^2*b^2*c^6*x^3*y^4*z^2 + a^2*c^8*x^3*y^4*z^2 + a^8*c^2*x^2*y^5*z^2 - 2*a^6*b^2*c^2*x^2*y^5*z^2 + a^4*b^4*c^2*x^2*y^5*z^2 - a^6*c^4*x^2*y^5*z^2 + 2*a^4*b^2*c^4*x^2*y^5*z^2 + a^2*b^4*c^4*x^2*y^5*z^2 - a^4*c^6*x^2*y^5*z^2 - 2*a^2*b^2*c^6*x^2*y^5*z^2 + a^2*c^8*x^2*y^5*z^2 - 3*a^6*c^4*x*y^6*z^2 + 3*a^4*b^2*c^4*x*y^6*z^2 - 3*a^4*c^6*x*y^6*z^2 - a^4*b^4*c^2*x^6*z^3 + a^2*b^6*c^2*x^6*z^3 + 2*a^2*b^4*c^4*x^6*z^3 - b^6*c^4*x^6*z^3 - b^4*c^6*x^6*z^3 + 2*a^4*b^4*c^2*x^5*y*z^3 - 4*a^2*b^6*c^2*x^5*y*z^3 + 2*b^8*c^2*x^5*y*z^3 + 2*a^2*b^4*c^4*x^5*y*z^3 + 2*b^6*c^4*x^5*y*z^3 - 4*b^4*c^6*x^5*y*z^3 + 2*a^6*b^2*c^2*x^4*y^2*z^3 - 3*a^2*b^6*c^2*x^4*y^2*z^3 + b^8*c^2*x^4*y^2*z^3 - 3*a^4*b^2*c^4*x^4*y^2*z^3 + 7*a^2*b^4*c^4*x^4*y^2*z^3 - b^6*c^4*x^4*y^2*z^3 - b^4*c^6*x^4*y^2*z^3 + b^2*c^8*x^4*y^2*z^3 + a^8*c^2*x^2*y^4*z^3 - 3*a^6*b^2*c^2*x^2*y^4*z^3 + 2*a^2*b^6*c^2*x^2*y^4*z^3 - a^6*c^4*x^2*y^4*z^3 + 7*a^4*b^2*c^4*x^2*y^4*z^3 - 3*a^2*b^4*c^4*x^2*y^4*z^3 - a^4*c^6*x^2*y^4*z^3 + a^2*c^8*x^2*y^4*z^3 + 2*a^8*c^2*x*y^5*z^3 - 4*a^6*b^2*c^2*x*y^5*z^3 + 2*a^4*b^4*c^2*x*y^5*z^3 + 2*a^6*c^4*x*y^5*z^3 + 2*a^4*b^2*c^4*x*y^5*z^3 - 4*a^4*c^6*x*y^5*z^3 + a^6*b^2*c^2*y^6*z^3 - a^4*b^4*c^2*y^6*z^3 - a^6*c^4*y^6*z^3 + 2*a^4*b^2*c^4*y^6*z^3 - a^4*c^6*y^6*z^3 - a^6*b^4*x^5*z^4 + 2*a^4*b^6*x^5*z^4 - a^2*b^8*x^5*z^4 - 3*a^2*b^6*c^2*x^5*z^4 + b^8*c^2*x^5*z^4 + 3*a^2*b^4*c^4*x^5*z^4 + b^6*c^4*x^5*z^4 - 2*b^4*c^6*x^5*z^4 - a^6*b^4*x^4*y*z^4 + 2*a^4*b^6*x^4*y*z^4 - a^2*b^8*x^4*y*z^4 + a^4*b^4*c^2*x^4*y*z^4 - 2*a^2*b^6*c^2*x^4*y*z^4 - b^8*c^2*x^4*y*z^4 + a^2*b^4*c^4*x^4*y*z^4 + 2*b^6*c^4*x^4*y*z^4 - b^4*c^6*x^4*y*z^4 + a^8*b^2*x^3*y^2*z^4 - a^6*b^4*x^3*y^2*z^4 - a^4*b^6*x^3*y^2*z^4 + a^2*b^8*x^3*y^2*z^4 + 7*a^4*b^4*c^2*x^3*y^2*z^4 - 3*a^2*b^6*c^2*x^3*y^2*z^4 - 3*a^4*b^2*c^4*x^3*y^2*z^4 + 2*a^2*b^2*c^6*x^3*y^2*z^4 + a^8*b^2*x^2*y^3*z^4 - a^6*b^4*x^2*y^3*z^4 - a^4*b^6*x^2*y^3*z^4 + a^2*b^8*x^2*y^3*z^4 - 3*a^6*b^2*c^2*x^2*y^3*z^4 + 7*a^4*b^4*c^2*x^2*y^3*z^4 - 3*a^2*b^4*c^4*x^2*y^3*z^4 + 2*a^2*b^2*c^6*x^2*y^3*z^4 - a^8*b^2*x*y^4*z^4 + 2*a^6*b^4*x*y^4*z^4 - a^4*b^6*x*y^4*z^4 - a^8*c^2*x*y^4*z^4 - 2*a^6*b^2*c^2*x*y^4*z^4 + a^4*b^4*c^2*x*y^4*z^4 + 2*a^6*c^4*x*y^4*z^4 + a^4*b^2*c^4*x*y^4*z^4 - a^4*c^6*x*y^4*z^4 - a^8*b^2*y^5*z^4 + 2*a^6*b^4*y^5*z^4 - a^4*b^6*y^5*z^4 + a^8*c^2*y^5*z^4 - 3*a^6*b^2*c^2*y^5*z^4 + a^6*c^4*y^5*z^4 + 3*a^4*b^2*c^4*y^5*z^4 - 2*a^4*c^6*y^5*z^4 - 2*a^6*b^4*x^4*z^5 + a^4*b^6*x^4*z^5 + a^2*b^8*x^4*z^5 + 3*a^4*b^4*c^2*x^4*z^5 - 3*a^2*b^6*c^2*x^4*z^5 - b^8*c^2*x^4*z^5 + 2*b^6*c^4*x^4*z^5 - b^4*c^6*x^4*z^5 - 4*a^6*b^4*x^3*y*z^5 + 2*a^4*b^6*x^3*y*z^5 + 2*a^2*b^8*x^3*y*z^5 + 2*a^4*b^4*c^2*x^3*y*z^5 - 4*a^2*b^6*c^2*x^3*y*z^5 + 2*a^2*b^4*c^4*x^3*y*z^5 + a^8*b^2*x^2*y^2*z^5 - a^6*b^4*x^2*y^2*z^5 - a^4*b^6*x^2*y^2*z^5 + a^2*b^8*x^2*y^2*z^5 - 2*a^6*b^2*c^2*x^2*y^2*z^5 + 2*a^4*b^4*c^2*x^2*y^2*z^5 - 2*a^2*b^6*c^2*x^2*y^2*z^5 + a^4*b^2*c^4*x^2*y^2*z^5 + a^2*b^4*c^4*x^2*y^2*z^5 + 2*a^8*b^2*x*y^3*z^5 + 2*a^6*b^4*x*y^3*z^5 - 4*a^4*b^6*x*y^3*z^5 - 4*a^6*b^2*c^2*x*y^3*z^5 + 2*a^4*b^4*c^2*x*y^3*z^5 + 2*a^4*b^2*c^4*x*y^3*z^5 + a^8*b^2*y^4*z^5 + a^6*b^4*y^4*z^5 - 2*a^4*b^6*y^4*z^5 - a^8*c^2*y^4*z^5 - 3*a^6*b^2*c^2*y^4*z^5 + 3*a^4*b^4*c^2*y^4*z^5 + 2*a^6*c^4*y^4*z^5 - a^4*c^6*y^4*z^5 - a^6*b^4*x^3*z^6 - a^4*b^6*x^3*z^6 + 2*a^4*b^4*c^2*x^3*z^6 + a^2*b^6*c^2*x^3*z^6 - a^2*b^4*c^4*x^3*z^6 - 3*a^6*b^4*x^2*y*z^6 - 3*a^4*b^6*x^2*y*z^6 + 3*a^4*b^4*c^2*x^2*y*z^6 - 3*a^6*b^4*x*y^2*z^6 - 3*a^4*b^6*x*y^2*z^6 + 3*a^4*b^4*c^2*x*y^2*z^6 - a^6*b^4*y^3*z^6 - a^4*b^6*y^3*z^6 + a^6*b^2*c^2*y^3*z^6 + 2*a^4*b^4*c^2*y^3*z^6 - a^4*b^2*c^4*y^3*z^6=0.
    ( Mostrar/Ocultar figura )


    Si P=H el punto de concurrencia de las perpendiculares es X125, centro de la hipérbola de Jerabek sobre la circunferencia de los nueve puntos.

  • jueves, 13 de junio del 2013

    Reflexiones de las bisectrices en la recta IO

    ( Anopolis #405Antreas P. Hatzipolakis)

    Sean ABC un triángulo, I su incentro, O su circuncentro, IaIbIc el triángulo excentral y La, Lb, Lc las reflexiones de la recta IO en las bisectrices AI, BI y CI, respectivamente.
    Las perpendiculares a La, Lb y Lc por Ia, Ib y Ic, respectivamente, concurren en el punto de coordenadas baricéntricas:

    ( a^9 - 2a^8(b+c) + 6a^7b*c + 2a^6(b-2c)(2b-c)(b+c) - a^5(4b^4+4b^3c-17b^2c^2+4c^3b+4c^4)- 2a^4(b-c)^2(b+c)(b^2-6b*c+c^2)+ a^3(b-c)^2(4b^4+2b^3c-11b^2c^2+2c^3b+4c^4)- 2a^2b*c(b-c)^2(b+c)(3b^2-5b*c+3c^2)- a(b-c)^6(b+c)^2 : ... : ... ),

    y tiene (6-9-13)-número de búsqueda en ETC: 26.33820686266028262904684.
    ( Mostrar/Ocultar figura )


    Más información sobre este punto (Randy Hutson, Anopolis #404):
    Es el Y476 del triángulo excentral y queda en la circunferencia circunscrita a éste.
    Es el antipodal del incentro en la circunferencia que pasa por los pies D, E y F de las perpendiculares trazadas desde los exincentros a las rectas La, Lb, Lc.

  • miércoles, 12 de junio del 2013

    Propiedad de una cúbica de Musselman

    (J.R.Musselman.- Some loci connected with a triangle. American Math. Monthly, p.354-361, June-July 1940)

    Sean ABC un triángulo, p una recta que pasa por el ortocentro H y P un punto (distinto de H) sobre p. Se denotan por D, E y F los puntos simétricos de H respecto a las perpendiculares por P a los lados BC, CA y AB, respectivamente, y consideramos los puntos:

    A'=BE∩CF     B'=CF∩AD     C'=AD∩BE,

    y las tres cónicas, (Ca) que pasa por B, C, H, P, A', (Cb) que pasa por C, A, H, P, B' y (Cc) que pasa por A, B, H, P, C'.
    Estas tres cónicas tienen un punto común Q. El lugar geométrico que describe Q cuando P se mueve sobre la recta p es una hipérbola rectangular (H) circunscrita a ABC.
    El segundo punto X de intersección de la hipérbola (H) son la recta p, describe la (tercera) cúbica de Musselman (K028 del catálogo de Bernard Gibert), cuando p gira alrededor de H.
    ( Mostrar/Ocultar figura )

    Si p es la recta de Euler, Q=X265 (para todo P) y X=X3.

    Información de Bernard Gibert:
    This description is a "remake" of CL055 since K028 is spK(X3, X5): a line through H meets the isogonal transform -hyperbola (H)- of its parallel at O at H and X.

  • sábado, 8 de junio del 2013

    Centro radical de circunferencias exinscritas

    ( Anopolis #365 Antreas P. Hatzipolakis)

    Sean ABC un triángulo y O su circuncentro. Consideramos las circunferencias exinscritas relativas al ángulo en O de los triángulos OBC, OCA y OAB; su centro radical es el punto de coordenadas baricéntricas:

    ( a(a^2-b^2-c^2)(2a^5(b+c)^2 - 4a^3(b^2+b*c+c^2)^2 + 2a(b-c)^2(b+c)^4 + 2S(b+c)(a^4-a^2(2b^2+5b*c+2c^2) + b^4-b^3c-c^3b+c^4)) : ... : ... ),

    donde S es el doble del área de ABC, y tiene (6-9-13)-número de búsqueda en ETC: 5.60319717911870517696190546.

    La circunferencia exinscrita a OBC, respecto al lado BC, toca a éste en su punto medio y tiene centro en el punto A1 = ( -2a^2(b*c+S) : b(a^2c+b^2c-c^3+2b*S) : c(a^2b-b^3+b*c^2+2c*S) ).
    ( Mostrar/Ocultar figura )

    Las paralelas por los vértices de ABC a los correspondientes ejes radicales se cortan en:

    ( a(a^2-b^2-c^2)(a^4-2a^2(b+c)^2 - 4a(b+c)S + (b^2-c^2)^2) : ... : ... ).

    con (6-9-13)-número de búsqueda en ETC: 6.13472549396753240388995633.

  • viernes, 7 de junio del 2013

    Centros de circunferencias de los nueve puntos y paralelas por los vértices del triángulo de referencia

    ( Anopolis #361 Antreas P. Hatzipolakis)

    Sean ABC un triángulo, P un punto y A'B'C' su triángulo ceviano. Adoptemos las siguientes notaciones:

    Ab y Ac son las proyecciones ortogonales de A sobre BB' y CC',
    Bc y Ba son las proyecciones ortogonales de B sobre CC' y AA',
    Ca y Cb son las proyecciones ortogonales de C sobre AA' y BB',
    A2 y A3 son las proyecciones ortogonales de A' sobre BB' y CC',
    B3 y B1 son las proyecciones ortogonales de B' sobre CC' y AA',
    C1 y C2 son las proyecciones ortogonales de C' sobre AA' y BB'.

    Na, Nb y Nc los centros de las circunferencia de los nueve puntos de los triángulos AAbAc, BBcBa, CCaCb,
    N1, N2 y N3 los centros de las circunferencia de los nueve puntos de los triángulos A'A2A3, B'B3B1, C'C1C2.

    Se verifica:
    Las rectas N1Na, N2Nb y N3Nc concurren en el punto P. De hecho, N1N2N3 es el triángulo ceviano de P respecto a NaNbNc.

    ( Mostrar/Ocultar figura )


    Sea la, lb y lc las paralelas a N1Na, N2Nb y N3Nc por A, B y C, resp. y las paralelas ma, mb y mc por A', B' y C', resp.

    El lugar geométrico de los puntos P, tales que las rectas la, lb y lc son concurrentes, son dos curvas algebraicas de grados 6, Σ6, ( Mostrar/Ocultar fórmula )
      Σ6:
      2*a^4*c^2*x^4*y^2 - 4*a^2*b^2*c^2*x^4*y^2 + 2*b^4*c^2*x^4*y^2 - 4*a^2*c^4*x^4*y^2 + b^2*c^4*x^4*y^2 + 2*c^6*x^4*y^2 + 4*a^4*c^2*x^3*y^3 - 8*a^2*b^2*c^2*x^3*y^3 + 4*b^4*c^2*x^3*y^3 - 3*a^2*c^4*x^3*y^3 - 3*b^2*c^4*x^3*y^3 - c^6*x^3*y^3 + 2*a^4*c^2*x^2*y^4 - 4*a^2*b^2*c^2*x^2*y^4 + 2*b^4*c^2*x^2*y^4 + a^2*c^4*x^2*y^4 - 4*b^2*c^4*x^2*y^4 + 2*c^6*x^2*y^4 - a^6*x^4*y*z + 3*a^4*b^2*x^4*y*z - 3*a^2*b^4*x^4*y*z + b^6*x^4*y*z + 3*a^4*c^2*x^4*y*z - 7*a^2*b^2*c^2*x^4*y*z + 4*b^4*c^2*x^4*y*z - 3*a^2*c^4*x^4*y*z + 4*b^2*c^4*x^4*y*z + c^6*x^4*y*z - a^6*x^3*y^2*z + 3*a^4*b^2*x^3*y^2*z - 3*a^2*b^4*x^3*y^2*z + b^6*x^3*y^2*z + 4*a^4*c^2*x^3*y^2*z - 9*a^2*b^2*c^2*x^3*y^2*z + 5*b^4*c^2*x^3*y^2*z - 5*a^2*c^4*x^3*y^2*z - 8*b^2*c^4*x^3*y^2*z + 2*c^6*x^3*y^2*z + a^6*x^2*y^3*z - 3*a^4*b^2*x^2*y^3*z + 3*a^2*b^4*x^2*y^3*z - b^6*x^2*y^3*z + 5*a^4*c^2*x^2*y^3* z - 9*a^2*b^2*c^2*x^2*y^3*z + 4*b^4*c^2*x^2*y^3*z - 8*a^2*c^4*x^2*y^3*z - 5*b^2*c^4*x^2*y^3*z + 2*c^6*x^2*y^3*z + a^6*x*y^4*z - 3*a^4*b^2*x*y^4*z + 3*a^2*b^4*x*y^4*z - b^6*x*y^4*z + 4*a^4*c^2*x*y^4*z - 7*a^2*b^2*c^2*x*y^4*z + 3*b^4*c^2*x*y^4*z + 4*a^2*c^4*x*y^4*z - 3*b^2*c^4*x*y^4*z + c^6*x*y^4*z + 2*a^4*b^2*x^4*z^2 - 4*a^2*b^4*x^4*z^2 + 2*b^6*x^4*z^2 - 4*a^2*b^2*c^2*x^4*z^2 + b^4*c^2*x^4*z^2 + 2*b^2*c^4*x^4*z^2 - a^6*x^3*y*z^2 + 4*a^4*b^2*x^3*y*z^2 - 5*a^2*b^4*x^3*y*z^2 + 2*b^6*x^3*y*z^2 + 3*a^4*c^2*x^3*y*z^2 - 9*a^2*b^2*c^2*x^3*y*z^2 - 8*b^4*c^2*x^3*y*z^2 - 3*a^2*c^4*x^3*y*z^2 + 5*b^2*c^4*x^3*y*z^2 + c^6*x^3*y*z^2 - a^6*x^2*y^2*z^2 + a^4*b^2*x^2*y^2*z^2 + a^2*b^4*x^2*y^2*z^2 - b^6*x^2*y^2*z^2 + a^4*c^2*x^2*y^2*z^2 - 24*a^2*b^2*c^2*x^2*y^2*z^2 + b^4*c^2*x^2*y^2*z^2 + a^2*c^4*x^2*y^2*z^2 + b^2*c^4*x^2*y^2*z^2 - c^6*x^2*y^2*z^2 + 2*a^6*x*y^3*z^2 - 5*a^4*b^2*x*y^3*z^2 + 4*a^2*b^4*x*y^3*z^2 - b^6*x*y^3*z^2 - 8*a^4*c^2*x*y^3*z^2 - 9*a^2*b^2*c^2*x*y^3*z^2 + 3*b^4*c^2*x*y^3*z^2 + 5*a^2*c^4*x*y^3*z^2 - 3*b^2*c^4*x*y^3*z^2 + c^6*x*y^3*z^2 + 2*a^6*y^4*z^2 - 4*a^4*b^2*y^4*z^2 + 2*a^2*b^4*y^4*z^2 + a^4*c^2*y^4*z^2 - 4*a^2*b^2*c^2*y^4*z^2 + 2*a^2*c^4*y^4*z^2 + 4*a^4*b^2*x^3*z^3 - 3*a^2*b^4*x^3*z^3 - b^6*x^3*z^3 - 8*a^2*b^2*c^2*x^3*z^3 - 3*b^4*c^2*x^3*z^3 + 4*b^2*c^4*x^3*z^3 + a^6*x^2*y*z^3 + 5*a^4*b^2*x^2*y*z^3 - 8*a^2*b^4*x^2*y*z^3 + 2*b^6*x^2*y*z^3 - 3*a^4*c^2*x^2*y*z^3 - 9*a^2*b^2*c^2*x^2*y*z^3 - 5*b^4*c^2*x^2*y*z^3 + 3*a^2*c^4*x^2*y*z^3 + 4*b^2*c^4*x^2*y*z^3 - c^6*x^2*y*z^3 + 2*a^6*x*y^2*z^3 - 8*a^4*b^2*x*y^2*z^3 + 5*a^2*b^4*x*y^2*z^3 + b^6*x*y^2*z^3 - 5*a^4*c^2*x*y^2* z^3 - 9*a^2*b^2*c^2*x*y^2*z^3 - 3*b^4*c^2*x*y^2*z^3 + 4*a^2*c^4*x*y^2*z^3 + 3*b^2*c^4*x*y^2*z^3 - c^6*x*y^2*z^3 - a^6*y^3*z^3 - 3*a^4*b^2*y^3*z^3 + 4*a^2*b^4*y^3*z^3 - 3*a^4*c^2*y^3*z^3 - 8*a^2*b^2*c^2*y^3*z^3 + 4*a^2*c^4*y^3*z^3 + 2*a^4*b^2*x^2*z^4 + a^2*b^4*x^2*z^4 + 2*b^6*x^2*z^4 - 4*a^2*b^2*c^2*x^2*z^4 - 4*b^4*c^2*x^2*z^4 + 2*b^2*c^4*x^2*z^4 + a^6*x*y*z^4 + 4*a^4*b^2*x*y*z^4 + 4*a^2*b^4*x*y*z^4 + b^6*x*y*z^4 - 3*a^4*c^2*x*y*z^4 - 7*a^2*b^2*c^2*x*y*z^4 - 3*b^4*c^2*x*y*z^4 + 3*a^2*c^4*x*y*z^4 + 3*b^2*c^4*x*y*z^4 - c^6*x*y*z^4 + 2*a^6*y^2*z^4 + a^4*b^2*y^2*z^4 + 2*a^2*b^4*y^2*z^4 - 4*a^4*c^2*y^2*z^4 - 4*a^2*b^2*c^2*y^2*z^4 + 2*a^2*c^4*y^2*z^4=0.
    y de grado 7, Σ7, ( Mostrar/Ocultar fórmula )
      Σ7:
      -a^2*c^4*x^4*y^3 + b^2*c^4*x^4*y^3 + c^6*x^4*y^3 - a^2*c^4*x^3*y^4 + b^2*c^4*x^3*y^4 - c^6*x^3*y^4 + a^4*c^2*x^4*y^2*z - b^4*c^2*x^4*y^2*z - 2*a^2*c^4*x^4*y^2*z + 2*b^2*c^4*x^4*y^2*z + c^6*x^4*y^2*z + 2*a^4*c^2*x^3*y^3* z - 2*b^4*c^2*x^3*y^3*z - 2*a^2*c^4*x^3*y^3*z + 2*b^2*c^4*x^3*y^3*z + a^4*c^2*x^2*y^4*z - b^4*c^2*x^2*y^4*z - 2*a^2*c^4*x^2*y^4*z + 2*b^2*c^4*x^2*y^4*z - c^6*x^2*y^4*z - a^4*b^2*x^4*y*z^2 + 2*a^2*b^4*x^4*y*z^2 - b^6*x^4*y*z^2 - 2*b^4*c^2*x^4*y*z^2 + b^2*c^4*x^4*y*z^2 - a^4*b^2*x^3*y^2*z^2 + 2*a^2*b^4*x^3*y^2*z^2 - b^6*x^3*y^2*z^2 + a^4*c^2*x^3*y^2*z^2 - 3*b^4*c^2*x^3*y^2*z^2 - 2*a^2*c^4*x^3*y^2*z^2 + 3*b^2*c^4*x^3*y^2*z^2 + c^6*x^3*y^2*z^2 + a^6*x^2*y^3*z^2 - 2*a^4*b^2*x^2*y^3*z^2 + a^2*b^4*x^2*y^3*z^2 + 3*a^4*c^2*x^2*y^3*z^2 - b^4*c^2*x^2*y^3*z^2 - 3*a^2*c^4*x^2*y^3*z^2 + 2*b^2*c^4*x^2*y^3*z^2 - c^6*x^2*y^3*z^2 + a^6*x*y^4*z^2 - 2*a^4*b^2*x*y^4*z^2 + a^2*b^4*x*y^4*z^2 + 2*a^4*c^2*x*y^4*z^2 - a^2*c^4*x*y^4*z^2 + a^2*b^4*x^4*z^3 - b^6*x^4*z^3 - b^4*c^2*x^4*z^3 - 2*a^4*b^2*x^3*y*z^3 + 2*a^2*b^4*x^3*y*z^3 - 2*b^4*c^2*x^3*y*z^3 + 2*b^2*c^4*x^3*y*z^3 - a^6*x^2*y^2*z^3 - 3*a^4*b^2*x^2*y^2*z^3 + 3*a^2*b^4*x^2*y^2*z^3 + b^6*x^2*y^2*z^3 + 2*a^4*c^2*x^2*y^2*z^3 - 2*b^4*c^2*x^2*y^2*z^3 - a^2*c^4*x^2*y^2*z^3 + b^2*c^4*x^2*y^2*z^3 - 2*a^4*b^2*x*y^3*z^3 + 2*a^2*b^4*x*y^3*z^3 + 2*a^4*c^2*x*y^3*z^3 - 2*a^2*c^4*x*y^3*z^3 + a^6*y^4*z^3 - a^4*b^2*y^4*z^3 + a^4*c^2*y^4*z^3 + a^2*b^4*x^3*z^4 + b^6*x^3*z^4 - b^4*c^2*x^3*z^4 - a^4*b^2*x^2*y*z^4 + 2*a^2*b^4*x^2*y*z^4 + b^6*x^2*y*z^4 - 2*b^4*c^2*x^2*y*z^4 + b^2*c^4*x^2*y*z^4 - a^6*x*y^2*z^4 - 2*a^4*b^2*x*y^2*z^4 + a^2*b^4*x*y^2*z^4 + 2*a^4*c^2*x*y^2*z^4 - a^2*c^4*x*y^2*z^4 - a^6*y^3*z^4 - a^4*b^2*y^3*z^4 + a^4*c^2*y^3*z^4=0.
    que pasa por los vértices de ABC (puntos triples), por I(incentro), por H (ortocentro) y por los pies de las cevianas de X69.


    El lugar geométrico de los puntos P tales que las rectas ma, mb y mc son concurrentes son dos curvas algebraicas de grados 6, Σ6 (que coincide con el caso anterior), y de grado 10, Δ10, ( Mostrar/Ocultar fórmula )
      Δ10:
      -a^2*c^4*x^6*y^4 + b^2*c^4*x^6*y^4 + c^6*x^6*y^4 - 2*a^2*c^4*x^5*y^5 + 2*b^2*c^4*x^5*y^5 - a^2*c^4*x^4*y^6 + b^2*c^4*x^4*y^6 - c^6*x^4*y^6 + a^4*c^2*x^6*y^3*z - 2*a^2*b^2*c^2*x^6*y^3* z + b^4*c^2*x^6*y^3*z - 2*a^2*c^4*x^6*y^3*z + 2*b^2*c^4*x^6*y^3* z + c^6*x^6*y^3*z + a^4*c^2*x^5*y^4*z - 2*a^2*b^2*c^2*x^5*y^4*z + b^4*c^2*x^5*y^4*z - 4*a^2*c^4*x^5*y^4*z + 4*b^2*c^4*x^5*y^4*z + 3*c^6*x^5*y^4*z - a^4*c^2*x^4*y^5*z + 2*a^2*b^2*c^2*x^4*y^5* z - b^4*c^2*x^4*y^5*z - 4*a^2*c^4*x^4*y^5*z + 4*b^2*c^4*x^4*y^5* z - 3*c^6*x^4*y^5*z - a^4*c^2*x^3*y^6* z + 2*a^2*b^2*c^2*x^3*y^6*z - b^4*c^2*x^3*y^6*z - 2*a^2*c^4*x^3*y^6*z + 2*b^2*c^4*x^3*y^6*z - c^6*x^3*y^6*z + a^6*x^5*y^3*z^2 - 3*a^4*b^2*x^5*y^3*z^2 + 3*a^2*b^4*x^5*y^3*z^2 - b^6*x^5*y^3*z^2 + 2*a^4*c^2*x^5*y^3*z^2 - 2*b^4*c^2*x^5*y^3* z^2 - 7*a^2*c^4*x^5*y^3*z^2 + 5*b^2*c^4*x^5*y^3*z^2 + 4*c^6*x^5*y^3*z^2 + 2*a^6*x^4*y^4*z^2 - 6*a^4*b^2*x^4*y^4*z^2 + 6*a^2*b^4*x^4*y^4*z^2 - 2*b^6*x^4*y^4*z^2 + 4*a^4*c^2*x^4*y^4*z^2 - 4*b^4*c^2*x^4*y^4*z^2 - 6*a^2*c^4*x^4*y^4*z^2 + 6*b^2*c^4*x^4*y^4*z^2 + a^6*x^3*y^5*z^2 - 3*a^4*b^2*x^3*y^5*z^2 + 3*a^2*b^4*x^3*y^5*z^2 - b^6*x^3*y^5*z^2 + 2*a^4*c^2*x^3*y^5*z^2 - 2*b^4*c^2*x^3*y^5*z^2 - 5*a^2*c^4*x^3*y^5*z^2 + 7*b^2*c^4*x^3*y^5*z^2 - 4*c^6*x^3*y^5*z^2 - a^4*b^2*x^6*y*z^3 + 2*a^2*b^4*x^6*y*z^3 - b^6*x^6*y*z^3 + 2*a^2*b^2*c^2*x^6*y*z^3 - 2*b^4*c^2*x^6*y*z^3 - b^2*c^4*x^6*y*z^3 - a^6*x^5*y^2*z^3 - 2*a^4*b^2*x^5*y^2*z^3 + 7*a^2*b^4*x^5*y^2*z^3 - 4*b^6*x^5*y^2*z^3 + 3*a^4*c^2*x^5*y^2*z^3 - 5*b^4*c^2*x^5*y^2*z^3 - 3*a^2*c^4*x^5*y^2*z^3 + 2*b^2*c^4*x^5*y^2*z^3 + c^6*x^5*y^2*z^3 - 4*a^4*b^2*x^4*y^3*z^3 + 8*a^2*b^4*x^4*y^3*z^3 - 4*b^6*x^4*y^3*z^3 + 4*a^4*c^2*x^4*y^3*z^3 - 6*b^4*c^2*x^4*y^3*z^3 - 8*a^2*c^4*x^4*y^3*z^3 + 6*b^2*c^4*x^4*y^3*z^3 + 4*c^6*x^4*y^3*z^3 + 4*a^6*x^3*y^4*z^3 - 8*a^4*b^2*x^3*y^4*z^3 + 4*a^2*b^4*x^3*y^4*z^3 + 6*a^4*c^2*x^3*y^4*z^3 - 4*b^4*c^2*x^3*y^4*z^3 - 6*a^2*c^4*x^3*y^4*z^3 + 8*b^2*c^4*x^3*y^4*z^3 - 4*c^6*x^3*y^4*z^3 + 4*a^6*x^2*y^5*z^3 - 7*a^4*b^2*x^2*y^5*z^3 + 2*a^2*b^4*x^2*y^5*z^3 + b^6*x^2*y^5*z^3 + 5*a^4*c^2*x^2*y^5*z^3 - 3*b^4*c^2*x^2*y^5*z^3 - 2*a^2*c^4*x^2*y^5*z^3 + 3*b^2*c^4*x^2*y^5*z^3 - c^6*x^2*y^5*z^3 + a^6*x*y^6*z^3 - 2*a^4*b^2*x*y^6*z^3 + a^2*b^4*x*y^6*z^3 + 2*a^4*c^2*x*y^6*z^3 - 2*a^2*b^2*c^2*x*y^6*z^3 + a^2*c^4*x*y^6*z^3 + a^2*b^4*x^6*z^4 - b^6*x^6*z^4 - b^4*c^2*x^6*z^4 - a^4*b^2*x^5*y*z^4 + 4*a^2*b^4*x^5*y*z^4 - 3*b^6*x^5*y*z^4 + 2*a^2*b^2*c^2*x^5*y*z^4 - 4*b^4*c^2*x^5*y*z^4 - b^2*c^4*x^5*y*z^4 - 2*a^6*x^4*y^2*z^4 - 4*a^4*b^2*x^4*y^2*z^4 + 6*a^2*b^4*x^4*y^2*z^4 + 6*a^4*c^2*x^4*y^2*z^4 - 6*b^4*c^2*x^4*y^2*z^4 - 6*a^2*c^4*x^4*y^2*z^4 + 4*b^2*c^4*x^4*y^2*z^4 + 2*c^6*x^4*y^2*z^4 - 4*a^6*x^3*y^3*z^4 - 6*a^4*b^2*x^3*y^3*z^4 + 6*a^2*b^4*x^3*y^3*z^4 + 4*b^6*x^3*y^3*z^4 + 8*a^4*c^2*x^3*y^3*z^4 - 8*b^4*c^2*x^3*y^3*z^4 - 4*a^2*c^4*x^3*y^3*z^4 + 4*b^2*c^4*x^3*y^3*z^4 - 6*a^4*b^2*x^2*y^4*z^4 + 4*a^2*b^4*x^2*y^4*z^4 + 2*b^6*x^2*y^4*z^4 + 6*a^4*c^2*x^2*y^4*z^4 - 6*b^4*c^2*x^2*y^4*z^4 - 4*a^2*c^4*x^2*y^4*z^4 + 6*b^2*c^4*x^2*y^4*z^4 - 2*c^6*x^2*y^4*z^4 + 3*a^6*x*y^5*z^4 - 4*a^4*b^2*x*y^5*z^4 + a^2*b^4*x*y^5*z^4 + 4*a^4*c^2*x*y^5*z^4 - 2*a^2*b^2*c^2*x*y^5*z^4 + a^2*c^4*x*y^5*z^4 + a^6*y^6*z^4 - a^4*b^2*y^6*z^4 + a^4*c^2*y^6*z^4 + 2*a^2*b^4*x^5*z^5 - 2*b^4*c^2*x^5*z^5 + a^4*b^2*x^4*y*z^5 + 4*a^2*b^4*x^4*y*z^5 + 3*b^6*x^4*y*z^5 - 2*a^2*b^2*c^2*x^4*y*z^5 - 4*b^4*c^2*x^4*y*z^5 + b^2*c^4*x^4*y*z^5 - a^6*x^3*y^2*z^5 - 2*a^4*b^2*x^3*y^2*z^5 + 5*a^2*b^4*x^3*y^2*z^5 + 4*b^6*x^3*y^2*z^5 + 3*a^4*c^2*x^3*y^2*z^5 - 7*b^4*c^2*x^3*y^2*z^5 - 3*a^2*c^4*x^3*y^2*z^5 + 2*b^2*c^4*x^3*y^2*z^5 + c^6*x^3*y^2*z^5 - 4*a^6*x^2*y^3*z^5 - 5*a^4*b^2*x^2*y^3*z^5 + 2*a^2*b^4*x^2*y^3*z^5 + b^6*x^2*y^3*z^5 + 7*a^4*c^2*x^2*y^3*z^5 - 3*b^4*c^2*x^2*y^3*z^5 - 2*a^2*c^4*x^2*y^3*z^5 + 3*b^2*c^4*x^2*y^3*z^5 - c^6*x^2*y^3*z^5 - 3*a^6*x*y^4*z^5 - 4*a^4*b^2*x*y^4*z^5 - a^2*b^4*x*y^4*z^5 + 4*a^4*c^2*x*y^4*z^5 + 2*a^2*b^2*c^2*x*y^4*z^5 - a^2*c^4*x*y^4*z^5 - 2*a^4*b^2*y^5*z^5 + 2*a^4*c^2*y^5*z^5 + a^2*b^4*x^4*z^6 + b^6*x^4*z^6 - b^4*c^2*x^4*z^6 + a^4*b^2*x^3*y*z^6 + 2*a^2*b^4*x^3*y*z^6 + b^6*x^3*y*z^6 - 2*a^2*b^2*c^2*x^3*y*z^6 - 2*b^4*c^2*x^3*y*z^6 + b^2*c^4*x^3*y*z^6 - a^6*x*y^3*z^6 - 2*a^4*b^2*x*y^3*z^6 - a^2*b^4*x*y^3*z^6 + 2*a^4*c^2*x*y^3*z^6 + 2*a^2*b^2*c^2*x*y^3*z^6 - a^2*c^4*x*y^3*z^6 - a^6*y^4*z^6 - a^4*b^2*y^4*z^6 + a^4*c^2*y^4*z^6=0.
    que pasa por los vértices de ABC (puntos cuádruples) y por H (ortocentro).
    ( Mostrar/Ocultar figura )

    Cuando el punto P recorre la séxtica Σ6, los triángulos N1N2N3 y NaNbNc degeneran: sus vértices yacen en una misma recta. Y todas la rectas la, lb, lc, ma, mb y mc son paralelas a ella.

    Si P=I, el punto de concurrencia de las rectas la, lb y lc tiene coordenadas baricéntricas:

    ( a^4 - 3a^3(b+c)+ 9a^2b*c + 3a(b-c)^2(b+c) - (b^2-c^2)^2 : ... : ... ).


    (con (6-9-13)-número de búsqueda en ETC: 2.733571194876240757704921053)

    Si P=H, el punto de concurrencia de las rectas la, lb y lc tiene coordenadas baricéntricas:

    ( a^2/(4a^4 - (b^2-c^2)^2 - 3a^2(b^2+c^2)) : b^2/(4b^4 - (c^2-a^2)^2 - 3b^2(c^2+a^2)) : c^2/(4c^4 - (a^2-b^2)^2 - 3c^2(a^2+b^2)) ).

    Este punto es el conjugado isogonal de X550, es decir, es el punto donde concurren las tangentes en los vértices de ABC a la isocúbica pK(X6,X550) (ver la sección La isocúbica pK(X6,X550))

    Si P=H, el punto de concurrencia de las rectas ma, mb y mc tiene coordenadas baricéntricas:

    ( a^2(a^2+b^2-c^2)(a^2-b^2+c^2) (a^4 - 2a^2(b^2+c^2) + b^4+3b^2c^2+c^4 ) (a^4(b^2+c^2) - 2a^2(b^4-3b^2c^2+c^4) + (b^2-c^2)^2(b^2+c^2)): ... : ... ).


    (con (6-9-13)-número de búsqueda en ETC: -336.44857544131622352057412)

  • jueves, 6 de junio del 2013

    Un centro de una circunferencia de los nueve puntos sobre la recta de Euler

    ( Anopolis #336 Antreas P. Hatzipolakis)

    Sean ABC un triángulo, A'B'C' el antipedal de N (centro de la circunferencia de los nueve puntos de ABC) y N1, N2, N3 los centros de la circunferencias de los nueve puntos de los triángulos NB'C', NC'A', NA'B', respectivamente.

    El centro de la circunferencia de los nueve puntos del triángulo N1N2N3 es el punto X5500 (8th Hatzipolakis-Montesdeoca Point) que está en la recta de Euler de ABC y sus coordenadas baricéntricas son:

    ( 2a^22
    - 15a^20(b^2+c^2)
    + 6a^18(8b^4+13b^2c^2+8c^4)-(b^2-c^2)^8(b^6+b^4c^2+b^2c^4+c^6)
    - a^16(81b^6+152b^4c^2+152b^2c^4+81c^6)
    + a^14(64b^8+111b^6c^2+128b^4c^4+111b^2c^6+64c^8)
    + a^12(14b^10+29b^8c^2+36b^6c^4+36b^4c^6+29b^2c^8+14c^10)
    - a^10(84b^12+67b^10c^2+56b^8c^4+48b^6c^6+56b^4c^8+67b^2c^10+84c^12)
    + a^8(82b^14-23b^12c^2-31b^10c^4-19b^8c^6-19b^6c^8-31b^4c^10-23b^2c^12+82c^14)
    - a^6(b^2-c^2)^2(34b^12+11b^10c^2-30b^8c^4-35b^6c^6-30b^4c^8+11b^2c^10+34c^12)
    + a^4(b^2-c^2)^4(b^10-2b^8c^2-22b^6c^4-22b^4c^6-2b^2c^8+c^10)
    + a^2(b^2-c^2)^6(4b^8+5b^6c^2+8b^4c^4+5b^2c^6+4c^8) : ... : ... ).


    ( Mostrar/Ocultar figura )


  • miércoles, 5 de junio del 2013

    Circunferencias coaxiales y reflexiones de la recta de Euler

    ( Anopolis #360 Antreas P. Hatzipolakis)

    Sean ABC un triángulo y l la recta de Euler, que corta a los lados BC, CA y AB en los puntos A', B' y C', respectivamente. Las circunferencias de diámetros AA', BB' y CC' son coaxiales y el eje d pasa por el ortocentro. Las tres circunferencias concurren en el punto X107 sobre la circunferencia circunscrita y en X125 (centro de la hipérbola de Jerabek: hipérbola equilátera circunscrita a ABC y que pasa por el circuncentro).

    ( Mostrar/Ocultar figura )

    Las reflexiones e1, e2 y e3 de la recta de Euler en los lados BC, CA y AB, respectivamente, se cortan en X110 (foco de la parábola de Kiepert), sobre la circunferencia circunscrita.

    Las reflexiones d1, d2 y d3 del eje d en los lados BC, CA y AB, respectivamente, se cortan en X1304, sobre la circunferencia circunscrita.

    Los puntos L11=e1∩d1, L22=e2∩d2 y L33=e3∩d3 son los vértices del triángulo circunceviano del ortocentro.

    Se definen los puntos L23=e2∩d3, L32=e3∩d2; L31=e3∩d1, L13=e1∩d3; L12=e1∩d2, L21=e2∩d1.
    Los triángulos L23L31L12 y L32L13L21 son perspectivos y su centro de perspectividad es el punto X de coordenadas baricéntricas:
    ( a^2(a^2-b^2)(a^2-c^2) (2a^6-4a^2(b^2-c^2)^2-a^4(b^2+c^2)+3(b^2-c^2)^2(b^2+c^2)): ... : ... ),
    (con (6-9-13)-número de búsqueda en ETC: 0.71653354827873787712696972)
    Incorporado a ETC como el punto X5502 (10th Hatzipolakis-Montesdeoca point).

  • martes, 4 de junio del 2013

    Circunferencias de los nueve puntos concurrentes

    ( Anopolis #354 Antreas P. Hatzipolakis)

    Sean ABC un triángulo, para todo punto P las circunferencias de los nueve puntos de los triángulos PBC, PCA y PAB son concurrentes, en un punto sobre la circunferencia de los nueve puntos de ABC.

    ( Mostrar/Ocultar figura )
    Si P=(u:v:w), en coordenadas baricéntricas, el punto de concurrencia Q (punto de Poncelet de de los puntos A, B, C y P) tiene coordenadas baricéntricas:
    ( u((a^2-b^2+c^2)v-(a^2+b^2-c^2)w)(b^2w(u+v)-c^2v(u+w)): v((b^2-c^2+a^2)w-(b^2+c^2-a^2)u)(c^2u(v+w)-a^2w(v+u)): w((c^2-a^2+b^2)u-(c^2+a^2-b^2)v)(a^2v(w+u)-b^2u(w+v)) ).

    Algunos ejemplos:
    Q=X11 si P=X1, X7, X8, X9, X21, X79, X80, X84, X90, X104, ...
    Q=X113 si P= X110, X112, ...
    Q=X114 si P= X99, ...
    Q=X115 si P= X2, X10, X13, X14, X17, X18, X76, X83, X94, X96, X98, ...
    Q=X116 si P= X103, ...
    Q=X117 si P= X109, ...
    Q=X118 si P= X101, ...
    Q=X119 si P= X100, ...
    Q=X122 si P= X20, ...
    Q=X124 si P=X58, X102, ...
    Q=X125 si P= X3, X6, X54, X64, X65, X66, X67, X68, X69, X70, X71, X72, X73, X74, ...
    Q=X127 si P=X22, ...
    Q=X130 si P=X51, ...
    Q=X133 si P=X107, ...
    Q=X134 si P=X52, ...
    Q=X135 si P=X34, ...
    Q=X137 si P=X5, X53, ...
    Q=X2679 si P=X32, ...
    Q=X3258 si P=X30, ...
    Q=X3259 si P=X56, ...
    Q=X5099 si P=X23, ...
    Q=X5139 si P=X25, ...
    Q=X5190 si P=X27, X92, ...

  • lunes, 3 de junio del 2013

    La isocúbica pK(X6,X550)

    ( Anopolis #351 Antreas P. Hatzipolakis)

    Sean ABC un triángulo, X un punto; N1, N2, N3 los centros de las circunferencias de los nueve puntos de los triángulos XBC, XCA, XAB, resp. and A'B'C' el triángulo antipedal de X. Los triángulos N1N2N3 y A'B'C' son perspectivos si y solo si X queda sobre la cúbica isogonal pK(X6,X550) (junto con la recta del infinito y la circunferencia circunscrita a ABC).

    ( Mostrar/Ocultar figura )

    Si X está sobre la cúbica pK(X6,X550) los triángulos N1N2N3 y A'B'C' son inversamente semejantes.

    El pivote P=X(550) de la isocúbica pK(X6,X550) es el punto medio del circuncentro y el punto de De Longchamps, X(20). La cúbica pK(X6,X550) es un elemento del Euler-haz de isocúbicas pivotales con pivote en la recta de Euler (OP = k OH, vectores, k=-1/2) y, por consiguiente, pasa por A, B , C, I=X(1), Ia, Ib, Ic (vértices del triángulo excentral), X(3), X(4), X(550),..; por los pies de las cevianas de X(550) (como en toda isocúbica, §1.4 Pivotal isocubics, Bernard Gibert.- Special Isocubics in the Triangle Plane).
    Las tangentes en los vértices A, B y C a la cúbica concurren en el punto P* (conjugado isogonal del pivote: 6º punto de intersección de la cúbica con la hipérbola equilátera circunscrita a ABC y que pasa por el circuncentro, hipérbola de Jerabek), de coordenadas baricéntricas:

    ( a^2(a^8 + a^6(b^2+c^2)+ a^4(-9b^4+19b^2c^2-9c^4) + 11a^2(b^2-c^2)^2(b^2+c^2) - (b^2-c^2)^2(4b^4+17b^2c^2+4c^4)): ... : ... ),

    que está sobre la cúbica y con (6-9-13)-número de búsqueda en ETC: -702.0040014335101057823010.

    Si X está en la circunferencia circunscrita a ABC su triángulo antipedal degenera en el punto antipodal de X.

  • domingo, 2 de junio del 2013

    Curvas algebraicas de grado cinco asociadas a centros de circunferencias de los nueve puntos

    ( Anopolis #344 Antreas P. Hatzipolakis)

    Sean ABC un triángulo, P un punto, N1, N2 y N3 los centros de las circunferencias de los nueve puntos de los triángulos PBC, PCA y PAB, respectivamente, O' el circuncentro de N1N2N3 y N' el centro de la circunferencia de los nueve puntos de N1N2N3.

    Si P=N (centro de la circunferencia de los nueve puntos de ABC), el punto O', que está en la recta de Euler de ABC, tiene coordenadas baricéntricas:

    ( -2a^16+ 9a^14(b^2+c^2)-
    a^10(b^6+b^4c^2+b^2c^4+c^6)+
    a^8(25b^8+10b^6c^2+8b^4c^4+10b^2c^6+25c^8) +
    a^6(-33b^10+31b^8c^2+11b^6c^4+11b^4c^6+31b^2c^8-33c^10)+
    a^4(b^2-c^2)^2(21b^8-20b^6c^2-25b^4c^4-20b^2c^6+21c^8)-
    a^2(b^2-c^2)^4(7b^6-13b^4c^2-13b^2c^4+7c^6)+
    (b^2-c^2)^6(b^4-4b^2c^2+c^4)-a^12(13b^4+18b^2c^2+13c^4): ... : ... )

    (con (6-9-13)-número de búsqueda en ETC: -3.187693716504994256890940). Incorporado a ETC como el punto X5501 (9th Hatzipolakis-Montesdeoca Point).

    Si P=O (centro de la circunferencia circunscrita a ABC), el punto N', que está en la recta de Euler de ABC, tiene coordenadas baricéntricas:

    ( 2a^10 - 5a^8(b^2+c^2) + 2a^6(b^4+5b^2c^2+c^4) + a^4(4b^6-5b^4c^2-5b^2c^4+4c^6) - a^2(b^2-c^2)^2(4b^4+5b^2c^2+4c^4) + (b^2-c^2)^4(b^2+c^2) : ... : ...)

    (con (6-9-13)-número de búsqueda en ETC: 4.7800096839999025703058)

    El lugar geométrico de P tal que N, P y O' están alineados es una curva algebraica de quinto grado que contiene a los vértices A, B y C (que son puntos doble) y pasa por los puntos X(1), X(4), X(5), ...
    Su ecuación baricéntrica es: ( Mostrar/Ocultar fórmula )
      a^6c^2x^3y^2 - 3a^4b^2c^2x^3y^2 + 3a^2b^4c^2x^3y^2 - b^6c^2x^3y^2 - 3a^4c^4x^3y^2 + 3a^2b^2c^4x^3y^2 + 3a^2c^6x^3y^2 + 2b^2c^6x^3y^2 - c^8x^3y^2 + a^6c^2x^2y^3 - 3a^4b^2c^2x^2y^3 + 3a^2b^4c^2x^2y^3 - b^6c^2x^2y^3 - 3a^2b^2c^4x^2y^3 + 3b^4c^4x^2y^3 - 2a^2c^6x^2y^3 - 3b^2c^6x^2y^3 + c^8x^2y^3 + a^2b^4c^2x^3yz - b^6c^2x^3yz - a^2b^2c^4x^3yz + b^2c^6x^3yz + 2a^4b^2c^2x^2y^2z - 2a^2b^4c^2x^2y^2z + a^6c^2xy^3z - a^4b^2c^2xy^3z + a^2b^2c^4xy^3z - a^2c^6xy^3z - a^6b^2x^3z^2 + 3a^4b^4x^3z^2 - 3a^2b^6x^3z^2 + b^8x^3z^2 + 3a^4b^2c^2x^3z^2 - 3a^2b^4c^2x^3z^2 - 2b^6c^2x^3z^2 - 3a^2b^2c^4x^3z^2 + b^2c^6x^3z^2 - 2a^4b^2c^2x^2yz^2 + 2a^2b^2c^4x^2yz^2 + 2a^2b^4c^2xy^2z^2 - 2a^2b^2c^4xy^2z^2 - a^8y^3z^2 + 3a^6b^2y^3z^2 - 3a^4b^4y^3z^2 + a^2b^6y^3z^2 + 2a^6c^2y^3z^2 + 3a^4b^2c^2y^3z^2 - 3a^2b^4c^2y^3z^2 + 3a^2b^2c^4y^3z^2 - a^2c^6y^3z^2 - a^6b^2x^2z^3 + 2a^2b^6x^2z^3 - b^8x^2z^3 + 3a^4b^2c^2x^2z^3 + 3a^2b^4c^2x^2z^3 + 3b^6c^2x^2z^3 - 3a^2b^2c^4x^2z^3 - 3b^4c^4x^2z^3 + b^2c^6x^2z^3 - a^6b^2xyz^3 + a^2b^6xyz^3 + a^4b^2c^2xyz^3 - a^2b^4c^2xyz^3 + a^8y^2z^3 - 2a^6b^2y^2z^3 + a^2b^6y^2z^3 - 3a^6c^2y^2z^3 - 3a^4b^2c^2y^2z^3 - 3a^2b^4c^2y^2z^3 + 3a^4c^4y^2z^3 + 3a^2b^2c^4y^2z^3 - a^2c^6y^2z^3=0
    ( Mostrar/Ocultar figura )

    El lugar geométrico del los puntos P tales que O (circuncentro de ABC), P y O' están alineados es una curva algebraica de grado cinco que pasa por los vértices A, B, C (que son puntos dobles), por los pies de las cevianas de X(5) y por los puntos X(3), X(4), X(5), ...
    Su ecuación baricéntrica es: ( Mostrar/Ocultar fórmula )
      a^4c^4x^3y^2 - a^2b^2c^4x^3y^2 - 2a^2c^6x^3y^2 - b^2c^6x^3y^2 + c^8x^3y^2 + a^2b^2c^4x^2y^3 - b^4c^4x^2y^3 + a^2c^6x^2y^3 + 2b^2c^6x^2y^3 - c^8x^2y^3 + a^6b^2x^3yz - 3a^4b^4x^3yz + 3a^2b^6x^3yz - b^8x^3yz - a^6c^2x^3yz + b^6c^2x^3yz + 3a^4c^4x^3yz - 3a^2c^6x^3yz - b^2c^6x^3yz + c^8x^3yz + a^8x^2y^2z - 2a^6b^2x^2y^2z + 2a^2b^6x^2y^2z - b^8x^2y^2z - 2a^6c^2x^2y^2z + 2b^6c^2x^2y^2z + a^4c^4x^2y^2z - b^4c^4x^2y^2z + a^8xy^3z - 3a^6b^2xy^3z + 3a^4b^4xy^3z - a^2b^6xy^3z - a^6c^2xy^3z + b^6c^2xy^3z - 3b^4c^4xy^3z + a^2c^6xy^3z + 3b^2c^6xy^3z - c^8xy^3z - a^4b^4x^3z^2 + 2a^2b^6x^3z^2 - b^8x^3z^2 + a^2b^4c^2x^3z^2 + b^6c^2x^3z^2 - a^8x^2yz^2 + 2a^6b^2x^2yz^2 - a^4b^4x^2yz^2 + 2a^6c^2x^2yz^2 + b^4c^4x^2yz^2 - 2a^2c^6x^2yz^2 - 2b^2c^6x^2yz^2 + c^8x^2yz^2 + a^4b^4xy^2z^2 - 2a^2b^6xy^2z^2 + b^8xy^2z^2 - 2b^6c^2xy^2z^2 - a^4c^4xy^2z^2 + 2a^2c^6xy^2z^2 + 2b^2c^6xy^2z^2 - c^8xy^2z^2 + a^8y^3z^2 - 2a^6b^2y^3z^2 + a^4b^4y^3z^2 - a^6c^2y^3z^2 - a^4b^2c^2y^3z^2 - a^2b^6x^2z^3 + b^8x^2z^3 - a^2b^4c^2x^2z^3 - 2b^6c^2x^2z^3 + b^4c^4x^2z^3 - a^8xyz^3 + a^6b^2xyz^3 - a^2b^6xyz^3 + b^8xyz^3 + 3a^6c^2xyz^3 - 3b^6c^2xyz^3 - 3a^4c^4xyz^3 + 3b^4c^4xyz^3 + a^2c^6xyz^3 - b^2c^6*xyz^3 - a^8y^2z^3 + a^6b^2y^2z^3 + 2a^6c^2y^2z^3 + a^4b^2c^2*y^2z^3 - a^4c^4y^2z^3=0
    ( Mostrar/Ocultar figura )

    El lugar geométrico del los puntos P tales que O (circuncentro de ABC), P y N' están alineados es una curva algebraica de grado cinco que pasa por los vértices A, B, C (que son puntos dobles), por los pies de las cevianas de X(3) y por los puntos X(3), X(4), ...
    Su ecuación baricéntrica es: ( Mostrar/Ocultar fórmula )
      a^2b^2c^4x^3y^2 - b^4c^4x^3y^2 + b^2c^6x^3y^2 + a^4c^4x^2y^3 - a^2b^2c^4x^2y^3 - a^2c^6x^2y^3 - a^6b^2x^3yz + 3a^4b^4x^3yz - 3a^2b^6x^3yz + b^8x^3yz + a^6c^2x^3yz + 2a^2b^4c^2x^3yz - 3b^6c^2x^3yz - 3a^4c^4x^3yz - 2a^2b^2c^4x^3yz + 3a^2c^6x^3yz + 3b^2c^6x^3yz - c^8x^3yz - a^8x^2y^2z + 2a^6b^2x^2y^2z - 2a^2b^6x^2y^2z + b^8x^2y^2z + 3a^6c^2x^2y^2z + a^4b^2c^2x^2y^2z - a^2b^4c^2x^2y^2z - 3b^6c^2x^2y^2z - 3a^4c^4x^2y^2z + 3b^4c^4x^2y^2z + a^2c^6x^2y^2z - b^2c^6x^2y^2z - a^8xy^3z + 3a^6b^2xy^3z - 3a^4b^4xy^3z + a^2b^6xy^3z + 3a^6c^2xy^3z - 2a^4b^2c^2xy^3z - b^6c^2xy^3z + 2a^2b^2c^4xy^3z + 3b^4c^4xy^3z - 3a^2c^6xy^3z - 3b^2c^6xy^3z + c^8xy^3z - a^2b^4c^2x^3z^2 - b^6c^2x^3z^2 + b^4c^4x^3z^2 + a^8x^2yz^2 - 3a^6b^2x^2yz^2 + 3a^4b^4x^2yz^2 - a^2b^6x^2yz^2 - 2a^6c^2x^2yz^2 - a^4b^2c^2x^2yz^2 + b^6c^2x^2yz^2 + a^2b^2c^4x^2yz^2 - 3b^4c^4x^2yz^2 + 2a^2c^6x^2yz^2 + 3b^2c^6x^2yz^2 - c^8x^2yz^2 + a^6b^2xy^2z^2 - 3a^4b^4xy^2z^2 + 3a^2b^6xy^2z^2 - b^8xy^2z^2 - a^6c^2xy^2z^2 + a^2b^4c^2xy^2z^2 + 2b^6c^2xy^2z^2 + 3a^4c^4xy^2z^2 - a^2b^2c^4xy^2z^2 - 3a^2c^6xy^2z^2 - 2b^2c^6xy^2z^2 + c^8xy^2z^2 + a^6c^2y^3z^2 + a^4b^2c^2y^3z^2 - a^4c^4y^3z^2 - a^4b^4x^2z^3 + a^2b^6x^2z^3 + a^2b^4c^2x^2z^3 + a^8xyz^3 - 3a^6b^2xyz^3 + 3a^2b^6xyz^3 - b^8xyz^3 - 3a^6c^2xyz^3 + 2a^4b^2c^2xyz^3 - 2a^2b^4c^2xyz^3 + 3b^6c^2xyz^3 + 3a^4c^4xyz^3 - 3b^4c^4xyz^3 - a^2c^6xyz^3 + b^2c^6xyz^3 - a^6b^2y^2z^3 + a^4b^4y^2z^3 - a^4b^2c^2y^2z^3=0
    ( Mostrar/Ocultar figura )


  • sábado, 1 de junio del 2013

    Perpendiculares a cevianas conjugadas isogonales

    ( Anopolis #343 Antreas P. Hatzipolakis)

    Sean ABC un triángulo y P un punto, P* su conjugado isogonal, A'B'C' y A"b"C" los triángulos cevianos de P y P*, respectivamente.

    A* es el punto de intersección de la perpendicular a AA' por A' y de la perpendicular a AA" por A",
    B* es el punto de intersección de la perpendicular a BB' por B' y de la perpendicular a BB" por B" y
    C* es el punto de intersección de la perpendicular a CC' por C' y de la perpendicular a CC" por C".

    Entonces, los triángulos ABC y A*B*C* son perspectivos, para cualquier punto P, y el centro de perspectividad es el circuncentro de ABC.

    ( Mostrar/Ocultar figura )

    Si las coordenadas baricéntricas de P son (u:v:w),
    A*=( a^2(v-w)-(b^2-c^2)(v+w))(a^2(c^2v-b^2w)+(b^2-c^2)(c^2v+b^2w)) : 2b^2(-a^2 + b^2 - c^2) (c^2v(v+w)+w(-a^2v+b^2(v+w))) : 2c^2(-a^2 - b^2 + c^2) (c^2v(v+w) +w(-a^2v +b^2(v+w))) )

    Por lo que la ecuación de la recta AA* es c^2(a^2+b^2-c^2)y - b^2(a^2-b^2+c^2)z=0, que pasa por el circuncentro de ABC.

  • viernes, 31 de mayo del 2013

    Caracterización geométrica de la cúbica de Napoleon-Feuerbach

    ( Anopolis #335 Antreas P. Hatzipolakis)

    Sean ABC un triángulo, P un punto, N, N1, N2, N3 los centro de las circunferencias de los nueve puntos de los triángulos ABC, PBC, PCA y PAB, respectivamente y N* el centro de la circunferencia de los nueve puntos del triángulo N1N2N3.

    Los puntos P, N y N* están alineados si y solo si P está en la circunferencia circunscrita a ABC o en la cúbica de Napoleon-Feuerbach (K005 del catálogo de Bernard Gibert)

    ( Mostrar/Ocultar figura )

    Si P varía sobre la circunferencia circunscrita a ABC, el lugar geométrico de N* es la circunferencia con centro en X140 y radio R/2.

  • jueves, 30 de mayo del 2013

    Reflexiones en cevianas y una curva algebraica de grado nueve

    Sea ABC un triángulo y A'B'C' el triángulo ceviano de un punto P. Se denota por
    L11 la perpendicular a AA' por A',
    L22 la perpendicular a BB' por B',
    L33 la perpendicular a CC' por C'.

    L12 la simétrica de L11 respecto a BB',
    L13 la simétrica de L11 respecto a CC'.

    A"=L12∩L13 y similarmente se definen B" y C".

    El lugar geométrico de los puntos P tal que los triángulos ABC y A"B"C" son perspectivos es una curva algebraica de grado nueve ( Mostrar/Ocultar ecuacíón )

      -2*a^2*c^4*x^5*y^4 + 2*b^2*c^4*x^5*
  y^4 + 2*c^6*x^5*y^4 - 
 2*a^2*c^4*x^4*y^5 + 2*b^2*c^4*x^4*
  y^5 - 2*c^6*x^4*y^5 - 
 a^4*c^2*x^5*y^3*z + 2*a^2*b^2*c^2*
  x^5*y^3*z - b^4*c^2*x^5*y^3*z + 
 2*a^2*c^4*x^5*y^3*z + 
 6*b^2*c^4*x^5*y^3*z - 
 c^6*x^5*y^3*z + a^4*c^2*x^3*y^5*z - 
 2*a^2*b^2*c^2*x^3*y^5*z + 
 b^4*c^2*x^3*y^5*z - 6*a^2*c^4*x^3*
  y^5*z - 2*b^2*c^4*x^3*y^5*z + 
 c^6*x^3*y^5*z - a^6*x^4*y^3*z^2 + 
 3*a^4*b^2*x^4*y^3*z^2 - 
 3*a^2*b^4*x^4*y^3*z^2 + 
 b^6*x^4*y^3*z^2 - a^4*c^2*x^4*y^3*
  z^2 + 2*a^2*b^2*c^2*x^4*y^3*z^2 - 
 b^4*c^2*x^4*y^3*z^2 + 
 5*a^2*c^4*x^4*y^3*z^2 + 
 3*b^2*c^4*x^4*y^3*z^2 - 
 3*c^6*x^4*y^3*z^2 - a^6*x^3*y^4*
  z^2 + 3*a^4*b^2*x^3*y^4*z^2 - 
 3*a^2*b^4*x^3*y^4*z^2 + 
 b^6*x^3*y^4*z^2 + a^4*c^2*x^3*y^4*
  z^2 - 2*a^2*b^2*c^2*x^3*y^4*z^2 + 
 b^4*c^2*x^3*y^4*z^2 - 
 3*a^2*c^4*x^3*y^4*z^2 - 
 5*b^2*c^4*x^3*y^4*z^2 + 
 3*c^6*x^3*y^4*z^2 + a^4*b^2*x^5*y*
  z^3 - 2*a^2*b^4*x^5*y*z^3 + 
 b^6*x^5*y*z^3 - 2*a^2*b^2*c^2*x^5*y*
  z^3 - 6*b^4*c^2*x^5*y*z^3 + 
 b^2*c^4*x^5*y*z^3 + a^6*x^4*y^2*
  z^3 + a^4*b^2*x^4*y^2*z^3 - 
 5*a^2*b^4*x^4*y^2*z^3 + 
 3*b^6*x^4*y^2*z^3 - 3*a^4*c^2*x^4*
  y^2*z^3 - 2*a^2*b^2*c^2*x^4*y^2*
  z^3 - 3*b^4*c^2*x^4*y^2*z^3 + 
 3*a^2*c^4*x^4*y^2*z^3 + 
 b^2*c^4*x^4*y^2*z^3 - 
 c^6*x^4*y^2*z^3 - 3*a^6*x^2*y^4*
  z^3 + 5*a^4*b^2*x^2*y^4*z^3 - 
 a^2*b^4*x^2*y^4*z^3 - 
 b^6*x^2*y^4*z^3 + 3*a^4*c^2*x^2*y^4*
  z^3 + 2*a^2*b^2*c^2*x^2*y^4*z^3 + 
 3*b^4*c^2*x^2*y^4*z^3 - 
 a^2*c^4*x^2*y^4*z^3 - 
 3*b^2*c^4*x^2*y^4*z^3 + 
 c^6*x^2*y^4*z^3 - a^6*x*y^5*z^3 + 
 2*a^4*b^2*x*y^5*z^3 - 
 a^2*b^4*x*y^5*z^3 + 6*a^4*c^2*x*y^5*
  z^3 + 2*a^2*b^2*c^2*x*y^5*z^3 - 
 a^2*c^4*x*y^5*z^3 + 2*a^2*b^4*x^5*
  z^4 - 2*b^6*x^5*z^4 - 
 2*b^4*c^2*x^5*z^4 + a^6*x^3*y^2*
  z^4 - a^4*b^2*x^3*y^2*z^4 + 
 3*a^2*b^4*x^3*y^2*z^4 - 
 3*b^6*x^3*y^2*z^4 - 3*a^4*c^2*x^3*
  y^2*z^4 + 2*a^2*b^2*c^2*x^3*y^2*
  z^4 + 5*b^4*c^2*x^3*y^2*z^4 + 
 3*a^2*c^4*x^3*y^2*z^4 - 
 b^2*c^4*x^3*y^2*z^4 - 
 c^6*x^3*y^2*z^4 + 3*a^6*x^2*y^3*
  z^4 - 3*a^4*b^2*x^2*y^3*z^4 + 
 a^2*b^4*x^2*y^3*z^4 - 
 b^6*x^2*y^3*z^4 - 5*a^4*c^2*x^2*y^3*
  z^4 - 2*a^2*b^2*c^2*x^2*y^3*z^4 + 
 3*b^4*c^2*x^2*y^3*z^4 + 
 a^2*c^4*x^2*y^3*z^4 - 
 3*b^2*c^4*x^2*y^3*z^4 + 
 c^6*x^2*y^3*z^4 + 2*a^6*y^5*z^4 - 
 2*a^4*b^2*y^5*z^4 + 2*a^4*c^2*y^5*
  z^4 + 2*a^2*b^4*x^4*z^5 + 
 2*b^6*x^4*z^5 - 2*b^4*c^2*x^4*z^5 - 
 a^4*b^2*x^3*y*z^5 + 6*a^2*b^4*x^3*y*
  z^5 - b^6*x^3*y*z^5 + 
 2*a^2*b^2*c^2*x^3*y*z^5 + 
 2*b^4*c^2*x^3*y*z^5 - 
 b^2*c^4*x^3*y*z^5 + a^6*x*y^3*z^5 - 
 6*a^4*b^2*x*y^3*z^5 + 
 a^2*b^4*x*y^3*z^5 - 2*a^4*c^2*x*y^3*
  z^5 - 2*a^2*b^2*c^2*x*y^3*z^5 + 
 a^2*c^4*x*y^3*z^5 - 2*a^6*y^4*z^5 - 
 2*a^4*b^2*y^4*z^5 + 2*a^4*c^2*y^4*
  z^5=0
    Esta curva contiene a A, B y C que son puntos cuádruples; dos de las cuatro tangentes en A son las dos bisectrices. Pasa por los pies de las alturas. Contiene al baricentro X2, y a los dos centros isogónicos X13 y X14.
    ( Mostrar/Ocultar figura )

    Cuando P=X2, las rectas AA", BB" y BB" son paralelas, con punto del infinito X524. Si P es uno de los centros isogónicos el centro de perspectividad Q de ABC y A"B"C" coincide con cada uno de ellos.

  • miércoles, 29 de mayo del 2013

    Circunferencias concurrentes con centros cocíclicos

    ( Anopolis #327 Antreas P. Hatzipolakis)

    Sea ABC un triángulo y A'B'C' el triángulo ceviano del incentro I. Se denota por
    L11 la perpendicular a AA' por A',
    L22 la perpendicular a BB' por B',
    L33 la perpendicular a CC' por C'.

    L12 la simétrica de L11 respecto a BB',
    L13 la simétrica de L11 respecto a CC'.

    M12 la paralela a L12 por B',
    M13 la paralela a L13 por C'.

    A"=M12∩M13 y similarmente se definen B" y C".

    O1 el circuncentro del triángulo A"B'C',
    O2 el circuncentro del triángulo B"C'A',
    O3 el circuncentro del triángulo C"A'B'.

    ( Mostrar/Ocultar figura )

    Entonces, los puntos I, O1, O2 y O3 son cocíclicos y el centro X de la circunferencia donde están tiene coordenadas baricéntricas:
    ( a(b+c)(a^5 - 2a^3(b^2+c^2)- a^2b c(b+c) + a(b^4-b^2c^2+c^4) + b(b-c)^2c(b+c) ) : ... : ... ).
    (con (6-9-13)-número de búsqueda en ETC: 0.4282485572315154394723079182). Incluido en la "Encyclopedia of Triangle Centers- ETC" con el número X5496="4th Hatzipolakis-Montesdeoca point".

    Además, las circunferencias circunscritas a los triángulos A"B'C', B"C'A' y C"A'B' son concurrentes en el punto Y:
    ( a(a^6-a^5(b+c) - a^4(b+c)^2+ a^3(2b^3+b^2c+b c^2+2c^3) - a^2(b^4-b^3c-3b^2c^2-b c^3+c^4) - a(b^5+b^4c+b c^4+c^5) + (b^2-c^2)^2(b^2+c^2) ) : ... : ... )
    (con (6-9-13)-número de búsqueda en ETC: 1.1800557516066828130300968705). Incluido en la "Encyclopedia of Triangle Centers- ETC" con el número X5497="5th Hatzipolakis-Montesdeoca point".

  • martes, 28 de mayo del 2013

    Circuncentros cocíclicos

    ( Anopolis #321 Antreas P. Hatzipolakis)

    Sea ABC un triángulo y A'B'C' el triángulo ceviano del incentro I. Se denota por
    L1 la perpendicular a AA' por A',
    L2 la perpendicular a BB' por B',
    L3 la perpendicular a CC' por C',
    (i.e. las rectas determinan el triángulo antipedal de I con respecto a A'B'C'),

    L11 la simétrica de L1 respecto a AA' (coincide con L1),
    L12 la simétrica de L1 respecto a BB',
    L13 la simétrica de L1 respecto a CC'.

    L21 la simétrica de L2 respecto a AA',
    L22 la simétrica de L2 respecto a BB' (coincide con L2),
    L23 la simétrica de L2 respecto a CC'.

    L31 la simétrica de L3 respecto a AA',
    L32 la simétrica de L3 respecto a BB',
    L33 la simétrica de L3 respecto a CC' (coincide con L3).

    O1 el circuncentro del triángulo AaAbAc acotado por las rectas L11, L12 y L13,
    O2 el circuncentro del triángulo BaBbBc acotado por las rectas L21, L22 y L23,
    O3 el circuncentro del triángulo CaCbCc acotado por las rectas L31, L32 y L33.

    ( Mostrar/Ocultar figura )

    Entonces, O1, O2, O3 y O (circuncentro de ABC) son cocíclicos en el punto X de coordenadas baricéntricas:
    ( a^2 ( a^7 (b + c) - a^6 (b^2 + c^2) - a^5(3 b^3 + 2 b^2 c + 2 b c^2 + 3 c^3) + a^4 (3 b^4 - b^3 c + 4 b^2 c^2 - b c^3 + 3 c^4) + a^3 (3 b^5 + b^4c + 2 b^3c^2 + 2 b^2 c^3 + b c^4 + 3 c^5) - a^2 (3 b^6 - 2 b^5 c - 2 b c^5 + 3 c^6) - a (b^7 - b^4 c^3 - b^3 c^4 + c^7) + (b^2 - c^2)^2(b^4 - b^3 c - b^2 c^2 - b c^3 + c^4)) : ... : ... ).
    (con (6-9-13)-número de búsqueda en ETC: 0.02511187325738577837)

    NOTA: Este punto ha sido incluido (28/05/2013) en la "Encyclopedia of Triangle Centers- ETC" con el número X5495="3rd Hatzipolakis-Montesdeoca point".

    El centro radical de las circunferencias circunscritas a los triángulos AaAbAc, BaBbBc y CaCbCc es X500, ortocentro del triángulo A'B'C'. (Randy Hutson)



  • domingo, 26 de mayo del 2013

    Concurrencia de rectas paralelas a rectas de Euler

    ( Anopolis #308 )

    Sea ABC un triángulo y A'B'C' el triángulo ceviano del incentro. Se denota por
    Ab, Ac las reflexiones de A' en BB' y CC', resp.
    Bc, Ba las reflexiones de B' en CC', AA', resp.
    Ca, Cb las reflexiones de C' en AA', BB', resp.
    La, Lb y Lc las rectas de Euler de los triángulos A'AbAc, B'BcBa, C'CaCb, resp.
    L'a, L'b, L'c las rectas paralelas a La, Lb, Lc por A, B y C, resp.

    Entonces, las rectas L'a, L'b, L'c concurren en X80 (simétrico del incentro respecto al punto de Feuerbach).

    ( Mostrar/Ocultar figura )


  • sábado, 25 de mayo del 2013

    Concurrencia de rectas simétricas de rectas de Euler

    ( Anopolis #301 )

    Sea ABC un triángulo y A'B'C' el triángulo ceviano del incentro. Se denota por
    Ab, Ac las reflexiones de A' en BB' y CC', resp.
    Bc, Ba las reflexiones de B' en CC', AA', resp.
    Ca, Cb las reflexiones de C' en AA', BB', resp.
    L, La, Lb y Lc las rectas de Euler de los triángulos ABC, AAbAc, BBcBa, CCaCb, resp.
    L'a, L'b, L'c las rectas simétricas de La, Lb, Lc respecto a AA', BB', CC', resp.

    Entonces, las rectas L,La, Lb, Lc son paralelas y las rectas L'a, L'b, L'c concurren en el punto de coordenadas baricéntricas:

    ( a(a^9 - a^8(b+c) - a^7(b-c)^2 + a^6(2b^3-b^2c-b*c^2+2c^3) - a^5(3b^4+b^3c-7b^2c^2+b*c^3+3c^4) + 4a^4b*c(b-c)^2(b+c) + a^3(b^2-c^2)^2(5b^2-4b*c+5c^2) - a^2(b-c)^2(2b^5+5b^4c+b^3c^2+b^2c^3+5b*c^4+2c^5) - a(b^2-c^2)^2(2b^4-3b^3c+5b^2c^2-3b*c^3+2c^4) + (b-c)^4(b+c)^3(b^2+c^2)) : ... : ... ).
    ( Mostrar/Ocultar figura )

    Este punto ha sido incluido (28/05/2013) en la "Encyclopedia of Triangle Centers- ETC" con el número X5494="2nd Hatzipolakis-Montesdeoca point".



  • viernes, 24 de mayo del 2013

    Circunferencias de Euler y triángulos ortológicos

    ( Anopolis #295 )

    Un triángulo ABC es ortológico respecto otro DEF si las perpendiculares por A, B y C a los lados de DEF se cortan en un punto. En tal caso, se verifica que, recíprocamente, DEF es ortológico respecto ABC: las perpendiculares por D, E y F a los lados de ABC también se cortan en un punto. A estos puntos de concurrencia se les llama centros de ortología.

    Sea ABC un triángulo y A'B'C', A"B"C" los triángulo órtico y medial, respectivamente.
    Se denota por:
    (Ab), (Ac) las circunferencias de los nueve puntos de A'BC", A'B"C, resp.
    (Bc), (Ba) las circunferencias de los nueve puntos de B'CA", B'C"A, resp.
    (Ca), (Cb) las circunferencias de los nueve puntos de C'AB", C'A"B, resp.
    Entonces, los triángulos ABC y DEF, el acotado por las recta (que pasan por los centros de las circunferencias) BcCb, CaAc, AbBa, son ortológicos.

    ( Mostrar/Ocultar figura )

    El centro de ortología de ABC y DEF es X1994. (Randy Hutson)

    El centro de ortología Y de DEF y ABC tiene coordenadas baricéntricas:

    ( 5a^8(b^2+c^2) - a^6(13b^4+10b^2c^2+13c^4) + a^4(9b^6-8b^4c^2-8b^2c^4+9c^6)+ a^2(b^2-c^2)^2(b^4+9b^2c^2+c^4) - 2(b^2-c^2)^4(b^2+c^2) : ... : ... ).


  • jueves, 23 de mayo del 2013

    Circunferencias de Euler y ejes radicales concurrentes

    ( Anopolis #280 )

    Sea ABC un triángulo y A'B'C', A"B"C" los triángulo órtico y medial, respectivamente.
    Se denota por:
    (Ab), (Ac) las circunferencias de los nueve puntos de A'BC", A'B"C, resp.
    (Bc), (Ba) las circunferencias de los nueve puntos de B'CA", B'C"A, resp.
    (Ca), (Cb) las circunferencias de los nueve puntos de C'AB", C'A"B, resp.
    r1 el eje radical de (Bc), (Cb);   r2 el eje radical de (Ca), (Ac);   r3 el eje radical de (Ab), (Ba)

    Entonces, r1, r2, r3 son concurrentes en el punto de X de coordenadas baricéntricas:

    ( 2a^6 - 4a^2b^2c^2 - 3a^4(b^2 + c^2) + (b^2 - c^2)^2 (b^2 + c^2) : ... : ... ).

    ( Mostrar/Ocultar figura )

    Las paralelas a r1, r2, r3 por A, B, C concurren en X1994.

    Las paralelas a r1, r2, r3 por A", B", C" concurren en el punto de Y de coordenadas baricéntricas:
    ( a^4(b^2+c^2) - 2a^2(b^4 + b^2c^2 + c^4)+ (b^2 - c^2)^2 (b^2 + c^2) : ... : ... ).
    Este punto es el complemento de X1994 y, además, el punto X obtenido antes es el punto medio del segmento YX1994.

  • miércoles, 22 de mayo del 2013

    Centro radical sobre la recta de Euler

    ( Anopolis #279 )

    Sea ABC un triángulo y A'B'C', A"B"C" los triángulo órtico y medial, respectivamente.
    Se denota por:
    (I11) la circunferencia exinscrita a A'B"C" respecto al ángulo B"A'C"
    (I22) la circunferencia exinscrita a A"B'C" respecto al ángulo C"B'A"
    (I33) la circunferencia exinscrita a A"B"C' respecto al ángulo A"C'B".

    ( Mostrar/Ocultar figura )

    Entonces, el centro radical de las circunferencia (I11), (I22), (I331) es X442, que está sobre la recta de Euler.

    Además, los triángulos ABC y I11I22I33 son perspectivos, con centro de perspectividad en X80 (simétrico del incentro respecto al punto de Feuerbach).
    Los puntos de contacto D, E y F de las circunferencia (I11), (I22) y (I33)con las rectas B"C", C"A" y A"B", resp., coinciden con las puntos de contacto de la circunferencia inscrita al triángulo medial con sus lados.

  • sábado, 11 de mayo del 2013

    Triángulos simétricos del triángulo preceviano de un punto, perspectivos con el triángulo de referencia

    (Advanced Plane Geometry #33 Paul Yiu)

    Dados un triángulo ABC y un punto P. sea A'B'C' el triángulo preceviano (anticeviano) de P, entonces el lugar geométrico de los puntos Q tales que ABC es perspectivo con el triángulo A"B"C", simétrico de A'B'C' respecto a Q, es la cónica biceviana C(G,P') del baricentro G y el tripolo P' de la recta pasando por P y por el cociente ceviano G/P, de G y P.

    ( Mostrar/Ocultar figura )

    Si P(p:q:r) y Q(x:y:z), en coordenadas baricéntricas, los vértices del triángulo anticeviano de P y los de su triángulo simétrico respecto a Q son:

    {A',B', C'} = {(-p : q : r), (p : -q : r), (p : q : -r)},
    {A", B", C"} = {(2(-p+q+r)x+p(x+y+z) : 2(-p+q+r)y-q(x+y+z) : 2(-p+q+ r)z-r(x+y+z)),   (2(p-q+r)x-p(x+y+z) : 2(p-q+r)y+q(x+y+z) : 2(p-q+r)z-r(x+y+z)),   (2(p+q-r)x-p(x+y+z) : 2(p+q-r)y-q(x+y+z) : 2(p+q-r)z+r(x+y+z))}.

    Las rectas AA", BB" y CC" son concurrentes si Q está en la cónica:

    qr(q-r)x^2 + rp(r-p)y^2 + pq(p-q)z^2 - p(p-q-r)(q-r)yz - q(q-r+p)(r-p)zx - r(r-p-q)(p-q)xy=0.

    Que se puede poner en la forma:
    \newfont{\euii}{eufm10 scaled \magstep4} 
{\hbox{\euii S}}_{ 
\begin{array}
 ppqr\\[-3pt]
 xyz
\end{array}}
\left[\frac{x^2}{\displaystyle\frac{1}{qr(q-r)}}
-\left(\frac{1}{\displaystyle\frac{1}{pq(p-q)}}+
\frac{1}{\displaystyle\frac{1}{rp(r-p)}}
\right)yz
\right]=0

    De donde se observa (Introduction to the Geometry of the Triangle §10.1.2) que es la ecuación de la cónica biceviana C(G,P') de los puntos G(1:1:1) y P'( 1/(qr(q-r)) : 1/((rp(r-p)) : 1/(pq(p-q)) ). Este último es el tripolo de recta que pasa por G y el cociente ceviano G/P( p(q+r-p) : q(r+p-q) : r(p+q-r) ); y también P' es el cuarto punto de intersección de las cónicas circunscritas a ABC de perspectores P y G/P (o equivalentemente —Introduction to the Geometry of the Triangle §9.3—, de las cónicas circunscritas de centro P y G/P).

    El lugar geométrico de los centros de perspectividad Q' de ABC y A"B"C", cuando Q varía en la cónica biceviana C(G,P'), es la cónica circunscrita que pasa por P y G/P.



  • viernes, 10 de mayo del 2013

    Triángulos inscritos en la circunferencia circunscrita a un triángulo, con circunferencias inscritas de mismo radio

    (Advanced Plane Geometry #29 Nikos Dergiades)

    Dado un triángulo ABC, se consideran los puntos A', B' y C', donde las paralelas por A, B y C a sus correspondientes lados opuestos, vuelven a cortar a la circunferencia circunscrita a ABC. Entonces, cada par de triángulos (AA'B, AA'C), (BB'C, BB'A) y (CC'A, CC'B) tienen circunferencias inscritas de mismo radio.

    ( Mostrar/Ocultar figura )

    Se tienen las siguientes coordenadas baricéntricas: A'(a^2: c^2-b^2 : b^2-c^2), B'(c^2-a^2 : b^2 : a^2-c^2) y C'(b^2-c^2 : a^2-b^2 : c^2).
    Los incentros de los triángulos AA'B y AA'C son, respectivamente: (a^2 : c(c-b)+a|b-c| : c(b-c)), y (a^2 : b(c-b) : b(b-c)+a|b-c|); éstos determinan las recta de ecuación |b-c|x-ay-az=0.
    El valor común del radio de las circunferencias inscritas a AA'B y AA'C es: bc(b^2-c^2)^2 / (2R(ab(b^2-c^2) + (b^2+c(a-c))|b^2-c^2|), con R el radio de la circunferencia circunscrita a ABC.

    Similarmente, los triángulos de las rectas que unen los incentros de los pares de triángulos restantes son: bx-|c-a|y+bz=0, cx+cy-|a-b|z=0.

    Estas tres rectas determinan un triángulo A1B1C1 homotético a ABC, con centro de homotecia:

    ( a(c+|a-b|)(b+|a-c|) : b(a+|b-c|)(c+|b-a|) : c(b+|c-a|)(a+|c-a|) ),

    y razón de homotecia: -(|a-b|(ab-|a-c||b-c|)+c(a|a-c| + b(2a+|b-c|)) / ((c+|a-b|)(b+|a-c|)(a+|b-c|)).

  • jueves, 09 de mayo del 2013

    Circunferencia lugar del incentro de un triángulo variable

    (Anopolis #229)

    Sean ABC un triángulo acutángulo y δ una recta que pasa por el ortocentro H. Denotemos por δa, δb y δc las reflexiones de δ en AH, BH, CH, resp.; δ'a, δ'b y δ'c las paralelas a δa, δb y δc a través de A, B, C, resp., y I' el incentro del triángulo A'B'C' acotado por las rectas δ'a, δ'b y δ'c.

    El lugar geométrico de I' cuando δ gira alrededor de H es la circunferencia de centro H y que pasa por X265, su radio es el de la circunferencia circunscrita a ABC.

    ( Mostrar/Ocultar figura )

    Ocurre además que los vértices de A'B'C' describen las circunferencias (B,C, X265), (C, A, X265) y (A, B, X265).



  • miércoles, 08 de mayo del 2013

    Recta de Euler e hipérbola de Jerabek

    (Anopolis #227)

    Sean ABC un triángulo, H su ortocentro y (u:v:w) las coordenadas baricéntricas de un punto P. Se denota por Ha, Hb, Hc los ortocentros de los triángulos PBC, PCA, PAB, resp.
    Las reflexiones de las rectas HHa en los lados de ABC concurren en P1:

    ( a²vw(SBv+SC(u+v))(SCw+SB(u+w)) : -b²uw(SBv+SC(u+v))(SBv-SCw) : c²uv(SBv-SCw)(SCw+SB(u+w)) ),
    situado sobre la circunferencia circunscrita a ABC.

    Las reflexiones de las rectas HHa en los lados de PBC en P'1:

    ( a²SBSCuvw(u+v+w) : SCw(SC²(u+v)² w+SB²v²(u+w)+SAu²(-SBv+SCw)-SBSCv( u²+uv-2vw)) : SBv(SC²(u+v) w²+SB²v(u+w)²+SAu²(SBv-SCw)- SBSCw(u²+uw-2vw)) ),
    situado sobre la circunferencia circunscrita a PBC. Donde se ha utilizado la notación de Conway.

    Siendo la ecuación de la recta P1P'1:
    u(a²SA+2SBSC)(SBv-SCw)x + a²vSB(SCw+SB(u+w))y - a²wSC(SBv+SC(u+v))z=0.

    Procediendo de forma similar, obtenemos:
    Las reflexiones de la recta HHb en los lados de ABC concurren en P2 y en los lados de PCA en P'2.
    Las reflexiones de la recta HHc en los lados de ABC concurren en P3 y en los lados de PAB en P'3.

    El lugar geométrico de los puntos P tales que los triángulos P1P2P3 y P'1P'2P'3 son perspectivos (junto con la recta del infinito) es la hipérbola de Jerabek (conjugada isogonal de la recta de Euler):

    a²(c²-b²)(b²+c²-a²)yz+ b²(a²-c²)(c²+a²-b²)zx + c²(b²-a²)(a²+b²-c²)xy = 0.

    • Cuando P se mueve en la hipérbola de Jerabek las rectas P1P'1, P2P'2 y P3P'3 concurren en Q, sobre la recta de Euler.

    ( Mostrar/Ocultar figura )

    • También, cuando P se mueve en la hipérbola de Jerabek las rectas paralelas a P1P'1, P2P'2 y P3P'3 a través de los vértices del triángulo de contacto interior, son concurrentes en el punto medio M de H y Q.



  • Un centro del triángulo de E. M. H. Lemoine

    (Advanced Plane Geometry #2 Paul Yiu)

    Dado un punto P en el plano del triángulo ABC, se traza la paralela por P al lado BC que interseca a AB en Ca y a AC en Ba. Similarmente, se construyen Ab, Cb sobre BC, BA, y Bc, Ac sobre CA, CB, respectivamente.

    (1) El punto P para el que BcCb, CaAc, y AbBa son todas tangentes a la circunferencia inscrita es el anticomplemento del punto de Nagel, X145 en ETC
    (E. M. H. Lemoine.- Étude sur de nouveaux points remarquables du plan d’un triangle, Journal de Math. Spéciales, ser. 2, 2 (1883) 3--6).

    Si P(x:y:z), coordenadas baricéntricas, entonces Ca(x : y+z : 0), Ba(x : 0 : y-z), Ab(0 : y : x+z), Cb(x+z : y : 0), Bc(x+y : 0 : z) y Ac(0 : x+y : z).

    Para que la recta BcCb sea tangente al circunferencia inscrita se ha de verificar que debe contener a su polo respecto a ella; por lo que el punto P ha de estar en las cónicas (la misma situación para las otras dos rectas CaAc, y AbBa):

    (a-b-c)x² + (a-b+c)y² + (a+b-c)z² + (a-b-c)yz + 2(a-c)zx + 2(a-b)xy=0,
    (b-c-a)y² + (b-c+a)z² + (b+c-a)x² + (b-c-a)zx + 2(b-a)xy + 2(b-c)yz=0,
    (c-a-b)z² + (c-a+b)x² + (c+a-b)y² + (c-a-b)xy + 2(c-b)yz + 2(c-a)zx=0.

    El punto común a estas tres cónicas es X145, simétrico del punto de Nagel respecto al incentro.
    ( Mostrar/Ocultar figura )

    (2) Los puntos de tangencia determinan un triángulo XYZ perspectivo con ABC, con centro de perspectividad el punto de coordenadas baricéntricas:

    ( (3a-b-c)² / (b+c-a) : (3b-c-a)² / (c+a-b) : (3c-a-b)² / (a+b-c) ).

    (con (6-9-13)-número de búsqueda en ETC: 0.02670313601037967069540658222).

    Este punto es conjugado isogonal del punto de concurrencia descrito en el problema O126:
    "O126. Let ABC be a triangle and let Ka be the A-mixtilinear incircle (the circle tangent to sides (AB),(AC) and internally tangent to the circumcircle Γ of triangle ABC). Denote by A' the tangency point of Ka with Γ and let A" be the diametrically opposed point of A' with respect to Ka . Similarly, define B" and C" . Prove that lines (AA") ,(BB") and (CC") are concurrent".
    Proposed by Cosmin Pohoata, NationalCollege ”Tudor Vianu”, Romania
    ( Mostrar/Ocultar figura )


    César Lozada da las coordenadas del punto de intersección de las rectas (AA"), (BB") y (CC") en ADGEOM #1098.

    (3) El triángulo XYZ es perspectivo con el triángulo de contacto interior DEF y el centro de perspectividad también es el X145.

    ( Mostrar/Ocultar figura )

    (4) Los seis puntos Ab, Ac, Bc, Ba, Ca y Cb están en una misma cónica:

    SumaCíclica[ 2(a+b-3c)(a-b-c)(a-3b+c) x^2 + (3a-b-c)(5a^2-2a(b+c)-7b^2+18bc-7c^2) yz]=0.
    De centro:
    Q = ( 39a3 - 43a2(b+c) + a(37b2-22bc+37c2) - 9b3+5b2c+5c2b-9c3 : ... : ... ),
    (con (6-9-13)-número de búsqueda en ETC: 0.512214699835464592430071714) y perspector (centro de perspectividad del triángulo ABC y el triángulo formado por las polares de sus vértices respecto a la cónica):


  • martes, 07 de mayo del 2013

    Reflexión de centros de circunferencias de los nueve puntos

    (Anopolis #224)

    Sean ABC un triángulo y P un punto, se denota por P11, P22, P33 los centros de las circunferencias de los nueve puntos de los triángulos PBC, PCA, PAB, resp.
    Consideremos los puntos:
    P12, P13 las reflexiones de P11 in PB, PC, resp.
    P23, P21 las reflexiones de P22 in PC, PA, resp.
    P31, P32 las reflexiones de P33 in PA, PB, resp.


    Los triángulos P11P12P13, P21P22P23, P31P32P33 son concéntricos, con circuncentro P.

    Sean Na, Nb, Nc los centros de las circunferencias de los nueve puntos de P11P12P13, P21P22P23, P31P32P33 , resp.

    El lugar geométrico de los puntos P tales que los triángulos P11P22P33 y NaNbNc son perspectivos (junto con la recta del infinito y la circunferencia circunscrita) es la cúbica de McCay, K003 en el catálogo de Bernard Gibert.

    Si P(u:v:w) son las coordenadas baricéntricas de un punto sobre la cúbica de McCay o en la circunferencia circunscrita, el centro de perspectividad Q de los triángulos P11P22P33 y NaNbNc es:

    a^6(b^2w(3v*w(v-w) + u^2(v+w) + u(v^2+2v*w-2w^2)) + c^2v(3v*w(w-v) + u^2(v+w) + u(w^2+2v*w-2v^2))) +
    a^4(c^4v(u^3 + 2v*w(v-2w) - u^2(v+w) + u(v^2+v*w-2w^2)) + b^4w(u^3 + 2v*w(w-2v) - u^2(v+w) + u(w^2+v*w-2v^2)) + b^2c^2(u^3(v+w)+2u(v-w)^2(v+w)-6u^2(v^2+w^2)-2v*w(v^2-4v*w+w^2))) -
    a^2(-c^6v(-2u^3+ u^2(v-w) + u*w(w-4v) + v*w(v+w)) - b^6w(-2u^3+ u^2(w-v) + u*v(v-4w) + v*w(v+w)) + b^4c^2(2u^3(2v+w) - u^2(5v^2+v*w-6w^2) + v*w(-v^2+v*w+2w^2) + u*w(10v^2-v*w+4w^2)) + b^2*c^4(2u^3(v+2w) + u^2(6v^2-v*w-5w^2) + v*w(2v^2+ v*w-w^2) + u*v(4v^2-v*w+10w^2))) +
    (b^2-c^2)^2u(c^4v(u^2 + u(w-v) + v(v+w)) + b^4w(u^2+u(v-w)+w(v+w)) + b^2c^2(-4u*v*w+3u^2(v+w)+2v*w(v+w)) ) : ... : ...

    Nota: Las coordenadas simétricas de Q han sido obtenidas usando el método descrito por Barry Wolk en el mensaje #19239 of Hyacinthos.

    Si P está sobre la circunferencia circunscrita a ABC, el centro de perspectividad Q de los triángulos P11P22P33 y NaNbNc es el punto del infinito de la recta OP (O circuncentro de ABC).
    ( Mostrar/Ocultar figura )


    Para puntos de la cúbica de McCay: ( P=I=X(1), Q=X(10) ); ( P=O=X(3), Q=X(3) ); ( P=H=X(4), Q=X(5) ).
    ( Mostrar/Ocultar figura )


  • lunes, 06 de mayo del 2013

    Dos nuevos centros del triángulo en la recta IO

    (Anopolis #220 Antreas P. Hatzipolakis)

    Consideremos un triángulo ABC de incentro I y denotamos por A'B'C' el triángulo ceviano de I.

    •  Sea W- el centro radical de las circunferencias A'(A'B), C'(C'A) y B'(B'C), y W+ el centro radical de las circunferencias A'(A'C), B'(B'A) y C'(C'B).
    Los puntos W+W- forman un par bicéntrico, por lo que el punto medio de W+W- es un centro del triángulo (sobre la recta IO), que tiene (6-9-13)-número de búsqueda en ETC: -4.58771580677070881713647893, y coordenadas baricéntricas:

    ( a(a4(b-c)2 + a3(b3+bc(b+c)+c3) - a2(b4-3bc(b2+c2)+c4) - a(b5+bc(b3+c3)+c5) - bc(b2-c2)2) : ... : ....)

    NOTA: Este punto ha sido incluido en la "Encyclopedia of Triangle Centers- ETC" con el número X5482="1st Hatzipolakis-Montesdeoca point".

    ( Mostrar/Ocultar figura )

    El punto del infinito de la recta W+W-, que es un centro, tiene (6-9-13)-número de búsqueda en ETC: 0.973211383408006530639583531 y coordenadas baricéntricas:
    ( a(b - c)(a4(b+c) - a3(b-c)2 - a2(b3 - 4bc(b+c) + c3) + a(b4+2bc(b2+c2)+c4) - bc(b+c)3) : ... : ...)


    Generalización:
    Si sustituimos el incentro por otro punto P(u:v:w), el punto medio de W+W- es:
    ( a4(u(v-w)2+2vw(v+w)) - a2(c2(u2(v-w)+2v2w+uw(w-3v))+b2(uv(v-3w)+2vw2+u2(w-v)))- (b2-c2)u(v+w)(b2(u+w)-c2(u+v)): ... : ... )

    Si P=H, el ortocentro, el punto medio de W+W- es X1147 (sobre la recta que une el ortocentro con el foco de la parábola de Kiepert):
    ( a4(a2-b2-c2)(a4-2a2(b2+c2)+b4+c4) : ... : ...)
    Y el punto del infinito de la recta W+W- es:
    ( a2(b2-c2)(a6 - a4(b2+c2) - a2(b4-4b2c2+c4) + b6-3b2c2(b2+c^2)+c6) : ... : ...)


    •  Otro caso de punto medio de centros radicales:   Sea W+ el centro radical de las circunferencias A'(A'B'), B'(B'C') y C'(C'A'), y W- el centro radical de las circunferencias A'(A'C'), B'(B'A') y C'(C'B').
    Los puntos W+W- forman un par bicéntrico y el punto medio de W+W-, situado sobre la recta IO, (con (6-9-13)-número de búsqueda en ETC: 2.182865285269536019160224004) es:

    ( a(3a5(b+c) + a4(b2+4bc+c2) - a3(b+c)(6b2-bc+6c2) -2a2(b4+3b3c+3b2c2+3bc3+c4)+ a(b-c)2(b+c)(3b2+5bc+3c2)+ (b-c)2(b+c)4) : ... : ...)
    ( Mostrar/Ocultar figura )


    Generalización:
    Si sustituimos el incentro por otro punto P(u:v:w), el punto medio de W+W- es:
    ( a^4vw(2v^2w^2+3u^3(v+w)+uvw(v+w)+ u^2(v^2+4vw+w^2)) - a^2u(b^2w(v^2(v-w)w+u^2(6v^2+5vw+w^2) + 2uv(v^2+3vw+2w^2)) + c^2v(vw^2(w-v) + 2uw(2v^2+3vw+w^2) + u^2(v^2+5vw+6w^2))) + (b^2-c^2)u^2(v+w)(b^2w(u(3v-w)+v(v+w))+c^2v(u(v-3w)-w(v+w))): ... : ... )

    Si P=G, el baricentro, el punto medio de W+W- es X140. Y el punto del infinito de la recta W+W- es X1499, punto de Biham.

    Si P=X7, el punto de Gergonne, el punto medio de W+W- es X942, inverso en la circunferencia inscrita del inverso del incentro en la circunferencia circunscrita. Y el punto del infinito de la recta W+W- es:
    ( a(b-c)(a^2- 2a(b+c) + b^2+4bc+c^2) : b(c-a)(b^2 - 2b(c+a) + c^2+4ca+a^2) : c(a-b)(c^2 - 2c(a+b) + a^2+4ab+b^2) ),

    con (6-9-13)-número de búsqueda en ETC: 5.739583333333333333.

  • viernes, 03 de mayo del 2013

    Triángulos homotéticos

    (Anopolis #212)

    • Consideremos un triángulo ABC de circuncentro O y denotamos por A'B'C' el triángulo ceviano de O. Sea da el eje radical de la circunferencia O(OA'), de centro O y radio OA', y de la circunferencia circunscrita a OBC; de forma similar se consideran los ejes radicales db y dc.
    El triángulo ABC y el acotado por las rectas da, db y dc son homotéticos

    El centro de homotecia tiene coordenadas baricéntricas:

    Es el cuadrado baricéntrico de X2167 y también el conjugado isogonal del cuadrado baricéntrico del centro de la circunferencia de los nueve puntos, X5.
    ( Mostrar/Ocultar figura )

    • Consideremos un triángulo ABC de circuncentro O y denotamos por A'B'C' su triángulo medial. Sea A"B"C" el triángulo circunceviano de O respecto a A'B'C', D=BC∩AA", E=CA∩BB" y F=AB∩CC". Denotamos por da el eje radical de la circunferencia A"(A"D), de centro A" y radio A"D, y de la circunferencia circunscrita a A"BC; de forma similar se consideran los ejes radicales db y dc.
    El triángulo ABC y el acotado por las rectas da, db y dc son homotéticos

    El centro de homotecia tiene coordenadas baricéntricas:

    (con (6-9-13)-número de búsqueda en ETC: 1.32064524330236458037845738)
    ( Mostrar/Ocultar figura )


  • miércoles, 1 de mayo del 2013

    Construcción de triángulo

    Problema 683. ( Laboratorio Virtual del Triángulo con Cabri)

    Construir un triángulo rectángulo con el baricentro en la circunferencia inscrita.
    Barroso, R. (2013): Comunicación personal.

    Mi aportación en HTML, o en PDF.

  • martes, 30 de abril del 2013

    Triángulos perspectivos

    Consideremos un triángulo ABC de circuncentro O y denotamos por DEF su triángulo medial. La circunferencia de centro en el punto medio Da de OD y que pasa por B y C, vuelve a cortar a las circunferencias de diámetros AC y AB en los puntos Ab y Ac respectivamente. Procediendo cíclicamente se definen los pares de puntos Bc y Ba, Ca y Cb.
    Las rectas AbAc, BcBa, CaCb determinan un triángulo A'B'C' perspectivo con ABC, cuyo centro de perspectividad es el centro del triángulo de (6,9,13)-número de búsqueda 1.87843618353880172287205618 en ETC y coordenadas baricéntricas:
    
\left(\frac{1}{S_A(10S_B^2S_C^2 + 17a^2S_AS_BS_C + 2S_A^2(3S_B^2 +  7S_BS_C + 3S_C^2))}, 
      \frac{1}{S_B(10S_A^2S_C^2 + 17b^2S_AS_BS_C + 2S_B^2(3S_A^2 +  7S_AS_C + 3S_C^2))}, 
      \frac{1}{S_C(10S_A^2S_B^2 + 17c^2S_AS_BS_C + 2S_C^2(3S_A^2 +  7S_AS_B + 3S_B^2))}\right)
    ( Mostrar/Ocultar figura )


  • lunes, 29 de abril del 2013

    Triángulos pedales y circunferencias centradas en los vértices de un triángulo

    El mensaje #159 del grupo Anopolis de Antreas Hatzipolakis sugiere el siguiente planteamiento, considerando pares de circunferencias con centros en los vértices de un lado de un triángulo:

    Consideremos un triángulo ABC y un punto U. Denotamos por UaUbUc su triángulo pedal, por O(R) su circunferencia circunscrita, por B(BUc) y C(CUb) las circunferencias centradas en B y C, de radios BUc y CUb respectivamente. Tomemos un punto P∈O(R), la recta PB corta a B(BUc) en B', B", la recta PC corta a C(PUb) en C', C". Entonces, las circunferencias (PB'C') y (PB"C") pasan por un punto fijo A1 sobre O(R), cuando el punto P varía. Así mismo, al variar P sobre O(R), las circunferencias (PB'C") y (PB"C') pasan por un punto A2, fijo sobre O(R). Similarmente se definen los puntos B1 y B2, C1 y C2, procediendo cíclicamente sobre los vértices de ABC.
    Las rectas A1A2, B1B2 y C1C2 son concurrentes si y sólo si el punto U está en O(R) o en la cúbica de Darboux (K004 en el catálogo de Bernard Gibert).
    ( Mostrar/Ocultar figura )

    •   Cuando U está en la circunferencia circunscrita, el punto de intersección de las rectas A1A2, B1B2 y C1C2 es el propio punto U.

    •   Cuando U se mueve en la cúbica de Darboux el lugar geométrico (?) de los puntos U' de intersección de las rectas A1A2, B1B2 y C1C2, contiene a los puntos A, B, C, antipodales de A, B, C en la circunferencia circunscrita, X3, X6, X22, X24, X1498, X1604, X1617, ...

    Puntos U' como centros de perspectividad de los triángulos tangencial y circunceviano de un punto V:
    (Clark Kimberling.- Triangle Centers and Central Triangles, (TCCT) §7.18, p. 201)

    U, en la cúbica de Darboux U', punto de intersección de las rectas A1A2, B1B2 y C1C2 V, tal que U' es el centro perspectividad de los triángulos tangencial y circunceviano de V
    X1 X1617 X57
    X4 X24 X4
    X20 X22 X2
    X40 X3 X1
    X64 ?
    X84 ?
    X1490 X1604 X9
    X1498 X1498 X83


  • viernes, 26 de abril del 2013

    Tríada de circunferencias centradas en los vértices de un triángulo

    El mensaje #159 del grupo Anopolis de Antreas Hatzipolakis sugiere el siguiente planteamiento:

    Consideremos un triángulo ABC y denotamos por O(R) su circunferencia circunscrita, por A(ρa), B(ρb) y C(ρc) circunferencias centradas en el vértice correspondiente y radios ρa, ρb y ρc, respectivamente. Tomemos un punto P∈O(R), la recta PB corta a B(ρb) en B', B", la recta PC corta a C(ρc) en C', C". Entonces, las circunferencias (PB'C') y (PB"C") pasan por un punto fijo A1 sobre O(R), cuando el punto P varía. Así mismo, al variar P sobre O(R), las circunferencias (PB'C") y (PB"C') pasan por un punto A2, fijo sobre O(R). Similarmente se definen los puntos B1 y B2, C1 y C2, verificándose que las rectas A1A2, B1B2 y C1C2 son siempre concurrentes.
    ( Mostrar/Ocultar figura )
      Tercer punto de Brocard


    Casos particulares

    •   Para las circunferencias centradas en los vértices del triángulo ABC y mutuamente tangentes, el punto común de las rectas A1A2, B1B2 y C1C2 es el X1617, centro de perspectividad de los triángulos tangencial y circunceviano de X57.

    •   Para las circunferencias A(a), B(b) y C(c), centradas en los vértices del triángulo ABC y de radios la longitud del lado opuesto, el punto común de las rectas A1A2, B1B2 y C1C2 es el X22, centro de perspectividad de los triángulos tangencial y circunceviano de X2.

    •   Para las circunferencias A(ma), B(mb) y C(mc), centradas en los vértices del triángulo ABC y de radios la longitud de la mediana correspondiente, el punto común de las rectas A1A2, B1B2 y C1C2 es el X1995 (como en el caso de las circunferencias que pasan por el baricentro).

    •   Para las circunferencias A(ha), B(hb) y C(hc), centradas en los vértices del triángulo ABC y tangentes al lado opuesto, el punto común de las rectas A1A2, B1B2 y C1C2 es el simediano, X6.

    •   Para las circunferencias A(AOa), B(BOb) y C(COc), centradas en los vértices del triángulo ABC y que pasan por los pies de las cevianas del circuncentro, el punto común de las rectas A1A2, B1B2 y C1C2 tiene primera coordenada baricéntrica:

    a^2 (a^18 - 6a^16(b^2+c^2) + 3a^14(5b^4+8b^2c^2+5c^4) - a^12(21b^6+37b^4c^2+37b^2c^4+21c^6) + a^10(21b^8+28b^6c^2+31b^4c^4+28b^2c^6+21c^8) - 3a^8(7b^10+2b^8c^2+3b^6c^4+3b^4c^6+2b^2c^8+7c^10) + a^6(b^2-c^2)^2(21b^8+22b^6c^2+24b^4c^4+22b^2c^6+21c^8) - a^4(b^2-c^2)^2(15b^10-5b^8c^2-6b^6c^4-6b^4c^6-5b^2c^8+15c^10) + 3a^2(b^2-c^2)^4(2b^8-b^4c^4+2c^8) - (b^2-c^2)^6(b^6+c^6) ) .

    •   Para las circunferencias A(AI), B(BI) y C(CI), centradas en los vértices del triángulo ABC y que pasan todas por el incentro (I=X1), el punto común de las rectas A1A2, B1B2 y C1C2 es el X56, centro de perspectividad de los triángulos tangencial y circunceviano de X266.

    •   Para las circunferencias A(AG), B(BG) y C(CG), centradas en los vértices del triángulo ABC y que pasan todas por el baricentro (G=X2), el punto común de las rectas A1A2, B1B2 y C1C2 es el X1995 (como en el caso de radio las medianas), centro de perspectividad de los triángulos tangencial y circunceviano de la raíz baricéntrica de X1383.

    •   Para las circunferencias A(AO), B(BO) y C(CO), centradas en los vértices del triángulo ABC y que pasan todas por el circuncentro (O=X3), el punto común de las rectas A1A2, B1B2 y C1C2 (mediatrices de ABC) es el X3, centro de perspectividad de los triángulos tangencial y circunceviano de X1.

    •   Para las circunferencias A(AH), B(BH) y C(CH), centradas en los vértices del triángulo ABC y que pasan todas por el ortocentro (H=X4), el punto común de las rectas A1A2, B1B2 y C1C2 es el X24, centro de perspectividad de los triángulos tangencial y circunceviano de X4.

    •   Para las circunferencias A(AK), B(BK) y C(CK), centradas en los vértices del triángulo ABC y que pasan todas por el simediano (K=X6), el punto común de las rectas A1A2, B1B2 y C1C2 es el X1384, centro de perspectividad de los triángulos tangencial y circunceviano del punto con coordenadas baricéntricas:

    ( a²/(a²-2(b²+c²))½ : b²/(b²-2(c²+a²))½ : c²/(c²-2(a²+b²))½ ) .


  • martes, 23 de abril del 2013

    Anopolis #147

    En el grupo Anopolis (mensaje #147) de Antreas Hatzipolakis se propone lo siguiente:

    Sea ABC un triángulo, P un punto, se designa por Ha, Hb, Hc los ortocentros de los triángulos PBC, PCA, PAB, resp. y por O1, O2, O3 los circuncentros de los triángulos HaBC, HbCA, HbAB, resp.
    ¿Cuál es el lugar geométrico de P para que los triángulos HaHbHc y O1O2O3 sean perspectivos?

    Mi respuesta (mensaje #150):

    El lugar geométrico de P tal que las rectas HaO1, HbO2 y HcO3 sean concurrentes consta de la circunferencia circunscrita a ABC y de la cúbica de McCay (K003 en el catálogo de Bernard Gibert).

    •  Si P está sobre la circunferencia circunscrita, las rectas HaO1, HbO2 y HcO3 son paralelas y su punto del infinito es SR(P,Q), donde Q es punto diametralmente opuesto a P.
    Una descripción del punto Simson-Rigby SR(R,U), con P y U puntos distintos sobre la circunferencia circunscrita al triángulo ABC, figura justo antes del punto X2677 en ETC.
    Si en coordenadas baricéntricas P = (p : q : r) y U = (u : v : w), entonces:
    SR(P,U)= ( (rv - qw)/(b²rw-c²qv) : (pw - ru)/(c²pu-a²rw) : (qu - pv)/(a²qv-b²pu) ).

    Si P se mueve en la circunferencia circunscrita, los puntos O1, O2, O3 son los simétrico de A, B, C, respecto al centro (X5) de la circunferencia NPC de los nueve puntos; y los triángulos HaHbHc son simétrico de ABC respecto al punto medio (sobre NPC) de P y el ortocentro.


    •  Si P queda sobre la cúbica de McCay, el punto de intersección de las rectas HaO1, HbO2 y HcO3 está sobre la tercera cúbica de Musselman (K028 en el catálogo de Bernard Gibert).


  • lunes, 22 de abril del 2013

    Puntos en cónicas inscritas

    Dado un triángulo ABC y un punto P, vamos a asociar a cada recta d que pasa por P un punto sobre la cónica inscrita en ABC de perspector P (conjugado isotómico de P). Además, daremos una propiedad geométrica relativa a dicho punto.
    ( Mostrar/Ocultar figura )
      Tercer punto de Brocard

    Si (u:v:w) son las coordenadas baricéntricas de P, respecto a ABC, la ecuación de una recta d variable por P la podemos expresar, en función de dos parámetros m y n, como sigue:
    (v - w) (m + v w n)x+ (w-u ) (m + wu n)y+ (u - v) (m + u v n)z=0,
    La cónica circunscrita d, conjugada isotómica de d, tiene por ecuación:
    (m(v - w) + n vw(v - w)) yz + (m(w - u) + n wu(w - u)) zx + (m(u - v) + n uv(u - v)) xy =0.
    Su perspector está en la tripolar de P (recta G-Dual de P).

    Sean Q el cuarto punto de intersección de d y la cónica circunscrita C(P) de perspector P, C(P,Q) la cónica que pasa por los pies Pa, Pb y Pc de las cevianas de P y Qa, Qb y Qc de Q (cónica biceviana de P y Q) y ℑ(P) la cónica inscrita de perspector P.

    El cuarto punto t(P,d) de intersección de ℑ(P) y C(P,Q) es el tripolo de la tangente a d en P (recta que une los perspectores de ℑ(P) y C(P,Q) respecto al triángulo ceviano de P). Las coordenadas de t(P,d) son:

    t(P,d) = ( u(v - w)²(m + n vw)² : v(w - u)²(m + n wu)² : w(u - v)²(n + n uv)² ).

    También: t(P,d) es el punto de tangencia con ℑ(P) de la tripolar del perspector de la cónica d (conjugada isotómica de la recta d).

    En particular, si P=(u:v:w), a la recta d=PP le corresponde el punto:
    ( u3(v2-w2)2 : v3(u2-w2)2 : w3(u2-v2)2 ).
    Se trata del punto de tangencia de la tripolar del punto de intersección de las tripolares de P y P con la cónica inscrita de perspector P .

    Una propiedad del punto t(P,d):
    Denotamos por Ca la cónica tangente en Pa a BC y que pasa por Qb, Qc y Q. Las cónicas Cb y Cc se definen de forma similar.
    Las tres cónicas Ca, Cb y Cc son tangentes en t(P,d) a la tripolar del perspector de la cónica d.


    Algunos casos particulares de obtención de puntos t(P,d):

    • P=X7 (punto de Gergonne), ℑ(X8) es la elipse inscrita de Mandart, de perspector el punto de Nagel.

      En el mensaje #14278 de Hyacinthos, Steve Sigur se extrañaba que en esta elipse hubiera sólo un punto de entre los 3000 primeros de ETC. Continuó comentando (mensaje #14291) que tal elipse podría no ser interesante ya que incluso Bernard Gibert en su excelente artículo en Forum Geometricurum, Volume 4 (2004) 177–198.(Generalized Mandart Conics) habla poco de ella. Por otra parte, Floor Van Lamoen menciona la siguiente propiedad (mensaje #14263): "La elipse de Mandart es cofocal con la elipse circunscrita de perspector el incentro. Todos los triángulos porísticos relativos a estas dos cónicas tienen el mismo perímetro, que es el máximo que un triángulo inscrito a la citada elipse circunscrita puede tener."

      Algunos puntos t(X7,d) en la elipse inscrita de Mandart, correspondientes a rectas por X7:
      • A la recta X1X7 (m=0) le corresponde el punto X4081.
      • A la recta X2X7 (n=0) le corresponde el punto X11, punto de Feuerbach.
      • A la recta X7X8 ( m=a+b+c, n=-(b+c-a)(a² - b² - c² + 2bc) ) le corresponde el punto X3271.
      • A la recta X7X21 ( m=a+b+c, n=(b+c-a)(a² - b² - c² + 2bc) ) le corresponde el punto X4092.
      • A la recta d que pasa por X7 de parámetros (m=área(ABC)=S/2, n=1) le corresponde el punto de coordenadas baricéntricas:

        ( (b-c)²(b+c-a)(15a-17(b+c))² : (c-a)²(c+a-b)(15b-17(c+a))² : (a-b)²(a+b-c)(15c-17(a+b))² ).



    • P=X8 (punto de Nagel), ℑ(X7) es la circunferencia inscrita (de perspector el punto de Gergonne).

      Algunos puntos t(X8,d) en la circunferencia inscrita, correspondientes a rectas por X8:
      • A la recta X1X8 (n=0) le corresponde el punto X11, punto de Feuerbach.
      • A la recta X3X8 ( m = a3 + b3 + c3 - a2b - ab2 - a2c - b2c - ac2 - bc2 + 6abc, n = -(a+b+c) ) le corresponde el punto X3326.
      • A la recta X4X8 ( m = 2(a3 - ab2 - ac2) - (a+b+c)(a2 - b2- c2 + 2bc ), n = a+b+c ) le corresponde el punto X1364.
      • A la recta d5=X5X8 le corresponde el punto:
        t(X8,d5) = ( (b+c-a)(b-c)2(2a3-a2(b+c)-2a(b2-bc+c2)+(b-c)2(b+c))2 : ... : ... ).

      • A la recta d6=X6X8 (m = (a+b+c)2, n=-1) le corresponde el punto de coordenadas baricéntricas:

        t(X8,d6) = ( (b+c-a)(b-c)²(b²+c²+a(b+c))² : (c+a-b)(c-a)²(c²+a²+b(c+a))² : (a+b-c)(a-b)²(a²+b²+c(a+b))² ).

      • A la recta X7X8 (m = a² + b² + c² - 2ab - 2ac - 2bc, n=1) le corresponde el punto X3022.
      • A la recta X8 X37 (m=(a+b+c)², n=0) le corresponde el punto X3026.
      • A la recta X8 X522 ( m = a5 - 2a4(b + c) + a3(b2 + 3bc + c2) - a(b - c)2(2b2 + bc + 2c2) + (b - c)2 (b3 + c3) + a2(b3 - 2b2c - 2bc2 + c3), n = a3 - a2(b + c) + (b - c)2(b + c) - a(b2 - 3bc + c2) ) le corresponde el punto X1317.
      • A la recta d que pasa por X8 de parámetros (m=r², n=1) le corresponde el punto de coordenadas baricéntricas:

        ( (b-c)((b-c)²-a²)(3a+5(b+c)) : (c-a)((c-a)²-b²)(3b+5(c+a)) : (a-b)((a-b)²-c²)(3c+5(a+b)) ).

      • A la recta d que pasa por X8 de parámetros (m=área(ABC)=S/2, n=1) le corresponde el punto de coordenadas baricéntricas:

        ( (b-c)²(34bc-30SA+8S) : (c-a)²(34ca-30SB+8S) : (a-b)²(34ab-30SC+8S) ).



    • P=X69, ℑ(X4) es la "orthic inconic" de perspector el ortocentro.

      Algunos puntos t(X69,d) en la "orthic inconic", correspondientes a rectas por X69:
      • A la recta d1 =X1X69 ( m = a4+2a3(b+c)+2a2(b2+3bc+c2)+2a(b+c)3+(b+c)2(b2+c2) , n = 4 ) le corresponde el punto de coordenadas baricéntricas:

        t(X69,d1) = ( (b-c)²(b²+c²-a²)(a²+(b+c)²)² : (c-a)²(c²+a²-b²)(b²+(c+a)²)² : (a-b)²(a²+b²-c²)(c²+(a+b)²)² ).

      • A la recta X2X69 (n=0) le corresponde el punto X125, centro de la hipérbola de Jerabek.
      • A la recta d3=X3X69 (m = 0) le corresponde el punto de coordenadas baricéntricas:

        t(X69,d3) = ( (b²-c²)² ⁄ (b²+c²-a²) : (c²-a²)² ⁄ (c²+a²-b²) : (a²-b²)² ⁄ (a²+b²-c²) ).

      • A la recta d4=X4X69 (m = S², n=-1) le corresponde el punto de coordenadas baricéntricas:

        t(X69,d4) = ( a4(b2-c2)2(b2+c2-a2)3 : b4(c2-a2)2(c2+a2-b2)3 : c4(a2-b2)2(a2+b2-a2)3 ).

      • A la recta d5=X5X69 (m = 2S², n=-1) le corresponde el punto de coordenadas baricéntricas:

        t(X69,d5) = ( SA(SB-SC)² (2S² -SBSC)² : SB(SC-SA)² (2S² -SCSA)² : SC(SA-SB)² (2S² -SASB)² ).

      • A la recta X7X69 (m = a^4-2abc(b+c)+(b²-c²)²-2a²(b²+bc+c²), n=4) le corresponde el punto X3270.
      • A la recta X63X69 (m = (b²+c²-a²)(a²+c²-b^2)(a²+b²-c²), n = -4(a+b+c)² ) le corresponde el punto X2969.


  • domingo, 21 de abril del 2013

    Cónicas conjugadas isotómicas de rectas que pasan por un punto

    Dados un triángulo ABC, un punto P y una recta d variable que gira alrededor de P, sea d la cónica circunscrita conjugada isotómica de d.
    Las rectas polares p de P respecto a las cónicas d, cuando d varía, pasan por un punto fijo Q.

    Los puntos de intersección de las restas d y p describen la hipérbola que pasa por G (baricentro), P y Q y las tangente en estos últimos se cortan en el conjugado isotómico P de P.

    Si P=(u:v:w), coordenadas baricéntricas respecto a ABC, las coordenadas de Q son:
    ( u(v²+w²-u²) : v(w²+u²-v²) : w(u²+v²-w²) ).

    Algunos casos de centros del triángulo:
    P X1 X3 X6 X9 X10 X30 X37
    Q X63 X1993 X22 X3870 X3995 X2 X4651

    De hecho, las polares de un punto del infinito, respecto a las cónica conjugadas isotómicas de rectas que pasan por él (paralelas), pasan por el baricentro; como es el caso de rectas paralelas a la recta de Euler, de punto del infinito X30.

  • jueves, 18 de abril del 2013

    In Memoriam (Juan Bosco Romero Márquez)

    El profesor Francisco Javier García Capitán le dedica el Problema 3721 de Crux Mathematicorum.
    tangente
    Addenda:
    Supongamos que D es un punto del plano del triángulo ABC y A' es el pie de la ceviana AD. Renombramos entonces los puntos P y Q del enunciado a Pa y Qa, respectivamente. Procediendo cíclicamente, definimos de forma análoga los puntos Pb y Pc, Qb y Qc.
    Entonces, las recta APa, BPb y CPc son concurrentes si y sólo si el punto D queda en la cúbica K003 (McCAY CUBIC, pK(X6, X3)) ó sobre la cúbica K006 ( ORTHOCUBIC, pK(X6, X4)).

    Las rectas PaQa, PbQb y PcQc son concurrentes si D está en la séxtica de ecuación baricéntrica:
    Σ [y z (b2 c2 x4 + 3 b2 c2 x3 y - a2 x2 y (-3 c2 y + (a2 - 2 b2) z))+ a4 y2 z2]=0.


  • martes, 16 de abril del 2013

    Puntos asociados al tercer punto de Brocard

    Sean ABC un triángulo, P un punto, P su conjugado isotómico y WaWbWc el triángulo ceviano del tercer punto de Brocard X76 (conjugado isotómico del simediano).
    Consideremos las cónicas circunscritas C(X76) de perspector X76 y la que pasa por X76 y P; denotemos por Q su cuarto punto de intersección y por QaQbQc su triángulo ceviano.
    Ahora, consideramos las cónicas:
    Ca tangente en Wa a BC y que pasa por Qb, Qc y Q,
    Cb tangente en Wb a CA y que pasa por Qc, Qa y Q,
    Cc tangente en Wc a AB y que pasa por Qa, Qb y Q.

    ( Mostrar/Ocultar figura )
      Tercer punto de Brocard
    Se verifica que las cuatro cónicas Ca, Cb y Cc y la cónica biceviana C(X76,Q) tienen en común el punto ω3(P), situado sobre la cónica inscrita de perspector X76, de coordenadas baricéntricas:
    ( a²(c²v - b²w)² : b²(c²u - a²w)² : c²(b²u - a²v)² ).
    Además, en este punto las tres cónicas Ca, Cb y Cc y la cónica inscrita de perspector X76 tienen recta tangente común.

    El punto ω3(P) es fijo para cualquier punto P sobre una recta que pasa por el simediano.
    Si P está en la recta X1X6, ω3(P)=X1086.
    Si P está en la recta X2X6, ω3(P)=X3124, "Danneels Perspector for X76".
    Si P está en la recta X3X6, ω3(P)=X338.
    Si P está en la recta X513X6, ω3(P)=X4437.

  • sábado, 13 de abril del 2013

    Cónica biceviana C(P,Q) pasando por P

    En un triángulo ABC, sean A1B1C1 y A2B2C2 los triángulos cevianos de los puntos P y Q, respectivamente. Por los seis puntos A1, B1, C1, A2, B2 y C2 pasa una cónica C(P,Q),cónica biceviana de P y Q. Supongamos que C(P,Q) pasa por P, entonces Q ha de estar en la cónica circunscrita de perspector P.

    Si P=(u:v:w) en coordenadas baricéntricas, un punto genérico de la cónica circunscrita de perspector P se puede poner en la forma:
    Q=(b²c²(b²u-a²v)(c²u-a²w)(a²b²+tuv)(a²c²+tuw): c²a²(c²v-b²w)(a²v-b²u)(b²c²+tvw)(b²a²+tvu): a²b²(a²w-c²u)(b²w-c²v)(c²a²+twu)(c²b²+twv)).

    Consideramos las cónicas:
    Ca tangente en A1 a BC y que pasa por B2, C2 y Q,
    Cb tangente en B1 a CA y que pasa por C2, A2 y Q,
    Cc tangente en C1 a AB y que pasa por A2, B2 y Q.

    Se verifica que las cuatro cónicas C(P,Q), Ca, Cb y Cc tienen en común el punto X de primera coordenada baricéntrica:
    u(-2a4b²c²vw + 2b²c²tu²vw + a²u(c²v+b²w)(b²c²-tvw))².

    Además, en este punto las tres cónicas Ca, Cb y Cc tiene como tangente común la recta de ecuación:
    tangente
    Un caso particular: Si P=X76 y t=0 entonces Q=X689. El punto común a las cuatro cónicas C(X76,X689), Ca, Cb y Cc es:
    (b2c2(b4 + c4 - 2a4 )2 : c2a2(a4 + c4 - 2b4)2 : a2b2(a4 + b4 - 2c4)2).


  • viernes, 12 de abril del 2013

    Cónica biceviana C(X76,X689)

    En un triángulo ABC, sean A1B1C1 y A2B2C2 los triángulos cevianos de los centros X76 y X689, respectivamente. Por los seis puntos A1, B1, C1, A2, B2 y C2 pasa una cónica C(X76,X689),cónica biceviana, que además pasa por X76; luego X689, al estar en la circunferencia circunscrita, es el cuarto punto de intersección de ésta con la cónica circunscrita de perspector X76.
    Consideramos las cónicas:
    Ca tangente en A1 a BC y que pasa por B2, C2 y X689,
    Cb tangente en B1 a CA y que pasa por C2, A2 y X689,
    Cc tangente en C1 a AB y que pasa por A2, B2 y X689.

    ( Mostrar/Ocultar figura )
      Cónica biceviana
    Se verifica que las cuatro cónicas C(X76,X689), Ca, Cb y Cc tienen en común el punto X de coordenadas baricéntricas:
    (b2c2(b4 + c4 - 2a4 )2 : c2a2(a4 + c4 - 2b4)2 : a2b2(a4 + b4 - 2c4)2).

    Además, en este punto las tres cónicas Ca, Cb y Cc tiene como tangente común la recta de coeficientes las coordenadas de X755 (recta G-Dual de X755).
    Generalización
    ¿Se tiene estos mismos resultados en otra cónica biceviana C(P,Q) que pasa por P?

    Las rectas que unen cada par de puntos de intersección de las cónicas Ca, Cb y Cc con los lados de ABC, distintos de A1, B1, C1, A2, B2 y C2, determinan un triángulo perspectivo con ABC, y su centro de perspectividad Y es:
    X?

    Propiedad geométrica de la cónica Ca:
    • Es el lugar geométrico del vértice D', del triángulo G-Dual D'E'F' de DEF (triángulo circunceviano de P), cuando P varía en la "hipérbola de Lemoine".
    • La tangente en el punto común a las cónicas Ca, Cb y Cc es la recta G-Dual de X755, cuarto punto de intersección de la circunferencia circunscrita y la "Hipérbola de Lemoine".

  • jueves, 11 de abril del 2013

    "Hipérbola de Lemoine"

    El lugar geométrico de los puntos P, tales que su triángulo circunceviano DEF y el triángulo G-Dual D'E'F' de éste, tienen como centro de perspectividad el centro X689, consta del Eje de Lemoine (tripolar del simediano) y de la hipérbola circunscrita (que podría denominarse "Hipérbola de Lemoine") de perspector X3005, punto de intersección del eje de Lemoine y la recta de De Longchamps (tripolar del conjugado isotómico del simediano).
    La "Hipérbola de Lemoine" contiene a los centros X(6), X(39), X(76), X(141), X(755), X(882), X(1843), X(2353) y X(3954)
    ( Mostrar/Ocultar figura (Caso Eje de Lemoine))
      Eje de Lemoine
    Para todo punto P sobre el Eje de Lemoine, el triángulo D'E'F' queda inscrito en la cónica circunscrita de perspector X76, que pasa por X689.
    El centro de perspectividad P'' de ABC y D'E'F' queda sobre recta de De Longchamps.

    ( Mostrar/Ocultar figura (Caso Hipérbola de Lemoine))
      Hipérbola de Lemoine
    La hipérbola a la que hemos denominado "Hipérbola de Lemoine", puede tener justificado su nombre por las siguientes propiedades:

    - Es la hipérbola circunscrita a ABC que pasa por el punto de Lemoine (simediano=X6) y por su conjugado isotómico (X76).
    - Su centro es X3124, "Danneels Perspector" para X6 (Eric Danneels.- "A Simple Perspectivity," Forum Geometricorum 6 (2006) 199-203).
    - Su perspector es X3005, punto de intersección del eje de Lemoine y de la polar trilineal del conjugado isotómico del simediano (recta de De Longchamps).
    - Para todo punto Y sobre la "Hipérbola de Lemoine" el centro de perspectividad de DEF y D'E'F' es el punto X689 (tripolo de la recta que pasa por X6 y X76).

    - Para todo punto Y sobre la "Hipérbola de Lemoine" el centro de perspectividad Y'' de ABC y D'E'F' queda sobre la cónica biceviana C(X76,X689) que pasa por los pies de las cevianas de X76 y X689; y como también pasa por X76 se verifica que X689 es el cuarto punto de intersección de la circunferencia circunscrita con la cónica circunscrita de perspector X76 (Bernard Gibert.- Bicevian Conics. Corollary 2. C(P,Q) also passes through P if and only if Q lies on the circum-conic with perspector P).

  • martes, 09 de abril del 2013

    Triángulo G-Dual

      Dado un triángulo ABC, para cada punto P de su plano se denota por dP la recta que tienen las mismas coordenadas baricéntricas que P, respecto a ABC. Si P no está en los lados de ABC, geométricamente dP es la tripolar del conjugado isotómico de P. Aunque algebraicamente dP siempre está bien definida. Diremos que dP es la recta G-Dual de P respecto al triángulo ABC.
       Cyril F. Parry en "The Isogonal Tripolar Conic" (Forum Geometricorum 1 (2001) 33–42) llama tripolar isotómica a la recta G-Dual.
       La recta G-Dual de P es la polar, respecto a la elipse circunscrita de Steiner, de la imagen de P mediante la homotecia de centro G y razón -2.
       En particular, si P está sobre una de las elipses de Steiner, su recta G-Dual es tangente a la otra elipse de Steiner.

       Dado un triángulo DEF, se considera el triángulo D'E'F' de vértices los puntos D'=dE∩dF, E'=dF∩dE y F'=dD∩dE. Diremos que D'E'F' es el triángulo G-Dual de DEF.

      Los triángulos DEF y D'E'F' son perspectivos. Además, si D, E y F están en una cónica circunscrita a ABC, el centro de perspectividad también está en la cónica.
      Si ABC y DEF son perspectivos, también son perspectivos ABC y D'E'F'.

    ( Mostrar/Ocultar figura)
      Triángulo G-Dual
    EJEMPLOS:
    • Sea DEF el triángulo circunceviano de un punto P=(u:v:w) y D'E'F' su triángulo G-Dual.
    El centro de perspectividad de DEF y D'E'F', que está en la circunferencia circunscrita a ABC, es:
    X2052


    El centro de perspectividad de ABC y D'E'F' es P''=(c²v + b²w: c²u + a²w: b²u + a²v).
    ( Mostrar/Ocultar figura)
      Triángulo G-dual Circunceviano


    • Sea DEF el triángulo ceviano de un punto P=(u:v:w) y D'E'F' su triángulo G-Dual.
    .
    El centro de perspectividad de ABC y D'E'F' es el conjugado isotómico de P.

    El centro de perspectividad de DEF y D'E'F' es:
    (vw(u²v² + u²w² - v²w²): wu(v²w² + v²u² - w²u²): uv(w²u² + w²v² - u²v²).

    Algunos pares (X,Y) de centros de ETC, tales que Y es el centro de perspectividad del triángulo ceviano de X y su triángulo G-Dual:
    (X2,X2), (X75,X63), (X76,X22), (X85,X3870), (X86,X3995), (X99,X2), ...
  • lunes, 08 de abril del 2013

    Propiedad geométrica de X(2052)

    X2052

    ( Mostrar/Ocultar figura)
      Propiedad X(2052)
    Parábola determinada por su directriz y dos tangentes (dttC_p)

    Otra consideración:
    Sean Ba y Ca los puntos de contacto con los lados AC y AB de la parábola de directriz BC; similarmente, se define los puntos Cb, Ab Ac y Bc de contacto de las otras dos parábolas con los correspondientes lados.
    Los puntos Ba, Ca, Cb, Ab Ac y Bc están en una misma cónica.

    Los puntos A, CbAb∩AcBc, BaAb∩CaAc están en una recta.
    Los puntos B, AcBc∩BaCa, CbBc∩AbBa están en una recta.
    Los puntos C, BaCa∩CbAb, AcCa∩BcCb están en una recta.
    Estas tres rectas concurren en el centro de coordenadas baricéntricas:

    X=(1/(SA2(a4 - SBSC) - SB2SC2): ... : ... )

    con (6-9-13)-número de búsqueda en ETC: 7.600894940587217504799796347
  • domingo, 07 de abril del 2013

    Polar trilineal del punto de Gergonne

    Sea A' el punto de intersección de las tangentes en los vértices a las dos parábolas con foco en A y que pasan por B y C; los puntos B' y C' se definen similarmente. Los puntos A',B' y C' están el la polar trilineal del punto de Gergonne.
    ( Mostrar/Ocultar figura)
      Tripolar_del_punto_de_Gergonne
    Parábolas dadas por el foco y dos puntos (FPPC_p)
Página Personal